Critical Care Medicine Review: 1000 Questions and Answers [1st ed.] 9781975103071

Covering all four critical care board exams (anesthesiology, surgery, internal medicine, and neurology), Critical Care M

7,901 779 23MB

English Pages 2101 Year 2019

Report DMCA / Copyright

DOWNLOAD FILE

Polecaj historie

Critical Care Medicine Review: 1000 Questions and Answers [1st ed.]
 9781975103071

  • Commentary
  • CONVERTED PDF

Table of contents :
Title......Page 2
Copyright......Page 5
Preface......Page 8
Contributors......Page 10
Table of Contents......Page 24
I. Neurologic Disorders......Page 38
1.Brain Death and Degenerative Diseases......Page 39
2.Cerebrovascular Diseases......Page 50
3.Seizure Disorder......Page 73
4.Neuromuscular Disorders......Page 87
5.Increased Intracranial Pressure......Page 98
6.Neurotrauma......Page 115
7.Spinal Cord Injury......Page 144
8.Encephalopathy and Delirium......Page 154
9.Clinical Syndromes......Page 170
10.Inflammatory and Demyelinating......Page 187
11.Neuro Oncology......Page 200
12.Analgesia, Sedation and Neuromuscular Blockade......Page 209
13.Neuro Monitoring and Diagnostic Modalities......Page 221
14.Management Strategies......Page 239
II. Cardiovascular Disorders......Page 259
15.Acute Coronary Syndrome......Page 260
16.Arrhythmias and Pacemaker......Page 286
17.Heart Failure......Page 307
18.Vascular Disorders......Page 318
19.Valvular Heart Disease......Page 337
20.Pericardial Diseases......Page 360
21.Myocardial Disease......Page 378
22.Congenital Heart Disease in Adults......Page 392
23.Shock States......Page 402
24.Mechanical Circulatory Support and the Transplanted Heart......Page 421
25.Calculated Cardiovascular Parameters......Page 435
26.Life Support and Resuscitation......Page 446
27.Imaging and Diagnostic Modalities......Page 460
28.Imaging (Ultrasound)......Page 477
29.Management Strategies (Coagulation, Vasoactive Medications)......Page 498
III. Pulmonary Disorders......Page 512
30.Respiratory Failure......Page 513
31.Hypoxemia and Oxygen Delivery......Page 534
32.Mechanical Ventilation......Page 555
33.Acute Respiratory Distress Syndrome......Page 576
34.Other Parenchymal Disease and Pulmonary Edema......Page 593
35.Airway Diseases......Page 610
36.Diseases of the Chest Wall......Page 630
37.Thromboembolic Disease and Hemoptysis......Page 643
38.Pleural Disorders......Page 655
39.Sleep Apnea......Page 662
40.Pulmonary Infections......Page 672
41.Neoplasm......Page 690
42.Lung Transplantation, Complications, and VV ECMO......Page 707
43.Respiratory Diagnostic Modalities and Monitoring......Page 724
IV. Renal, Electrolyte and Acid Base Disorders......Page 752
44.Acute Renal Failure......Page 753
45.Oliguria and Polyuria......Page 770
46.Renal Replacement Therapy......Page 782
47.Drug Dosing in Renal Failure......Page 796
48.Renal Transplantation......Page 809
49.Diagnosis and Monitoring in Renal Failure......Page 817
50.Sodium......Page 833
51.Potassium......Page 852
52.Calcium, Phosphate, and Magnesium......Page 862
53.Acid Base Disorders......Page 875
V. Endocrine Disorders......Page 903
54.Diabetes Mellitus......Page 904
55.Thyroid......Page 918
56.Parathyroid and Calcium......Page 929
57.Pitutary......Page 938
58.Endocrine Tumors......Page 952
59.Renin-Angiotensin-Aldosterone System......Page 961
60.Hypothalamic-Pituitary-Adrenal Axis......Page 971
61.Management During Critical Illness......Page 986
VI. Infections and Immunologic Disease......Page 1003
62.Systemic Infections......Page 1004
63.CNS Infections......Page 1021
64.Head and Neck, Upper Airway Infections......Page 1044
65.Cardiovascular Infections......Page 1054
66.Gastrointestinal and Intra-abdominal Infections......Page 1063
67.Genitourinary Infection......Page 1084
68.Soft-Tissue, Bone, Joint Infections......Page 1093
69.Antimicrobial Therapy and Resistance......Page 1102
70.Immune Suppression: Congenital, Acquired, Drugs......Page 1118
71.Infections in the Immunocompromised Host......Page 1136
72.Immunological Effects of Infections......Page 1160
73.Bioterrorism......Page 1173
74.Hospital Infection Control, Hospital Acquired Infections......Page 1182
75.Immunological Diseases......Page 1197
VII. Hematologic and Oncologic Disorders......Page 1213
76.RBC Disorders......Page 1214
77.White Blood Cell Disorders......Page 1229
78.Platelet Disorders......Page 1244
79.Coagulopathies......Page 1255
80.Hypercoagulable States......Page 1270
81.Transfusion Medicine......Page 1280
82.Solid Tumors......Page 1290
83.Oncological Syndromes......Page 1300
84.Hemopoietic Cell Transplantation......Page 1311
85.Complications of Immunosuppressive Drugs and Chemotherapy......Page 1323
VIII. Gastrointestinal, Nutrition and Genitourinary Disorders......Page 1332
86.Esophagus......Page 1333
87.Stomach......Page 1343
88.Small Intestine......Page 1352
89.Large Intestine......Page 1363
90.Liver......Page 1375
91.Gallbladder and Biliary Tract......Page 1389
92.Pancreas......Page 1398
93.Genitourinary......Page 1413
94.Diagnostic and Management Modalities......Page 1422
95.Nutrition in Critical Illness......Page 1438
IX. Surgery, Trauma, and Transplantation......Page 1448
96.Cardiothoracic and Vascular Surgery......Page 1449
97.Abdominal and Gastrointestinal Surgery......Page 1480
98.Skin, Soft Tissue, and Extremities......Page 1496
99.Polytrauma......Page 1512
100.Hemorrhage and Resuscitation......Page 1538
101.Environmental Injury......Page 1555
102.Burns......Page 1571
103.Disaster Management......Page 1587
104.Transplantation......Page 1606
X. Pharmacology and Toxicology......Page 1622
105.Basic Pharmacologic Principles......Page 1623
106.Adverse Effects of Drugs......Page 1638
107.Toxins and Poisoning......Page 1652
108.Drug Overdoses......Page 1679
109.Metabolism and Drug Interactions......Page 1691
110.Toxicology and Drugs of Addiction......Page 1708
111.Psychoactive Medications......Page 1727
XI. Research, Administration, and Ethics......Page 1736
112.Research and Biostatistics......Page 1737
113.Administration......Page 1754
114.Teaching......Page 1778
115.Psychosocial Issue Among Providers......Page 1785
116.Ethical Considerations......Page 1791
117.Patient Confidentiality, Healthcare Policy......Page 1801
118.Palliative Care and End of Life......Page 1811
119.Organ Donation......Page 1828
XII. Miscellaneous......Page 1836
120.Procedures......Page 1837
121.Pregnancy......Page 1860
122.Dermatological Disorders......Page 1873
123.Thermoregulatory Disorders......Page 1891
Appendix: Grab Bag......Page 1902
Index......Page 2027

Citation preview

Critical Care Medicine Review 1000 Questions and Answers Abraham Sonny, MD, FASE Assistant Professor Department of Anesthesia, Critical Care and Pain Medicine Massachusetts General Hospital Harvard Medical School, Boston, Massachusetts

Edward A. Bittner, MD, PhD, MS.Ed, FCCM Associate Professor Program Director, Critical Care Anesthesiology Fellowship Department of Anesthesia, Critical Care and Pain Medicine Massachusetts General Hospital Harvard Medical School, Boston, Massachusetts

Ryan J. Horvath, MD, PhD Instructor in Anesthesia Department of Anesthesia, Critical Care and Pain Medicine Massachusetts General Hospital Harvard Medical School, Boston, Massachusetts

Sheri M. Berg, MD Instructor in Anesthesia Medical Director, Post Anesthesia Care Units

Department of Anesthesia, Critical Care and Pain Medicine Massachusetts General Hospital Harvard Medical School, Boston, Massachusetts ASSOCIATE EDITOR

Hassan Farhan, MD Anesthesia Resident Department of Anesthesia, Critical Care and Pain medicine Massachusetts General Hospital Harvard Medical School, Boston, Massachusetts.

Copyright Acquisitions Editor: Keith Donnellan Development Editor: Ashley Fischer Editorial Coordinator: Tim Rinehart Production Project Manager: Kim Cox Design Coordinator: Stephen Druding Manufacturing Coordinator: Beth Welsh Prepress Vendor: TNQ Technologies Copyright © 2020 Wolters Kluwer. All rights reserved. This book is protected by copyright. No part of this book may be reproduced or transmitted in any form or by any means, including as photocopies or scanned-in or other electronic copies, or utilized by any information storage and retrieval system without written permission from the copyright owner, except for brief quotations embodied in critical articles and reviews. Materials appearing in this book prepared by individuals as part of their official duties as US government employees are not covered by the abovementioned copyright. To request permission, please contact Wolters Kluwer at Two Commerce Square, 2001 Market Street, Philadelphia, PA 19103, via email at [email protected], or via our website at shop.lww.com (products and services). 9 8 7 6 5 4 3 2 1 Printed in China Library of Congress Cataloging-in-Publication Data ISBN-13: 978-1-975102-90-6 Cataloging in Publication data available on request from publisher. This work is provided “as is,” and the publisher disclaims any and all warranties, express or implied, including any warranties as to accuracy, comprehensiveness, or currency of the content of this work.

This work is no substitute for individual patient assessment based upon healthcare professionals’ examination of each patient and consideration of, among other things, age, weight, gender, current or prior medical conditions, medication history, laboratory data, and other factors unique to the patient. The publisher does not provide medical advice or guidance, and this work is merely a reference tool. Healthcare professionals, and not the publisher, are solely responsible for the use of this work including all medical judgments and for any resulting diagnosis and treatments. Given continuous, rapid advances in medical science and health information, independent professional verification of medical diagnoses, indications, appropriate pharmaceutical selections and dosages, and treatment options should be made and healthcare professionals should consult a variety of sources. When prescribing medication, healthcare professionals are advised to consult the product information sheet (the manufacturer ’s package insert) accompanying each drug to verify, among other things, conditions of use, warnings, and side effects and identify any changes in dosage schedule or contraindications, particularly if the medication to be administered is new, infrequently used, or has a narrow therapeutic range. To the maximum extent permitted under applicable law, no responsibility is assumed by the publisher for any injury and/or damage to persons or property, as a matter of products liability, negligence law or otherwise, or from any reference to or use by any person of this work. shop.lww.com

Preface This book was conceived as a result of feedback from our critical care fellows on the absence of a comprehensive question and answer book to learn from, during fellowship training. Subsequently, we compiled this book, over a span of one year, to bridge this gap in educational resource. Our goal was to create a resource equipped to help all trainees in critical care, irrespective of their primary discipline. The content of this book was developed from keywords in critical care published by various boards, specifically American board of Anesthesiology, Internal Medicine, Neurology, and Surgery. The chapters were contributed by critical care fellows and junior faculty from various reputed institutions across the United States, working in conjunction with senior authors who are recognized experts in their discipline. This book covers all topics pertinent to the practice of critical care in a question and answer format, divided into twelve section and 123 chapters. A large majority of the questions are clinically oriented with case scenarios, making it pertinent to your clinical practice. After each question, the readers are directed toward relevant references and resources, for additional reading on a certain topic. Furthermore, this book also provides a “grab bag” chapter which contains a random collection of questions from various common topics in critical care. We hope this book can improve your knowledge in critical care medicine especially during fellowship training and also serve as reference guide in future. Abraham Sonny, MD, FASE Edward A. Bittner, MD, PhD, MS.Ed, FCCM Ryan J. Horvath, MD, PhD Sheri M. Berg, MD Boston, Massachusetts

Contributors Noor Abdalla MD Resident, Lahey Hospital, Burlington, Massachusetts Fatima I. Adhi MD Fellow Physician, Critical Care Medicine, Respiratory Institute, Cleveland Clinic, Cleveland, Ohio Avneep Aggarwal MD Staff physician, Anesthesiology Institute, Center for Critical Care Medicine, Cleveland Clinic, Cleveland, Ohio Abdulaziz S. Almehlisi MBBS Assistant Professor, Department of Emergency Medicine, King Saud University, Riyadh, Saudi Arabia Reem Almuqati MD Critical care fellow, Department of Anesthesiology, Cleveland clinic foundation, Cleveland, Ohio John Andre MD Chief of Skills and Simulation, Department of General Surgery, Loyola University Medical Center, Maywood, Illinois Daniel Austin MD Resident, Department of Anesthesia & Perioperative Care, University of California, San Francisco, California Ji Sun Christina Baek MD Department of Anesthesiology, University of California,San Diego, California Theresa Barnes MD, MPH Associate Professor, Department of Anesthesiology, Emory University, Atlanta, Georgia Sean M. Baskin DO, MA Resident, Department of Anesthesiology and Perioperative Medicine, Penn

State Milton S. Hershey Medical Center, Hershey, Pennsylvania DaMarcus Baymon MD Resident, Department of Emergency Medicine, Massachusetts General Hospital - Harvard, Boston, Massachusetts Lisa M. Bebell MD Instructor, Harvard Medical School, Assistant in Medicine, Massachusetts General Hospital, Infectious Diseases Unit, Boston, Massachusetts William J. Benedetto MD Assistant Professor, Department of Anesthesia, Critical Care and Pain Medicine, Massachusetts General Hospital, Boston, Massachusetts Sheri M. Berg MD Instructor in Anesthesia, Medical Director, Post Anesthesia Care Units, Department of Anesthesia, Critical Care and 
Pain Medicine, Massachusetts General Hospital, Harvard Medical School, Boston, Massachusetts Lorenzo Berra MD Reginald Jenney Associate Professor of Anaesthesia, Harvard Medical School, Massachusetts General Hospital, Boston, Massachusetts Leah N. Bess MD Resident, Department of Anesthesiology and Perioperative Medicine, Penn State University Milton S. Hershey Medical Center, Hershey, Pennsylvania Annie van Beuningen MD Fellow in Cardiovascular Medicine, Department of Medicine, Massachusetts General Hospital, Boston, Massachusetts Somnath Bose MD Instructor of Anesthesiology, Harvard Medical School, Department of Anesthesiology, Critical Care and Pain Medicine, Beth Israel Deaconess Medical Center, Boston, Massachusetts Jason K. Bowman MD Chief Resident, Departments of Emergency Medicine, Massachusetts General Hospital, Brigham and Women’s Hospital, Boston, Massachusetts

Joanna Brenneman MD Staff Anesthesiology & Critical Care Medicine, Cleveland Clinic Akron General, Akron, Ohio Edward A. Bittner MD, PhD, MS.Ed, FCCM Associate Professor, Program Director, Critical Care Anesthesiology Fellowship, Department of Anesthesia, Critical Care and 
Pain Medicine, Massachusetts General Hospital, Harvard Medical School, Boston, Massachusetts Erika Lore Brinson MD Assistant Clinical Professor of Anesthesia, Department of Anesthesia and Perioperative Care, University of California, San Francisco, San Francisco, California Lundy Campbell MD Professor, Department of Anesthesiology, Chief, Division of Cardiothoracic Anesthesiology, University of California, San Francisco, California Marvin G. Chang MD Faculty, Department of Anesthesia, Massachusetts General Hospital, Boston, Massachusetts Anoop Chhina MD Anesthesiologist and Intensivist, Department of Anesthesiology, Henry Ford Hospital, Detroit, Michigan Christine Choi MD Assistant Professor, Department of Anesthesiology, University of California, San Diego, San Diego, California Margaret R. Connolly MD Resident, Department of Surgery, Massachusetts General Hospital, Boston, Massachusetts Jennifer Cottral MD Clinical Fellow in Anaesthesia, Department of Anesthesia, Critical Care, and Pain Medicine, Massachusetts General Hospital, Boston, Massachusetts Phat Tan Dang MD

Anesthesiology Critical Care Fellow, Department of Anesthesiology, University of California, San Diego, La Jolla, California Christopher Dinh MD Critical Care Fellow, Department of Anesthesiology, University of California, San Diego, San Diego, California David M. Dudzinski MD, JD Director, Cardiac Intensive Care Unit, Massachusetts General Hospital, Assistant Professor, Harvard Medical School, Boston, Massachusetts Brett Elo DO Department of Anesthesia and Critical Care, Cleveland Clinic, Cleveland, Ohio Faith Natalie Factora MD Medical Director, Surgical Intensive Care Unit, Cleveland Clinic, Cleveland, Ohio Peter Fagenholz MD Assistant Professor of Surgery, Harvard Medical School, Attending Surgeon, Division of Trauma, Emergency Surgery, and 
Critical Care, Department of Surgery, Massachusetts General Hospital, Boston, Massachusetts Hassan Farhan MD Anesthesia Resident, Department of Anesthesia, Critical Care and 
Pain medicine, Massachusetts General Hospital, Harvard Medical School, Boston, Massachusetts Raffaele Di Fenza MD Resident, School of Anesthesia, Critical Care and 
Pain Medicine, University of Milan-Bicocca, Milan, Italy Rachel C. Frank MD Cardiovascular Medicine Fellow, Division of Cardiology, Massachusetts General Hospital, Boston, Massachusetts Kevin E. Galicia MD, MA Resident, Department of General Surgery, Loyola University Medical Center, Maywood, Illinois

Mariya Geube MD, FASE Assistant Professor, Cleveland Clinic Lerner College of Medicine, Department of Cardiothoracic Anesthesiology, Cleveland Clinic Foundation, Cleveland, Ohio Jeffrey Gotts MD, PhD Assistant Professor, Department of Medicine, University of California San Francisco, San Francisco, California Ngoc-Tram Ha MD Pulmonary and Critical Care Fellow, Department of Pulmonary and Critical Care Medicine, Geisinger Medical Center, Danville, Pennsylvania Dusan Hanidziar MD, PhD Instructor in Anesthesia, Department of Anesthesia, Critical Care and Pain Medicine, Massachusetts General Hospital, Boston, Massachusetts Charles Corey Hardin MD, PhD Assistant Professor of Medicine, Division of Pulmonary and Critical Care, Massachusetts General Hospital, Boston, Massachusetts Qasim AlHassan MBBS Anesthesiology Critical Care Fellow, Department of Anesthesiology, Cleveland Clinic Foundation, Cleveland, Ohio Kathryn A. Hibbert MD Instructor in Medicine, Director, Medical ICU, Massachusetts General Hospital, Harvard Medical School, Boston, Massachusetts Kristen Holler DO Anesthesiology Institute, The Cleveland Clinic Foundation, Cleveland, Ohio Ryan J. Horvath MD, PhD Instructor in Anesthesia, Department of Anesthesia, Critical Care and 
Pain Medicine, Massachusetts General Hospital, Harvard Medical School, Boston, Massachusetts Steven Hur MD Fellow, Department of Anesthesia & Perioperative Care, University of California, San Francisco, San Francisco, California

John O. Hwabejire MD, MPH Clinical Fellow in Trauma, Acute Care Surgery, and Surgical Critical Care, Massachusetts General Hospital and Harvard Medical School, Boston, Massachusetts Saef Izzy MD Neurocritical Care faculty, Divisions of Stroke, Cerebrovascular, and 
 Critical Care Neurology, Assistant Professor in Neurology, Department of Neurology, Brigham and Women’s hospital, Harvard Medical School, Boston, Massachusetts Ceena N. Jacob MD Internal Medicine, Cleveland Clinic Foundation, Cleveland, Ohio Todd A. Jaffe MD Emergency Medicine Resident, Harvard Affiliated Emergency Medicine Residency, Brigham and Women’s/Massachusetts General Hospital, Boston, Massachusetts Paul S. Jansson MD, MS Critical Care Medicine Fellow, Brigham and Women’s Hospital, Harvard Medical School, Boston, Massachusetts Teny M. John MD Assistant Professor, Department of Infectious Disease, Infection Control & Employee Health, The University of Texas MD Anderson Cancer Center, Houston, Texas Sonia John MD Critical Care Fellow, Department of Anesthesiology, Massachusetts General Hospital, Boston, Massachusetts Sneha Kannan MD Resident, Department of Medicine, Massachusetts General Hospital, Boston, Massachusetts Kunal Karamchandani MD, FCCP Associate Professor, Department of Anesthesiology and Perioperative Medicine, Penn State Health Milton S. Hershey Medical Center, Hershey,

Pennsylvania Riaz M. Karukappadath MD Assistant Professor, Department of Anesthesiology and Critical Care, University of Alabama at Birmingham, Birmingham, Alabama Sandeep Khanna MD Assistant Professor, Cleveland Clinic Lerner College of Medicine, Staff, Department of General Anesthesiology, Department of Outcomes Research, Cleveland Clinic Foundation, Ohio Mina Khorashadi MD Department of Anesthesia and Perioperative Care, University of California San Francisco, San Francisco, California Thomas J. Krall MD Assistant Professor, Department of Anesthesia and Perioperative Care, University of California - San Francisco, San Francisco, California Nitin Das Kunnathu Puthanveedu MD Fellow Infectious Disease, Cleveland clinic, Cleveland, Ohio Jean Kwo MD Assistant Professor, Department of Anesthesia, Critical Care, and 
Pain Medicine, Massachusetts General Hospital, Boston, Massachusetts Yvonne Lai MD Clinical Instructor, Associate Residency Program Director, Department of Anesthesia, Critical Care, and Pain, Management Massachusetts General Hospital, Boston, Massachusetts Jarone Lee MD, MPH, FCCM Associate Professor, Harvard Medical School, Boston, Massachusetts Nathan M. Lee MD Assistant Professor, Department of Anesthesia and Critical Care, University of Chicago Medical Center, Chicago, Illinois Brian P. Lemkuil MD Associate Professor, Department of Anesthesiology, University of California,

San Diego, California David P. Lerner MD Assistant Professor, Department of Neurology, Lahey Hospital and Medical Center, Burlington, Massachusetts Casey McBride Luckhurst MD Surgical Critical Care Fellow, Massachusetts General Hospital, Boston, Massachusetts Jason H. Maley MD Fellow, Pulmonary and Critical Care Medicine, Massachusetts General Hospital, Boston, Massachusetts Francisco Jesús Marco Canosa MD Fellow, Infectious Disease Department, Cleveland Clinic Foundation, Cleveland, Ohio Maram Marouki MD Critical Care Anesthesia Fellow, Anesthesia Institute, Cleveland Clinic Foundation, Cleveland, Ohio Lydia R. Maurer MD General Surgery Resident, Department of Surgery, Massachusetts General Hospital, Boston, Massachusetts Zeb McMillan MD Associate Professor, Department of Anesthesiology, Division of Critical Care, University of California San Diego, San Diego, California Jenna McNeill MD Pulmonary and Critical Care Fellow, Department of Pulmonary and Critical Care, Massachusetts General Hospital, Boston, Massachusetts April E. Mendoza MD, MPH Instructor, Department of Surgery, Massachusetts General Hospital, Boston, Massachusetts Nino Mihatov MD Fellow in Cardiovascular Medicine, Chief Medical Resident, Massachusetts

General Hospital, Boston, Massachusetts Yuk Ming Liu MD, MPH Assistant Professor, Department of Surgery, Loyola University Medical Center, Maywood, Illinois Anushirvan Minokadeh MD Professor, Department of Anesthesiology, UC San Diego Health, San Diego, California Ilan Mizrahi MD Instructor of Anesthesia, Department of Anesthesia, Critical Care, and 
 Pain Medicine, Massachusetts General Hospital, Boston, Massachusetts Christoph G. S. Nabzdyk MD, MEd Cardiothoracic and Critical Care Anesthesia Fellow, Department of Anesthesia, Critical Care and Pain Medicine, Massachusetts General Hospital, Boston, Massachusetts Revati Nafday MD Adult Cardiothoracic Anesthesiology Fellow, Department of Anesthesia and Perioperative Care, University of California, San Francisco, San Francisco, California Alexander Nagrebetsky MD, MSc Assistant Professor, Department of Anesthesia, Critical Care and 
Pain Medicine, Massachusetts General Hospital, Harvard Medical School, Boston, Massachusetts Alan S. Nova DO Neurocritical Care Fellow, Department of Neurosciences, University of California San Diego, La Jolla, California Nandini C. Palaniappa MD Assistant Professor, Department of Anesthesia and Perioperative Care, University of California, San Francisco, San Francisco, California Riccardo Pinciroli MD Assistant Professor of Anesthesia, University of Milan-Bicocca, School of Medicine and Surgery, Anesthesiologist and Intensivist, Department of

Anesthesia and Critical Care, Niguarda Hospital, Milan, Italy Alexandra Plichta MD Critical Care Fellow, Department of Anesthesia, Critical Care, and 
Pain Medicine, Massachusetts General Hospital, Boston, Massachusetts Kenneth Potter MD Assistant Professor, Department of Anesthesiology, Virginia Commonwealth University Health System, Richmond, Virginia Irfan Qureshi MD, MS Clinical instructor Trauma Surgery, Department of Surgery, Colorado Plains Medical Center, Morgan, Colorado Jeremy T. Rainey DO Fellow, Center for Critical Care Medicine Anesthesiology Institute, Cleveland Clinic, Cleveland, Ohio Phillip Ramirez MD Anesthesia Institute, Cleveland Clinic Foundation, Cleveland, Ohio Kimberly S. Robbins MD Associate Clinical Professor, Department of Anesthesiology and Critical Care, UC San Diego Medical Center, La Jolla, California Martin G. Rosenthal MD Instructor in Surgery, Department of Trauma, Emergency Surgery, Surgical Critical Care, Massachusetts General Hospital, Boston, Massachusetts Galen Royce-Nagel MD Department of Anesthesia, Critical Care, and 
Pain Medicine, Massachusetts General Hospital, Boston, Massachusetts Ofer Sadan MD, PhD Department of Neurology and Neurosugery, Division of Neurocritical Care, Emory University Hospital, Atlanta, Georgia Debdoot Saha MD Fellow, Critical Care Medicine, Geisinger Medical Center, Danville, Pennsylvania

Ulrich Schmidt MD, PhD, MBA Vice Chair Critical Care Medicine, Clinical Professor of Anesthesiology, University of California San Diego, San Diego, California Milad Sharifpour MD, MS Assistant Professor, Department of Anesthesiology and Critical Care Medicine, Emory University Hospital, Atlanta, Georgia Archit Sharma MD, MBA Fellowship Director, Critical Care Fellowship, Department of Anesthesiology, University of Iowa Carver College of Medicine, Iowa City, Iowa Hasan Khalid Siddiqi MD Fellow, Division of Cardiovascular Medicine, Brigham and Women’s Hospital, Boston, Massachusetts Wendy Smith MD Assistant Clinical Professor, Department of Anesthesia & Perioperative Care, University of California at San Francisco, San Francisco, California Abraham Sonny MD, FASE Assistant Professor, Department of Anesthesia, Critical Care and 
Pain Medicine, Massachusetts General Hospital, Harvard Medical School, Boston, Massachusetts Jamie Sparling MD Associate in Anesthesia, Department of Anesthesia, Critical Care and 
Pain Medicine, Boston, Massachusetts Roshni Sreedharan MD Program Director, Anesthesiology Critical Care Medicine Fellowship, Assistant Professor of Anesthesiology, CCLCM, Faculty, Center for Excellence in Healthcare Communication, Department of General Anesthesiology, Anesthesiology Institute/Center for Critical Care Medicine, Cleveland Clinic, Cleveland, Ohio, Chair, In-training section of the SCCM Rachel Steinhorn MD Resident physician, Massachusetts General Hospital, Boston, Massachusetts Alex T. Suginaka DO

Fellow in Critical Care Medicine, Department of Anesthesiology, University of Iowa Hospitals & Clinics, Iowa City, Iowa Jaya Prakash Sugunaraj MD Assistant Professor, Department of Pulmonary & Critical Care, Geisinger, Danville, Pennsylvania Kristina Sullivan MD Professor, Department of Anesthesia and Perioperative Care, University of California, San Francisco, San Francisco, California Madiha Syed MD Clinical Assistant Professor, Anesthesiology Institute and Center for Critical Care, Cleveland Clinic Foundation, Cleveland, Ohio Maryam Bita Tabrizi MD, FACS Clinical Instructor Harvard University, Department of Surgery, Massachusetts General Hospital, Boston, Massachusetts Kevin C. Thornton MD Clinical Professor, Department of Anesthesia and Perioperative Care, University of California San Francisco, San Francisco, California Minh Hai Tran MBBS Assistant Clinical Professor, Department of Anesthesiology, University of California San Diego, San Diego, California William J. Trudo MD Resident Physician, Department of Anesthesiology, Emory School of Medicine, Atlanta, Georgia Aaron C. Tyagi MD Emergency Medicine Critical Care Fellow, Department of Anesthesia, University of Iowa Hospitals and Clinics Bharathram Vasudevan MBBS, MD Critical care fellow, Department of Anesthesiology, University of Iowa Hospitals and Clinics, Iowa City, Iowa Anand Venkatraman MD

Resident, Department of Neurology, Massachussets General Hospital/Brigham and Women’s Hospital/Harvard Medical School, Boston, Massachusetts Brett J. Wakefield MD Critical Care Medicine Fellow, Department of Anesthesiology, Washington University - Barnes Jewish Hospital, St Louis, Missouri Anureet K. Walia MD Clinical Assistant Professor, Department of Anesthesiology, Department of Psychiatry, University of Iowa Carver College of Medicine, Iowa city, Iowa Daniel P. Walsh MD Instructor in Anesthesia, Beth Israel Deaconess Medical Center, Harvard Medical School, Boston, Massachusetts Sarah Welch PharmD, BCCCP Surgical Intensive Care Pharmacy Specialist, Department of Pharmacy, Cleveland Clinic, Cleveland, Ohio Jeanine P. Wiener-Kronish MD Henry Isaiah Dorr Professor of Research and Teaching in Anaesthetics and Anaesthesia, Department of Anesthesia, Critical Care and Pain Medicine, Harvard Medical School, Anesthetist-in-Chief, Massachusetts General Hospital, Boston, Massachusetts Dario Winterton MD School of Anesthesia Critical Care and Pain Medicine, University of MilanBicocca, Milano, Italy Amanda S. Xi MD, MSE Anesthesia Critical Care Fellow, Department of Anesthesia, Critical Care and Pain Medicine, Massachusetts General Hospital, Boston, Massachusetts Howard Zee MD Resident, Department of Anesthesia, Critical Care and Pain Medicine, Massachusetts General Hospital, Boston, Massachusetts

Table of Contents I. Neurologic Disorders 1.Brain Death and Degenerative Diseases David P. Lerner, Anand Venkatraman, and Saef Izzy 2.Cerebrovascular Diseases David P. Lerner and Saef Izzy 3.Seizure Disorder David P. Lerner, Anand Venkatraman, and Saef Izzy 4.Neuromuscular Disorders David P. Lerner and Saef Izzy 5.Increased Intracranial Pressure David P. Lerner and Saef Izzy 6.Neurotrauma David P. Lerner, Anand Venkatraman, and Saef Izzy 7.Spinal Cord Injury Leah N. Bess and Kunal Karamchandani 8.Encephalopathy and Delirium Alexander Nagrebetsky and Jeanine P. Wiener-Kronish 9.Clinical Syndromes Archit Sharma and Alex T Suginaka

10.Inflammatory and Demyelinating Minh Hai Tran, Brian P. Lemkuil, and Ulrich Schmidt 11.Neuro Oncology Milad Sharifpour and Ofer Sadan 12.Analgesia, Sedation and Neuromuscular Blockade Daniel P. Walsh and Somnath Bose 13.Neuro Monitoring and Diagnostic Modalities David P. Lerner, Noor Abdalla, and Saef Izzy 14.Management Strategies Ofer Sadan and Milad Sharifpour

II. Cardiovascular Disorders 15.Acute Coronary Syndrome Nino Mihatov and David M. Dudzinski 16.Arrhythmias and Pacemaker Christoph G. S. Nabzdyk and Yvonne Lai 17.Heart Failure Christoph G. S. Nabzdyk and Yvonne Lai 18.Vascular Disorders Kristen Holler and Mariya Geube 19.Valvular Heart Disease Brett J. Wakefield and Mariya Geube

20.Pericardial Diseases Hasan Khalid Siddiqi and David M. Dudzinski 21.Myocardial Disease Rachel C. Frank and David M. Dudzinski 22.Congenital Heart Disease in Adults Archit Sharma and Aaron C. Tyagi 23.Shock States Jason H. Maley and Kathryn A. Hibbert 24.Mechanical Circulatory Support and the Transplanted Heart Annie van Beuningen and David M. Dudzinski 25.Calculated Cardiovascular Parameters Revati Nafday, Steven Hur, and Lundy Campbell 26.Life Support and Resuscitation Yuk Ming Liu and John Andre 27.Imaging and Diagnostic Modalities Sneha Kannan and David M. Dudzinski 28.Imaging (Ultrasound) Thomas J. Krall and Kevin C. Thornton 29.Management Strategies (Coagulation, Vasoactive Medications) Christoph G. S. Nabzdyk and Yvonne Lai

III. Pulmonary Disorders

30.Respiratory Failure Jeffrey Gotts 31.Hypoxemia and Oxygen Delivery Christine Choi and Jeanine P. Wiener-Kronish 32.Mechanical Ventilation Jason H. Maley and Kathryn A. Hibbert 33.Acute Respiratory Distress Syndrome Rachel Steinhorn and Jeanine P. Wiener-Kronish 34.Other Parenchymal Disease and Pulmonary Edema Raffaele Di Fenza, Riccardo Pinciroli, and 
Lorenzo Berra 35.Airway Diseases Jenna McNeill and Charles Corey Hardin 36.Diseases of the Chest Wall Charles Corey Hardin and Jenna McNeill 37.Thromboembolic Disease and Hemoptysis Maram Marouki, Joanna Brenneman, and Roshni Sreedharan 38.Pleural Disorders Galen Royce-Nagel 39.Sleep Apnea Dario Winterton, Riccardo Pinciroli, and Lorenzo Berra 40.Pulmonary Infections

Jason H. Maley and Kathryn A. Hibbert 41.Neoplasm Jenna McNeill and Charles Corey Hardin 42.Lung Transplantation, Complications, and VV ECMO Archit Sharma and Bharathram Vasudevan 43.Respiratory Diagnostic Modalities and Monitoring Phat Tan Dang, Christopher Dinh, Abdulaziz S. Almehlisi, and Ulrich Schmidt

IV. Renal, Electrolyte and Acid Base Disorders 44.Acute Renal Failure Qasim AlHassan, Madiha Syed, and Roshni Sreedharan 45.Oliguria and Polyuria Abdulaziz S. Almehlisi, Phat Tan Dang, 
Ji Sun Christina Baek, Anushirvan Minokadeh, 
Alan S. Nova, Zeb McMillan, Kimberly S. Robbins, and Ulrich Schmidt 46.Renal Replacement Therapy Riaz M. Karukappadath, Faith Natalie Factora, and Roshni Sreedharan 47.Drug Dosing in Renal Failure Abdulaziz S. Almehlisi, Phat Tan Dang, 
Ji Sun Christina Baek, and Ulrich Schmidt 48.Renal Transplantation Hassan Farhan

49.Diagnosis and Monitoring in Renal Failure Abdulaziz S. Almehlisi, Phat Tan Dang, 
Ji Sun Christina Baek, Anushirvan Minokadeh, 
Alan S. Nova, Zeb McMillan, Kimberly S. Robbins, and Ulrich Schmidt 50.Sodium Jaya Prakash Sugunaraj and Ngoc-Tram Ha 51.Potassium Jaya Prakash Sugunaraj and Debdoot Saha 52.Calcium, Phosphate, and Magnesium Jaya Prakash Sugunaraj and Debdoot Saha 53.Acid Base Disorders Jaya Prakash Sugunaraj and Ngoc-Tram Ha

V. Endocrine Disorders 54.Diabetes Mellitus Jean Kwo 55.Thyroid Kenneth Potter and Kunal Karamchandani 56.Parathyroid and Calcium Nathan M. Lee 57.Pitutary Nathan M. Lee 58.Endocrine Tumors

Nathan M. Lee 59.Renin-Angiotensin-Aldosterone System Nathan M. Lee 60.Hypothalamic-Pituitary-Adrenal Axis Ilan Mizrahi 61.Management During Critical Illness Ilan Mizrahi

VI. Infections and Immunologic Disease 62.Systemic Infections Lisa M. Bebell 63.CNS Infections Nitin Das Kunnathu Puthanveedu and Fatima I. Adhi 64.Head and Neck, Upper Airway Infections Ilan Mizrahi 65.Cardiovascular Infections Marvin G. Chang 66.Gastrointestinal and Intra-abdominal Infections Erika Lore Brinson and Kristina Sullivan 67.Genitourinary Infection Lisa M. Bebell

68.Soft-Tissue, Bone, Joint Infections Ilan Mizrahi 69.Antimicrobial Therapy and Resistance Lisa M. Bebell 70.Immune Suppression: Congenital, Acquired, Drugs Rachel C. Frank and Dusan Hanidziar 71.Infections in the Immunocompromised Host Francisco Jesús Marco Canosa, Fatima I. Adhi, 
Ceena N. Jacob, and Teny M. John 72.Immunological Effects of Infections Rachel C. Frank and Dusan Hanidziar 73.Bioterrorism Jean Kwo 74.Hospital Infection Control, Hospital Acquired Infections Lisa M. Bebell 75.Immunological Diseases Jamie Sparling

VII. Hematologic and Oncologic Disorders 76.RBC Disorders Jean Kwo 77.White Blood Cell Disorders Jean Kwo

78.Platelet Disorders Sean M. Baskin and Kunal Karamchandani 79.Coagulopathies Phillip Ramirez and Somnath Bose 80.Hypercoagulable States Hassan Farhan 81.Transfusion Medicine Nandini C. Palaniappa and Kevin C. Thornton 82.Solid Tumors Milad Sharifpour and Ofer Sadan 83.Oncological Syndromes Milad Sharifpour and Ofer Sadan 84.Hemopoietic Cell Transplantation Jeffrey Gotts 85.Complications of Immunosuppressive Drugs and Chemotherapy Jeffrey Gotts

VIII. Gastrointestinal, Nutrition and 
Genitourinary Disorders 86.Esophagus Irfan Qureshi and Yuk Ming Liu 87.Stomach Margaret R. Connolly and Peter Fagenholz

88.Small Intestine Lydia R. Maurer and Peter Fagenholz 89.Large Intestine Maryam Bita Tabrizi and Martin G. Rosenthal 90.Liver Daniel P. Walsh and Somnath Bose 91.Gallbladder and Biliary Tract Casey McBride Luckhurst and Peter Fagenholz 92.Pancreas Casey McBride Luckhurst and Peter Fagenholz 93.Genitourinary Maryam Bita Tabrizi and Martin G. Rosenthal 94.Diagnostic and Management Modalities Maryam Bita Tabrizi and Martin G. Rosenthal 95.Nutrition in Critical Illness Galen Royce-Nagel

IX. Surgery, Trauma, and Transplantation 96.Cardiothoracic and Vascular Surgery Mina Khorashadi, Daniel Austin, Wendy Smith, 
Revati Nafday, Steven Hur, and Lundy Campbell 97.Abdominal and Gastrointestinal Surgery April E. Mendoza

98.Skin, Soft Tissue, and Extremities Casey McBride Luckhurst and April E. Mendoza 99.Polytrauma Casey McBride Luckhurst, and April E. Mendoza 100.Hemorrhage and Resuscitation Rachel Steinhorn and Galen Royce-Nagel 101.Environmental Injury Lydia R. Maurer and April E. Mendoza 102.Burns John Andre, Kevin E. Galicia, and Yuk Ming Liu 103.Disaster Management Todd A. Jaffe and Jarone Lee 104.Transplantation Reem Almuqati, Sandeep Khanna, and 
Roshni Sreedharan

X. Pharmacology and Toxicology 105.Basic Pharmacologic Principles Sarah Welch and Avneep Aggarwal 106.Adverse Effects of Drugs Anoop Chhina and Avneep Aggarwal 107.Toxins and Poisoning Paul S. Jansson and Jarone Lee

108.Drug Overdoses DaMarcus Baymon and Jarone Lee 109.Metabolism and Drug Interactions Jeremy T. Rainey and Avneep Aggarwal 110.Toxicology and Drugs of Addiction Alexandra Plichta and Sheri M. Berg 111.Psychoactive Medications Archit Sharma and Anureet K. Walia

XI. Research, Administration, and Ethics 112.Research and Biostatistics Edward A. Bittner 113.Administration Jason K. Bowman and Jarone Lee 114.Teaching Ryan J. Horvath 115.Psychosocial Issue Among Providers Jennifer Cottral and William J. Benedetto 116.Ethical Considerations Theresa Barnes, William J. Trudo, and Avneep Aggarwal 117.Patient Confidentiality, Healthcare Policy Ryan J. Horvath

118.Palliative Care and End of Life Jennifer Cottral and William J. Benedetto 119.Organ Donation Nandini C. Palaniappa and Kevin C. Thornton

XII. Miscellaneous 120.Procedures Jennifer Cottral and William J. Benedetto 121.Pregnancy Brett Elo and Mariya Geube 122.Dermatological Disorders Howard Zee and Abraham Sonny 123.Thermoregulatory Disorders Sonia John and Abraham Sonny Appendix: Grab Bag​ Alexandra Plichta, Amanda S. Xi, John O. Hwabejire, 
Rachel Steinhorn, Edward A. Bittner, Sheri M. Berg, and Abraham Sonny Index​​

Neurologic Disorders 1.Brain Death and Degenerative Diseases  3​ 2.Cerebrovascular Diseases  7​ 3.Seizure Disorder  15​ 4.Neuromuscular Disorders  20​ 5.Increased Intracranial Pressure  24​ 6.Neurotrauma  30​ 7.Spinal Cord Injury  40​ 8.Encephalopathy and Delirium  43​ 9.Clinical Syndromes  49​ 10.Inflammatory and Demyelinating  56​ 11.Neuro Oncology  61​ 12.Analgesia, Sedation and Neuromuscular Blockade  64​ 13.Neuro Monitoring and Diagnostic Modalities  68​ 14.Management Strategies  75​

C H AP T E R 1

Brain Death and Degenerative Diseases David P. Lerner, Anand Venkatraman and Saef Izzy

1. An 84-year-old man with coronary artery disease and atrial fibrillation had a ST-segment elevation myocardial infarction that was complicated by a ventricular fibrillation cardiac arrest with 45 minutes of pulselessness. His initial management included evaluation in the cardiac catheterization lab with placement of a bare metal stent into the right coronary artery. He underwent 24 hours of cooling post cardiac arrest, but following this he has had limited neurologic recovery. Post arrest day 6, neurology is consulted. Which of the following is MOST correct?

A. The most accurate prognostic test for poor neurologic outcome is electroencephalography. B. If there are no corneal responses 6 days post cardiac arrest, there is no anticipated neurologic recovery. C. Neuron-specific enolase is not affected by cooling and can be used for prognosis at 6 days post cardiac arrest. D. A magnetic resonance imaging (MRI) can assist with prognosis, and the most commonly affected area of the brain is the cortical region.

2. A 54-year-old woman is admitted to the intensive care unit (ICU) with a subarachnoid hemorrhage due to a left middle cerebral artery aneurysm rupture. Early external ventricular drain was placed because of a poor neurological examination. Over the course of 7 days, there have been ongoing issues with refractory elevated intracranial pressure and poor neurologic examination, progressing to no cranial nerve responses. Her examination is as follows: pupils 5 mm and nonreactive, absent oculocephalic reflexes, absent corneal

reflexes, absence of facial grimace, absent gag, and absent cough. The only evoked motor response is minimal triple flexion in the bilateral legs. An apnea test was completed and there were no spontaneous respirations with an increase in pCO2 20 mm Hg more than baseline. Which of the following is MOST true?

A. Because of the motor response present, the patient does not meet criteria for brain death. B. An ancillary test (EEG, cerebral angiogram, nuclear scan) should be completed to diagnose brain death. C. We need to repeat the apnea test. D. The current examination is consistent with brain death.

3. A 28-year-old previously healthy male is admitted to the ICU with altered mental status and hypoxic respiratory failure following a traumatic brain injury. There is limited history of his actual injury, but the night before his admission, he was intoxicated with friends when he got in an altercation and was hit on left side of his head and did have a brief loss of consciousness. His friends took him home, and at that time he was confused, complaining of a headache but was still talking and walking. He went to bed and he was checked on 10 hours later and was unresponsive. Emergency medical services (EMS) was called, and on arrival he was unresponsive with fixed, dilated pupils and no movements to painful stimulation and was intubated without sedation or paralytic. A head computed tomography (CT) was completed on arrival to the emergency department and demonstrated a 1.4 cm left-sided holocephalic subdural with 1.2 cm of left-to-right midline shift including uncal herniation and midbrain compression. His examination demonstrates lack of brain stem responses, no spontaneous breathing, and no movement to painful stimulation. An apnea test was attempted to evaluate for brain death but was unable to be completed because of hemodynamic instability. A whole brain positron emission tomography (PET) scan was completed and showed no activity in the brain stem and cortex. What is the MOST

correct statement regarding the patient?

A. The patient does not meet brain death criteria because the ancillary study does not support the diagnosis of brain death. B. The patient does not meet brain death criteria because he was unable to complete an apnea test. C. The patient does meet brain death criteria because his clinical examination is consistent with this and did not need an ancillary study. D. The patient does meet brain death criteria because his clinical examination is consistent with this and the ancillary study supports this diagnosis.

4. A 58-year-old woman with amyotrophic lateral sclerosis (ALS) presents to the emergency department complaining of increased weakness and difficulty with feeding herself. Her daughter inquires if there are any interventions which have been proven to increase life expectancy in ALS patients. Which of these is the MOST appropriate answer?

A. Amantadine B. Noninvasive ventilation C. Colostomy D. Prophylactic antibiotics E. Indwelling nasogastric tube

5. A 74-year-old man with hypertension, coronary artery disease, and Parkinson disease is admitted to the ICU for management of pneumonia. He is intubated and started on broad-spectrum antibiotics. Six days after admission, he is found to have worsening fever, rigidity, and is no longer following commands. What is the MOST likely etiology for his condition?

A. Status epilepticus B. Meningitis C. Serotonin syndrome D. Neuroleptic malignant syndrome (NMS) E. Infective endocarditis

Chapter 1 Answers 1. Correct Answer: D Rationale: The extent of brain injury is the key factor for prognostication after cardiac arrest. Clinical examination has been the staple of prognosis— absent pupillary reflexes, absent corneal reflexes, motor response of extensor posturing, or no movement. These findings have come into question in the era of therapeutic cooling. Most importantly, absence of corneal reflexes does not necessarily portent a poor prognosis. Although electroencephalography is for the detection of seizures, the prognostication value has not been validated. Somatosensory evoked potentials (SSEPs) have been studied and the largest study evaluated 407 patients with cardiac arrest, and of the patients with bilaterally absent cortical sensory responses, all had poor neurologic outcome. Pertinent biomarkers include neuron-specific enolase, which was studied in the same SSEP study and levels higher than 33 µg/L predicting a poor outcome, but this study was performed before cooling. Hypothermia can attenuate release of neuron-specific enolase, and there are reports of good outcome with levels greater than 100 µg/L. Although there are limitations to imaging, many use MRI with most useful imaging coming 3 to 5 days post arrest. Common findings are diffuse cortical diffusion restriction and changes in the basal ganglia. Consideration of Evaluation for Prognosis after Cardiac Arrest 0-24 Hours

24-48 Hours

48-72 Hours (Postarrest or Post Rewarming)

3-5 Days (Postarrest or Post Rewarming)

Clinical examination

Clinical examination

Clinical examination

Clinical examination

Continuous EEG

Continuous EEG

SSEP

EEG (if indicated)

Consider NSE

CT

MRI

EEG (if indicated)

Consider NSE

CT, computed tomography; EEG, electroencephalography; MRI, magnetic resonance imaging; NSE, neurospecific enolase; SSEP, somatosensory evoked potential. Adopted from Greer DM. Cardiac arrest and postanoxic encephalopathy. Continuum. 2015;21:13841396.

References

1. Wijdicks EF, Hijdra A, Young GB, et al. Practice parameter: prediction of outcome in comatose survivors after cardiopulmonary resuscitation (an evidence-based review): report of the Quality Standards Subcommittee of the American Academy of Neurology. Neurology. 2006;67:203-210.

2. Booth CM, Boone RH, Tomlinson G, et el. Is this patient dead, vegetative, or severely neurologically impaired? Assessing outcome for comatose survivors of cardiac arrest. JAMA. 2004;291:870-879.

3. Sandroni C, Cavallaro F, Callaway CW, et al. Predictors of poor neurological outcome in adult comatose survivors of cardiac arrest: a systematic review and meta-analysis. Part 1: patients not treated with therapeutic hypothermia. Resuscitation. 2013;84:1310-1323.

2. Correct Answer: D Rationale: The clinical findings necessary to confirm irreversible cessation of all functions of the entire brain, including the brain stem: coma, absence of brain stem reflexes, and apnea. The above patient does demonstrate all three of these criteria consistent with brain death. There are multiple reported reflexive movements that can be seen in patients with the diagnosis of brain death. Patient’s apnea test is consistent with brain death criteria, which is based on increase pCO2 >60 mm Hg or 20 mm Hg more than baseline. Single apnea test is required. No ancillary tests are required if the full clinical examination is consistent with brain death. In some institutions, two assessments of brain stem reflexes are required before

declaring brain death. References

1. Wijdisks EF, Varelas PN, Gronseth GS, et al. Evidence-based guideline update: determining brain death in adults. Neurology. 2010;74:19111918.

2. Han SG, Kim GM, Lee KH, et al. Reflex movements in patients with brain death: a prospecitve study in a tertiary medical center. J Korean Med Sci. 2006;21:588-590.

3. Correct Answer: D Rationale: The diagnosis of brain death is primarily clinical. However, ancillary tests are performed when the clinical criteria cannot be applied reliably. Irreversible coma that is explained by neuroimaging, lack of other etiology that could explain brain death (CNS-depressant drugs, paralytics, electrolyte abnormalities, profound hypothermia, hypotension), and clinical examination are all criteria required to make the diagnosis. The diagnosis of brain death can be challenging in cases when there is unreliable clinical examination or it is not possible to perform apnea test, for which ancillary tests should be performed. In this patient’s case, apnea test was not completed because of hemodynamic instability for which ancillary test is required. There are several ancillary tests including EEG, cerebral angiography, nuclear scan, transcranial Dopplers, CT angiography, and magnetic resonance angiography (MRA). The ideal ancillary test is one with no confounding effects from sedatives or metabolic disturbances and preferable with no false positives. Evaluating cerebral perfusion with fourvessel cerebral angiography and nuclear scan of blood flow is commonly utilized. However, CT angiography and MRA may soon be found to be equally suitable. References

1. Wijdisks EF, Varelas PN, Gronseth GS, et al. Evidence-based guideline update: determining brain death in adults. Neurology. 2010;74:1911-

1918.

2. Karantanas AH, Hadjigeorgiou GM, Paterakis K, et al. Contribution of MRI and MR angiography in early diagnosis of brain death. Eur Radiol. 2002;12:2710-2716.

3. Saposnik G, Maurino J, Bueri J. Movements in brain death. Eur J Neurol. 2001;8:209-213.

4. Mandel S, Arenas A, Scasta D. Spinal automatism in cerebral death. N Engl J Med. 1982;307:501.

4. Correct Answer: B Rationale: Patients with ALS have progressive degeneration of both upper and lower motor neurons causing weakness, difficulty swallowing, and respiratory insufficiency. Although many different approaches have been tried to manage these patients, there are only a few which have shown benefits in increasing life expectancy. Noninvasive ventilation, specifically with optimized bilevel positive airway pressure (BiPAP) protocols, helps to avoid hypercarbia, secondary to diaphragm and respiratory muscle weakness. Other interventions include riluzole, a medication that is thought to work on the neuronal level and shown to increase survival by few months and delay the onset of tracheostomy and ventilator dependence in selected ALS patients. Amantadine is used to promote alertness in patients with neurological injury; however, it has not been shown to be of benefit in ALS. Although patients with ALS often get gastrostomy tubes for safer feeding, colostomy and indwelling nasogastric tubes have not been shown to increase life expectancy. Prophylactic antibiotics are not recommended for ALS as there is no immune suppression. References

1. Amyotrophic Lateral Sclerosis (ALS) Fact Sheet. 2019. Available at https://www.ninds.nih.gov/Disorders/Patient-CaregiverEducation/Fact-Sheets/Amyotrophic-Lateral-Sclerosis-ALS-Fact-Sheet. Retrieved January 24, 2019.

2. Karam CY, Paganoni S, Joyce N, et al. Palliative care issues in amyotrophic lateral sclerosis: an evidenced-based review. Am J Hosp Palliat Care. 2016;33(1):84-92.

3. Hardiman O, Al-Chalabi A, Chio A, et al. Amyotrophic lateral sclerosis. Nat Rev Dis Primers. 2017;3:17071.

5. Correct Answer: D Rationale: NMS is an uncommon but often life-threatening illness characterized by fever, rigidity, obtundation, and autonomic instability. Elevated serum creatine kinase levels can also be seen. Though the pathogenesis is not well understood, it is thought to be related to a sudden decrease in dopaminergic signaling, which may be caused by stopping dopaminergic drugs such as levodopa as well as the use of neuroleptics. It is commonly confused with serotonin syndrome, which is a much more rapidly developing condition caused by excess serotonin signaling. This patient is unlikely to have status epilepticus or stroke, as that is unlikely to cause worsening fevers with rigidity. Infective endocarditis would present with fevers and potentially with neurological deficits if it leads to septic emboli to the brain; however, rigidity, obtundation, and autonomic instability are not typical for infective endocarditis. References

1. Hashimoto T, Tokuda T, Hanyu N, et al. Withdrawal of levodopa and other risk factors for malignant syndrome in Parkinson’s disease. Parkinsonism Relat Disord. 2003;9:25-30.

2. Guzé BH, Baxter LR. Neuroleptic malignant syndrome. N Engl J Med. 1985;313(3):163-166.

C H AP T E R 2

Cerebrovascular Diseases David P. Lerner and Saef Izzy

1. A 49-year-old man with no past medical history is admitted to the medicine service for 2 weeks of intermittent night sweats, myalgia, and progressive headache. Other than febrile, his vital signs are normal at the time of admission. His neurologic examination at the time of admission is normal, and basic laboratory workup is unrevealing. A lumbar puncture is performed with a normal opening pressure, pleocytosis with 41 white blood cells/microL (94% polymorphonuclear cells), 5 red blood cells/microL, glucose 58 mg/dL, and protein 53 mg/dL. There was concern for potential infectious meningitis, so vancomycin, ceftriaxone, and acyclovir were started. One day following the lumbar puncture, the patient had acute onset of marked expressive aphasia and right facial weakness. A head computed tomography (CT) was completed and demonstrated in the figure that follows.

Additional workup was completed to determine the etiology of his stroke, and he was found to have a mobile target on the anterior leaflet of the mitral valve, concerning for endocarditis (shown in the figure that follows), and the mitral valve has severe mitral valve regurgitation.

What is the next best step in management of the patient’s possible endocarditis? A. The patient should undergo urgent (within 5 days) mitral valve replacement. B. The patient should undergo urgent (within 5 days) mitral valve repair. C. The patient should undergo delayed (∼4 weeks) mitral valve replacement. D. The patient should undergo delayed (∼4 weeks) mitral vale repair. E. Only antibiotics therapy is needed, and current antibiotics should remain the same.

2. A 53-year-old woman is postbleed day 8 from a subarachnoid hemorrhage (SAH) from a, now secured, right middle cerebral artery (MCA) aneurysm. Since admission, the patient has been closely watched and the data from her external ventricular drain

(EVD), brain tissue oxygenation monitor, and microdialysis catheter are all monitored. Recordings from the previous day shows PbtO2 (partial pressure of brain tissue O2) to be consistently greater than 25 and lactate/pyruvate (L/P) ratio less than 35, while her most recent readings from this morning are noted in the table that follows.

7 am

8 am

9 am

10

11 am 29

PbtO2

22

15

15

am 28

Brain temp Cerebral perfusion pressure (CPP) Lactate (L) Pyruvate (P) L/P ratio

37.9 86

38.2 74

38.2 78

38.2 101

37.6 109

6.3 150 42.1

7.9 172 46

8.1 180 45

8.2 221 36.9

7 199 35.1

What is the best next step in management to reduce the patient’s risk of delayed cerebral ischemia? A. Continue to maintain increased cerebral perfusion pressure (CPP) B. Targeted temperature management C. Transfuse one unit of packed red blood cells D. Start continuous electroencephalogram (cEEG) monitoring

3. A 47-year-old male with untreated hypertension was brought to the emergency department for loss of consciousness while at home and an episode of emesis. A head CT demonstrated a diffuse SAH with intraventricular extension and early signs of hydrocephalus. A CT angiogram demonstrated a 6 mm fusiform aneurysm from the distal right posterior inferior cerebellar artery (PICA). He was admitted to the intensive care unit for ongoing management. Over the course of the evening, there was progressive somnolence and an EVD was placed with an elevated opening pressure. Once placed, the EVD remained clamped. Given the need to delay definitive management of the aneurysm, transexamic acid (TXA) was started. Within 8 hours of admission, there was acute worsening of the examination and an acute increase in the intracranial pressure (ICP), and on opening the

EVD, blood actively drained. What of the following is MOST true of aneurysmal rerupture?

A. Aneurysm rerupture does not change functional outcome of those who survive. B. Posterior circulation aneurysms are more common to have rerupture than anterior circulation aneurysms. C. The use of TXA decreases the risk of rebleeding and improves clinical outcomes. D. Placement of an EVD increases the risk of rebleeding. E. Rerupture more commonly occurs within the first 3 to 5 days, following the initial aneurysm rupture.

4. A 67-year-old female has had progressive tinnitus over the last 4 years. Initial laboratory workup has been unrevealing, so additional workup with brain imaging was completed. The CT angiogram is shown in the figure that follows.

Which of the following is MOST true regarding incidentally discovered aneurysms? A. Saccular aneurysms are most amenable to treatment with intraarterial coiling. B. Incidentally discovered aneurysms without any symptoms do not

warrant further evaluation or intervention. C. Aneurysms located in the posterior circulation have a higher risk of rupture than the anterior circulation. D. The average annual rupture rate of all incidentally discovered aneurysm is around 10%.

5. A 69-year-old male, who has not seen a doctor in at least 10 years, presents to the emergency department for left-sided numbness and weakness. He initially had symptoms 1 day before presentation that lasted for 30 minutes with complete recovery. A head CT did not demonstrate any ischemic changes. A magnetic resonance imaging (MRI) did not demonstrate any infarction. A magnetic resonance angiogram (MRA) of the intracranial and neck vessels demonstrated severe stenosis of the right MCA. What is the next best step management?

A. Consultation to endovascular service for angioplasty and placement of a stent across the right MCA stenosis B. Start intravenous (IV) heparin infusion with bolus of heparin C. Start dual antiplatelet therapy with aspirin and clopidogrel D. Start single antiplatelet therapy with aspirin following a loading dose

6. A 73-year-old woman, with prior parietal intraparenchymal hemorrhage approximately 6 months prior and ischemic stroke approximately 2 years prior, presents to the emergency department after being found slumped in a chair at home and unresponsive. She was intubated for airway protection, and a head CT demonstrated a large left frontal intracerebral hemorrhage with intraventricular extension and SAH as well as 5 mm of left-to-right midline shift. Her pertinent medications at home are metoprolol 25 mg daily and aspirin 81 mg daily. Her vital signs in the emergency department are heart rate (HR) 86, blood pressure (BP) 124/68, and SpO2 98% on 40% FiO2. Her basic metabolic panel and complete blood count are

normal. Her only medication at the time of evaluation is propofol for sedation. What medications/treatments should be added to the patient’s current regimen?

A. Platelet transfusion B. Platelet transfusion and levetiracetam C. Platelet transfusion, levetiracetam, and labetalol infusion D. Levetiracetam and labetalol infusion E. No additional medications are needed

7. A 64-year-old man with hypertension is brought to the emergency department for acute-onset (within the last 60 minutes) left face, arm, and leg weakness. A noncontrast head CT is completed and does not demonstrate a hemorrhage or early ischemic changes. His vitals are BP 174/120, HR 76, and SpO2 99% on room air. A fingerstick blood glucose was obtained and was 127, but other labs are pending. What is the next BEST step in management for the patient?

A. Administration of tissue plasminogen activator (tPA) at 0.9 mg/kg with initial bolus of 10% total dose and 90% via infusion. B. Await coagulation profile (international normalized ratio [INR] and partial thromboplastin time [PTT]) and platelet count before treatment. C. Place a nasogastric tube and Foley catheter followed by administration of tPA. D. Administration of 182 mg rectal aspirin, given patient’s dysarthria. E. Administration of labetolol 10 mg IV push.

8. A 69-year-old female with hypertension presents to the emergency department from home following acute onset of slurred speech and left facial droop. She was with her family watching television when her daughter noted the symptoms. Emergency medical services (EMS) was called and noted left facial droop, left arm weakness, and

dysarthria. Her initial vitals were unremarkable other than a BP of 212/92. She was treated with IV labetolol with BP improvement and was treated with IV tPA. Thirty minutes into the infusion she complained of a headache and became less responsive. Her BP was 190/86, and the tPA was stopped. A repeat head CT was completed and is shown in the figure that follows.

Axial noncontrast head CT. There is a large right MCA acute ischemic stroke with edema and effacement of the sulci. There is hemorrhage present within the area of ischemia centered in the right basal ganglia and insula.

Other than BP management, what is the next best step in management of

her current neurologic issue? A. Administration of 10 mg IV vitamin K followed by 5 mg IV daily for 3 days B. Administration of at least 2500 units of prothombin complex concentrate C. Administration of aminocaproic acid (Amicar) 10 g IV in 250 mL NS IV over 1 hour or TXA with load of 1 g over 10 minutes and 1 g over the following 8 hours D. Administration of cryoprecipitate E. Administration of fresh frozen plasma

9. A 54-year-old man with no past medical history was brought into the emergency department by his wife for altered mental status. On arrival to the emergency department, the only pertinent history and findings were an ongoing holocephalic headache and some confusion. A noncontrast head CT demonstrated a right frontoparietal intraparenchymal hemorrhage. The patient was stabilized and taken for a diagnostic angiogram, which is shown in the figure that follows. The figure is a right internal carotid injection projected as an anterior-posterior view. He was diagnosed with an arteriovenous malformation (AVM).

Which of the following statements is true regarding this patient and his AVM? A. This AVM is not concerning, given that it does not arise directly from the internal carotid artery. B. Because the AMV has already bled, there is a lower risk of rehemorrhage in the future. C. AVMs are typically congenital and increase the lifetime risk of seizure and intracerebral hemorrhage.

D. An AVM is a direct connection between arteries and veins with normal brain tissue surrounding the abnormal vessels.

Chapter 2 Answers 1. Correct Answer: D Rationale: The patient meets Duke criteria for possible endocarditis, given he has one major criteria (transthoracic echocardiography [TTE] with new regurgitation and mobile target) and two minor criteria (fever and emboli). The patient has a severe mitral regurgitation, which will require surgical repair. There are mixed criteria for early intervention, but the American Heart Association recommends early intervention if any of the following is observed: Valve dysfunction causing symptoms or signs of heart failure Paravalvular extension of infection with development of annular or aortic abscess Destructive or penetrating lesion causing heart block Infection from a difficult-to-treat pathogen such as fungal or highly resistant organism Persistent infection after the start of appropriate antibiotics

This patient does not meet the criteria above, and there is no need to repair his mitral valve urgently. In addition, given his intraparenchymal hemorrhage, undergoing anticoagulation is not an option at this acute time of his presentation. Overall, endocarditis patients with intracerebral hemorrhage are at high risk for clinical worsening during the first month after symptom onset and have a higher mortality than those without (75% vs. 40%). References

1. Durack DT, Lukes AS, Bright DK. New criteria for diagnosis of infective endocarditis: utilization of specific echocardiographic findings. Duke Endocarditis Service. Am J Med. 1994;96:200-209.

2. Baddour LM, Wilson WR, Bayer AS, et al. Infective endocarditis in adults: diagnosis, antimicrobial therapy, and management of complications: a scientific statement for healthcare professional from the American Heart Association. Circulation. 2015;132:1435-1486.

2. Correct Answer: A

Rationale: All SAH patients are at risk for delayed cerebral ischemia and cerebral infarction during the acute and subacute stages of their disease. Some centers use multimodality monitoring for evaluation of delayed cerebral ischemia as well as evaluation of interventions attempting to prevent or treat this disease. This particular patient has a brain tissue sensor which measures the partial pressure of oxygen in the brain interstitial cortical tissue. PbtO2 is a balance between oxygen delivery and oxygen consumption in brain cells. It can be affected by a number of parameters, such as cerebral metabolism, cerebral blood flow, sedation, low inspired oxygen, ICP and CPP changes, and other traumatic changes in the cellular environment. The microdialysis catheter is a semipermeable membrane that allows diffusion of water and solutes down the concentration gradient, and is used to measure the concentrations of these solutes. This particular example has both glucose, lactate, and pyruvate as markers of cerebral metabolism. During times of oligemia, glucose level will drop and lactate levels will rise, marking a shift to anaerobic metabolism. In this scenario, PbtO2 and lactate-pyruvate ratio (LPR) were within goal the day before and changed significantly this morning to show low brain oxygenation and increased LPR as shown in the table. The goal with PbtO2 monitoring is to assess which change will have the greatest effect on PbtO2 trends. In this case, increasing CPPs (as shown in the table) reversed the changes in PbtO2 and LPR (this morning) and improved brain oxygenation (PbtO2) and metabolism. References

1. Helbok R, Olson DM, Le Roux PD, Vespa P. Intracranial pressure and cerebral perfusion pressure monitoring in non-TBI patients: special considerations. Neurocrit Care. 2014;21(suppl 2):S85-S94.

2. Ngwenya LB, Burke JF, Manley GT. Brain tissue oxygen monitoring and the intersection of brain and lung: a comprehensive review. Resp Care. 2016;69:1232-1244.

3. de Lima Oliveira M, Kairalla AN, Fonoff ET, et al. Cerebral microdialysis in traumatic brain injury and subarachnoid hemorrhage: state of the art. Neurocrit Care. 2014;21:152-162.

3. Correct Answer: E Rationale: This SAH patient has suffered a rerupture of his PICA aneurysm. Rebleeding is a major complication of SAH and a major cause of mortality, which can occur at any time during the course. Early rebleeding has been commonly reported to take place between 3 and 5 days, yet the exact period of great risk is still debated. The correlation between risk of rebleeding and predictors such as poor clinical SAH grades, loss of consciousness, external ventricular drainage, and size of aneurysm has been debated. Posterior circulation aneurysms are more likely to rupture, yet overall they are not associated with an increased risk of rerupture. In an attempt to decrease the risk of rebleeding, TXA has been evaluated in SAH patients and has shown to decrease the rate of early rebreeding; however, it did not associate with improvement in clinical outcome. References

1. Naidech AM, Janjua N, Kreiter KT, et al. Predictors and impact of aneurysmal rebleeding after subarachnoid hemorrhage. Arch Neurol. 2005;62:410-416.

2. Starke RM, Connolly ES Jr; Participants in the International MultiDisciplinary Consensus Conference on the Critical Care Management of Subarachnoid Hemorrhage. Rebleeding after aneurysmal subarachnoid hemorrhage. Neurocrit Care. 2011;15:24106.

4. Correct Answer: C Rationale: The tinnitus is likely an unrelated symptom, but there is CT angiogram evidence of an incidentally discovered broad-based unruptured aneurysm arising from the left vertebral artery. The overall annual incidence of aneurysm rupture is 1.1% to 1.4%. Patient factors that increase the risk of aneurysmal rupture include smoking, female sex, and posterior circulation, and patient age inversely (younger patients at higher risk) increased the risk of rupture. Aneurysm factors that increase the risk of rupture include

larger size in anterior circulation >7 and >6 mm in posterior circulation, multilobulated aneurysm, posterior circulation, and aneurysm growth in serial imaging are associated with increased risk of rupture. The different approaches to management of unruptured, and ruptured, aneurysm are open surgery with ligation or wrapping of the aneurysm and endovascular therapy with coiling of the aneurysm with or without stent assistance. References

1. Juvela S, Poussa K, Lehto H, et al. Natural history of unruptured intracranial aneurysms: a long-term follow-up study. Stroke. 2013;44:2414-2421.

2. Wermer MJH, van der Schaff IC, Algra A, et al. Risk of rupture of unruptured intracranial aneurysms in relation to patient and aneurysm characteristics. Stroke. 2007;38:1404-1410.

5. Correct Answer: C Rationale: This patient presents with a crescendo transient ischemic attack (TIA), defined as recurrent episodes of TIAs over hours to as long as 1 week. The potential mechanism may be embolic or hemodynamic. There are prior studies that examine patients with stroke or TIA attributed to stenosis of 70% to 99% diameter of a major intracranial artery—SAMPRIS trial. In this trial, patients were randomized to percutaneous transluminal angioplasty and stenting with aggressive medical management versus aggressive medical management alone (aspirin and clopidogrel, management of primary and secondary risk factors including lifestyle modification). In those undergoing stenting there was increased risk of stroke and death at 30 days and 1 year compared to medical therapy alone. A second trial (POINT) evaluated the use of dual antiplatelet therapy versus single antiplatelet agent in patients with small ischemic stroke or TIAs due to intracranial atherosclerotic disease in China. Dual antiplatelet therapy is associated with a reduction in stroke recurrence from 11.7% to 8.2%. Although some advocate for anticoagulation in crescendo TIA, there is limited evidence of efficacy, but there are data that heparinization is safe.

References

1. Johnston DCC, Hill MD. The patient with transient cerebral ischemia: a golden opportunity for stroke prevention. CMAJ. 2004;107:1134-1137.

2. Chimowitz MI, Lynn MJ, Derdeyn CP, et al. Stenting versus aggressive medical therapy for intracranial arterial stenosis. N Engl J Med. 2011;365:993-1003.

3. Wang Y, Wang Y, Zhao X. Clopidogrel with aspiring in acute minor stroke or ischemic attack. N Engl J Med. 2013;369:11-19.

6. Correct Answer: E Rationale: There are no additional medications that are needed at this time. Given her intracerebral hemorrhage, she should have aggressive BP management to maintain a systolic BP goal of 185/>110. Treatment of the BP to below this level can then allow for tPA administration, so option E is the right choice. Absolute Contraindication

Relative Contraindication

Acute intracranial hemorrhage History of intracranial hemorrhage Severe uncontrolled hypertension Serious head trauma or stroke within 3 mo Thrombocytopenia or coagulopathy Including use of direct thrombin inhibitor, factor Xa inhibitor, and heparinoids unless able to demonstrate lack of drug Severe hypo/hyperglycemia

Early ischemic changes on head CT Advanced age (>75 y old) Mild or improving symptoms Severe stroke and coma Recent major surgery

Arterial puncture of noncompressible vessel Recent gastrointestinal or genitourinary hemorrhage Seizure at onset Recent myocardial infarction Central nervous system lesion Neoplasm, AVM, or aneurysm Dementia

References

1. Jauch EC, Saver JL, Adams HP Jr, et al. AHA/ASA guideline: guidelines for the early management of patients with acute ischemic stroke. Stroke. 2013;44:870-947.

2. Fugate JE, Rabinstein AA. Absolute and relative contraindications to IV rt-PA for acute ischemic stroke. Neurohospitalist. 2015;5:110-121.

8. Correct Answer: D Rationale: For every 100 patients treated with tPA, 1 patient will experience a severely disabled or fatal outcome as a result of tPA-related hemorrhage. The treatment of symptomatic hemorrhage following tPA administration has not been studied in a randomized fashion. In patients with hypofibrinoemia (level ipsilateral weakness B. Subfalcine herniation—contralateral > ipsilateral leg weakness C. Transtentorial (uncal) herniation—contralateral weakness and anisocoria

D. Tonsillar herniation—marked depressed level of consciousness and bulbar dysfunction

2. A 22-year-old male has an external ventricular drain placed for intracranial pressure monitoring following a traumatic brain injury. The patient continues to have a poor neurologic examination. There are episodes of time when he has an elevation in his intracranial pressure to 20 mm Hg that will last for 10 to 20 minutes at a time. During this time there are no changes to other vital signs. A tracing of the external ventricular drain (EVD) is shown below. What is the phenomenon depicted?

A. Lundberg A wave B. Lundberg B wave C. Lundberg C wave

D. Autonomic storming

3. An 18-year-old male is brought in to the emergency department after a motor vehicle crash. He was on a motorcycle and was found 30 feet away from his vehicle in unconscious state without a helmet. He has noted trauma to his left forearm, active bleeding from the left posterior portion of his scalp. He was intubated in the field with EMS. On arrival he is tachycardic (heart rate 120s), blood pressure 110/62. He was recently paralyzed for intubation and does not have twitches present on train of four testing. A head CT is completed which demonstrates a large left-sided subdural hematoma with 6 mm midline shift. In the interval, as neurosurgery team is taking the patient to the OR, hyperventilation is started to decrease cerebral edema. How does hyperventilation decrease intracranial pressure?

A. Change in CSF pH results in constriction of vascular smooth muscle B. Change in intracranial blood pH results in constriction of vascular smooth muscle C. Change in CSF pH results in electrical quiescence of neurons, decreased metabolic demand, and decreased cerebral blood flow D. Change in pCO2 sensed by carotid body chemoreceptors results in vasoconstriction E. A drop in pCO2 leads to a large osmotic gradient and worsening interstitial edema

4. A 32-year-old female with chronic alcoholism and cirrhosis was brought to the emergency department following a night of binge drinking. She was found unresponsive at home and EMS was called. On arrival to the emergency department she was intubated for airway protection. She did not require sedation for intubation and is not currently on any sedation. A head CT was completed and demonstrated diffuse cerebral edema with effacement of the sulci and ventricular system. Her lab results were remarkable for an

elevation in AST and ALT (2000, 1000 units/L respectively), total bilirubin 5.6 mg/dL, and ammonia 3642 µm/L. Her examination remains poor, with only extensor posturing to motor stimulation. Her pupils are 5 mm and sluggishly reactive to light. Given the findings on head CT what is the next best step in management?

A. Hyperventilate the patient with goal pCO2 20 mm Hg B. Start sedation with a midazolam infusion at 1 mg/h C. Placement of an intraparenchymal monitor for intracranial pressure monitoring and guidance of therapy D. Infusion of mannitol at 1 g/kg and if needed repeat every 6 hours E. Emergent liver transplantation

5. A 62-year-old male was admitted with diffuse subarachnoid hemorrhage, intraventicular extension of the hemorrhage, and early signs of hydrocephalus. An EVD was placed without complication. The EVD has functioned well and following placement the patient’s examination improved. The figure that follows is the waveform produced. What occurs during the P3 peak?

A. P3 represents the arterial pulse B. P3 represents the cerebral compliance C. P3 represents the respiratory ICP wave D. P3 represents the dicrotic notch

6. A 59-year-old female with no past medical history is admitted to the neurointensive care unit following a large right middle cerebral artery ischemic stroke. She was not a candidate for intravenous tPA nor intra-arterial therapy. She has had progressive somnolence and anisocoria with a right larger than left pupil that was not responsive to direct or consensual light testing. She was started on hyperosmolar therapy followed by a decompressive hemicraniectomy. She is now poststroke day 5 and continues to have

malignant cerebral edema. Prior to her next dose of mannitol her lab values are as follows:

What is her osmolar gap? A. 0 B. 6 C. 14 D. 53

7. A 53-year-old, right-handed, previously independent male with no past medical history was found down at home. He was brought to the emergency department where he was found to have a large, wellestablished L MCA ischemic stroke on CT with a proximal L M1 thrombus on CT angiogram. He was not a candidate for IV tPA or intra-arterial therapy given the well established infarction and his last known well-being over 12 hours prior to presentation. He was admitted to the intensive care unit for ongoing management. Over the course of the next 24 hours he has progressive decline in his mental status and required intubation. A repeat head CT shows evolution of the ischemic stroke with left-to-right midline shift. What is the best description of the anticipated outcome following a hemicraniectomy for malignant ischemic stroke?

A. A hemicraniectomy can potentially be a life-saving intervention and will improve neurologic recovery B. A hemicraniectomy can potentially be a life-saving intervention but will not improve neurologic recovery C. Because the ischemic stroke is on the dominate hemisphere, hemicraniectomy should be completed as there is a higher chance of recovery D. Hemicraniectomy without durotomy is preferred as there is a lower risk of infection as compared to hemicraniectomy with durotomy

Chapter 5 Answers 1. Correct Answer: B Rationale: The head CT above demonstrates an acute on subacute left-sided subdural hemorrhage with left-to-right midline shift and compression of the left lateral ventricle and potential trapping of the right lateral ventricle. The timing of the subdural hemorrhage is based on the density of the blood product. Acute blood appears hyperdense to brain parenchyma. After approximately 3 days the blood product begins to break down and will reach an isodense characteristic compared to brain parenchyma, typically around 10 to 14 days. After 21 days, the blood product becomes hypodense to brain tissue and is similar to cerebrospinal fluid. The herniation demonstrated on this image is subfalcine with left frontal lobe shift under the falx into the right hemisphere. Subfalcine herniation results with cingulate gyrus herniation across the falx and compression of the pericallosal arteries resulting in contralateral (and commonly bilateral) leg weakness. Transtentorial or uncal herniation occurs with the medial temporal lobe and uncus compresses the ipsilateral cerebral peduncle resulting in compression of the ipsilateral third cranial nerve resulting in an enlarged and fixed or sluggishly reactive pupil and contralateral weakness. Tonsillar herniation occurs when there is a pressure within the cerebellum resulting in herniation of the cerebellar tonsil into the foramen magnum and compression of the fourth ventricle resulting in noncommunicating hydrocephalus and obtundation. Fungating herniation occurs when there is a skull defect and herniation of brain tissue outside the cranial vault. All herniation occurs due to a pressure gradient from one compartment (left-right hemisphere, or supra-infratentorial compartment). References

1. Brant WE, Helms CA. Fundamentals of Diagnostic Radiology. Lippincott Williams & Wilkins; 2007.

2. Blumenfeld H. Neuroanatomy Through Clinical Cases. 2nd ed. Sinauer Associates; 2010.

2. Correct Answer: A Rationale: The intracranial pressure (ICP) waveform shown is the Lundberg A, which are a sustained elevation in ICP with amplitude of 50 to 100 mm Hg (higher than Lundberg B and C waves). They typically last for 5 to 20 minutes and they are always pathological as they represent reduced cerebral compliance and increase intracranial pressure. Lundberg B waves are short elevation of ICP with amplitude of 5 to 20 mm Hg at a frequency of 0.5 to 2 waves/min, which last for 1 to 5 minutes. They are thought to be normal waves with probably some association with unstable ICP and vasospasm. Lundberg C waves are rapid oscillations of 4 to 8 waves/min with low amplitude of P2 > P3. If P2 > P1, it is indicated that ICP is likely elevated and the intracranial compliance is likely decreased. The respiratory ICP waveform correlates with the respiratory cycle. References

1. Cardoso ER, Rowan JO, Galbraith S. Analysis of the cerebrospinal fluid pulse wave in intracranial pressure. J Neurosurg. 1983;59:817-821.

2. Doyle DJ, Mark PW. Analysis of intracranial pressure. J Clin Monit. 1992;8:81-90.

3. Oshio K, Onodera J, Uchida M, et al. Assessment of brain compliance using ICP waveform analysis in water intoxication rat model. Acta Neurochir Suppl. 2013;118:219-221.

4. Kirkness CJ, Mitchell PH, Curr RL, et al. Intracranial pressure waveform analysis: clinical and research implications. J Neurosci Nurs. 2000;32:271-277.

6. Correct Answer: A Rationale: The osmolar gap estimates the unknown osmotic agent in the blood, which in this patient’s case is mannitol. To calculate the osmolar gap we need to calculate the calculated osms.

Mannitol is a hypertonic solution of sugar that is used to treat cerebral edema. The medication works as an osmotic diuretic which causes large volume urinary output due to high concentrated urine within the distal collecting duct and allows for extraction of extracellular fluid into the bloodstream due to the osmotic gradient between the intravascular and extravascular, extracellular compartment. There are a number of complications that can occur with mannitol therapy including volume depletion, electrolytes imbalance such as hyponatremia, and metabolic acidosis. Acute kidney injury can also occur secondary to dehydration and mannitol accumulation. Therefore, mannitol should only be given within specific parameters including osmolar gap 20 cm H2O E. None of the above

5. A 40-year-old male is admitted to the ICU after being stabbed in the back during a bar fight. Neurological examination reveals loss of all sensation at the T8 level, loss of proprioception and vibration below T8 on one side, and loss of pain and temperature sensation below T8 on the other side. Motor strength is impaired on the same side as

loss of proprioception and vibration. Which of the following syndrome best describes the findings on neurological examination in this patient?

A. Brown-Sequard syndrome (BSS) B. Central cord syndrome C. Anterior cord syndrome D. Complete spinal cord injury (SCI)

Chapter 7 Answers 1. Correct Answer: C Rationale: SCI can lead to neurogenic shock, which consists of bradycardia and severe arterial hypotension. It is due to autonomic nervous system malfunction and is caused by the lack of sympathetic activity, through loss of supraspinal control and unopposed parasympathetic tone via intact vagus nerve. Lesions between T1-T4 interrupt the cardiac accelerator fibers resulting in significant bradycardia along with hypotension, decreased vascular tone, and venous pooling. Lesions at or above T7 cause impaired functioning of intercostal muscles, which causes reduction in VC and expiratory reserve volume, leading to hypoventilation and hypoxia. Disruption of the sympathetic nervous system due to SCI also results in impaired thermoregulatory function secondary to interruption of signal transmission to the hypothalamic temperature regulating center. This leads to hypothermia, which is characteristic of neurogenic shock. References

1. Krassioukov AV, Claydon VE. The clinical problems in cardiovascular control following spinal cord injury: an overview. Prog Brain Res. 2007;152:223-229.

2. Grigorean VT, Sandu AM, Popescu M, et al. Cardiac dysfunctions following spinal cord injury. J Med Life. 2009;2(2):133-145.

3. Miller RD, Miller ED, Reves JG, et al. Anesthesia. Vols. 297–298. 7th ed. New York, NY: Churchill Livingstone; 2009:2299.

2. Correct Answer: D Rationale: This patient has most likely suffered spinal cord concussion (SCC), which is a variant of mild SCI, clinically designated as transient paraplegia or neurapraxia, and characterized by variable degrees of sensory

impairment and motor weakness that typically resolve within 24 to 72 hours without permanent deficits. Many patients show signs of recovery with the first few hours after injury and completely recover within 24 hours. Spinal cord injuries are classified as concussions if they met three criteria: (1) spinal trauma immediately preceded the onset of neurological deficits; (2) neurological deficits were consistent with spinal cord involvement at the level of injury; and (3) complete neurological recovery occurred within 72 hours after injury. SCC is predominantly a sport-related injury occurring in a wide variety of contact sports in adult and pediatric athletes including wrestling, hockey, gymnastics, and diving, but most commonly in American football. Because the injury is self-resolving, no further treatment is needed. There is, however, controversy over whether players who suffer SCC have a higher likelihood of sustaining SCI in future and whether they should be cleared for return-to-play. References

1. Fischer I, Haas C, Raghupathi R, Jin Y. Spinal cord concussion: studying the potential risks of repetitive injury. Neural Regen Res. 2016;11(1):5860.

2. Zwimpfer TJ, Bernstein M. Spinal cord concussion. J Neurosurg. 1990;72(6):894-900.

3. Correct Answer: D Rationale: Spinal cord instability results when at least two of the three spinal columns (anterior, middle, and posterior) are disrupted. The most common mechanism of injury is blunt force involving acceleration-deceleration; these patients should be approached with a high degree of suspicion until injuries have been ruled out radiographically. Injuries to the thoracolumbar region are common in the setting of flexion forces and typically involve T11-L3. Bilateral calcaneus fractures typically result from high impact forces and are also associated with an increased incidence of spinal fractures and require a thorough thoracolumbar evaluation. In contrast to blunt spinal cord trauma, penetrating injuries are less likely to

result in spinal instability and may not require placement of c-collars and immobilization. Damage caused by penetrating injuries occur at the time of the initial trauma making the risk of subsequent exacerbation less likely than with blunt spinal cord trauma. References

1. Stuke LE, Pons PT, Guy JS, Chapleau WP, Butler FK, McSwain NE. Prehospital spine immobilization for penetrating trauma–review and recommendations from the prehospital Trauma Life Support Executive Committee. J Trauma. 2011;71(3):763-769; discussion 769-770.

4. Correct Answer: C Rationale: SCI results in significant morbidity and mortality. Improving neurological recovery by reducing secondary injury is a major principle in the management of SCI. To minimize secondary injury, maintaining adequate spinal cord perfusion using blood pressure (BP) augmentation has been advocated. Spinal cord perfusion pressure (SCPP) is the difference between the diastolic blood pressure (DBP) and Intraspinal pressure (ISP) or intracranial pressure (ICP). [SCPP = DBP−ISP/ICP]. Increasing the DBP, potentially increases the SCPP, thus improving perfusion to the injured spinal cord. Current recommendations according to the guidelines of the American Association of Neurological Surgeons/Congress of Neurological Surgeons (AANS/CNS) Joint Section on Spine and Peripheral Nerves advise correcting hypotension and maintaining a MAP goal of 85 to 90 mm Hg for 7 days postinjury. The use of methylprednisolone after acute SCI is debatable, and there is unclear evidence about the efficacy and clinical impact of methylprednisolone in recovery from SCI. Consensus statements consider methylprednisolone as a treatment option for acute SCI, but not a standard of care based on available evidence. SBP or CVP has limited impact on improving spinal cord perfusion. References

1. Breslin K, Agrawal D. The use of methylprednisolone in acute spinal cord injury: a review of the evidence, controversies, and recommendations. Pediatr Emerg Care. 2012;28(11):1238-1245.

2. Walters BC, Hadley MN, Hurlbert RJ, et al. Guidelines for the management of acute cervical spine and spinal cord injuries: 2013 update. Neurosurgery. 2013;60(suppl 1):82-91.

3. Saadeh YS, Smith BW, Joseph JR, et al. The impact of blood pressure management after spinal cord injury: a systematic review of the literature. Neurosurg Focus. 2017;43(5):E20.

5. Correct Answer: A Rationale: Incomplete SCI is defined as partial injury to the cord that results in varying degrees of residual sensory and motor function. The site of the injury dictates the findings on neurological examination. BSS or lateral hemi-section syndrome represents a spinal cord hemisection in its pure form. It involves injury to the dorsal column, corticospinal tract, and spinothalamic tract unilaterally, which results in weakness, loss of vibration, and proprioception ipsilateral to, and loss of and temperature sensation contralateral to the injury. Sensory loss of all modalities at the level of the lesion is often seen. BSS is usually secondary to penetrating SCI but can be rarely seen from transverse myelitis after influenza vaccination or a ruptured pheochromocytoma. Management is conservative with aggressive early rehabilitation. Surgical intervention is indicated in the presence of cerebrospinal fluid leak, persistent spinal cord/root compression, or progressive deterioration. BSS demonstrates a favorable prognosis compared with other types of incomplete spinal cord injuries. Central cord syndrome is the most common of the clinical syndromes, often seen in individuals with underlying cervical spondylosis who sustain a hyperextension injury (most commonly from a fall) and may occur with or without fracture and dislocations. This clinically presents as an incomplete injury with greater weakness in the upper limbs than in the lower limbs. The anterior cord syndrome is a relatively rare syndrome that historically

has been related to a decreased or absent blood supply to the anterior twothirds of the spinal cord. The dorsal columns are spared, but the corticospinal and spinothalamic tracts are compromised. The clinical symptoms include a loss of motor function, pain sensation, and temperature sensation at and below the injury level with preservation of light touch and joint position sense. Complete SCI would lead to complete paralysis and absence of sensation below the level of the injury. References

1. Moskowitz E, Schroeppel T. Brown-Sequard syndrome. Trauma Surg Acute Care Open. 2018;3:e000169.

2. Kirshblum SC, Burns SP, Biering-Sorensen F, et al. International standards for neurological classification of spinal cord injury (revised 2011). J Spinal Cord Med. 2011;34:535-546.

C H AP T E R 8

Encephalopathy and Delirium Alexander Nagrebetsky and Jeanine P. Wiener-Kronish

1. A 34-year-old previously healthy female was admitted to the hospital in labor. She was hypertensive on admission and complained of right upper quadrant pain. Fifteen minutes after delivery she developed a generalized onset motor seizure and was intubated and admitted to the ICU. Her seizure is MOST likely:

A. A late initial manifestation of a preexisting epilepsy B. A complication of delivery before normalization of blood pressure C. Associated with subcortical vasogenic cerebral edema on imaging D. A complication of high dermatomal level of epidural analgesia/anesthesia E. Associated with thrombocytosis and cerebral vascular occlusion

2. A 63-year-old man with a history of hypertension controlled with three agents and type 2 diabetes treated with metformin presents with new-onset confusion, nausea, and vomiting. His daughter states that he had self-discontinued his antihypertensive medications. He is normoglycemic but hypertensive with systolic blood pressure consistently above 200 mm Hg. Brain imaging did not show evidence of acute hemorrhage or ischemic changes; chest imaging was unremarkable. Assuming that the patient’s symptoms are due to hypertension, the recommended goal for blood pressure reduction during the first hour is:

A. 120 mm Hg. Confusion, nausea, and vomiting suggest hypertensive encephalopathy as the most likely diagnosis. The current American College of Cardiology/American Heart Association (ACC/AHA) guidelines from 2017 for management of hypertension in adults recommend reducing blood pressure by a maximum of 25% over the first hour. The blood pressure goal for the following 2 to 6 hours is 160/100 to 110 mm Hg. Blood pressure should be normalized over the following 24 to 48 hours. Blood pressure goals for the first hour are different in patients with severe preeclampsia or pheochromocytoma crisis (8 mm are associated with impaired consciousness and shifts >11 mm are typically consistent with coma. In uncal herniation syndrome, lateral forces lead to asymmetric herniation of the temporal uncus. The ipsilateral oculomotor nerve (CN III) is displaced and stretched, leading to pupillary dilation, downward and outward eye deviation (C). Subsequently, contralateral pupil reactivity may be lost with midbrain damage (B). Hemiplegia occurs with compression of the corticospinal tract, not the spinothalamic tract (D). Vestibulo-ocular reflexes are present and normal early until brainstem compression occurs (E). Clinically, these patients develop signs of elevated ICP as identified in the Cushing triad (hypertension, bradycardia, irregular respirations). Papilledema is

consistent with ICP elevation but not a part of the triad (A). References

1. Edlow JA, Rabinstein A, Traub SJ, Wijdicks EF. Diagnosis of reversible causes of coma. Lancet. 2014;384:2064.

2. Simonetti F, Uggetti C, Farina L, et al. Uncal displacement and intermittent third nerve compression. Lancet. 1993;342:1431.

3. Wijdicks EF, Giannini C. Wrong side dilated pupil. Neurology. 2014;82:187.

C H AP T E R 1 0

Inflammatory and Demyelinating Minh Hai Tran, Brian P. Lemkuil and Ulrich Schmidt

1. A 21-year-old male presents to the emergency department with a history of fevers to 100.4°F, headache, nausea, and vomiting for the last 48 hours. He has a history of tonic-clonic seizures for which he takes phenytoin. He has recently started taking ibuprofen for his headaches. He has no allergies. He has not had his flu shot this year, and no one else is unwell in his family. On examination, he is lying down in a dark room and requests for you to avoid turning on the light. He is somnolent but has no focal weakness. He is unable to flex his neck without discomfort.

The emergency room physician had empirically started ceftriaxone and vancomycin and performed a lumbar puncture with the following results. CSF

Normal Ranges

Results

Color WBC RBC Protein Glucose Gram stain

Clear 10 L of urine output per day. Lumbar puncture reveals clear cerebrospinal fluid (CSF) with WBC count 22 cells/mm3, protein 280 mg/dL, and glucose 25 mg/dL. The gram stain and culture were negative. A slice of her MRI brain imaging showing leptomeningeal enhancement (arrows) is shown in the following figure.

(Image courtesy of Smith JK, Matheus MG, Castillo M. Imaging Manifestations of Neurosarcoidosis. Am J Roentgenol. 2004;182(2):289-295.)

Which test would be MOST LIKELY to assist with the diagnosis of her condition?

A. Tensilon test B. Chest computed tomography C. Synacthen test D. 24-hour urinary electrolytes

3. A homeless 35-year-old alcoholic male with a body mass index of 18 kg/m2 is admitted for seizures, encephalopathy, and dysarthria. On presentation, he had a serum sodium of 108 mmol/L, potassium 2.4 mmol/L, chloride 98 mmol/L, alanine aminotransferase 356 IU/L, aspartate aminotransferase 450 IU/L, and gamma-glutamyl transpeptidase 1200 IU/L. He was started on lactulose, thiamine replacement, and saline infusions. CT scan of brain was unremarkable. He recovered within 36 hours, coincident with a sodium correction to 130 mmol/L. The patient then deteriorated on day 6 to a catatonic state with flaccid paralysis of all extremities. Which of the following MOST LIKELY caused his deterioration?

A. Administration of thiamine before glucose B. Nonconvulsive status epilepticus C. Aggressive nutritional support and elevated phosphate D. Saline infusion

4. A 22-year-old Scandinavian female presents with acute symptoms of vision loss, headaches, fatigue, and leg weakness. She reports that her mother has a history of multiple sclerosis (MS). MRI demonstrates inflammatory lesions of the same age found in multiple areas. Despite various treatment modalities, the patient progressed to death 6 months later.

Which of the following MS variant diseases is MOST likely? A. Balo concentric sclerosis B. Marburg variant

C. Schilder disease D. Devic disease

5. A 5-year-old male presents with irritability, ataxia, headaches, and progressive somnolence. His mother reports fevers after receiving his vaccinations for measles, mumps, and rubella, 3 weeks ago. The pediatrician is highly concerned for acute disseminated encephalomyelitis (ADEM). The mother believes that the recent vaccinations are the cause of the child’s symptoms.

Which of the following statements is LEAST true? A. The disease is typically monophasic. B. Recovery can occur in 50% to 75% of cases. C. Vaccines have been linked to ADEM. D. First-line therapy is plasmapheresis.

Chapter 10 Answers 1. Correct Answer: C Rationale: Aseptic meningitis refers to the patient population that have negative CSF gram stain and cultures but laboratory and clinical evidence of meningeal irritation. The most common causes are the enteroviruses such as coxsackie virus and echovirus. Additional etiologies form a fairly extensive list which includes other infections (mycobacteria, fungi, spirochetes), parameningeal infections (HIV, herpes simplex, varicella zoster, Epstein-Barr virus, cytomegalovirus, human herpes virus-6, and adenoviruses), medications (ibuprofen), and malignancies (lymphoma). CSF from bacterial meningitis would have positive gram stains, greatly elevated WBCs >150 cells/mm3, high protein, and reduced glucose levels. Seizures may be associated with CSF pleocytosis and transiently elevated CSF protein levels, however, the clinical presentation does not fit this. Drug-induced aseptic meningitis is primarily a diagnosis of exclusion and unlikely to have elevated WBCs and RBCs. A fungal infection would be a very rare cause of aseptic meningitis in an immunocompetent individual; however, HIV testing would likely be warranted in this diagnostic workup. References

1. Connolly KJ, Hammer SM. The acute aseptic meningitis syndrome. Infect Dis Clin North Am. 1990;4(4):599.

2. Jarrin I, Sellier P, Lopes A, et al. Etiologies and management of aseptic meningitis in patients admitted to an internal medicine department. Medicine. 2016;95:e2372.

3. Chatzikonstantinou A, Ebert AD, Hennerici MG. Cerebrospinal fluid findings after epileptic seizures. Epileptic Disord. 2015;17(4):453-459.

4. Kupila L, Vuorinen T, Vainionpää R, Hukkanen V, Marttila RJ, Kotilainen P. Etiology of aseptic meningitis and encephalitis in an adult population. Neurology. 2006;66(1):75.

2. Correct Answer: B Rationale: Sarcoidosis is a rare multisystem noncaseating granulomatous disease,

which affects 3 to 10 per 100 000 among Caucasians and 35 to 80 per 100 000 among African Americans. 5% to 10% of patients with systemic sarcoidosis will have neurological involvement whereas only 17% of patients will solely present with neurosarcoidosis. Any component of the central or peripheral nervous system can be affected. Cranial mononeuropathies are common, in particular, the peripheral facial nerve is affected in 25% to 50% of neurosarcoidosis cases. Neuroendocrine dysfunction may manifest as polyuria or disturbances in temperature, libido, or appetite due to hypothalamic inflammation. Diabetes insipidus may either be central from hypopituitarism or nephrogenic from activated macrophage calcitriol causing hypercalcemia. Lung parenchyma and mediastinal lymph nodes are the most common sites affected peripherally with reported involvement in 24% to 68% of individuals presenting with neurologic sarcoidosis. The diagnosis is based on clinical symptoms, CSF evidence of inflammation (pleocytosis, elevated CSF protein), and/or MRI imaging demonstrating leptomeningeal enhancement especially around the base of the brain (arrows); however, neurosarcoidosis can involve the bone, dura mater, nerve roots, and brain parenchyma. Confirmation of systemic sarcoidosis may occur either through positive histology or two indirect indicators (gallium scan imaging, chest imaging demonstrating enlarged lymphadenopathy, or elevated angiotensin-converting enzyme [ACE] levels). Corticosteroids remain the mainstay of initial treatment followed by immunomodulatory therapy. Chest CT scan with evidence of hilar adenopathy or parenchymal changes is consistent with sarcoidosis and may also guide a tissue biopsy diagnosis, and thus is the correct answer. Tensilon test is useful in diagnosis of myasthenia gravis and is performed by administrating edrophonium which prevents acetylcholine breakdown causing improvement in symptoms. Synacthen test (or cosyntropin test) uses adrenocorticotropin hormone (ACTH) to evaluate adrenal gland function. 24-hour urinary electrolytes may help confirm suspicion of diabetes insipidus but would not assist with the diagnosis of neurosarcoidosis. References

1. Burns TM. Neurosarcoidosis. Arch Neurol. 2003;60(8):1166. 2. Joseph FG, Scolding NJ. Neurosarcoidosis: a study of 30 new cases. J Neurol Neurosurg Psychiatry. 2009;80(3):297.

3. Stuart CA, Neelon FA, Lebovitz HE. Disordered control of thirst in hypothalamic-pituitary sarcoidosis. N Engl J Med. 1980;303(19):1078.

4. Smith JK, Matheus MG, Castillo M. Imaging manifestations of neurosarcoidosis. Am J Roentgenol. 2004;182:289-295.

5. Pawate S, Moses H, Sriram S. Presentations and outcomes of neurosarcoidosis: a study of 54 cases. Q J Med. 2009;102:449-460.

3. Correct Answer: D Rationale: Rapid correction of serum sodium with saline infusions (or hypertonic saline) in patients with chronic hyponatremia can cause central pontine myelinolysis (CPM) or osmotic demyelination syndrome. CPM was originally described in 1959 as a disease associated with malnourished alcoholics. Extrapontine myelinolysis was recognized in 1962, and the link with rapid correction of hyponatremic patients was found in 1982. It remains a rare disease with a biphasic presentation. Patients typically are encephalopathic or seizing upon presentation. Laboratory testing usually reveals profound hyponatremia (Na 70 microV), positive transient followed by a negative deflection with an anterior-posterior gradient and anterior-posterior delay. Triphasic waves were initially described in hepatic encephalopathy but have been described in multiple metabolic derangements, and they can be associated with nonconvulsive seizures. Treatment of triphasic waves with benzodiazepine will result in improvement in the discharges, but it may not change the clinical picture of the patient. A trial of benzodiazepine might help in the diagnosis of nonconvulsive status. A positive benzodiazepine test if there is resolution of potentially ictal EEG pattern and improvement in clinical state or appearance of previously absent normal EEG pattern. If there is demonstration of the above, then treatment with an AED would be appropriate. Imipenem and other carbapenem medications have been associated with increased risk of seizures; with imipenem use, 4/1000 patients have seized. References

1. Brigo F, Storti M. Triphasic waves. Am J Electroneurodiagnostic Technol. 2011;51:16-25.

2. Fountain NB, Waldman WA. Effects of benzodiazepines on triphasic waves: implications for non-convulsive status epilepticus. 
J Clin Neurophysiol. 2001;18:345-352.

3. Jirsch J, Hirsch LJ. Nonconvulsive seizures: developing a rational approach to the diagnosis and management in the critically ill population. Clin Neurophysiol. 2007;118:1660-1670.

4. Cannon JP, Lee TA, Clark NM, et al. The risk of seizures among the carbapenems: a meta-analysis. J Antimicrob Chemother. 2014;69:20432055.

2. Correct Answer: D

Rationale: MuSK is a tyrosine kinase receptor found on muscle which is important in the maintenance of the neuromuscular junction. The repetitive stimulation presented in the question demonstrates the typical findings of myasthenia gravis, a decrement of >33% at 3 Hz cycling. Other findings on electrodiagnostic testing include increased jitter on single fiber electromyography. MuSK-positive myasthenia gravis can present with crisis as their initial presentation and typically have predominately ocular and bulbar symptoms. Treatment of crisis is the same as for other myasthenia gravis patients, which is initial treatment with a rapidly acting intervention, either IVIG or plasmapheresis, and concurrent or shortly after with chronic immunosuppressive therapy. MuSK positive myasthenia patients appear to have improved early response with 93% responding to plasmapheresis and only 61% responding to IVIG. Although initiation of steroids is the mainstay of treatment of other myasthenia gravis patients, MuSK antibody patients do not respond as well. These patients respond better to rituximab for chronic immunosuppressive therapy. References Hoch W, McConville J, Helms S, et al. Auto-antibodies to the receptor tyrosine kinase MuSK in patients with myasthenia gravis without acetylcholine receptor antibodies. Nat Med. 2001;7:365-368. Schwartz MS, Stalberg E. Myasthenia gravis with features of the myasthenic syndrome. An investigation with electrophysiologic methods including single-fiber electromyography. Neurology. 1975;25:80-84. Nuptial JT, Sanders DB, Evoli A. Anit-MuSK antibody myasthenia gravis: clinical findings and response to treatment in two large cohorts. Muscle Nerve. 2011;44:36-40. Sanders DB, Guptill JT. Myasthenia gravis and Lambert-Eaton myasthenic syndrome. Continuum. 2014;20. Nuptial JT, Sanders DB. Update on muscle-specific tyrosine kinase antibody positive myasthenia gravis. Cure Opin Neurol. 2010;23:530-535.

3. Correct Answer: C

Rationale: Transcranial Doppler ultrasound (TCD) is a noninvasive, easily reproducible test that can be used to monitor for cerebral artery vasospasm following aneurysmal subarachnoid hemorrhage. It can be used for other disease evaluation including emboli detection, brain death evaluation, and sickle cell disease. TCD uses low-frequency (2 MHz) pulse Doppler to evaluate velocity of blood flow through the proximal intra- and extracranial arteries. Although a good screening tool, it has limited sensitivity (90%), specificity (70%), and positive predictive value (57%) when compared to digital subtraction angiography. Early studies using TCD evaluated the diameter of blood vessels and the mean velocity. Mean velocities >120 cm/s correlated with decrease in blood vessel diameter by 50%. The patient’s transcranial Doppler ultrasounds are normal despite having angiographic and clinical vasospasm which can be seen given the sensitivity of the testing. More importantly, there are limitations in the ability to TCD to detect vasospasm in major blood vessels other than the middle cerebral and basilar arteries. Although not present here, elevated velocities may not be a sign of vasospasm. At times there can be hyperemia resulting in increased velocities throughout. The advent of the Lindegaard ratio can be used to further evaluate elevation in velocities by comparing the mean velocity in the middle cerebral artery with that of the ipsilateral extracranial internal carotid artery (MEANmca/MEANeica) with ratios of 3 to 6 consistent with mild spasm and >6 moderate spasm while a ratio of 3 and mean velocity >120 cm/s correlated with clinical vasospasm 85% and angiographic vasospasm 83.2%. There are many treatment options for angiographic vasospasm and verapamil works by blocking L-type calcium channels and results in relaxation of smooth muscle. Following treatment with verapamil mean TCD velocities will decrease rather than increase. References

1. Purykayastha S, Sorond F. Transcranial Doppler ultrasound: technique and application. Semin Neurol. 2012;32:411-420.

2. White H, Venkatesh B. Application of transcranial Doppler in the ICU: a review. Intensive Care Med. 2006;32:981-994.

3. Kumar G, Shahripour RB, Harrrigan MR. Vasospasm on transcranial Doppler is predictive of delayed cerebral ischemia in aneurysmal subarachnoid hemorrhage: a systematic review and meta-analysis. J Neurosurg. 2016;124:1257-1264.

4. Lindegaard KF, Nornes H, Bakke SJ, et al. Cerebral vasospasm diagnosis by means of angiography and blood velocity measurements. Acta Neurochir (Wien). 1989;100:12-24.

5. Gonzalez NR, Boscardin WJ, Glenn T, et al. Vasospasm probability index: a combination of transcranial doppler velocities, cerebral blood flow and clinical risk factors to predict cerebral vasospasm after aneurysmal subarachnoid hemorrhage. J Neurosurg. 2007;107:11011112.

6. Sayama CM, Jiu JK, Caldwell WT. Update on endovascular therapies for cerebral vasospasm induced by aneurysmal subarachnoid hemorrhage. Neurosurg Focus. 2006;21:E12.

4. Correct Answer: C Rationale: The patient presents with a subarachnoid hemorrhage in the setting of a basilar tip aneurysm which is seen on the sagittal reformatted images shown. The description of the patient’s presentation is best described by scales that have been previously developed that assist with long-term outcome as well as anticipated complications during the hospitalization. The Hunt and Hess score was developed as a tool for assessment of death. There are five grades with the lowest scores having better prognosis and higher having worse prognosis (Hunt and Hess Grade table). With the course of time and improved treatment strategies, the prognosis has improved, but those with the highest grade have a high mortality (71%). The Hunt and Hess grading system is based on the clinical exam of the patient. The second commonly used grading system for subarachnoid hemorrhage (SAH) is the modified Fisher scale (Modified Fisher Grade table). This is a radiographic-based scale which uses the thickness of subarachnoid blood and intraventricular extension to predict clinically relevant vasospasm. Vasospasm is a well-defined complication of subarachnoid hemorrhage which results in vasoconstriction of the cerebral

vasculature through both calcium-dependent and calcium-independent pathways and results in cerebral ischemia. Hunt and Hess Grade Grade Description

Mortality (%)

FollowUp Mortality (%)

I

11

3

26

3

37

9

71

24

100%

71

Asymptomatic or minimal headache and slight nuchal rigidity Moderate or severe headache, nuchal rigidity, no neurologic deficit other than cranial nerve palsy Drowsiness, confusion, or mild focal deficit Stupor, moderate to severe hemiparesis, possibly early decerebrate rigidity and vegetative disturbance Deep coma, decerebrate posturing, moribund appearance

II

III IV

V

Modified Fisher Grade Grade 1 2 3 4

Description

Vasospasm (%)

Focal or diffuse thick SAH, without intraventricular hemorrhage Focal or diffuse thin SAH, with intraventricular hemorrhage Thick SAH without intraventricular hemorrhage Thick SAH with intraventricular hemorrhage

24 33 33 40

References

1. Hunt WE, Hess RM. Surgical risks as related to time of intervention in the repair of intracranial aneurysms. J Neurosurg. 1968;28:14-20.

2. Antigua H, Ortega-Gutierrez S, Schmidt JM, et al. Subarachnoid hemorrhage: who dies, and why? Crit Care. 2015;31;309-316.

3. Frontera JA, Claaassen J, Schmidt JM, et al. Prediction of symptomatic vasospasm after subarachnoid hemorrhage: the modified Fisher Scale.

Neurosurgery. 2006;59:21-27.

4. Kolas AG, Sen J, Belli A. Pathogenesis of cerebral vasospasm following aneurysmal subarachnoid hemorrhage: putative mechanisms and novel approaches. J Neurosci. 2009;87:1-11.

5. Correct Answer: B Rationale: Given the findings on MRI, the most likely cause of her cerebellar hemorrhage is hypertension. This location is one of the common places for hypertensive bleeds (other being thalamus, basal ganglia, and pons). This is thought to be due to degeneration of the internal elastic lamina and resultant lipohyalinosis of the small vessels of the brain. The MRI of patients with hypertensive angiopathy will demonstrate areas of microhemorrhage within the same vascular regions as macrohemorrhages. Cerebral amyloid angiopathy is the most common cause of lobar intraparenchymal hemorrhage in older adults. This is the result of betaamyloid protein deposition within the intima and media of the large and medium-sized intracranial vasculature. Autoimmune vasculitis is another important etiology of intraparenchymal hemorrhage. The typical imaging finding is both ischemic and hemorrhagic infarcts of varying age. The clinical presentation of autoimmune vasculitis can mimic the presentation above, but there is typically a prodromal phase with progressive symptoms. Lastly, infective endocarditis results in intraparenchymal hemorrhage from formation of mycotic aneurysms. Infected embolic material from the affected valve will embolize and can result in formation of microhemorrhages at the sites as well as macrohemorrhages if the aneurysms rupture. Vessel imaging with CT angiography, MR angiography, or digital subtraction arteriogram are best for evaluating an aneurysm. References

1. Campbell GJ, Roach M. Fenestration in the internal elastic lamina at bifurcations of human cerebral arteries. Stroke. 1981;12:489-496.

2. Fisher CM. Pathologic Observations in hypertensive cerebral hemorrhage. J Neuropathol Exp Neurol. 1971;30:536-550.

3. Tsushima Y, Aoki J, Endo K. Brain microhemorrhages detected on T2*weighted gradient-cheo MR images. AJNR Am J Neuroradiol. 2003;24(1):88-96.

4. Rosand J, Greenberg SM. Cerebral amyloid angiopathy. Neurologist. 2000;6:315-325.

5. Pomper MG, Miller TJ, Stone JH, et al. CNS vasculitis in autoimmune disease: MR imaging findings and correlation with angiography. Am J Neuroradiol. 1999;20:75-85.

6. Correct Answer: C Rationale: The differential diagnosis for subarachnoid hemorrhage is broad, including aneurysm, trauma, vascular tumors, dural arteriovenous fistula, and arteriovenous malformation. The head CT above demonstrates an isolated perimesencephalic subarachnoid hemorrhage. Although less common than the typical basilar subarachnoid hemorrhage, perimesecephalic subarachnoid hemorrhage can be the result of an arterial aneurysm (typically verterbrobasilar), but 95% are idiopathic despite extensive evaluation. Patients presenting with isolated perimesencephalic subarachnoid hemorrhage appear similar with thunderclap headache, nausea, emesis, but typically do not have a loss of consciousness or decreased level of consciousness. Despite CT angiograms being negative, they will undergo typically multiple digital subtraction angiograms to evaluate for an underlying vascular lesion and/or aneurysm. There is no standard of care practice on evaluation of perimesencephalic subarachnoid hemorrhage, but personal practice includes angiogram at the time of presentation and then at day 7 of hospitalization as small aneurysms may collapse at the time of rupture, and allowing for an extended time interval to pass it may reexpand. This type of subarachnoid hemorrhage has lower rates of complications than other subarachnoid hemorrhages including vasospasm and hydrocephalus. Although there is no definitive etiology for perimesencephalic subarachnoid hemorrhage, it is thought to be due to a venous tear and hemorrhage rather than arterial. References

1. Boswell S, Thorell W, Gogela S, et al. Angiogram-negative subarachnoid hemorrhage: outcomes data and review of the literature. J of Stroke and Cerebro Dis. 2013;22:750-757.

2. Coelho LG, Costa JM, Silva EI. Non-aneurysmal spontaneous subarachnoid hemorrhage: perimesencephalic versus 
nonperimesencephalic. Rev Bras Ter Intensiva. 2016;28:141-146.

3. van Gijn J, van Dongen KJ, Vermeulen M, et al. Perimesencphalic hemorrhage: a nonaneurysmal and benign form of subarahnoid hemorrhage. Neurology. 1985;35:493-497.

4. Gupta SK, Gupta R, Khosla VK, et al. Nonaneurysmal nonperimesencephalic subarachnoid hemorrhage: is it a benign entity? Surg Neurol. 2009;71:571-572.

5. Jung JY, Kim YB, Lee JW, et al. Spontaneous subarachnoid haemorrhage with negative initial angiography: a review of 143 cases. J Clin Neurosci. 2006;13(10):1011-1017.

6. van der Schaaf IC, Velthuis BK, Gouw A, et al. Venous drainage in perimesencephalic hemorrhage. Stroke. 2004;35:1614-1618.

C H AP T E R 1 4

Management Strategies Ofer Sadan and Milad Sharifpour

1. A 31-year-old man, with no past medical history, is admitted to the ICU with acute liver failure (ALF) following ingestion of an unknown herbal supplements. On initial examination, he is awake, oriented only to self, follows simple commands and has mild asterixis but no focal motor deficits. Notable laboratory test results include AST 1734, ALT 1567, T. Bilirubin 2.3, and Ammonia 110. On day 2 of ICU admission, he has a witnessed generalized tonic-clonic seizure. Two milligrams of lorazepam are administered intravenously and the convulsions are terminated. A Stat Head CT is performed, which shows diffuse cerebral edema and no ischemic or hemorrhagic changes. What is the MOST appropriate medication to administer at this time?

A. Fosphenytoin B. Valproate C. Levetiracetam D. Midazolam infusion

2. A 76-year-old woman, with past medical history of hypertension, coronary artery disease, atrial fibrillation, and Parkinson disease is admitted to the ICU with progressively worsening mental status. She suffered a ground level fall 2 days before her admission. Her home medications are amlodipine, lisinopril, levodopa, and apixaban. On physical examination, she is somnolent, only opens her eyes to painful stimuli, pupils are equal in size and briskly reactive to light, does not vocalize, and withdraws from noxious stimuli in all four extremities. The patient is emergently intubated for airway protection and admitted to the ICU. On arrival to the ICU, a head CT is obtained (see figure that follows).

What will be the MOST appropriate next step in treatment?

A. Administer stat IV levetiracetam B. Administer stat IV mannitol C. Administer stat IV NaCl 3% D. Neurosurgical consultation for hematoma evacuation

3. A 45-year-old woman, who is an active smoker and has history of untreated hypertension, presented following a sudden “thunderclap” headache. Head CT revealed (see figure that follows) subarachnoid hemorrhage of an anterior communicating artery aneurysm. She underwent successful endovascular coiling of the aneurysm, and an external ventricular drain was placed for hydrocephalus. Five days after presentation, she acutely became somnolent. Vital signs are T 37.4, HR 94, R 24, BP 124/75 (MAP 91), and O2 saturation 95% on room air. On examination she is somnolent but easily arousable and follows simple commands with all four extremities, but there is a clear drift of the right hand and leg.

What is the most appropriate next step in management? A. Administer stat IV Levetiracetam B. Bolus 1 liter of fluid C. Consult interventional radiology for stat angioplasty D. Administer empiric IV Vancomycin and Ceftriaxone in meningitis

doses.

4. A 57-year-old man, with past medical history of hypertension and hyperlipidemia, presented with an acute onset of slurred speech and right-sided weakness. He was diagnosed with an acute left middle cerebral artery (LMCA) stroke and IV tPA was administered. He was then admitted to the ICU for close monitoring. On day 3 poststroke, his focal deficits persist, and it is noted that one of his pupils is larger than the other. He subsequently becomes unresponsive and is emergently intubated and hyperventilated. Stat head CT demonstrated increasing cerebral edema and midline shift of 9.6 mm and uncal herniation without signs of cerebral hemorrhage. What is the most appropriate next step in treatment?

A. Transfuse to keep the hemoglobin>10 mg/dL B. Administer IV dexamethasone C. Administer hypertonic saline D. Readminister IV tPA at half the original dose

5. A 76-year-old man with history of COPD and stage 3 chronic kidney disease underwent elective craniotomy for tumor resection. His postoperative course was complicated by the development of status epilepticus and respiratory failure. On postoperative day 5, he developed a fever of 38.7°C and his WBC increased from 12 000 to 19 500. Blood, urine, and respiratory cultures were obtained. A CSF sample revealed: Nucleated cells 230 cells/mL3, RBC: 1500 cells/mL3, Protein 220, and Glucose 34 mg/dL (systemic glucose 110 mg/dL). Which empiric antibiotic regimen is most appropriate to initiate at this time?

A. Ceftazidime and vancomycin B. Ampicillin, ceftriaxone, and vancomycin C. Vancomycin, ceftriaxone, and metronidazole

D. Cefepime and vancomycin

6. A 24-year-old woman, with history of recent upper respiratory infection, was admitted to the ICU with a week of progressive ascending weakness. On initial examination she was awake, alert, oriented, and cooperative. She had quadriparesis and areflexia. She subsequently developed respiratory distress and was intubated. Head and cervical spine CT are read as normal. A lumber puncture is performed, which reveals the following CSF profile: 0 WBCs, 0 RBCs, Protein 120 mg/dL, and Glucose 80 mg/dL. What is the next best step in management?

A. Administer IV methylprednisolone B. Administer intravenous immunoglobulin C. Perform stat nerve conduction and electromyography studies D. Administer IV Acyclovir

5. A 38-year-old man suffers multisystem trauma and hemorrhagic shock after a motor vehicle accident. Initial CT imaging revealed a left-sided depressed skull fracture with underlying subdural hematoma (SDH) with contrecoup intracerebral hemorrhage, multiple bilateral rib fractures, lung contusions, and a splenic laceration. After initial damage control surgery, he was admitted to the ICU, where an increased intracranial pressure (ICP) monitor was placed. The patient remained sedated and ventilated, with limited neurological examination. On ICU day 2, he developed acute kidney injury with oliguria, and by day 3 he became anuric unresponsive to diuretic therapy. On the same day he had a sustained elevation in ICP to 27 mm Hg. Basic metabolic panel on day 3: Na 142; K 4.7; Cl 109, BUN 45, creatinine 3.5. What is the next most appropriate step to manage the ICP elevation?

A. Administer IV mannitol

B. Administer dexamethasone C. Administer NaCl 23.4% D. Neurosurgical consultation for an emergent decompressive hemicraniectomy

8. A 45-year-old man with a history poorly controlled hypertension is admitted to the ICU after elective endovascular repair of a descending thoracic aortic aneurysm. His intraoperative course was uneventful but on arrival in the ICU, he complains of lower limb weakness. His vital signs are within normal limits with blood pressure 110/65. On physical examination, he is awake, alert, and oriented. His cranial nerves and upper limb strength are intact. There is a clear nearly symmetric motor deficit in the bilateral lower limbs, with 3/5 weakness in the proximal muscles, and 2/5 in the distal ones. Tone is flaccid, and there is no Babinski sign or sensory deficit. In addition to increasing the blood pressure what is the MOST appropriate next step in management?

A. Increase the blood pressure and insert a lumbar drain for CSF drainage B. Obtain a stat MRI brain and spine C. Initiate high-dose steroid treatment D. Obtain a stat angiogram of the lumbar vessels

9. A 78-year-old woman with past medical history of hypertension, COPD, and coronary artery disease is admitted to the ICU with respiratory failure due to community-acquired pneumonia. She is intubated and mechanically ventilated. Empiric antibiotic therapy for pneumonia was initiated following obtaining cultures. On the morning of her third ICU day, she his noted to be less arousable when sedation is decreased. On examination she does not follow commands, has a gaze preference to the left, and only moves her left side spontaneously. A stat head CT does not show any acute findings. The last documented normal neurological examination was

at the shift change the day prior. The patient’s home meds include a baby aspirin (81 mg) and a statin. What is the MOST appropriate intervention at this time?

A. Administer stat IV tPA B. No acute intervention C. Administer stat fosphenytoin D. Obtain a stat CT angiogram and CT perfusion study for possible thrombectomy

10. A 32-year-old woman is admitted to the hospital with a 2-week course of progressively worsening confusion. She has no past medical history except for a mild upper respiratory infection two weeks before admission. On examination, she is awake, not oriented to time, place, or person and is only able to follow simple commands. There are no obvious focal neurological deficits. During the assessment, she develops a generalized tonic-clonic seizure. The seizure does not break despite three 2 mg doses of IV lorazepam and is emergently intubated and a propofol infusion is initiated, which terminates the convulsions. She is treated empirically with antibiotics for bacterial meningitis and acyclovir for viral encephalitis. EEG monitoring is notable for left temporal focal nonconvulsive status epilepticus, and fosphenytoin is administered with resolution of the seizures is administered. A head CT is obtained but does not show any acute abnormality. An MRI of the brain shows bitemporal T2 hyperintensities and no sign of a space occupying lesion, hemorrhage, or stroke. A lumbar puncture is performed, and CSF content shows 8 WBCs, 0 RBCs, Glucose 80 mg/dL (systemic 120 mg/dL), and protein 95 mg/dL. PCR for HSV 1&2, CMV, EBV, and VZV are negative, and acyclovir is discontinued. Gram stain and cultures are negative. What is the MOST appropriate therapeutic intervention at this time?

A. Administer methylprednisolone

B. Re-administer IV acyclovir C. Administer valproic acid D. Wean propofol to obtain a neurological examination

Chapter 14 Answers 1. Correct Answer: C Rationale: The patient in this question suffers from ALF because of a toxic exposure. A common complication of ALF is cerebral edema and occasionally seizures. The patient responded to the first line treatment for the seizure (IV lorazepam) and now needs prophylaxis as the underlying cause of the seizure remains uncontrolled. When choosing an antiepileptic regimen, one should consider clearance and potential side effects. Fosphenytoin is primarily metabolized by the liver and could reach toxic levels in administered in the setting of ALF (Answer A is incorrect). Valporate is also metabolized by the liver and is known to induce hyperammonemia, even with normal functioning liver, and therefore it should not be used when hyperammonemia already exists (Answer B in incorrect). Levetiracetam is a relatively safe and effective medication, which is not primarily metabolized by the liver, and therefore the best from the above-stated options (Answer C is correct). Benzodiazepines, propofol, ketamine, etc, are indicated only if a patient fails treatment with other anti-epileptic drugs (Answer D is incorrect). References

1. Polsen J, Lee WM; American Association for the Study of Liver Disease. AASLD position paper: The management of acute liver failure. Hepatology. 2005;41:1179-1197.

2. Glauser T, Glauser T, Gloss D, et al. Evidence-based guidelines: treatment of convulsive status epilepticus in children and adults: report of the guideline committee of the American Epilepsy Society. Epilepsy Curr. 2016;16:48-61.

3. Lacerda G, Krummel T, Sabourdy C, Ryvlin P, Hirsch E. Optimizing therapy of seizures in patients with renal or hepatic dysfunction. Neurology. 2006;67:S28-S33.

2. Correct Answer: D

Rationale: This patient has the classic presentation of SDH, with the gradual neurological deterioration after a fall. Imaging reveals a L. hemispheric SDH, primarily acute but probably with some chronic components (lesshyperintense areas). Head CT also demonstrates left-sided cerebral edema with left to right midline shift. Although seizures are very common in SDH patients, the description does not suggest a current seizure. It is common to administer prophylactic, not therapeutic doses of an antiepileptic medication (Answer A is incorrect). Although imaging does show significant cerebral edema with a midline shift, osmotherapy is generally avoided as it may cause further shrinkage of the brain tissue, which will expand the subdural space and exacerbate the bleed (Answers B and C are incorrect). The correct answer is D: in an acute, symptomatic SDH, the solution is surgical evacuation. The patient requires a clot evacuation in the operating room as soon as possible. References

1. Fomchenko EI, Gilmore EJ, Matouk CC, et al. Management of subdural hematomas: part I. medical management of subdural hematomas. Curr Treat Options Neurol. 2018;20:28.

2. Fomchenko EI, Gilmore EJ, Matouk CC, et al. Management of subdural hematomas: part II. surgical management of subdural hematomas. Curr Treat Options Neurol. 2018;20:34.

3. Correct Answer: B Rationale: This patient has a classic presentation of symptomatic cerebral vasospasm and delayed cerebral ischemia after sub-arachnoid hemorrhage. She has several risk factors for vasospasm, including: sex, smoking history, and the blood pattern on the head CT (blood in the cistern and the presence of intraventricular blood). The highest incidence of vasospasm occurs between post-bleed days 4 to 10 (although it can occur up to three weeks post bleed). Although angioplasty is a definitive treatment for

vasospasm, the first bedside intervention should be fluid administration to increase the mean arterial pressure (MAP) to improve cerebral perfusion. The description is not suggestive of a seizure and therefore choice A is incorrect. The description also does not suggest a new infection, and therefore Answer D is incorrect. Reference

1. Francoeur CL, Mayer SA. Management of delayed cerebral ischemia after subarachnoid hemorrhage. Crit Care. 2016;20(91):277.

4. Correct Answer: C Rationale: The patient suffered an extensive MCA stroke and did not improve with IV thrombolytics. Brain edema and ICP are often associated with occlusion of large intracranial arteries. Edema of the brain begins to develop during the first 24 to 48 hours and reaches a maximum extent of 3 to 5 days from the occurrence of acute ischemic stroke. The presentation described is classic for uncal herniation secondary to increasing edema, with pressure on midbrain causing a CN III palsy manifested by a blown pupil. Because post-stroke edema is cytotoxic in nature and vasogenic edema occurs secondarily (as opposed to perineoplastic changes), steroids are not beneficial (Option B is incorrect). In fact, it has been shown that steroid administration in the setting of acute stroke worsens outcomes. Although increasing cerebral edema could cause secondary cerebral ischemia by compressing healthy brain tissue, there is no indication to increase transfusion threshold (Option A is incorrect). Redosing tPA could be detrimental in the settings of a large stroke and should not be attempted (Option D is incorrect). Treatment of stroke-related cerebral edema is osmotherapy, such as hypertonic saline (Option C is correct) or mannitol. Following this initial intervention, the patient should be evaluated for possible hemicraniectomy. References

1. Ayata C, Ropper AH. Ischaemic brain oedema. J Clin Neurosci.

2002;9(2):113-124.

2. Bar B, Biller J. Select hyperacute complications of ischemic stroke: cerebral edema, hemorrhagic transformation, and orolingual angioedema secondary to intravenous alteplase. Expert Rev Neurother. 2018;18(10):749-759.

5. Correct Answer: A Rationale: The patient presents with post craniotomy meningitis. According to the Infectious Diseases Society of America 2017 guidelines, coverage for grampositive bacteria, and gram-negative bacteria including antipseudomonal coverage is required. Vancomycin should be aggressively dosed to achieve a trough concentration of 15 to 20 µg/mL. The recommendation for gramnegative coverage includes: cefepime, ceftazidime, or meropenem. Although cefepime is an option, there is increased risk of seizures associated with its use compared with other beta-lactams. The risk is especially significant in older patients with renal impairment. For these reasons it is preferable not to choose cefepime in this elderly patient with chronic kidney disease and seizures (option D is incorrect). Option B represents a common empiric coverage regimen for community acquired bacterial meningitis, which is not relevant for this patient. Option C does not cover pseudomonas. The correct answer is Option A: ceftazidime and vancomycin. References

1. Tunkel AR, Hasbun R, Bhimraj A, et al. 2017 Infectious disease Society of America’s clinical practice guidelines for healthcare associated ventriculitis and meningitis. Clin Infect Dis. 2017 64:e34-e65.

2.

https://www.fda.gov/Drugs/DrugSafety/ucm309661.htm.

6. Correct Answer: B Rationale:

The patient described has a classic presentation of Guillain-Barré syndrome. This disease often appears following a febrile illness or a gastrointestinal tract infection. Typical features include ascending flaccid paralysis and a CSF sample with no WBCs and high protein. Nerve conduction and electromyography studies can be helpful to confirm the diagnosis; however, they are usually not needed to initiate treatment (Option C is incorrect). High-dose steroid therapy is not effective for Guillain-Barré syndrome (Option A is incorrect). First line treatment includes intravenous immunoglobulin or plasma exchange (Option B is correct). The patient’s clinical presentation is not suggestive of encephalitis as there are no clear signs of CNS involvement (no cortical deficits such as aphasia, or upper motor neuron pattern of injury) therefore Option D is incorrect. Reference

1. Willison HJ, Jacobs BC, van Doorn PA. Guillain Barre syndrome. Lancet. 2016;388(10045):717-727.

7. Correct Answer: C Rationale: T Cerebral edema and ICP are complications of severe TBI, which occurred in this patient. Cerebral edema in the settings of TBI is mainly cytotoxic and not vasogenic; therefore, steroids are not indicated (Option B is incorrect). The fastest and most effective intervention to reduce increased ICP in patients with TBI is osmotherapy. Although surgery might be indicated, osmotherapy is the initial therapy of choice (Option D is incorrect). There is no difference in outcome between the use of mannitol and 23.4% hypertonic saline in patients with increased ICP. However because mannitol’s effect is through diuresis, it may lead to systemic volume overload in an anuric patient. The correct answer is C. Hypertonic saline is the appropriate therapy to reduce intracranial pressure in a patient with TBI and ATN. References

1. Winkler EA, Minter D, Yue JK, Manley GT. Cerebral edema in traumatic brain injury: pathophysiology and prospective therapeutic targets. Neurosurg Clin N Am. 2016;27:473-488.

2. Continuum: lifelong learning in neurology. Neurocrit Care. 2015;21:12991323.

8. Correct Answer: A Rationale: A devastating complication of thoracic aortic aneurysm repair is spinal cord ischemia. In open thoracic aortic aneurysm repair, cross clamping the aorta causes decreased blood flow to the spinal cord resulting in ischemia. In endovascular repair, such as in this vignette, there is no aortic crossclamp, but the endostent deployed can occlude small collateral arteries that perfuse the anterior spinal cord, which can result in hypoperfusion and spinal ischemia. Because of the anatomic blood supply of the spinal cord, the anterior motor fibers are supplied primarily by a single anterior spinal artery commonly resulting in bilateral leg weakness. Although there are no large randomized control trials, the common treatment aims at improving spinal cord perfusion by increasing MAP and inserting a lumbar drain for CSF removal. The result of these interventions is an increase in spinal cord perfusion pressure (SCPP), which is the difference between MAP and intraspinal pressure (ISP), i.e., SCPP=MAPISP. Although this treatment may be more effective when performed prophylactically before surgery, it may also be used as a rescue treatment when postoperative cord ischemia occurs (Option A is correct). Obtaining an MRI may be important to rule out other causes of spinal cord ischemia, such as spinal cord hematoma but would not be the immediate next step (Answer B is incorrect). Steroids have no demonstrated role in spinal cord ischemia (Option C is incorrect). The mechanism of spinal cord ischemia in this case is due to the endo-stent occlusion of perfusing arteries resulting in decreased anterior spinal cord perfusion; therefore, IR-guided thrombectomy will not treat the underlying problem (Option D is incorrect). References

1. Coselli JS, LeMaire SA, Köksoy C, et al. Cerebrospinal fluid drainage reduces paraplegia after thoracoabdominal aortic aneurysm repair: Results of a randomized controlled trial. J Vasc Surg. 2002;35:631-639.

2. Khan NR, Smalley Z, Nesvick CL, et al. The use of lumbar drains in preventing spinal cord injury following thoracoabdominal aortic aneurysm repair: an updated systematic review and meta-analysis. J Neurosurg Spine. 2016;25:383-393.

9. Correct Answer: D Rationale: The patient described in the vignette demonstrates evidence of an ischemic stroke, an infrequent but important complication in critically ill patients. Timely detection is challenging because physical examination can be limited by sedation, delirium, or other medications, which can make acute neurologic changes difficult to detect. The patient probably has a LMCA syndrome based on the description. Because her last known normal examination was over 4.5 hours before the diagnosis, she is out of the window for IV thrombolytic therapy (Answer A is incorrect). However, in the era of endovascular thrombectomy, there are still urgent interventions that could change the course and outcome. Although a seizure is in the differential when an acute neurological change occurs, the gaze deviation (to the left in this case) suggests that this is a stroke and not a seizure (Option C is incorrect). According to the 2018 American Stroke Association, there is enough level I data to recommend pursuing a CT angiogram and CT perfusion study to assess the stroke burden even up to 24 hours from the time of last known normal examination. Several randomized controlled trials have shown the benefit of patient-selective approach to find the correct candidate for mechanical thrombectomy. Therefore, it is not necessarily too late for this patient to receive timely and potentially beneficial intervention (Option D is correct). Reference

1. Powers WJ, Rabinstein AA, Ackerson T, et al. 2018 Guidelines for the early management of patients with acute ischemic stroke: a guideline

for healthcare professionals from the American Heart Association/American Stroke Association. Stroke. 2018;49:e46-e99.

10. Correct Answer: A Rationale: The patient in this scenario presents with a clinical picture of autoimmune encephalitis. The subacute progressive process, new onset seizures, and the temporal findings on imaging, all point toward the diagnosis. As part of the workup, it is critical to rule out other causes including infections (viral and bacterial), neoplastic etiology, toxic metabolic syndromes, posterior reversible encephalopathy syndrome (PRES), and a central manifestation of a systemic autoimmune disease (SLE, Sjögren, Behçet, etc). Once other potential diagnoses are excluded, early treatment with high-dose steroids and plasma exchange or IVIg is recommended (Option A is correct). Although it is possible to have an early false negative PCR for HSV, this patient has been symptomatic for two weeks, and therefore the chances of false negative PCR are low (Option B is incorrect). The patient is treated with a second and third line agents to control her seizures (phenytoin and propofol), with a good response. There is no advantage at this point in adding another antiepileptic agent, particularly valproic acid because of the significant interactions it has with phenytoin (Option C is incorrect). As for seizure control, the aim is to control all seizures, focal or generalized, with medications, and one cannot “tolerate” focal status epilepticus to achieve other clinical goals such as an examination (Option D is incorrect). References

1. Graus F, Titulaer MJ, Balu R, et al. A clinical approach to diagnosis of autoimmune encephalitis. Lancet Neurol. 2016;15:391-404.

2. Lancaster E. The diagnosis and treatment of autoimmune encephalitis. J Clin Neurol. 2016;12:1-13.

3. Perruca E. Clinically relevant drug interactions with antiepileptics. Br J Clin Pharmacol. 2006;61:246-255.

Cardiovascular Disorders 15.Acute Coronary Syndrome  85​ 16.Arrhythmias and Pacemaker  96​ 17.Heart Failure  104​ 18.Vascular Disorders  108​ 19.Valvular Heart Disease  116​ 20.Pericardial Diseases  125​ 21.Myocardial Disease  132​ 22.Congenital Heart Disease in Adults  137​ 23.Shock States  140​ 24.Mechanical Circulatory Support and the Transplanted Heart  147​ 25.Calculated Cardiovascular Parameters  152​ 26.Life Support and Resuscitation  156​ 27.Imaging and Diagnostic Modalities  162​ 28.Imaging (Ultrasound)  169​ 29.Management Strategies (Coagulation, Vasoactive Medications)  177​

C H AP T E R 1 5

Acute Coronary Syndrome Nino Mihatov and David M. Dudzinski

1. A 46-year-old man with atrial fibrillation, hypertension, hyperlipidemia, and diabetes is admitted with unstable angina. He undergoes coronary angiography which demonstrates a 90% stenosis of the mid left circumflex in a left dominant system. A single drug-eluting stent is deployed with an excellent angiographic result, and the patient is given aspirin and ticagrelor. The patient is admitted to the cardiac service and shortly after arriving to the floor reports new onset chest pain. A 12-lead electrocardiogram is performed:

Which of the following is the next BEST step in management? A. Intravenous nitroglycerin B. Observation C. Urgent repeat coronary angiography

D. Transthoracic echocardiogram (TTE)

2. A 55-year-old male with known coronary artery disease with prior percutaneous coronary intervention of the right coronary artery 2 years prior to presentation, hyperlipidemia, and hypertension presents to the emergency department with 2 days of exertional chest pain which culminated in acute onset of “stabbing” substernal chest pain radiating to the left shoulder. Vital signs on presentation are:

Temperature: 36.3°F Blood pressure: 200/89 mm Hg Heart rate: 81 beats per minute Respiratory rate: 18 per minute Oxygen saturation: 93% on room air Laboratory studies are notable for a cardiac troponin-T that is undetectable. A 12-lead electrocardiogram demonstrates normal sinus rhythm, Qwaves in inferior leads, and nonspecific ST segment changes. A plain film chest radiograph is obtained (see figure that follow).

A bedside point-of-care ultrasound demonstrates a moderate-sized circumferential pericardial effusion.

Which of the following is the most appropriate next step in management? A. Coronary angiography B. Contrasted computed tomography (CT) of the chest C. Pericardiocentesis D. Transesophageal echocardiography (TEE)

3. A 66-year-old male with hypertension and hyperlipidemia presents with substernal chest pain. His presenting electrocardiogram demonstrates inferior ST elevations with reciprocal changes in the

high lateral leads. Emergent coronary angiography is pursued with percutaneous coronary intervention undertaken on a subtotally occluded right coronary artery. Aspirin, ticagrelor, metoprolol, and atorvastatin are initiated. He is admitted to the intensive care unit for postintervention monitoring after repeat electrocardiogram shows resolution of previous ST segment elevations.

Twelve hours later, the patient develops subacute significant shortness of breath following medication administration. The patient’s oxygen saturation is 99% on room air and his physical exam is unremarkable. A repeat electrocardiogram is unchanged from postintervention. Contrasted computed tomography (CT) of the chest is negative for pulmonary embolism. Which one of his medications is a potential culprit of his dyspnea symptoms? A. Aspirin B. Metoprolol C. Ticagrelor D. Atorvastatin

4. A 65-year-old female with hypertension and hyperlipidemia develops substernal chest pressure with dyspnea.

Physical exam is notable for the following:

Blood pressure: 126/62 mm Hg, pulse: 76 beats per minute, oxygen saturation of 97% on 4 L nasal cannula General: Sitting up with increased work of breathing Heart: Regular rate and rhythm. A III/VI holosystolic murmur is heard at the cardiac apex, which is nondisplaced on chest palpation Lung: Posterior diffuse crackles Abdomen: Soft, nontender, and nondistended

Extremities: Warm, without edema 12-lead electrocardiogram reveals the following:

What is the most likely mechanism of this patient’s dyspnea? A. Pulmonary embolism B. Mitral regurgitation C. LV systolic dysfunction D. Tamponade

5. A 96-year-old female is admitted to the intensive care unit for closer hemodynamic monitoring following uncomplicated deployment of a drug-eluting stent to the first obtuse marginal, via the left radial artery, in the context of a presentation consistent with an ST elevation myocardial infarction (STEMI). Four hours postprocedure, the patient develops acute hypotension necessitating vasopressor support.

A 12-lead electrocardiogram is performed.

Compared to her postintervention electrocardiogram, ST segment elevations in leads II, III, aVF, V5, and V6 persist but are less prominent. Otherwise, there are no significant changes. Assessment of her left radial access site is unrevealing. What is the best next step in management? A. Repeat diagnostic coronary angiography B. Urgent surface echocardiogram C. Computed tomography imaging of the abdomen D. Right heart catheterization

6. A 57-year-old male with a history of hyperlipidemia is admitted to the intensive care unit with hypotension necessitating vasopressor support. He had been in his usual state of health, but in the preceding 24 hours, he developed progressively worsening dyspnea, culminating in respiratory failure and necessitating intubation.

His admission electrocardiogram is shown here:

His presenting troponin-T is 2.52 ng/mL (reference 2.0 D. Mannitol 0.5 g/kg before aortic cross clamp

6. A 64-year-old male with a past medical history significant for hypertension, heart failure with reduced ejection fraction, myocardial infarction 2 years prior, and chronic kidney disease presents to the ICU following open AAA repair. He was restarted on his home medications on postoperative day 1. Three days later, his vitals and pertinent laboratory values include: blood pressure 150/100 mm Hg, heart rate 94 bpm, potassium 2.9 mmol/L, serum creatinine 2.6 mg/dL. The following electrocardiogram is obtained 24 hours after admission:

Which of the following medications was the patient most likely taking before surgery? A. Atenolol B. Digoxin

C. Lisinopril D. Furosemide

7. Which of the following findings on physical examination are most likely observed in post-thoracic aortic aneurysm repair spinal cord infarction?

A. Bilateral loss of proprioception in lower extremities B. Inability to flex the knee but maintained motor function of ankle and feet bilaterally C. Loss of sensation, motor function, and dorsalis pedis pulse in the right lower extremity D. Flaccid paralysis at T8 level with complete loss of sensory and motor function

8. Which of the following potential complications IS NOT paired correctly with the corresponding classification of aortic dissection?

A. Elevated lactate, elevated INR and AKI—DeBakey Type II B. Elevated troponin—Stanford Type A C. New onset diastolic murmur heard at the right second intercostal space—DeBakey Type III D. New onset right hemiplegia—DeBakey Type I

9. A 65-year-old male is admitted to the ICU with confusion, headache, nausea, vomiting, and hypertension. He underwent right carotid endarterectomy 8 hours ago. The first set of vital signs obtained in the ICU showed blood pressure 171/94 mm Hg, heart rate 84 bpm, respiratory rate 22 per minute, and oxygen saturation 96% on room air. Which of the following IS NOT recommended as treatment for his complication?

A. Hypertonic saline B. Mannitol C. Labetalol D. Levetiracetam

Chapter 18 Answers 1. Correct Answer: D Rationale: It is important to distinguish the true from the false lumen in an acute aortic dissection, especially when the dissection involves the ascending aorta, as there is a potential in compromising the patency of the coronaries and head and neck vessels, or distally, the visceral arteries as these may originate from the false lumen. Additionally, identification of the true lumen is critical to guide aortic wire placement during interventions. There are certain characteristics that help to identify the true lumen on echocardiography. In the descending aorta, the true lumen is usually smaller than the false lumen (A) and expands during systole (B). In the ascending aorta, the true lumen tends to be larger than the false lumen because proximal aortic pressures (closer to the left ventricle) are higher and thus keep the true lumen pressurized. The false lumen has a concave appearance compared to the convex appearance of the true lumen in systole. Echo findings of cobwebs (fibrinous remnants sheared from the intima during separation from media) are 100% specific for the false lumen in acute aortic dissection (C). Color-flow Doppler pattern is always inphase with the cardiac cycle as opposed to the out-of-phase pattern in the false lumen (D). Echocardiographic features of true and false lumen are important to recognize compression of the true lumen, which may result in organ malperfusion to identify the origin of important aortic branches (if they originate from the true vs false lumen) and provide live guidance for cannula placement in the true lumen. References

1. Evangelista A, Frank A, Erbel R, et al. Echocardiography in aortic diseases: EAE recommendations for clinical practice. Eur J Echocardiogr. 2010;11(8):645-658. doi:10.1093/ejechocard/jeq056.

2. Armstrong WF, Ryan T. Feigenbaum’s Echocardiography. Lippincott Williams & Wilkins; 2010:646-653.

2. Correct Answer: B Rationale: Based on the 2018 Society for Vascular Surgery practice guidelines, the strongest level recommendation is to pursue elective surgical repair in patients with AAA >5.5 cm, saccular aneurysms, and any aneurysm that is symptomatic (B) (back pain, abdominal pain), as these incur the highest risk of rupture. Likewise, strong evidence exists to serially monitor aortic dilation 5.5 cm, symptomatic aneurysms, and saccular aneurysms, as these carry the highest risk of rupture. References

1. Gloviczki P, Lawrence PF, Forbes TL. Update of the Society for Vascular Surgery abdominal aortic aneurysm guidelines. J Vasc Surg. 2018;67(1):1. doi:10.1016/j.jvs.2017.11.022.

2. Filardo G, Powell JT, Martinez MA, Ballard DJ. Surgery for small asymptomatic abdominal aortic Aaneurysms. Cochrane Database Syst Rev. 2015;(2):CD001835. doi:10.1002/14651858.cd001835.pub4.

3. Correct Answer: C Rationale: The risk of SCI following thoracic endovascular aortic repair (TEVAR) is ∼10%. Early identification and treatment of this devastating complication is critical in preventing permanent neurologic deficit. Prior aneurysm repair (A), magnitude of the repair, coverage of the left subclavian artery (B), and pre-existing chronic renal insufficiency (D) have all been shown to have higher incidences of SCI post TEVAR. Spinal cord perfusion is dependent on one anterior and two posterior spinal arteries as well as a cervical vascular network proximally and pelvic vascular network distally. Proximal supply to the cervical vascular network is via the subclavian arteries that give rise to the vertebral arteries and then the anterior spinal artery. Thus, left carotid subclavian artery bypass should be considered before TEVAR when the proximal stent graft is expected to cover the origin of the left subclavian artery. The distal spinal cord is supplied by a pelvic vascular network, which arises from the lumbar and sacral arteries and forms a collateral network with branches of the inferior mesenteric and hypogastric arteries. Disruptions of either the proximal or distal collateral networks can place watershed areas of the spinal cord at risk of ischemia. Strategies that increase spinal cord perfusion pressure (mean arterial pressure minus cerebrospinal fluid [CSF] pressure) as well as decrease metabolism and oxygen demand can reduce SCI post repair. Mild hypothermia (C) decreases metabolism and oxygen demand and would therefore be protective rather than a risk factor for developing postoperative ischemia. CSF drainage is one intervention that can increase spinal cord perfusion pressure and potentially decrease the incidence of SCI after TEVAR. A recent Cochrane review in 2012 based on three randomized controlled trials of 287 patients examined the role perioperative drainage of cerebrospinal fluid in patients undergoing thoracoabdominal and thoracic aortic aneurysm repair. It is the mainstay of neuroprotection along with

additional strategies that increase spinal cord perfusion pressure and oxygen delivery, such as augmentation of the mean arterial pressure and correction of severe anemia. Postoperative SCI is a potentially devastating complication following TEVAR. Maintaining or augmenting spinal perfusion pressure and reducing metabolic demands are the mainstays of therapy for both prevention and treatment. References

1. Ullery BW, Cheung AT, Fairman RM, et al. Risk factors, outcomes, and clinical manifestations of spinal cord ischemia following thoracic endovascular aortic repair. J Vasc Surg. 2011;54(3):677-684.

2. Feezor R, Martin T, Hess P, et al. Extent of aortic coverage and incidence of spinal cord ischemia after thoracic endovascular aneurysm repair. Ann Thorac Surg. 2008;86:1809-1814.

4. Correct Answer: C Rationale: Consideration should be given with regard to preoperative placement of prophylactic lumbar spinal drain in patients at high risk for postoperative SCI; however, in patients who present for emergency repair of aortic aneurysm or dissection, a drain may not be placed because of time constraints. Postoperative rescue management of SCI includes therapies aimed at optimizing spinal cord perfusion pressure (the difference between the mean arterial pressure and either CSF pressure or central venous pressure, whichever is higher). This includes drainage of CSF via a subarachnoid drain, augmenting arterial pressure, and reducing central venous pressure or a combination of the three. The American Society of Regional Anesthesia and Pain Medicine (ASRA) published updated guidelines in 2018 for anticoagulation interruption before performing neuraxial techniques, which includes newer oral anticoagulation agents. The table that follows includes recommendations from the most updated ASRA guidelines as of 2018.

Minimum Time Between Last Dose of Anticoaguland and Neuraxial Procedure Traditional Anticoagulants Warfarin Heparin IV Heparin 5000 sq bid Heparin 5000 sq tid Fondaparinux Enoxaparin 1 mg/kg sq Enoxaparin 40 mg sq qd Direct Thrombin Inhibitors Argatroban Bivalirudin Oral Antiplatelet Agents Aspirin/NSAIDs Clopidogrel Prasugrel Ticlopidine GPIIB/IIIA Inhibitors Abciximab Tirofiban Direct Factor Xa Inhibitors Rivaroxaban Apixaban

when INR 1.5 cm

8. An 85-year-old woman underwent a transapical transcatheter aortic valve replacement (TAVR) for severe symptomatic aortic stenosis. After an uneventful procedure, she is brought to the ICU intubated and sedated. In the 30 minutes after arrival to the ICU she develops progressive tachycardia and hypotension, which prompts the placement of a TEE probe. The mid-esophageal 4-chamber view is shown in the figure that follows.

Based on the findings in the image, what is the most appropriate next step in management? A. Left-sided chest tube placement B. Emergent surgical pericardial evacuation C. Pericardial drain placement

D. CT angiography of the chest

9. A 35-year-old woman with no prior medical history presents to your hospital’s emergency department with a complaint of dyspnea that started acutely 2 hours ago. She appears moderately distressed and has a room air oxygen saturation of 85%, so you are consulted for possible ICU admission. Bedside cardiac ultrasound reveals grossly normal biventricular size and function. Thoracic ultrasound shows lung sliding and A lines in all fields bilaterally. There is no evidence of B lines, consolidated lung, or pleural effusion. Which of the following ultrasound examinations is most indicated next?

A. Compression and color Doppler of the femoral and popliteal veins B. Focused assessment of the mitral valve with color Doppler C. Abdominal ultrasound to assess for the presence of free fluid D. Tissue Doppler quantification of diaphragmatic contraction velocity to assess for the risk of imminent respiratory failure

10. Which of the following thoracic ultrasound patterns will most likely be present in severe ARDS?

A. B lines in all fields B. Large hypoechoic spaces in bilateral subpleural regions C. Absence of lung sliding anteriorly, with an area of lung sliding moving in and out of the frame through the respiratory cycle D. Anterior fields with B lines, posterior fields with ultrasonographically visible lung containing patchy areas of hyperechogenicity

Chapter 28 Answers 1. Correct Answer: D Rationale: Bedside cardiac ultrasound is indicated in critically ill patients when unexplained hypotension is present. If TEE views are not adequate, then a TEE examination is indicated. This patient has unexplained hypotension following liver transplantation despite high doses of vasopressors and has not demonstrated volume responsiveness. Cardiac ultrasound is required to rule out pathology such as right or left ventricular dysfunction, acute valvular disorders, and hemodynamically significant pericardial effusion. Because no transthoracic views could be obtained, transesophageal echocardiography would offer insight into cardiac function. According to the ASE/SCA Guidelines for Performing a Comprehensive Transesophageal Echocardiographic Examination, the absolute contraindications for TEE probe placement include: Perforated viscus Esophageal stricture Esophageal tumor Esophageal perforation, laceration Esophageal diverticulum Active upper GI bleed

The esophageal diverticulum would represent the absolute contraindication in this patient. The remainder of the patient’s pathology falls under relative contraindications. Of note, at many centers transesophageal echocardiography is a standard monitor during liver transplantation, even in patients with nonbleeding esophageal varices. A series of small studies has shown relatively good safety of TEE in patients with varices, though the risk of bleeding is higher than in the general population. References

1. Dalia AA, Flores A, Chitilian H, Fitzsimons MG. A comprehensive review of transesophageal echocardiography during orthotopic liver transplantation. J Cardiothorac Vasc Anesth. 2018;32(4):1815-1824.

2. Burger-Klepp U, Karatosic R, Thum M, et al. Transesophageal echocardiography during orthotopic liver transplantation in patients with esophagoastric varices. Transplantation. 2012;94(2):192-196.

3. Spier BJ, Larue SJ, Teelin TC. et al. Review of complications in a series of patients with known gastro-esophageal varices undergoing transesophageal echocardiography. J Am Soc Echocardiogr. 2009;22(4):396-400.

4. Hahn RT, Abraham T, Adams MS, et al. Guidelines for performing a comprehensive transesophageal echocardiographic examination: recommendations from the American Society of Echocardiography and the society of cardiovascular anesthesiologists. J Am Soc Echocardiogr. 2013;26(9):921-964.

2. Correct Answer: C Rationale: The diastolic picture shows a mid-esophageal long axis view of the left atrium, left ventricle, mitral valve, and aortic valve. The basal septum appears disproportionately thick, but without more information one cannot distinguish between disproportionate upper septal thickening and hypertrophic cardiomyopathy (HCM). In the systolic frame, one can see several abnormalities. The anterior leaflet of the mitral valve has moved into the left ventricular outflow tract (LVOT). A visible space is seen between the anterior and posterior mitral leaflets, which should have a tight coaptation point in systole. The aortic valve, which should be widely open during systole, appears only partially open. Taken together these findings lead to the diagnosis of systolic anterior motion (SAM) of the mitral valve. In SAM, the mitral valve moves into the LVOT during systole, creating an outflow tract obstruction. The force of the blood moving through the LVOT may pull the anterior mitral leaflet away from the posterior leaflet, leading to mitral regurgitation. The interrupted systolic ejection can lead to early aortic valve closure or partial opening in systole. SAM is most commonly seen in HCM or after mitral valve repair. However, it can also be seen in hyperdynamic or hypovolemic states, when small LV size combined with excessive inotropy can lead to SAM in otherwise normal hearts.

Medical management of SAM hinges around volume loading, reducing inotropy, and increasing peripheral resistance. Therefore, stopping the dobutamine infusion would be the first step of the options provided. Adding epinephrine would likely worsen the SAM through increased inotropy and reduced diastolic filling time. An intra-aortic balloon pump reduces LV afterload and would not be beneficial here. Finally, the aortic valve likely has no pathology and only opens incompletely due to a small stroke volume (SV). Valve replacement is not indicated. Reference

1. Ibrahim M, Rao C, Ashrafian H, Chaudhry U, Darzi A, Athanasiou T. Modern management of systolic anterior motion of the mitral valve. Eur J Cardiothorac Surg. 2012;41(6):1260-1270.

3. Correct Answer: C Rationale: Ultrasound assessment of the retro-hepatic IVC can provide insights into a patient’s volume status. The assessment hinges around changes in intrathoracic pressure throughout the respiratory cycle being transmitted to the IVC via the right atrium. As a nonmechanically ventilated patient inhales, pressure in the thorax is reduced. This pressure reduction is transmitted to the thin-walled right atrium and then to the IVC. Patients who are volume-responsive tend to have low right-sided filling pressures and an IVC that has not reached its maximum distensibility. When the right atrial pressure decreases during inspiration, the IVC luminal pressure decreases relative to the intra-abdominal pressure and the vessel collapses. In patients who are not volume responsive, the rightsided filling pressures are higher relative to the changes in thoracic pressure and the vessel size changes less. The exact cutoff of IVC diameter change for predicting volume responsiveness is not well defined. Some studies identify >40% to 45% collapse during tidal breathing as a cutoff for volume responsiveness in spontaneously breathing nonventilated patients. However, these studies exclude many patients where the technique is prone to drawing the wrong conclusion. If the intra-abdominal pressure is elevated, as in choice C, then

the IVC may collapse even in a patient who would not be volume responsive. This would call the fellow’s conclusion into question and makes choice C the correct answer. Both a hemodynamically significant pneumothorax and pericardial effusion would increase the right atrial pressure without changing the abdominal pressure, leading to a distended IVC even if a patient would indeed be volume responsive. A unilateral DVT would not alter venous return enough to be the correct answer in this case. References

1. Preau S, Bortolotti P, Colling D, et al. Diagnostic accuracy of the inferior vena cava collapsibility to predict fluid responsiveness in spontaneously breathing patients with sepsis and acute circulatory failure. Crit Care Med. 2017;45(3).

2. Muller L, Bobbia X, Toumi M, et al. Respiratory variations of inferior vena cava diameter to predict fluid responsiveness in spontaneously breathing patients with acute circulatory failure: need for a cautious use. Critical Care. 2012;16(5):R188.

3. Zhang Z, Xu X, Ye S, Xu L. Ultrasonographic measurement of the respiratory variation in the inferior vena cava diameter is predictive of fluid responsiveness in critically ill patients: systematic review and meta-analysis. Ultrasound Med Biol. 2014;40(5):845-853.

4. Zhang J, Critchley LAH. Inferior vena cava ultrasonography before general anesthesia can predict hypotension after induction. Anesthesiology. 2016;124(3):580-589.

5. Airapetian N, Maizel J, Alyamani O, et al. Does inferior vena cava respiratory variability predict fluid responsiveness in spontaneously breathing patients? Critical Care. 2015;19(1):1-8.

4. Correct Answer: A Rationale: The image in this case demonstrates the utility of cardiac ultrasound in assessing placement of percutaneous left ventricular assist devices. The devices are frequently placed under fluoroscopy in the cardiac

catheterization laboratory, but fluoroscopy is rarely available in the ICU. A parasternal long axis view shows the Impella crossing the aortic valve and entering the LV. The device draws blood from the LV inflow port and pumps it into the aortic root via the outflow port. The inflow port should be positioned 3.5 to 4 cm beyond the aortic valve. Thus, in this case, the device should be advanced by 1.5 cm. It is important that the pigtail portion of the device is not included in the measurement. If the device is pulled back into the aorta too far, both the inflow and outflow ports will be on the same side of the aortic valve and the device will not provide hemodynamic support. Additionally, turbulence from the aortic valve interfering with blood inflow can lead to hemolysis. If the device is advanced too far into the LV, the papillary muscles and aortic valve can interfere with inflow and outflow, respectively, leading to hemolysis. The pigtail can tangle with the mitral apparatus, leading to worsening mitral regurgitation. Reference

1. Stainback RF, Estep JD, Agler DA, et al. Echocardiography in the management of patients with left ventricular assist devices: recommendations from the American Society of Echocardiography. J Am Soc Echocardiogr. 2015;28(8):853-909.

5. Correct Answer: B Rationale: Cardiac ultrasound provides an opportunity to estimate cardiac output in a noninvasive manner. This can be a useful tool in the assessment of undifferentiated shock and correlates very well with thermodilution cardiac output measurements by PA catheter. The technique involves obtaining the velocity time integral (VTI) at the LVOT. When pulsed-wave Doppler is applied to a specific area of blood flow, a tracing is produced that has velocity of blood flow on the y-axis and time on the x-axis, as shown in the image below. This is the blood flow velocity versus time at that specific location, in this case the LVOT. When the integral (area under the curve) of that tracing is obtained for a single systolic period, the result is a specific distance. This is the distance that a disc that has the cross

sectional area of the LVOT travels in systole. Thus, we end up with a 2D area of the LVOT and a height, allowing us to calculate the LV SV. The LVOT area is calculated by measuring the diameter in the parasternal long axis (transthoracic echocardiogram) or mid esophageal long axis (TEE). Assuming there is no aortic regurgitation, the SV calculated here multiplied by the heart rate provides us the cardiac output. LVOT Area = (LVOT diameter/2)2 × 3.14SV = LVOT VTI × LVOT AreaCO = HR × SV To accurately measure the LVOT VTI, we must align the probe with the direction of blood flow. In typical hearts, this is best done in the apical 5chamber view (transthoracic echocardiogram) and the deep transgastric 5chamber view (TEE), making B the correct answer here. In the other answer choices, the Doppler beam will not align with the direction of blood flow and therefore the VTI will be underestimated. Reference

1. Mercado P, Maizel J, Beyls C, et al. Transthoracic echocardiography: an accurate and precise method for estimating cardiac output in the critically ill patient. Critical Care. 2017;21:136.

6. Correct Answer: D Rationale: This ultrasound image shows a dilated RV and an interventricular septum that is shifted toward the left ventricle. Acute RV failure from pressure overload is associated with a dilated and septal shifting toward the LV. In a short-axis view, this results in the LV forming a “D” shape. In this patient with cardiogenic shock, a history of mitral valve replacement, and a dilated, hypokinetic RV, the two most likely differential diagnoses are pulmonary embolism and mitral stenosis. The PE was ruled out by the CT chest already. To investigate the gradient across the mitral valve, continuous-wave Doppler should be placed across the valve, making D the correct answer. In mitral stenosis, the pressure gradient across the stenotic valve will result in high velocity flow through the valve. In severe mitral stenosis, the elevated left atrial pressures result in postcapillary pulmonary hypertension that can result in right-sided heart failure.

Demonstration of lower-extremity venous thrombus would not be useful after a PE has already been ruled out. Color Doppler of the main PA would likely not show any abnormality, even if a PE were present. The IVC will likely be plethoric, but again will not tell us the specific diagnosis. Reference

1. Zoghbi WA, Chambers JB, Dumesnil JG, et al. Recommendations for evaluation of prosthetic valves with echocardiography and doppler ultrasound. J Am Soc Echocardiogr. 2009;22(9):975-1014.

7. Correct Answer: B Rationale: Cardiac ultrasound is extremely sensitive for the detection of pericardial effusions. It is also useful for determining the hemodynamic significance of pericardial effusions. In the setting of a pericardial effusion, signs of cardiac tamponade include IVC dilation, RA and RV diastolic collapse, elevated respiratory variation in trans-mitral and trans-tricuspid inflow, and septal “bounce.” Of these, RA collapse in greater than one-third of the cardiac cycle is the most sensitive and specific for cardiac tamponade, making it the correct answer in this case. Collapse of a cardiac chamber in tamponade occurs when the pericardial pressure exceeds the pressure in that chamber. As pericardial pressure increases, the right atrium will collapse first in early diastole. The longer

the chamber remains collapsed, the longer the pericardial pressure exceeds the atrial pressure and the worse the tamponade. The IVC will generally be plethoric in tamponade, but this can also be seen in other cardiac pathologies and is not specific to tamponade. The same applies to inspiratory variation in AV valve inflow. The size of the pericardial effusion correlates poorly with hemodynamic significance. A large pericardial effusion does not necessarily indicate tamponade. Reference

1. Klein AL, Abbara S, Agler DA, et al. American Society of Echocardiography clinical recommendations for multimodality cardiovascular imaging of patients with pericardial disease: endorsed by the Society for Cardiovascular Magnetic Resonance and Society of Cardiovascular Computed Tomography. J Am Soc Echocardiogr. 2013;26(9):965-1012.e15.

8. Correct Answer: B Rationale: The image above shows a focal pericardial fluid collection adjacent to the left atrium. The fluid collection clearly distorts the left atrial anatomy and is responsible for the hypotension in this case. This is a good example of focal tamponade that can occur after cardiac surgery. That patient population tends not to have the classic hypoechoic and free-flowing pericardial effusion seen in classic tamponade, instead showing localized areas of semi-thrombosed blood products that impinge on specific cardiac chambers. Note the swirling echogenicity within the fluid collection here that is typical of static blood products. The most common cause of pericardial bleeding after TAVR is RV perforation. However, aortic root rupture or LA/LV bleeding is possible and should be considered in surgical planning for this case. The correct management here is surgical evacuation of the fluid collection. The posterior and lateral location of the fluid collection precludes percutaneous drainage. Because the fluid is pericardial and not pleural, chest tube placement would not be appropriate.

Reference

1. Fassa A, Himbert D, Vahanian A. Mechanisms and management of TAVR-related complications. Nat Rev Cardiol. 2013;10:685.

9. Correct Answer: A Rationale: Thoracic ultrasound provides a tool for the rapid assessment of patients with acute dyspnea. Lichtenstein and colleagues proposed an algorithmic diagnostic approach to respiratory failure. The first step is to look for lung sliding bilaterally. If present, lung sliding essentially rules out a pneumothorax. Then, evidence of interstitial or alveolar edema in the form of B lines is sought. If there is no evidence of pneumothorax, pulmonary edema, consolidation, or pleural effusion on lung ultrasound, the next differential to consider is pulmonary embolism. The next step in this case would be to look for venous thrombosis. If venous thrombus is demonstrated, then pulmonary embolism is very likely. An absence of DVT, however, does not rule out pulmonary embolism and further studies must be pursued. Mitral regurgitation, which would be seen on color Doppler assessment, would result in pulmonary edema and B lines on imaging, which were absent on the examination. Abdominal fluid would not explain her acute dyspnea in the absence of other symptoms. Tissue Doppler assessment of diaphragmatic contraction velocity has been preliminarily investigated to predict ventilator weaning failure but has not been validated in the assessment of acute dyspnea. References

1. Lichtenstein DA, Mezière GA. Relevance of lung ultrasound in the diagnosis of acute respiratory failure. Chest. 2008;134(1):117-125.

2. Lichtenstein D. Novel approaches to ultrasonography of the lung and pleural space: where are we now? Breathe. 2017;13(2):100-111.

10. Correct Answer: D

Rationale: Cross-sectional CT imaging of the noncardiogenic pulmonary edema seen in ARDS tends to show ground-glass opacities in the anterior (nondependent) lung fields and consolidation in the posterior (dependent) fields. On ultrasound examination, this presents as B lines in the areas containing interstitial edema and the appearance of consolidated lung in the dependent fields. Consolidated lung on ultrasound will have a variable appearance depending on the pathophysiology. The first principle to remember is that normal lung is never ultrasonically visible. If lung can be “imaged” on ultrasound, then the alveoli are no longer gas filled. In the dense atelectasis that accompanies pleural effusions, the lung has a fine-textured uniform hypoechoic appearance similar to that of liver and spleen. The fluid-filled alveoli in pneumonia and ARDS create a different appearance on ultrasound. A hallmark of ARDS is the significant heterogeneity in consolidation density and aeration between lung units. This leads to areas of hypoechoic lung immediately adjacent to areas of better-aerated lung or bronchus, which will appear hyperechoic. The borders between these areas tend to be irregular and jagged, so the presence of a jagged interface between consolidated and aerated lung is known as the “shred sign.” Of the answer choices above, diffuse B lines without evidence of consolidation is more compatible with cardiogenic pulmonary edema. Large hypoechoic spaces subpleurally is consistent with pleural effusions. Answer choice C describes “lung point,” which is a specific finding in pneumothorax. This is the ultrasound appearance of lung sliding (due to apposed visceral and parietal pleura) moving into and out of the imaged area of pneumothorax (lack of lung sliding) through the respiratory cycle. References

1. Copetti R, Soldati G, Copetti P. Chest sonography: a useful tool to differentiate acute cardiogenic pulmonary edema from acute respiratory distress syndrome. Cardiovascular Ultrasound. 2008;6(1):16.

2. Sekiguchi H, Schenck LA, Horie R, et al. Critical care ultrasonography differentiates ARDS, pulmonary edema, and other causes in the early course of acute hypoxemic respiratory failure. Chest. 2015;148(4):912918.

3. Volpicelli G. Sonographic diagnosis of pneumothorax. Intensive Care Med. 2011;37(2):224-232.

C H AP T E R 2 9

Management Strategies (Coagulation, Vasoactive Medications) Christoph G. S. Nabzdyk and Yvonne Lai

1. A 67-year-old male patient had a non-ST elevation myocardial infarction and is currently on a heparin drip and requires coronary artery bypass grafting. His baseline activated coagulation time (ACT) is 189 seconds. He is 78 kg and received a total of 28 000 units of heparin (350 Units/kg). Three minutes later, a second ACT is drawn and repeat ACT is 286 seconds. Another 10 000 units of heparin is administered, and targeted ACT is still not achieved. You suspect heparin resistance. What is NOT a predictor for heparin resistance?

A. Antithrombin activity level B. Disseminated intravascular coagulation C. Prior heparin therapy D. Sepsis E. Factor IIa level

2. A 58-year-old female with diverticulitis developed free air and required intensive care unit admission. In the intensive care unit, she went into persistent atrial fibrillation and the team decided to start her on a heparin drip for stroke prevention. Her preoperative platelet count was 343 000/µL, and on hospital day 5, her platelet count is 86 000/µL. You suspect heparin-induced thrombocytopenia (HIT) but still need to provide anticoagulation. What is NOT an appropriate drug to administer?

A. Argatroban B. Bivalirudin C. Enoxaparin D. Desirudin E. Hirudin

3. A 29-year-old male is hypotensive in the emergency department after a motor vehicle accident 2 hours ago despite fluid resuscitation. You suspect intra-abdominal hemorrhage and have heard that tranexamic acid may reduce mortality and death from hemorrhage. You decide to administer tranexamic acid knowing the possible complications from it. How is tranexamic acid metabolized?

A. Renal B. Liver C. Lungs D. Plasma esterases E. CYP 450

4. A 68-year-old female in the intensive care unit on total parenteral nutrition due to ileus from recent abdominal surgery now has an STelevation myocardial infarction and was emergently taken to the cardiac catheterization laboratory. The cardiology team performs percutaneous coronary intervention in one of her coronary arteries and recommends continuation of antiplatelet agents. Because she cannot tolerate oral aspirin currently, which other medication can be used?

A. Prasugrel B. Ticagrelor

C. Cangrelor D. Clopidogrel E. Argatroban

5. You have decided to place a patient with a history of HIT on a bivalirudin infusion for anticoagulation. Which laboratory value shown below should you be able to monitor?

A. Activated partial thromboplastin time (PTT) B. Prothrombin time (PT) C. Fibrinogen D. Anti-Xa E. Platelets

6. A 52-year-old male with a history of hypertrophic cardiomyopathy is nil per oral for right heart catheterization procedure. The patient is lightly sedated and the cardiology team obtains these numbers:

Blood pressure Pulmonary capillary wedge pressure Pulmonary artery pressure Central venous pressure Cardiac output

80/42/55 mm Hg 10 mm Hg 28/10 mm Hg 6 mm Hg 5.5 L/min

Based on the systemic vascular resistance and the rest of his numbers, what would be the best choice of vasopressor for this patient? A. Epinephrine B. Norepinephrine C. Phenylephrine D. Dobutamine E. Dopamine

7. A 43-year-old female presenting with urosepsis has persistent hypotension despite fluid resuscitation. You do a bedside echocardiogram on the patient showing normal biventricular function and measure an inferior vena cava size of 2.1 cm with minimal respiratory variation. Her blood pressure is 92/43 mm Hg on norepinephrine 30 µg/min, and you decide to add vasopressin 0.04 Units/min. What is the receptor that you are targeting by adding vasopressin?

A. V1 B. V2 C. V3 D. Alpha-1 E. D2

8. A 56-year-old male with liver cirrhosis and hypertension (on angiotensin-converting enzyme inhibitors) presents to the intensive care unit postoperatively after a Whipple for pancreatic cancer. You note that he is on norepinephrine 30 µg/min, vasopressin 0.04 Units/min, and epinephrine 2 µg/min to maintain a mean arterial pressure of 58 mm Hg and heart rate 99 beats/min. His arterial line shows no pulse pressure variation. You do a bedside echocardiogram which shows hyperdynamic left ventricular function and measures inferior vena cava of 1.9 cm with 50 is as high as 50% E. Narcan should be administered

8. A 24-year-old man with a history of severe asthma with multiple intubations presents to the ED with several days of worsening dyspnea despite the frequent use of albuterol nebs. The same morning, he visited a friend who has a cat and his dyspnea rapidly worsened. CXR shows hyperinflation and the ED physician gives solumedrol, continuous albuterol nebs, and initiates critical care consultation because of persistent accessory muscle use after an hour of care in the ED. Which of the following statements about severe asthma exacerbations is correct?

A. Peak expiratory flow (PEF) is predictive of arterial oxygen saturation B. Intravenous magnesium is not recommended C. The use of heliox (helium-oxygen mixtures) is well supported in the literature D. Following intubation, the respiratory rate should be set 14 to 20 breaths/min E. Increasing extrinsic PEEP may help improve breath triggering during the resolution phase

9. A 60-year-old man with very severe emphysema who is noncompliant with prescribed home oxygen therapy presents to the ED with a bleeding traumatic laceration on his foot. Triage vitals

reveal T 36 C, HR 90, BP 120/50, RR 18, SpO2 71% RA. On further questioning he complains of chronic dyspnea on exertion but does not feel any worse than normal. Laboratory test results are notable for a hematocrit of 60%. Supplemental oxygen with a nonrebreathing mask is administered with O2 sat quickly rising to 100%. Given the high acuity and census in the ED he is placed in the hallway to await physician evaluation and suturing. Thirty minutes later the patient is noted by the nurse to be unarousable, ABG 7.05/130/140 with bicarb 45. Which of the following statements is true?

A. The mechanism of somnolence in acute hypercapnia is decreased cerebral blood flow B. Narcan is likely to be effective in restoring consciousness C. The Haldane effect is partially to blame D. Acetazolamide should be prescribed at discharge E. The acute increase in PaCO2 is due almost entirely to reduced minute ventilation

10. The patient described in the preceding question is intubated, quickly regains his baseline level of alertness, and is extubated 4 hours later. He is alarmed by his need for mechanical ventilation and before discharge he asks you about his life expectancy. Which of the following measures is the best predictor of survival in COPD?

A. FEV1 B. BODE index C. Success in smoking cessation D. Age E. Presence of Diabetes Mellitus

Chapter 30 Answers 1. Correct Answer: D Rationale: The ARMA trial randomly assigned 861 patients with ARDS to low tidal volume ventilation (6 mL/kg predicted body weight) or 12 mL/kg PBW. The 6 mL/kg group had a lower risk of 28-day mortality (RR 0.74, CI 0.61-0.88). A subsequent meta-analysis found similar results. Importantly, the ARMA protocol targeted both a tidal volume of 6 mL/kg and plateau pressure 1 g/d D. Urine output 1 g/d are other signs of acute rejection. Diagnosis is confirmed with allograft biopsy. Treatment is with pulse methylprednisolone in most centers, with the dose and duration of pulse therapy depending on the grade of rejection. Maintenance immunosuppression therapy may also be intensified (eg, aim for higher tacrolimus levels). Antithymocyte globulin is typically used as a secondline agent and is coadministered with glucocorticoid therapy in higher grade rejection or rejection refractory to glucocorticoid pulse therapy. The expected reversal rate for a first episode of acute rejection is 60% to 70% with this regimen. Plasmapharesis can be effective in antibody-mediated rejection, but it is not first-line therapy. Therapy success is indicated by increases in urine output and a decrease in serum creatinine within 5 days of initiating treatment. References

1. Lamarche C, Côté JM, Sénécal L, Cardinal H. Efficacy of acute cellular rejection treatment according to banff score in kidney transplant recipients: a systematic review. Transplant Direct. 2016;2:e115.

2. Vineyard GC, Fadem SZ, Dmochowski J, et al. Evaluation of corticosteroid therapy for acute renal allograft rejection. Surg Gynecol Obstet. 1974;138:225.

3. Burton SA, Amir N, Asbury A, et al. Treatment of antibody-mediated rejection in renal transplant patients: a clinical practice survey. Clin Transplant. 2015;29:118.

2. Correct Answer: D Rationale: Transplant renal artery stenosis (TRAS) usually occurs between 3 months and 2 years after renal transplantation, but it can present at any time. It is a potentially curable cause of posttransplant hypertension, allograft dysfunction, and graft loss. Renal artery stenosis usually occurs close to the allograft renal artery surgical anastomosis. Risk factors include difficulties in procurement and surgical technique, atherosclerotic disease, cytomegalovirus infection, and DGF. It presents in most cases as refractory hypertension and allograft dysfunction. Although noninvasive imaging modalities such as Doppler ultrasonography, spiral computed tomography, and magnetic resonance angiography are useful in screening for TRAS, arteriography is the definitive diagnostic and treatment modality when TRAS is suspected based on noninvasive tests or clinical presentation. Once identified by arteriography, the stenosis can be corrected directly with angioplasty and stenting. It is important to rule out rejection before arteriography to optimize chances of graft function recovery. References

1. Bruno S, Remuzzi G, Ruggenenti P. Transplant renal artery stenosis. J Am Soc Nephrol. 2004;15:134.

2. Audard V, Matignon M, Hemery F, et al. Risk factors and long-term outcome of transplant renal artery stenosis in adult recipients after treatment by percutaneous transluminal angioplasty. Am J Transplant. 2006;6:95.

3. Correct Answer: C Rationale: Renal artery thrombosis is a devastating posttransplant complication

that usually results in graft loss. Fortunately, it is an uncommon complication occurring in less than 1% of patients. Early identification and intervention is most important. It usually presents with sudden cessation of urine output and a tender, swollen graft. Risk factors include hypotension, hypercoagulable state, and multiple renal arteries. Diagnosis is usually made with color flow Doppler studies. Once the diagnosis is made, urgent surgical exploration and thrombectomy is indicated. Outcomes are unfavorable as the transplanted kidney does not have collateral vessels, and its tolerance of warm ischemia is poor. References

1. Humar A, Matas AJ. Surgical complications after kidney transplantation. Semin Dial. 2005;18:505.

2. Bakir N, Sluiter WJ, Ploeg RJ, et al. Primary renal graft thrombosis. Nephrol Dial Transplant. 1996;11:140.

4. Correct Answer: A Rationale: Although there are over 10 definitions of DGF in the literature, it is most commonly defined as the need for dialysis within 7 days of transplantation (69% of studies reviewed between 1984 and 2007 use this criteria for their definition). This definition offers a standard by which centers can report outcomes and define a clinical entity that can be studied to help improve graft and patient survival. It occurs in 20% to 50% of patients receiving a first cadaveric graft. It is characterized by acute tubular necrosis following renal transplantation. DGF occurs more commonly among recipients of deceased donor transplants compared with live donor transplants. DGF has significant effects on graft and patient survival as it can be associated with both acute and chronic allograft nephropathy and increased risk of graft failure. Risk factors include deceased donor and prolonged allograft ischemia times. Studies are currently ongoing to look at pretransplant, intraoperative, and posttransplant interventions that may reduce the risk of DGF and subsequent graft failure. These treatments focus on immunosuppression, ischemic preconditioning, and vasodilatory agents.

References

1. Perico N, Cattaneo D, Sayegh MH, Remuzzi G. Delayed graft function in kidney transplantation. Lancet. 2004;364:1814-1827.

2. Siedlecki A, Irish W, Brennan DC. Delayed graft function in the kidney transplant. Am J Transplant. 2011;11(11):2279-2296.

C H AP T E R 4 9

Diagnosis and Monitoring in Renal Failure Abdulaziz S. Almehlisi, Phat Tan Dang, Ji Sun “Christina” Baek, Anushirvan Minokadeh, Alan S. Nova, Zeb McMillan, Kimberly S. Robbins and Ulrich Schmidt

1. You are asked to consult on a previously healthy 68-year-old woman who presented with malaise and one episode of hematuria. She visited her primary care doctor 5 days ago for a “bladder infection” and was prescribed trimethoprim-sulfamethoxazole, which she has been taking. She is alert and oriented, and her physical examination is within normal limits. Her vital signs are normal. Her laboratory data are unremarkable except for elevated eosinophils, creatinine of 3 mg/dL, and urea 41 mg/dL.

What would you expect to see in urine analysis? A. Muddy brown cast B. Red blood cell cast C. White blood cell cast D. Envelopelike crystals

2. What management should you pursue next?

A. Administer fomepizole B. Discontinue trimethoprim-sulfamethoxazole C. Administer 500 mL bolus of normal saline D. Obtain a computed tomography (CT) abdomen/pelvis with intravenous (IV) contrast

3. Which of the electrolyte abnormalities is associated with advanced chronic kidney disease?

A. Hyponatremia B. Hypophosphatemia C. Hypomagnesemia D. Hypokalemia

4. You are working as an intensivist in a rural hospital where the emergency physician calls you to evaluate a patient for intensive care unit (ICU) admission. When you come to the emergency department (ED), you see a disheveled, cachectic, old gentleman who was brought in by his neighbor for altered mental status. His neighbor reports that the patient has been complaining about back pain, and he was taking some “over-the-counter ” (OTC) pain medication. You cannot elicit any history from the patient, and his physical examination is unremarkable. His laboratory data show elevated creatinine of 2.1 mg/dL, sodium 147 mEq/L, potassium 5.8 mEq/L, chloride 113 mEq/L, bicarbonate 22 mEq/L, albumin 2 mg/dL, and glucose 98 mg/dL. His arterial blood gas shows pH 7.39 and pCO2 38.

What is the next test that you would order? A. Obtain an acetaminophen level B. Obtain a salicylate level C. Obtain an alcohol level D. Obtain a ketone level

5. What is the BEST next step of management?

A. Administer N-acetylcysteine B. Start an insulin drip C. Start emergent hemodialysis D. Proceed with urine alkalization

6. A 42-year-old man with a history of insulin-dependent diabetes mellitus, hypertension, and end-stage renal disease on hemodialysis is admitted to the surgical ICU after a motor vehicle collision resulting in femur fracture and subdural hematoma. The patient is alert and oriented. His vital signs are blood pressure (BP) 185/100 mm Hg, heart rate (HR) 110 beats per minute, respiratory rate 12/min, and SpO2 95% on room air. His blood work is significant for creatinine 4 mg/dL, urea 80 mg/dL, and potassium 5.8 mEq/L. The patient states that he missed a dialysis session two days ago and that he was on his way to the dialysis center when he had the car accident.

Which of the following dialysis modalities is INAPPROPRIATE for this patient? A. Intermittent hemodialysis (iHD) B. Continues renal replacement therapy (CRRT) C. Sustained low efficiency dialysis (SLED) D. Extended daily dialysis (EDD)

7. A 64-year-old male with a history of cirrhosis secondary to hepatitis C is being evaluated for abdominal pain. The patient reports worsening generalized abdominal pain and fever for the past 4 days. He denies hematemesis or melena. His BP was 110/60 mm Hg, and HR was 95 beats per minute. His physical examination is significant for icterus, ascites, and generalized abdominal tenderness. Laboratory analysis is notable for white blood cell count of 18 000/ µL, creatinine 1.1 mmol/L (baseline 0.8 mmol/L), blood urea nitrogen 36 mg/dL, and total bilirubin is >4.3 mg/dL. The patient

received broad-spectrum antibiotics.

Which of the following drugs is MOST appropriate to administer to reduce risk for renal failure in this patient? A. Terlipressin 1 mg IV bolus now and then every 6 hours plus 1 g/kg of albumin B. Albumin 1.5 g per kg now and 1.0 g/kg on day 3 C. Midodrine 7.5 mg orally now and then every 8 hours plus 1 g/kg of albumin D. Octreotide subcutaneous injection 100 µg now and then every 8 hours plus 1 g/kg of albumin

8. A 22-year-old man presented to the ED with vomiting, altered mental status, and fever. The patient’s roommate states that the patient has not been feeling well for 2 days, yesterday was complaining of fever and headache. This morning the patient had a new-onset seizure and altered mental status. The patient’s roommate is not aware of any history of drug abuse other than marijuana. In the ED, a CT scan was done and did not show any acute intracranial pathology. Urine toxicology screen was positive for cannabis. A lumbar puncture was done in the ED and resulted cell count consistent with viral meningitis/encephalitis. Cerebrospinal fluid herpes simplex virus (HSV) PCR and bacterial cultures were ordered, and results are pending. The patient was started on empirical vancomycin, ceftriaxone, and acyclovir. The patient was admitted to the ICU for monitoring. The next day, the patient’s mental status improved and the patient was transferred out of the ICU to the medical floor. The following day (48 hours after admission), the patient starts to have nausea, oliguria, abdominal, and flank pain. Repeated blood works were significant for creatinine 3.2 mg/dL, urea 56 mg/dL, potassium 5.3 mEq/L, and sodium 142 mEq/L. Urine analysis shows white blood cells 5 cells/HPF, red blood cells 5 cells/HPF, protein 100 mg/dL, and crystals.

Which of the following is the MOST LIKELY cause of acute kidney injury (AKI)/failure in this patient? A. Vancomycin B. Synthetic marijuana C. Acyclovir D. Ceftriaxone

9. A 75-year-old male is in postoperative day 3 in the ICU status post liver resection. He was aggressively resuscitated with IV fluids in the perioperative period. Currently, he is mechanically ventilated and hemodynamically stable; however over the last 24 hours, his urine output has decreased significantly to 5 mL/h. What is the best test to assess his AKI?

A. Serum urea/creatinine B. Urine sodium C. Renal ultrasound with Doppler D. Abdominal CT scan

Chapter 49 Answers 1. Correct Answer: C Rationale: Acute interstitial nephritis (AIN) is a rare cause of AKI. Most patients present with nonspecific signs and symptoms of acute kidney failure. The classical triad of fever, rash, and eosinophilia occurs only in 10% of the population. Moreover, patients can be oliguric or nonoliguric, and hematuria can occur in 5% of them. The most common cause of AIN is drug-associated, such as trimethoprim-sulfamethoxazole that the patient has been taking. Other causes include infection, idiopathic, or associated with systemic autoimmune diseases. Laboratory results usually show an increase in creatinine level, eosinophilia or eosinophiluria, white blood cells or white blood cell casts in urine (question 1—choice C), and a variable degree of proteinuria. Muddy brown cast is usually seen in acute tubular necrosis (ATN), which is associated with prolonged prerenal insult or nephrotoxin induced. Red blood cell cast is associated with acute glomerulonephritis, in which immunological mechanisms cause glomerular inflammation. These mechanisms could be infection-related, cancer, or exposure to drugs of toxins. Enveloplike, calcium oxalate crystals are seen in patients with ethylene glycol poisoning. As the patient is healthy and does not have any risk factors, and her examination is normal, ATN, acute glomerulonephritis, or ethylene glycol poisoning are very unlikely. Reference

1. Moledina DG, Perazella MA. Drug-induced acute interstitial nephritis. Clin J Am Soc Nephrol. 2017;12:2046-2049.

2. Correct Answer: B Rationale: Treatment of AIN includes discontinuing the offending agent (question 2—choice B) or treating the underlying disease. In severe cases of biopsy-

confirmed AIN, steroids can be administered. Answer A is incorrect because fomepizole is the treatment for ethylene glycol poisoning, not AIN. Her urea/creatinine ratio is less than 20, making prerenal cause unlikely, so fluid bolus would not be needed here. Also, CT scan with IV contrast would not be the best choice as there is no indication for this study, and IV contrast should be avoided in patients with AKI (answer D). References

1. Praga M, Appel GB. Clinical Manifestations and Diagnosis of Acute Interstitial Nephritis. Retrieved from Uptodate. https://www.uptodate.com/contents/clinical-manifestations-anddiagnosis-of-acute-interstitial-nephritis.

2. Roberts PR, Todd SR. Acute and chronic renal failure and management (including hemodialysis and continuous renal replacement. therapies). In: Comprehensive Critical Care: Adult. 2nd ed. USA: Society of Critcal Care Medicine; 2017:365-376:Chap 33.

3. Kshirsagar AV, Falk RJ. Treatment of Acute Interstitial Nephritis. Retrieved from Uptodate. https://www.uptodate.com/contents/treatment-ofacute-interstitial-nephritis.

4. Ronco P. Mechanisms of Immune Injury of the Glomerulus. Retrieved from Uptodate. https://www.uptodate.com/contents/mechanisms-ofimmune-injury-of-the-glomerulus.

5. Sivilotti MLA. Methanol and Ethylene Glycol Poisoning. Retrieved from Uptodate. https://www.uptodate.com/contents/methanol-and-ethyleneglycol-poisoning.

3. Correct Answer: A Rationale: Kidneys play an essential role in the body to maintain normal acid-base status and electrolyte levels. In addition, kidneys excrete acids in the form of ammonium chloride. As kidney disease progresses, patients lose the ability to effectively neutralize and excrete acids, leading to metabolic acidosis. As the glomerular filtration decreases, renal failure patients lose the

ability to concentrate urine, which can lead to hyponatremia. Studies suggest that a decreased clearance of vasopressin also contributes to the development of dysnatremia. It is not uncommon for hyperkalemia, hyperphosphatemia, and hypermagnesemia to occur as the kidneys lose their ability to excrete these electrolytes. References

1. Barash PG. Clinical Anesthesia. 7th ed. Philadelphia, PA: Wolters Kluwer Health/Lippincott Williams & Wilkins; 2013:1410.

2. Cadnapaphornchai MA, Schrier RW.“Pathogenesis and management of hyponatremia.” Am J Med. 2000;109(8):688-692.

3. Dhondup T, Qian Q. “Electrolyte and acid-base disorders in chronic kidney disease and end-stage kidney failure.” Blood Purif. 2017;43(1– 3):179-188. doi:10.1159/000452725. Epub 2017 January 24.

4. Correct Answer: B Rationale: Because we cannot obtain any useful history from the patient, we need to look for clues from his physical examination and laboratory results. His anion gap (corrected for serum albumin) is 147 − 113 − 22 + 2.5(4 − 2) = 17, indicating a high-gap metabolic acidosis. Based on the Winter ’s formula, his expected pCO2 should be 1.5 × 22 + 8 = 41 mm Hg. However, as his pCO2 is 38, he also has a respiratory alkalosis. In the presence of a high-gap metabolic acidosis, we need to identify if there is a third process. The patient’s Δ gap = 17 – 12 = 5, and Δ bicarbonate = 24 – 22 = 2. As Δ gap > Δ bicarbonate, he also has a metabolic alkalosis. The history of taking OTC medications and the combination of metabolic acidosis and respiratory alkalosis make aspirin toxicity high on the differential diagnosis. Aspirin could also cause altered mental status, confusion, and possible seizure at a toxic dose. Moreover, aspirin could cause GI upset, which could explain his metabolic alkalosis. Although euglycemic diabetes ketoacidosis has been described in the literature, a normal glucose level makes diabetic ketoacidosis very unlikely.

Similarly, alcohol intoxication is less likely to cause respiratory alkalosis, which makes it lower in the differential diagnosis. Because of its availability OTC and the potential combination with aspirin in some formulary, acetaminophen level should be checked; however, it would not be the best choice as it does not typically cause mixed metabolic acidosis and respiratory alkalosis. Moreover, acetaminophen overdose typically presents with gastrointestinal signs and symptoms. Reference

1. O’Malley GF. Emergency department management of the salicylatepoisoned patient. Emerg Med Clin North Am. 2007;25:333-346.

5. Correct Answer: C Rationale: In a patient with salicylate toxicity, the presence of altered mental status is an indication for emergent dialysis. Other indications for emergent dialysis are pulmonary edema, seizure, decreased renal function impairing salicylate elimination (consider when creatinine >2 mg/dL or 1.5 mg/dL for elderly or glomerular filtration rate 90 mg/dL. Urine alkalization is indicated in salicylate poisoning to enhance its elimination; however, the presence of altered mental status and AKI makes dialysis the best choice for this patient. References

1. Boyer EW, Weibrecht KW. Salicylate (Aspirin) Poisoning in Adults. Retrieved from Uptodate. https://www.uptodate.com/contents/salicylate-aspirin-poisoning-inadults.

2. Heard K, Dart R. Acetaminophen (Paracetamol) Poisoning in Adults: Treatment. Retrieved from Uptodate. https://www.uptodate.com/contents/acetaminophen-paracetamolpoisoning-in-adults-treatments.

6. Correct Answer: A Rationale: Intermittent hemodialysis (iHD) is associated with an increased risk of causing dialysis disequilibrium syndrome (DDS). DDS is characterized by different neurological symptoms of varying severity. Its symptoms range from nausea, headache, dizziness to seizure, coma, and death. DDS is primarily caused by fluid shifts that result in brain edema. Removal of urea across the blood-brain barrier occurs at a much slower rate than urea removal from plasma. This cause the brain cells to be “relatively” hyperosmolar to plasm and promotes water movement to brain cells. Moreover, patients with end-stage renal disease are in a chronic hyperosmolar state; this leads to the development of “idiogenic osmoles,” which add to the “relative” hyperosmolarity of the brain following dialysis. It has been reported that brain volume increases by an average of 3% after hemodialysis, which makes iHD inappropriate for patients with head trauma, intracranial bleeding, stroke, or any intracranial pathology that leads to increased intracranial pressure. The use of dialysis modality that removes solute and fluid at lower rates do not cause significant elevation of intracranial pressure and lower the risk for DDS. CRRT, SLED, and EDD can be used for patients at risk for DDS, and they all have similar effects on intracranial pressure and hemodynamics. References

1. Davenport A. Practical guidance for dialyzing a hemodialysis patient following acute brain injury. Hemodial Int. 2008;12(3):307-312. doi:10.1111/j.1542-4758.2008.00271.x.

2. Kumar A, Cage A, Dhar R. Dialysis-induced worsening of cerebral edema in intracranial hemorrhage: a case series and clinical perspective. Neurocrit Care. 2015;22:283-287.

3. Wu VC, Huang TM, Shiao CC, et al. The hemodynamic effects during sustained low-efficiency dialysis versus continuous veno-venous hemofiltration for uremic patients with brain hemorrhage: a crossover study. J Neurosurg. 2013;119:1288-1295. doi:10.3171/2013.4.JNS122102.

4. Khwaja A. KDIGO clinical practice guidelines for acute kidney injury. Nephron Clin Pract. 2012;120(4):c179-c184.

7. Correct Answer: B Rationale: The patient in the vignette is presenting with signs and symptoms concerning for spontaneous bacterial peritonitis (SBP). Cirrhosis patients with SBP are at high risk of developing renal failure secondary to hepatorenal syndrome. The risk can be reduced (from 30% to 10%) with IV albumin infusion. Albumin appears to be most effective in patients with serum creatinine >1 mg/dL and total bilirubin >4 mg/dL, but its effect is unclear in patients who had lower creatinine and bilirubin levels. The European Association for the Study of the Liver guidelines recommend starting albumin infusion in all patients with SBP. The recommended dose is 1.5 g per kg at the time of diagnosis and 1.0 g/kg body weight on day 3. Terlipressin, midodrine, and octreotide used as treatment options for hepatorenal syndrome but not for prevention. References

1. European Association for the Study of the Liver. EASL clinical practice guidelines on the management of ascites, spontaneous bacterial peritonitis, and hepatorenal syndrome in cirrhosis. J Hepatol. 2010;53(3):397-417. doi:10.1016/j.jhep.2010.05.004. https://www.ncbi.nlm.nih.gov/pubmed/20633946.

2. Sort P, Navasa M, Arroyo V, et al. Effect of intravenous albumin on renal impairment and mortality in patients with cirrhosis and spontaneous bacterial peritonitis. N Engl J Med. 1999;341:403-409. https://www.ncbi.nlm.nih.gov/pubmed/10432325.

8. Correct Answer: C Rationale: This patient presented encephalitis/meningitis. The

to the ED patient was

with possible HSV started on empirical

antimicrobial, including acyclovir. After appropriate treatment with acyclovir, the patient mental status improved but developed AKI. Acyclovir and vancomycin, both are nephrotoxic. Acyclovir can cause AKI by forming crystals that precipitate in renal tubules. Vancomycin, on the other hand, does not cause crystal-induced nephropathy. Acyclovir crystal– induced nephropathy can be asymptomatic or present with nausea, abdominal pain, flank pain, asterixis, multifocal myoclonus, seizures, hallucination, and altered mental status. Symptoms typically within 24 to 48 hours after therapy. Crystal-induced nephropathy can be avoided by appropriate volume repletion before starting acyclovir infusion, slow IV acyclovir infusion over 1 to 2 hours, and dose adjustment for patients with renal impairment. Treatment of acyclovir crystal–induced nephropathy range from IV hydration and loop diuretics to hemodialysis depending on the severity of symptoms. There are case reports for AKI associated with synthetic marijuana. There are reports of calcium oxalate crystal on kidney biopsy of patients who had renal impairment associated with synthetic marijuana abuse. As the patient does not have a history of synthetic marijuana abuse, it is less likely to be the cause of his renal impairment. Conventional urine drug screen does not test for synthetic marijuana. References

1. Gentry JL III, Peterson C. Death delusions and myoclonus: acyclovir toxicity. Am J Med. 2015;128:692-694. doi:10.1016/j.amjmed.2015.03.001.

2. Perazella M, Palevsky P, Forman J. Crystal-Induced Acute Kidney Injury. Uptodate. www.uptodate.com/contents/crystal-induced-acute-kidneyinjury. Accessed 11/1/2018.

3. Perazella MA, Crystal-induced acute renal failure. Am J Med. 1999;106(4):459.

4. Kazory A, Aiyer R. Synthetic marijuana and acute kidney injury: an unforeseen association. Clin Kidney J. 2013;6:330-333. doi:10.1093/ckj/sft047.

9. Correct Answer: C Rationale:

Renal ultrasound with Doppler can easily detect obstruction and can be performed at the bedside. In addition, Doppler measurements allow for assessment of renal perfusion. The renal resistive index (RI) is defined as peak systolic velocity—end diastolic velocity/peak systolic velocity and is measured at the renal arcuate or interlobar arteries. It has a specificity and sensitivity of about 90% to discern between states of no AKI, kidney injury, and persistent AKI. It can also guide therapy to optimize renal perfusion. Normal RI is approximately 0.58 ± 0.10 and values >0.70 are considered to be abnormal and a high renal RI on ICU admission may be predictive for developing AKI. RI is unaffected by changes in sodium or creatinine in urine or serum after diuretics or hemodialysis. Serum urea and creatinine are late markers of renal failure and are not predictive of permanent renal failure. These laboratory parameters in conjunction with urine sodium might potentially help to predict whether the patient is in a prerenal or renal state, however this information is often unreliable in critically ill patients. Abdominal CT scan will show renal pathology, however this necessitates transport of the patient. Additionally, contrast is often required for best images and this might further decrease renal function. References

1. Lerolle N, Guerot E, Faisy C, Bornstain C, Diehl JL, Fagon JY. Renal failure in septic shock: predictive value of Doppler-based renal arterial resistive index. Intensive Care Med. 2006;32:1553.

2. Darmon M, Schortgen F, Vargas F, et al. Diagnostic accuracy of Doppler renal resistive index for reversibility of acute kidney injury in critically ill patients. Intensive Care Med. 2011;37:68-76.

3. Deruddre S, Cheisson G, Mazoit JX, Vicaut E, Benhamou D, Duranteau J. Renal arterial resistance in septic shock: effects of increasing mean arterial pressure with norepinephrine on the renal resistive index assessed with Doppler ultrasonography. Intensive Care Med. 2007;33:1557.

4. Bellomo R, Bagshaw S, Langenberg C, Ronco C. Pre-renal azotemia: a flawed paradigm in critically ill septic patients? Contrib Nephrol. 2007;156:1.

C H AP T E R 5 0

Sodium Jaya Prakash Sugunaraj and Ngoc-Tram Ha

1. A 78-year-old man is brought to the hospital after being found down by his daughter at his home this morning. He was functional at baseline and last seen normal yesterday. He is a life-time smoker with chronic obstructive pulmonary disease (COPD) and was recently diagnosed with lung cancer. On physical examination, he is lethargic, has unsteady gait, and is confused. His temperature is 37.4°C, blood pressure is 127/94 mm Hg, pulse rate is 74 beats/min, and respiratory rate is 11 breaths/min. On examination, he has a normal jugular venous pressure, but he has decreased air entry at the lung bases. A CT head obtained shows age-related atrophic changes. Laboratory studies obtained are as follows:

Sodium 120 mEq/L (mmol/L) BUN 9 mg/dL Bicarbonate 30 mmol/L Creatinine 1.0 mg/dL Urine osmolality 275 mOsm/kg H2O Urine sodium 45 mEq/L (mmol/L) Serum osmolality 264 mOsm/kg H2O Glucose 84 mg/dL (4.6 mmol/L) Which of the following is the MOST appropriate next step in management of this patient? A. Fluid restriction to 800 mL B. Desmopressin C. Isotonic saline infusion D. Tolvaptan E. Hypertonic saline infusion

2. A 17-year-old college student is admitted to the hospital after sustaining a traumatic hit to the head during football practice, resulting in subdural hemorrhage. Upon arrival, his Glasgow Coma Scale is 4. He is started on hypertonic saline and undergoes emergent neurosurgical intervention. His exam remains unchanged overnight, but during morning rounds, the nurse reports that his urine output increased to over 300 mL/h.

Laboratory studies obtained show serum sodium 167 mEq/L, specific gravity 1.013, random urine sodium 55 mEq/L (mmol/L), random urine creatinine 51 mg/dL, urine osmolality 199 mOsm/kg H2O, and serum osmolality 338 mOsm/kg H2O. Which is the MOST likely cause of his increased urine output? A. Syndrome of inappropriate antidiuretic hormone secretion (SIADH) B. Central diabetes insipidus (DI) C. Cerebral salt wasting (CSW) syndrome D. Nephrogenic diabetes insipidus (NDI) E. Osmotic diuresis

3. A 68-year-old man presents to his primary care physician complaining of frequent urination at night. He reports a strong urinary stream without any feeling of incomplete emptying. His medications include aspirin, pravastatin, lithium, and amlodipine. On physical examination, he is afebrile, his blood pressure 138/75 mm Hg, pulse rate 74 beats/min, and respiratory rate 18 breaths/min. Which of the following laboratory studies are MOST consistent with the patient’s clinical presentation?

A.

Plasma Sodium

Urine Osmolality

Plasma Osmolality

133 mg/dL

281 mOsm/kg H2O

210 mOsm/kg H2O

B.

149 mg/dL

370 mOsm/kg H2O

280 mOsm/kg H2O

C.

144 mg/dL

262 mOsm/kg H2O

310 mOsm/kg H2O

D.

134 mg/dL

239 mOsm/kg H2O

234 mOsm/kg H2O

4. A 58-year-old woman with past medical history of alcohol abuse and bipolar schizophrenia presents to the hospital after a fall complicated by numerous rib fractures seen on chest x-ray. She is currently not taking any medications. On physical examination, her temperature is 37.1°C, blood pressure is 138/88 mm Hg, pulse rate is 99 beats/min, and respiratory rate is 14 breaths/min, oxygen saturation is 95% on room air. Her neurological, cardiovascular, and abdominal examinations are normal. She exhibits tenderness to palpation over the left chest wall with decreased bibasilar breath sounds. Her chemistry panel is as follows:

Sodium 122 mEq/L (mmol/L) Potassium 2.8 mEq/L (mmol/L) BUN 5 mg/dL Creatinine 0.7 mg/dL Urine osmolality 117 mOsm/kg Urine sodium 18 mEq/L (mmol/L) Serum osmolality 266 mOsm/kg Glucose 105 mg/dL Thyroid stimulating hormone 2.20 mIU/L Morning cortisol 16 µg/dL Total cholesterol 140 mg/dL HDL cholesterol 55 mg/dL LDL cholesterol 124 mg/dL Triglycerides 162 mg/dL What is the MOST appropriate management of her hyponatremia? A. Conivaptan

B. Desmopressin C. Normal saline infusion D. Fluid restriction E. Observation with repeat laboratory testing in 4 to 6 hours

5. A 62-year-old woman is brought to the hospital with sudden onset headache followed by nausea and vomiting. Upon arrival to the emergency room, she is lethargic with a Glasgow Coma Scale of 5. Significant vital signs include a blood pressure of 220/130 mm Hg. She is emergently intubated. CT scan of the head reveals subarachnoid hemorrhage in the basilar cisterns. She undergoes placement of a right frontal extraventricular device and coiling with improvement of her neurological examination. Over the following days, she is weaned off propofol. However, on the seventh day, her urine output increases to 4 L/d and she becomes hypotensive with blood pressure of 89/60 mm Hg and pulse rate of 118 beats/min. Laboratory data show hyponatremia with a sodium level of 130 mEq/L, potassium of 3.3 mEq/L, plasma osmolality of 269 mOsm/kg, urine sodium concentration of 71 mEq/L, urine osmolality of 93 mmol/L, glucose 172 mg/dL, TSH 3.1 mIU/L, and triglyceride of 118 mg/dL.

What is the MOST likely cause for the patient’s acute changes? A. CSW syndrome B. Central DI C. Osmotic diuresis D. SIADH E. Pseudohyponatremia

6. A 68-year-old male with long-standing smoking and alcohol use history presents to his primary care physician. His wife noticed progressively worsening jaundice and poor appetite with an associated weight loss of 35 lbs over the past 2 months. He also

reports early satiety and vague abdominal pain.

His temperature is 37.5°C, blood pressure is 110/75 mm Hg, pulse rate is 84 beats/min, and respiratory rate is 12 breaths/min. An ultrasound of his abdomen confirms a mass in the head of the pancreas. He is admitted for surgical intervention, and laboratory data obtained show the following:

Sodium 131 mEq/L (mmol/L) Potassium 3.8 mEq/L (mmol/L) BUN 7 mg/dL Creatinine 1.0 mg/dL Total bilirubin 8.7 mg/dL Amylase 90 U/L Lipase 67 U/L Total cholestrol 485 mg/dL LDL cholesterol 157 mg/dL HDL cholesterol 42 mg/dL Triglycerides 349 mg/dL Plasma osmolality 295 mOsm/kg Urine osmolality 420 mOsm/kg TSH 2.1 mIU/L Glucose 93 mg/dL What is the MOST likely cause for the patient’s hyponatremia? B. Beer potomania B. Psychogenic polydipsia C. SIADH D. Adrenal insufficiency E. Pseudohyponatremia

7. A 24-year-old woman is brought to the emergency department by her boyfriend for worsening lethargy. He reports that she has no known medical history and was doing well until 2 days ago after

they returned home from a hiking trip on the Appalachian trail. She started experiencing diarrhea after drinking from the fresh springs. Vital signs are as follows: temperature is 37.8°C, blood pressure is 97/64 mm Hg, pulse rate is 112 beats/min, and respiratory rate is 14 breaths/min. On physical examination, abdominal examination reveals tenderness in the left lower quadrant and weak radial pulses.

Sodium 144 mEq/L (mmol/L) Potassium 3.9 mEq/L (mmol/L) Chloride 110 (mmol/L) Bicarbonate 18 (mmol/L) BUN 42 mg/dL Creatinine 1.2 mg/dL Glucose 123 mg/dL Urine sodium 24 mEq/L What is the next BEST step in management of this patient’s condition? A. Normal saline with 20 KCl mEq/L B. Ringer lactate C. ½ normal saline D. ½ normal saline with D5W E. Normal saline with D5W

8. A 38-year-old male is brought to the hospital after a witnessed seizure. The patient’s medical history is only significant for bipolar disease. His sister also reports that he recently started a new diet regimen using herbal supplements that he purchased online. On physical examination, his vital signs are unremarkable. He appears unkempt, and there is a small laceration noted over his tongue. His serum sodium is 128 mEq/L (mmol/L). Other laboratory findings before and after water deprivation test (WDT) are shown below:

Before WDT

After WDT

Serum osmolality Urine osmolality Specific gravity

268 mOsm/kg 137 mOsm/kg 1.009

305 mOsm/kg 780 mOsm/kg 1.023

What is the MOST likely diagnosis in this patient? A. Nephrogenic DI B. Pseudohyponatremia C. Psychogenic polydipsia D. SIADH E. Central DI

9. A 19-year-old woman with past medical history of type I diabetes mellitus and seizures on oxcarbazepine is admitted for fever, chills, and myalgia. She works as a nurse assistant in the local skilled nursing facility where a few patients recently have been diagnosed with the flu. Her appetite has been poor since her symptoms started, and she also reports bouts of diarrhea. On physical examination, she is febrile with a temperature of 38.7°C, blood pressure is 102/76 mm Hg, pulse rate is 120 beats/min, and respiratory rate is 12 breaths/min. Laboratory values show the following:

Sodium 130 mEq/L (mmol/L) Potassium 4.1 mEq/L (mmol/L) Chloride 95 (mmol/L) Bicarbonate 9 (mmol/L) BUN 9 mg/dL Creatinine 0.7 mg/dL Glucose 623 mg/dL What is the MOST likely underlying cause for her hyponatremia? A. Poor solute intake B. SIADH C. Medication related

D. Hyperglycemia E. Diarrhea

10. A 56-year-old man with a past medical history of hypertension, hyperlipidemia, and type II diabetes mellitus presents to his primary care physician for a follow-up visit. He started working as a welder 6 months ago and was recently diagnosed with hypersensitivity pneumonitis after complaining of worsening shortness of breath at the time. He was prescribed a medication whose name he is unable to recall but stopped taking them abruptly after 2 months because of his busy schedule. His other medications include amlodipine, atorvastatin, and hydrochlorothiazide though he admits being noncompliant with those, too.

Today, he reports fatigue, unintentional weight loss of 7 lbs in 2 weeks, nausea, and lightheadedness. Pertinent vital signs include a temperature of 37.6°C, blood pressure of 117/58 mm Hg, pulse rate of 92 beats/min, and respiratory rate of 12 breaths/min. What are the MOST likely laboratory findings in this patient?

A. B. C. D. E.

Serum Sodium

Serum Potassium

Serum Urine Osmolality Sodium

130 mg/dL 128 mg/dL 145 mg/dL 149 mg/dL 132 mg/dL

4.7 mg/dL 5.4 mg/dL 3.8 mg/dL 4.0 mg/dL 5.2 mg/dL

295 mOsm/kg 260 mOsm/kg 265 mOsm/kg 285 mOsm/kg 280 mOsm/kg

32 mEq/L 45 mEq/L 48 mEq/L 53 mEq/L 25 mEq/L

Chapter 50 Answers 1. Correct Answer: E Rationale: The patient’s history and laboratory studies are consistent with hypotonic euvolemic hyponatremia. The important differentials are SIADH, adrenocortical insufficiency, polydipsia, physiological stimulus antidiuretic hormone (ADH) release (nausea, pain, anxiety), and hypothyroidism. Based on presentation, the most likely etiology of hyponatremia in this patient is SIADH. Patients with lung cancer, particularly small cell lung cancer, have a reported incidence of up 18.9%. SIADH is caused by the secretion of ADH from the posterior pituitary gland or unregulated ectopic production by tumor cells. Elevated levels of ADH lead to hyponatremia and hypoosmolality by decreasing the renal excretion of free water. While not all cases of hyponatremia require correction with hypertonic saline, this patient has acute onset, severe hyponatremia and has moderate symptoms. Symptoms of hyponatremia include headache, nausea, vomiting, confusion, disorientation, and seizures. Though his sodium needs correction with hypertonic saline, the sodium should not be corrected by more than 9 mEq/L in 24 hours to avoid osmotic demyelination syndrome (ODS). It is important to note that neurological effects related to ODS can take up to 1 week to manifest, including dysarthria and dysphagia. References

1. Fiordoliva I, Meletani T, Baleani MG, et al. Managing hyponatremia in lung cancer: latest evidence and clinical implications. Ther Adv Med Oncol. 2017;9:711-719.

2. Hoorn EJ, Zietse R. Diagnosis and treatment of hyponatremia: compilation of the guidelines. J Am Soc Nephrol. 2017;28:1340-1349.

2. Correct Answer: B Rationale: Central DI occurs in the setting of inadequate production of ADH. Normally, ADH is secreted by the posterior pituitary gland. However, in patients who sustained traumatic brain injury, a decrease or cessation of ADH production can occur, thereby decreasing reabsorption of water

leading to hypernatremia. In otherwise healthy patients, the lack of ADH will cause increased thirst and lead to polydipsia. These symptoms might not be apparent in a critically ill patient. Water deprivation test and administration of desmopressin, as depicted in the flowchart below, may be used to confirm the diagnosis. However, the test is relatively contraindicated in hypovolemic and hypernatremic patients due to risk of exacerbating these. Further testing is not needed in our patient, since the clinical presentation and laboratory studies are consistent with central DI. Diagnostic evaluation for suspected DI includes serum sodium, serum, and urine osmolality, which are expected to show hypernatremia with decreased urine osmolality 295 mOsm/kg. In patients with central DI, the urine osmolality is expected to increase by >100% after administration of desmopressin.

References

1. Robertson GL. Diabetes insipidus. Endocrinol Metab Clin North Am. 1995;24(3):549-572.

2. Makaryus AN, McFarlane AI. Diabetes insipidus: diagnosis and treatment of a complex disease. Clev Clin J Med. 2006;73(1):65-71.

3. Correct Answer: C Rationale: The patient’s history and presentation is consistent with lithium-induced NDI. In NDI, the kidney’s ability to concentrate the urine is decreased despite the presence of sufficient ADH, due to insufficient water reabsorption in the collecting duct. Vasopressin is responsible for regulating the water permeability in the collecting duct and water reabsorption based on the tonicity of the medullary interstitium. Lithium causes NDI by entering the principal cells in the collecting tubules and interfering with aquaporin function, thereby decreasing the ability to concentrate urine. Other causes for NDI include hypercalcemia, hypercalciuria, and obstructive uropathy. In the setting of a positive water deprivation test, the administration of desmopressin can further differentiate primary polydipsia from central and nephrogenic DI. Patients with NDI will not respond to the administration of desmopressin, whereas patients with central DI will have an increased urine osmolality. In NDI, the urine osmolarity is typically lower than serum osmolarity (option C). Since water intake is maintained by compensatory polydipsia, patients do not typically develop overt hypernatremia. References

1. Bockenhauer D, Bichet D. Pathophysiology, diagnosis and management of nephrogenic diabetes insipidus. Nat Rev Nephrol. 2015;11:576-588.

2. Sands JM, Bichet DG. Nephrogenic diabetes insipidus. Ann Intern Med. 2006;144:186-194.

4. Correct Answer: D Rationale: This patient’s hyponatremia is due to low solute intake associated with increased consumption of beer which is also known as beer potomania. Beer is low in solutes and electrolytes. Excessive consumption with an otherwise nutritionally poor diet leads to impaired water clearance and therefore dilutional hyponatremia. Typical laboratory findings include low urine sodium, low urine osmolality, and low ADH levels. However, it should be noted, that the concomitant use of diuretics can make the diagnosis of beer potomania more difficult as diuretics will cause

increased sodium excretion in urine and thus result in higher than expected urine sodium and osmolality concentrations. References

1. Joshi R, Chou S. Beer potomania: a view on the dynamic process of developing hyponatremia. Cureus. 2018;10:3024.

2. Rafei H, Yunus R, Khurana P. Beer potomania: a challenging case of hyponatremia. J Endocrinol Metab. 2016;6:123-126.

5. Correct Answer: A Rationale: The pathophysiology of CSW syndrome remains poorly understood. There are two proposed mechanisms that lead to CSW. One of the mechanisms involves the disruption of the sympathetic neural input which normally promotes the reabsorption of sodium. The other mechanism is natriuresis induced by natriuretic peptides that are released in patients with brain injury. Both of these mechanisms lead to decreased activation of the renin-angiotensin-aldosterone system, leading to decreased sodium absorption at the proximal tubules. CSW syndrome and SIADH both have similar laboratory values, including low serum osmolality, high urine osmolality, and high urine sodium levels. The most important distinguishing feature is the extracellular fluid volume status. Patients with CSW will have low extracellular fluid, reflected by their hypotension, whereas SIADH patients are generally euvolemic. Evaluation of 24 hour uric acid excretion may also aid in the diagnosis as patients with SIADH. Uric acid is normally resorbed in the proximal tubule along with sodium. Both SIADH and CSW results in loss of uric acid via urine. However, an important differentiating feature is that in patients with SIADH, serum uric acid level and fractional excretion of uric acid normalize after correction of the serum sodium level, whereas the uric acid level remains low and uric acid excretion remains elevated in patients with CSW, despite correction of hyponatremia. Common causes for CSW include subarachnoid hemorrhage, intracranial tumors and infections. Overall, the prevalence of CSW is less common than SIADH, but it is important to exclude as the treatment for CSW and SIADH vary vastly. The treatment for CSW includes

administration of fluids and mineralocorticoids. In contrast, SIADH is treated with water restriction. References

1. Maesaka JK, Imbriano L, Mattana J, Gallagher D, Bade N, Sharif S. Differentiating SIAD from cerebral/renal salt wasting: failure of the volume approach and need for a new approach to hyponatremia. J Clin Med. 2014;3:1373-1385.

2. Momi J, Tang CM, Abcar A, Kujubu DA, Sim JJ. Hyponatremia – What is cerebral salt wasting? Perm J. 2010;12(2):62-65.

6. Correct Answer: E Rationale: While the patient’s calculated plasma osmolality is 270 mOsm/kg H2O, his measured plasma osmolality is normal at 295 mOsm/kg H2O, making the diagnosis of pseudohyponatremia most likely. Plasma osmolality (mOsm/kg H2O) = 2 × serum sodium (mEq/L) + plasma glucose (mg/dL)/18 + blood urea nitrogen (mg/dL)/2.8. Plasma consists of 93% water and 7% lipids and proteins. However, in the presence of other substances, such as elevated lipids or paraproteins, the aqueous fraction of plasma will be diluted and thereby falsely lower the serum sodium concentration while the actual serum sodium concentration remains normal. Pseudohyponatremia is due to a laboratory error in the measurement of the serum sodium that leads to low reported sodium with normal plasma osmolality in the presence of total serum cholesterol and lipoprotein X as in patients with obstructive jaundice. Other causes for pseudohyponatremia include severe hypertriglyceridemia, diabetic ketoacidosis, plasma cell dyscrasia, such as in patients with multiple myeloma, and obstructive jaundice as in this patient. References

1. Hussain I, Ahmad Z, Garg A. Extreme hypercholesterolemia presenting with pseudohyponatremia – a case report and review of the literature. J Clin Lipidol. 2015;9(2):260-264.

2. Vo I, Gosmanov AR, Garcia-Rosell M, Wall BM. Pseudohyponatremia in acute liver disease. Am J Med Sci. 2013;345(1):62-64.

7. Correct Answer: B Rationale: In patients who cannot tolerate oral rehydration therapy (ORT), the usage of isotonic fluids such as lactated Ringers (LR) is the most appropriate. The lactate found in LR is converted to bicarbonate in the liver and replaces the bicarbonate that is lost with diarrhea (which contains high level of sodium, bicarbonate and potassium). Thus, persistent diarrhea will lead to a hypokalemic, hyperchloremic metabolic acidosis in most patients which should be replaced by isotonic fluids, such as LR. However, close monitoring of potassium should be performed as the correction of the metabolic acidosis will lead to an intracellular shift of potassium leading to hypokalemia. Reference Pandya S. Practical Guidelines on Fluid Therapy. 2nd ed. Bhalani Medical Book House; 2012.

8. Correct Answer: C Rationale: The cause for his hyponatremia is due to increased free water intake which may manifest in patients with underlying psychiatric disorders, such as schizophrenia, depression, and bipolar disorder in particular. When patients present with polyuria, WDT can help differentiate between psychogenic polydipsia and DI. It is believed that patients with psychogenic polydipsia have a dysregulated thirst mechanism with a reduced osmotic threshold for thirst compared to the ADH threshold, causing them to drink more than needed. It rarely occurs as the amount of free water intake needed to cause hyponatremia is greater than 7 L/d. In patients with polydipsia, an increase in serum osmolarity from water deprivation leads to a marked increase in urine osmolality. On the other hand, urine osmolality will remain low in a patient with DI. Other important differentiating feature is that DI causes hypernatremia while patients with polydipsia are either eunatremic or mildly hyponatremic. Desmopressin has no effect since endogenous release is intact. SIADH and

pseudohyponatremia do not cause polyuria. References

1. Goldman MB. The influence of polydipsia on water excretion in hyponatremic, polydipsic, schizophrenic patients. Endocrinol Metab. 1996;81(4):1465-1470.

2. Hariprasad MK, Eisinger RP, Nadler IM. Hyponatremia in psychogenic polydipsia. Arch Intern Med. 1980;140(12):1639-1642.

3. Trimpou P, Olsson DS, Ehn O, Ragnarsson O. Diagnostic value of the water deprivation test in the polyuria-polydipsia syndrome. Hormones (Athens). 2017;16(4):414-422.

9. Correct Answer: D Rationale: Glucose is an osmotically active solute. In the presence of hyperglycemia, the serum osmolality increases which leads to water movement out of the cells. This leads to a dilutional hyponatremia. Generally, for any glucose level greater that 100 mg/dL, an additional 1.6 mEq/L should be added to the measured serum sodium level. In this case, the patient’s corrected sodium level is approximately 138 mEq/L. Reference Liamis G, Liberopoulos E, Barkas F, Elisaf M. Diabetes mellitus and electrolyte disorders. World J Clin Cases. 2014;2(10):488-496.

10. Correct Answer: B Rationale: Sudden withdrawal of prolonged steroid therapy can lead to low adrenocorticotropic hormone (ACTH) levels due to suppression of the hypothalamic-pituitary-adrenal axis, leading to mineralocorticoid and glucocorticoid deficiency. The patient likely developed adrenal insufficiency after abrupt cessation of his prednisone that he was prescribed for treatment for hypersensitivity pneumonitis. The symptoms vary based on the severity of adrenal insufficiency; however, they most commonly manifest with fatigue, weight loss, and GI symptoms, such as nausea and vomiting. Mineralocorticoid deficiency results in excessive

sodium loss and insufficient potassium excretion in the urine. Significant laboratory findings include hyponatremia, hyperkalemia, and anemia which are all due to the mineralocorticoid and glucocorticoid deficiency due to suppressed hypothalamic-pituitary dysfunction. Reference Broersen LHA, Pereira AM, Jørgensen JO, Dekkers OM. Adrenal insufficiency in corticosteroids use: systematic review and metaanalysis. J Clin Endocrinol Metab. 2015;100(6):2171-2180.

C H AP T E R 5 1

Potassium Jaya Prakash Sugunaraj and Debdoot Saha

1. A 78-year-old lady was found at her home after a fall earlier in the day by her daughter. Medication history is significant for aspirin, statin, glipizide, and acetaminophen use for chronic low back pain. She recently visited her primary care physician with significant weight loss and failure to thrive. She is currently admitted in the intensive care unit (ICU) for multiple rib fractures and flail chest. Her urine analysis and labs are as follows:

pH—7.26

pCO2—16

pO2—101

HCO3—10

Na—136 mEq/L Cl—101 mEq/L Ketones—absent

K—5.7 mEq/L Glucose—234 mg/dL Lactate—0.9

What is the most likely cause for her acid-base abnormality and hyperkalemia? A. Ketones from starvation ketosis B. 5-Oxoproline C. Isopropyl alcohol D. Lactic acidosis

2. A 55-year-old male with a long-standing diabetes, heart failure with reduced ejection fraction, and open-angle glaucoma presents to the emergency room. He reports loose stools for last few days. His list of medications includes metoprolol, acetazolamide, atorvastatin, aspirin, and metformin.

Serum:

1. 2. 3. 4. 5. 6. 7.

pH—7.30 pCO2—40 mm Hg Na—145 mEq/L K—3.0 mEq/L HCO3—10 mEq/L Cl—125 mEq/L Albumin—4 g/dL

Urine: 1. 2. 3.

Na—56 mEq/L K—10 mEq/L Cl—76 mEq/L

The most likely cause of this patient’s acidosis is: A. Diarrhea B. Renal tubular acidosis C. Spironolactone use D. Acetazolamide use

3. A 59-year-old female with a history of hypertension and gout is admitted in the ICU for observation status post thrombolysis for ischemic stroke. Her outpatient medications include metoprolol, colchicine, aspirin, metformin, and meloxicam. Vitals are normal except for sinus tachycardia with a heart rate (HR) of 108 beats per minute. Low bicarbonate is noted on labs prompting an arterial blood gas (ABG), and patient is found to be mildly acidotic. Lab values are given below:

pH

7.32

PaCO2

38 mm Hg

Na Cl BUN Glucose Albumin

140 mEq/L 110 mEq/L 18 mg/dL 76 mg/dL 4 g/dL

K Bicarb Cr Lactic acid

5.9 mEq/L 18 mEq/L 1.1 mg/dL 1.0

Urine electrolytes:

1. 2. 3.

Na—56 mEq/L K—10 mEq/L Cl—56 mEq/L

Which of the following can most likely be expected to be the cause of this? A. Chronic diarrhea B. Type 1 renal tubular acidosis C. Type 4 renal tubular acidosis D. Bartter syndrome

4. A 55-year-old female presents with headaches and generalized weakness. Her mental status is intact. Her vital signs are blood pressure (BP) 170/80 mm Hg, HR 120/min, respiratory rate (RR) 18/min, and temperature 36.8°C.

Lab values: pH

7.48

PaCO2

46 mm Hg

Na Cl

152 mEq/L 100 mEq/L

K Bicarb

3.1 mEq/L 34 mEq/L

What is the next best test to determine the cause of this acid-base abnormality? A. Serum cortisol B. Urine electrolytes C. Serum ionized calcium D. Liver function test

5. A 30-year-old male with history of alcohol abuse presents with nausea and vomiting. He is jaundiced, agitated, and endorsing visual hallucinations. Vital signs are as follows:

BP HR

105/100 mm Hg 110/min

RR Temperature

22/min 38°C

Labs and ABG values are shown below: pH

7.48

PaCO2

28 mm Hg

Na Cl PaO2

138 mEq/L 80 mEq/L 100 mm Hg

K Bicarb Albumin

3.0 mEq/L 22 mEq/L 4 g/dL

Which of the following best describes the acid-base disorder? A. Metabolic acidosis/respiratory alkalosis B. Respiratory alkalosis C. Combined respiratory alkalosis, metabolic acidosis, and metabolic alkalosis D. Metabolic alkalosis and respiratory alkalosis

Chapter 51 Answers 1. Correct Answer: B Rationale: This patient has high anion gap metabolic acidosis with resultant acidosis-induced hyperkalemia. One of the overlooked causes of metabolic acidosis is elevation of serum oxoproline levels, seen more commonly in undernourished patients taking acetaminophen regularly. The patient has no lactate or ketones in their lab workup. Isopropyl alcohol toxicity causes an osmolar gap only and no anion gap (choices A, C, and D are incorrect). Causes of high anion gap metabolic acidosis can be remembered with the mnemonic—GOLDMARK Glycols Oxoproline L-Lactate D-Lactate Methanol Aspirin Renal failure Ketoacidosis When a source of high anion gap metabolic acidosis is not obvious, elevated 5-oxoproline level (also called pyroglutamic acid) should be considered, especially if there is a history of acetaminophen use. References

1. Mehta A, Emmett JB, Emmett M. GOLD MARK: an anion gap mnemonic for the 21st century. Lancet. 2008;372(9642):892.

2. Aronson PS, Giebisch G. Effects of pH on potassium: new explanations for old observations. J Am Soc Nephrol. 2011; 22:1981.

2. Correct Answer: A Rationale: The pH suggests acidosis. The anion gap is 10, which is appropriate for

an albumin of 4. There is no respiratory-driven acidosis as denoted by the normal pCO2 of 40. Therefore, this patient has normal anion gap metabolic acidosis (NAGMA). The next step is to determine the etiology using a calculation of the urine anion gap (UAG) A positive numerical value on UAG in NAGMA can be seen with renal tubular acidosis, spironolactone, and acetazolamide use. Diarrhea typically results in numerically negative UAG. Typically acidosis is associated with hyperkalemia unlike in this scenario, which can be explained by GI loss of potassium, resulting in hypokalemia in spite of academia. Reference

1. Goldstein MB, Bear R, Richardson RMA, et al. The urine anion gap: a clinically useful index of ammonium excretion. Am J Med Sci. 1986;292(4):198-202.

3. Correct Answer: C Rationale: The blood gas suggests a metabolic acidosis. The anion gap is 12, which is appropriate for an albumin of 4. There is no respiratory-driven acidosis component as denoted by normal pCO2 of 38 mm Hg. This patient has NAGMA. The next step is to calculate the UAG A positive numerical value on UAG in NAGMA can be seen in renal tubular acidosis (type 1 and type 4), spironolactone, and acetazolamide use. This patient has serum chemistry consistent with renal 
tubular acidosis type 4 with hyperkalemia not explained by any other etiology. Types of renal tubular acidosis with their respective differentiating features are depicted in the table below.

Defect

Type 1 Distal

Type 2 Proximal

Type 4

Impaired H+ excretion in distal tubule

Impaired HCO3—reabsorption in the proximal tubule

Impaired cation exchange in distal

Urine pH

Usually >5.5

Usually 300 pg/mL) are treated with cinacalcet. Several case reports have suggested cinacalcet can be effective in the management of this condition. References

1. Nigwekar SU, Thadhani R, Brandenburg VM. Calciphylaxis. N Engl J Med. 2018;378:1704-1714.

2. Nigwekar SU, Brunelli SM, Meade D, et al. Sodium thiosulfate therapy for calcific uremic arteriolopathy. Clin J Am Soc Nephrol. 2013;8:11621170.

3. Zitt E, König M, Vychytil A, et al. Use of sodium thiosulphate in a multiinterventional setting for the treatment of calciphylaxis in dialysis patients. Nephrol Dial Transplant. 2013;28:1232-1240.

4. Velasco N, MacGregor MS, Innes A, et al. Successful treatment of calciphylaxis with cinacalcet-an alternative to parathyroidectomy? Nephrol Dial Transplant. 2006;21:1999.

5. Robinson MR, Augustine JJ, Korman NJ. Cinacalcet for the treatment of calciphylaxis. Arch Dermatol. 2007;143:152.

6. Sharma A, Burkitt-Wright E, Rustom R. Cinacalcet as an adjunct in the successful treatment of calciphylaxis. Br J Dermatol. 2006;155:1295.

5. Correct Answer: C Rationale: IV calcium replacement is recommended only when severe effects of hypocalcemia-like neurological symptoms (tetany and seizures), hypotension, prolonged QT interval, or in asymptomatic patients with an acute decrease in serum corrected calcium to ≤7.5 mg/dL. This patient does not meet criteria for treatment. Reference

1. Kraft MD, Btaiche IF, Sacks GS, Kudsk KA. Treatment of electrolyte disorders in adult patients in the intensive care unit. Am J Health Syst Pharm. 2005;62:1663-1682.

6. Correct Answer: A Rationale: The management of Torsades de Pointes begins with assessing if the patient is hemodynamically stable. Most episodes of torsades are selflimiting. However, the danger lies in those patients who go on to develop ventricular fibrillation. Synchronized cardioversion should be performed on a hemodynamically unstable patient in torsades who has a pulse (100 J monophasic, 50 J Biphasic). Pulseless torsades should be defibrillated. This patient is hemodynamically stable. Studies have shown that levels of magnesium drop precipitously a short time after a bolus infusion. It is recommended to start an infusion soon after the bolus dose of magnesium or supplement with intermittent magnesium pushes and follow labs for magnesium levels and monitor patient for magnesium overdose effects. References

1. Biesenbach P, Mårtensson J, Lucchetta L, et al. Pharmacokinetics of magnesium bolus therapy in cardiothoracic surgery. J Cardiothorac Vasc Anesth. 2018;32(3):1289-1294.

2. Tzivoni D, Banai S, Schuger C, et al. Treatment of torsade de pointes with magnesium sulfate. Circulation. 1988;77(2):392-397.

7. Correct Answer: A Rationale: Low serum phosphorus has been strongly associated with diaphragmatic weakness. Since the duration of steroid therapy was short, the probability of steroid induced myopathy is low. Reference

1. Aubier M, Murciano D, Lecocguic Y, et al. Effect of hypophosphatemia on diaphragmatic contractility in patients with acute respiratory failure. N Engl J Med. 1985;313(7):420-424.

8. Correct Answer: B Rationale: The patient has refeeding syndrome. Studies suggest significant benefit in decreasing the rate of tube feed in patients who develop refeeding syndrome after initiation of tube feeds, correcting electrolyte disturbances and only then increasing back to a target rate after electrolytes have stabilized. Reference

1. Doig GS, Simpson F, Heighes P, et al. Restricted versus continued standard caloric intake during the management of refeeding syndrome in critically ill adults: a randomised, parallel-group, multicentre, single-blind controlled trial. Lancet Respir Med. 2015;3(12):943-952.

C H AP T E R 5 3

Acid Base Disorders Jaya Prakash Sugunaraj and Ngoc-Tram Ha

1. A 33-year-old woman presents to the primary care physician for a month-long history of palpations and diarrhea. She has no known medical history. She denies taking any prescribed medication and has not traveled anywhere recently; however, she reports being exposed to “lots of sick people” as part of her job as a nurse. On physical examination, she appears anxious, but the rest of her examination results, including vital signs, are within normal limits. Her BMI is 21 kg/m2. Laboratory studies are as follows:

Sodium 139 mEq/L (mmol/L) Potassium 3.6 mEq/L (mmol/L) Chloride 117 mEq/L (mmol/L) Bicarbonate 16 mEq/L (mmol/L) Glucose 135 mg/dL BUN 12 mg/dL Creatinine 0.7 mg/dL Albumin 4.2 g/dL Which of the following is the MOST likely cause for her laboratory abnormalities? B. Laxative abuse B. Bulimia nervosa C. Factious disorder D. Exogenous insulin use E. Diuretic use

2. A 43-year-old man is brought by the police to the emergency

department after being found unresponsive on the street. He appears unkempt and disheveled. On physical examination, his temperature is 36.0°C, blood pressure is 146/92 mm Hg, pulse rate is 84 beats/min, and respiratory rate is 10 breaths/min. He is not oriented to time and place. Laboratory data show the following:

Sodium 144 mEq/L (mmol/L) Potassium 4.3 mEq/L (mmol/L) Chloride 108 mEq/L (mmol/L) Bicarbonate 8 mEq/L (mmol/L) Glucose 135 mg/dL BUN 18 mg/dL Creatinine 1.1 mg/dL Albumin 4.0 g/L Lactate 0.8 mmol/L TSH 3.60 mIU/L An arterial blood gas is also obtained which shows a pH 7.28, pCO2 18 mm Hg, and plasma osmolality 278 mOsm/kg H2O. Which of the following is the MOST likely diagnosis in this patient? A. Ethylene glycol poisoning B. Starvation ketoacidosis C. Lactic acidosis D. Isopropyl ingestion E. Propylene glycol toxicity

3. An 87-year-old lady is being evaluated in the nursing home after being found to be more lethargic. She had been complaining of decreased appetite due to abdominal pain and ongoing diarrhea for the last 3 days. Her past medical history is significant for hypertension, type II diabetes mellitus, chronic back pain, and diverticulosis. Her medications include metformin, insulin, lisinopril, and naproxen. On physical examination, her temperature

is 37.8°C, blood pressure is 118/82 mm Hg, pulse rate is 104 beats/min, and respiratory rate is 10 breaths/min.

Laboratories obtained show the following:

Sodium 140 mEq/L (mmol/L) Potassium 3.4 mEq/L (mmol/L) Chloride 99 mEq/L (mmol/L) Bicarbonate 20 mEq/L (mmol/L) Glucose 243 mg/dL BUN 22 mg/dL Creatinine 1.6 mg/dL (baseline 1.1) Albumin 3.7 g/L Lactate 1.0 mmol/L ABG pH 7.52 pCO2 20 mm Hg Which of the following is the MOST likely diagnosis in this patient? A. Metabolic acidosis with compensation B. Respiratory acidosis with metabolic alkalosis C. Respiratory alkalosis with increased anion gap metabolic acidosis D. Respiratory alkalosis with acute compensation E. Mixed disorder

4. A 32-year-old male is referred to the nephrologist for abnormal laboratory values. He was recently seen by his primary care physician for his annual physical examination. During his visit, he was told that his “urine was abnormal.” The patient does not have any known medical history except for a 26-year pack-year smoking history. He is normothermic with a blood pressure of 128/90 mm Hg, pulse rate of 58 beats/min, and respiratory rate of 12 breaths/min. The rest of his physical examination is unremarkable. His family

history is only significant for arthritis and “thyroid disease.” Further laboratory studies obtained show the following:

Sodium 142 mEq/L (mmol/L) Potassium 3.3 mEq/L (mmol/L) Chloride 117 mEq/L (mmol/L) Bicarbonate 10 mEq/L (mmol/L) Glucose 243 mg/dL BUN 22 mg/dL Creatinine 1.3 mg/dL (baseline is 1.1) Albumin 4.2 g/L Urine pH 5.6 Urine sodium 62 mEq/L Urine potassium 85mEq/L Urine chloride 126 mEq/L Urine calcium 378 mg/dL What is the MOST likely diagnosis in this patient? A. Gitelman syndrome B. Renal tubular acidosis (RTA) type 1 C. RTA type 2 D. Bartter syndrome E. Salicylate toxicity

5. A 64-year-old woman is being treated in the local intensive care unit (ICU) for septic shock due to Streptococcus pneumoniae. She weighs 120 kg. She is started on empiric antibiotics and resuscitated with 30 mL/kg of normal saline without significant improvement. She is therefore temporarily started on vasopressor support. She improves after 4 days and is weaned off all vasopressors. She extubated to noninvasive ventilation. However, her voice remains hoarse, and she fails her swallow evaluation. She remains on intravenous fluids for hydration for 3-day history of diarrhea.

Laboratory data obtained on admission:

Sodium Potassium Chloride s Bicarbonate BUN Creatinine Albumin Lactate pH pCO2

On Admission

On the Sixth Day of Admission

141 mEq/L (mmol/L) 4.3 mEq/L (mmol/L) 102 mEq/L (mmol/L) 24 mEq/L (mmol/L) 32 mg/dL 1.2 mg/dL 3.7 g/L 5.2 mg/dL 7.29 30 mm Hg

148 mEq/L (mmol/L) 3.8 mEq/L (mmol/L) 120 mEq/L (mmol/L) 19 mEq/L (mmol/L) 23 mg/dL 1.4 mg/dL 2.8 g/L 2.0 mg/dL 7.31 24 mm Hg

What is the MOST likely cause for her laboratory abnormalities? A. Contraction alkalosis B. Diuresis C. Fluid administration D. RTA E. Hypoalbuminemia

6. A 62-year-old man with past medical history of benign prostate hyperplasia, congestive heart failure, and hyperlipidemia presents to his primary care physician for a routine follow-up visit. Today, he complains of progressively worsening fatigue and back pain for the past 4 weeks. He states that there are days that he feels so tired that he does not leave the house. He no longer enjoys playing golf or fishing on the weekends. On physical examination, his temperature is 37.2°C, blood pressure is 134/72 mm Hg, pulse rate is 74 beats/min, and respiratory rate is 12 breaths/min. Pallor conjunctiva and point tenderness are noted over his lumbar spine. The rest of the physical examination is unremarkable. His medications include pravastatin, furosemide, and tamsulosin. He also has been taking aspirin daily for his back pain. Laboratory data show the following:

Sodium 133 mEq/L (mmol/L) Potassium 3.4 mEq/L (mmol/L) Chloride 104 mEq/L (mmol/L) Bicarbonate 16 mEq/L (mmol/L) Glucose 243 mg/dL BUN 28 mg/dL Creatinine 1.7 mg/dL Albumin 2.9 g/L Calcium 11.3 mg/dL Phosphorous 3.0 mg/dL Total protein 6.2 g/dL WBC 8200/µL Hemoglobin 8.9 g/dL Hematocrit 27.3 g/dL Platelets 250,000/µL ABG pH 7.42 pCO2 38 mm Hg What is the MOST likely cause for the patient’s acid-base disturbance? A. Starvation ketoacidosis B. d-lactic acidosis C. Normal anion gap metabolic acidosis D. High anion gap metabolic acidosis E. Dehydration

7. A 54-year-old male was brought to the emergency department with nausea and vomiting. He has a long-standing history of alcohol abuse and cardiomyopathy. He complains of severe abdominal pain. On physical examination, his temperature is 37.6°C, blood pressure is 94/67 mm Hg, pulse rate is 122 beats/min, and respiratory rate is 20 breaths/min. His BMI is 17. He is tender to palpation over the epigastrium, and guarding is noted. He is kept NPO and treated with aggressive fluid resuscitation. Given his poor respiratory

status, he is intubated and admitted to the ICU. After 8 days, he is clinically improving but remains unable to tolerate enteral feeds for which he is started on total parenteral nutrition (TPN) for 5 days.

Laboratory data obtained:

Sodium Potassium Chloride Bicarbonate BUN Creatinine Albumin Hemoglobin Hematocrit Lipase Magnesium Phosphorus pH pCO2

On Admission

On the 13th Day of Admission

137 mEq/L (mmol/L)

142 mEq/L (mmol/L)

3.1 mEq/L (mmol/L) 110 mEq/L (mmol/L) 22 mEq/L (mmol/L) 32 mg/dL 1.2 mg/dL 2.8 g/L 17.1 mg/dL 54.8% 620 U/L 2.7 mg/dL 3.2 mg/dL 7.36 38 mm Hg

4.4 mEq/L (mmol/L) 97 mEq/L (mmol/L) 34 mEq/L (mmol/L)

15 mg/dL 0.9 mg/dL 3.1 g/L 13.2 mg/dL 48.0% 120 U/L 1.9 mg/dL 2.2 mg/dL 7.48 42 mm Hg

Based on the information provided, what is the BEST explanation for his acid-base abnormalities? A. Excessive vomiting B. Administration of normal saline C. Malnutrition D. TPN E. Acute respiratory distress syndrome

8. A 72-year-old woman with a past medical history of hypertension, hyperlipidemia, and congestive heart failure (most recent echocardiogram 3 months ago showed an ejection fraction of 45%) is intubated in the ICU for septic shock in the setting of Escherichia coli bacteremia. She received 9 L total of IV fluids, and on examination, she is awake and cooperative, though she has anasarca. She is

unable to be weaned off the ventilator due to high respiratory rate, and a chest x-ray obtained shows bilateral vascular congestion. She is given furosemide over the next few days and started on enteral feedings.

After 3 days, repeat laboratory data show the following:

Sodium 146 mEq/L (mmol/L) Potassium 3.2 mEq/L (mmol/L) Chloride 110 mEq/L (mmol/L) Bicarbonate 16 mEq/L (mmol/L) Glucose 143 mg/dL BUN 27 mg/dL Creatinine 1.2 mg/dL Albumin 4.1 g/L Calcium 9.8 mg/dL ABG pH 7.48 pCO2 45 mm Hg Which of the following is MOST likely the cause of her acid-base disturbances? A. Bartter syndrome B. Diuretic usage C. Primary respiratory acidosis D. Milk-Alkali syndrome E. RTA type 2

9. An 89-year-old man with a past medical history of chronic kidney disease stage III, diabetes mellitus type II, and hypertension who is 4 days status post small bowel resection for a small bowel obstruction. He has a persistent ileus with nasogastric decompression. On the fifth day, he develops palpitations and lethargy. An ECG and laboratory parameters are shown below:

Sodium 146 mEq/L (mmol/L) Potassium 3.0 mEq/L (mmol/L) Chloride 90 mEq/L (mmol/L) Bicarbonate 18 mEq/L (mmol/L) Glucose 243 mg/dL BUN 22 mg/dL Creatinine 1.8 mg/dL Albumin 3.7 g/L Lactate 1.0 mmol/L ABG pH 7.28 pCO2 28 HCO3 33 PO2 98% Based on this, what do you expect his urine pH value to be? A. 4.1 B. 8.9 C. 6.0

D. 7.4 E. 3.1

10. A 23-year-old man is brought to the hospital by his girlfriend. She states that he has not been feeling well, complaining of nausea and abdominal pain for the past 2 days. His younger sister was recently treated for rotavirus, though she denies that anyone else had any other symptoms. Given his lethargy, he has not taken any of his medications. She also admits that he “binge drinks” on occasion but has not consumed any alcohol over 2 months.

On physical examination, his temperature is 36.8°C, blood pressure is 92/60 mm Hg, pulse rate is 124 beats/min, and respiratory rate is 10 breaths/min. His oral mucosa is dry, and his pulse is palpable though thready.

Sodium 132 mEq/L (mmol/L) Potassium 4.2 mEq/L (mmol/L) Chloride 102 mEq/L (mmol/L) Bicarbonate 10 mEq/L (mmol/L) Glucose 656 mg/dL BUN 42 mg/dL Creatinine 1.4 mg/dL Albumin 4.1 g/L Calcium 9.8 mg/dL ABG pH 7.48 pCO2 45 mm Hg Urine ketones Urine glucose Urine protein Specific gravity Urine pH Urine blood

Positive Positive Negative 1.029 7.0 Negative

Urine nitrite Urine leukocyte esterase Urine clarity Urine color Urine urobilinogen Urine bilirubin

Negative Negative Clear Yellow 0.5 Negative

Which of the following is MOST likely the cause of his acid-base disturbances? A. Starvation ketoacidosis B. Acute kidney injury C. Alcoholic ketoacidosis D. d-Lactic acidosis E. Diabetic ketoacidosis (DKA)

11. A 74-year-old man presents to his primary care physician for worsening lower extremity edema. He has advanced chronic obstructive pulmonary disease and benign prostatic hyperplasia and is a former alcoholic. He is prescribed a diuretic and returns to the office 1 week after. His swelling has improved, but he is complaining about worsening shortness of breath. Given that, an ABG on room air is obtained and shows the following.

ABG pH 7.47 PaO2 82 mm Hg PCO2 53 mm Hg HCO3 38 mEq/L What is the MOST likely acid-base disturbance? A. Chronic respiratory alkalosis B. Acute respiratory alkalosis with metabolic compensation C. Acute metabolic alkalosis with respiratory compensation D. Mixed acid-base disorder E. Chronic metabolic alkalosis

12. A previously healthy 23-year-old woman is taken to the hospital by her boyfriend for weakness since 2 weeks. They recently returned from a trip to Caribbean after which she developed an upper

respiratory infection. She was given Levofloxacin by the urgent care provider. While her cough improved, she noticed weakness in her feet that subsequently traveled up to her arms. On physical examination, vital signs are unremarkable except for a respiratory rate of 8 breaths/min. Deep tendon reflexes are diminished bilaterally.

Sodium 137 mEq/L (mmol/L) Potassium 3.9 mEq/L (mmol/L) Chloride 109 mEq/L (mmol/L) Bicarbonate 32 mEq/L (mmol/L) Glucose 121 mg/dL BUN 16 mg/dL Creatinine 0.8 mg/dL Arterial blood gas pH 7.26 PCO2 72 mm Hg What is the MOST likely acid-base disturbance in this patient? A. Acute on chronic respiratory acidosis B. Respiratory alkalosis with increased anion gap metabolic acidosis C. Mixed disorder D. Respiratory acidosis with metabolic alkalosis E. Uncompensated acute respiratory acidosis

13. A 78-year-old man presents from his skilled nursing facility for generalized weakness and a “funny feeling in his ears.” He recently sustained a fall complicated by a left femur fracture. He underwent open reduction and internal fixation a week ago and has been undergoing physical rehabilitation. He has medical history of hypertension, coronary artery disease, and chronic obstructive pulmonary disease (COPD).

On physical examination, his blood pressure is 152/86 mm Hg, pulse rate is 64 beats/min, and respiratory rate is 22 breaths/min. He is confused to time and place. Laboratory data show the following:

Sodium 140 mEq/L (mmol/L) Potassium 3.8 mEq/L (mmol/L) Chloride 111 mEq/L (mmol/L) Bicarbonate 10 mEq/L (mmol/L) Glucose 95 mg/dL BUN 22 mg/dL Creatinine 1.4 mg/dL (baseline 1.0 mg/dL) Albumin 3.7 g/L Lactate 1.0 mmol/L An arterial blood gas (ABG) is also obtained which shows a pH 7.21, pCO2 38 mm Hg, and plasma osmolality 288 mOsm/kg H2O. What is the MOST likely acid-base disturbance in this patient? A. Anion gap metabolic acidosis and metabolic alkalosis B. Anion gap metabolic acidosis and respiratory acidosis C. Respiratory acidosis and metabolic alkalosis D. Normal anion gap metabolic acidosis E. Respiratory acidosis

14. A 67-year-old man is brought to the local hospital for severe onset of acute abdominal pain. He has chronic obstructive pulmonary disease and hypertension and is an active smoker. On physical examination, his vital signs are unremarkable except for a blood pressure of 188/94 mm Hg and pulse rate of 118 beats/min. He is tender to abdominal palpation and unable to lie still due to pain. A CT angiogram reveals an aortic dissection for which he is taken to the operating room emergently. He receives a total of 12 units of packed red blood cells during the case. He remains intubated postoperatively, and laboratory data obtained after surgery reveal

the following:

Sodium 147 mEq/L (mmol/L) Potassium 3.2 mEq/L (mmol/L) Chloride 100 mEq/L (mmol/L) Bicarbonate 37 mEq/L (mmol/L) BUN 18 mg/dL Creatinine 1.2 mg/dL Calcium 7.8 mg/dL Urine pH 5.8 ABG pH 7.52 pCO2 48 mm Hg What is the MOST appropriate treatment for his acid-base disturbance? A. Furosemide B. Bicarbonate infusion C. Lactated Ringer D. Normal saline E. Acetazolamide

15. A 43-year-old woman is brought to the emergency department by her neighbor by ambulance after being found unresponsive. She has a known past medical history of bipolar disease, seizures, and previous suicide attempts. Her medications include lithium and levetiracetam. She was in her usual state of health prior to this; however, he recalls she was recently treated with “some antibiotic” for acute bronchitis. On physical examination, her temperature is 37.2°C, blood pressure is 110/72 mm Hg, pulse rate is 98 beats/min, and respiratory rate is 8 breaths/min. She is lethargic and unable to follow commands or answer questions. A tongue bite mark and a soiled underwear are also noted. Laboratory data obtained after 6 days are shown below:

Sodium 134 mEq/L (mmol/L) Potassium 4.2 mEq/L (mmol/L) Chloride 102 mEq/L (mmol/L) Bicarbonate 12 mEq/L (mmol/L) Glucose 99 mg/dL BUN 24 mg/dL Creatinine 1.6 mg/dL Lactate 4.4 mmol/L Plasma osmolality 289 mOsm/kg H2O ABG pH 7.48 pCO2 45 mm Hg Urine ketones Urine glucose Urine protein Specific gravity

Negative Negative Negative 1.010

Urine nitrite Urine leukocyte esterase Urine clarity Urine color

Urine pH Urine blood Urine RBC

6.5 Negative Positive

Urine urobilinogen Urine bilirubin

What is the MOST likely cause of her acid-base disturbance? A. Methanol overdose B. Acute kidney injury C. Rhabdomyolysis D. Aspirin-induced E. Isopropyl poisoning

Trace Negative Cloudy Dark yellow 0.7 Negative

Chapter 53 Answers 1. Correct Answer: A Rationale: This patient has a normal anion gap metabolic acidosis which can be seen in patients abusing laxatives. Stool contains a significant amount of bicarbonate along with potassium and sodium. With increased amounts of diarrhea, bicarbonate is lost, leading to a metabolic acidosis. Other causes of normal anion gap metabolic acidosis include renal causes, such as RTA, which emphasizes the importance of measurement of the urine anion gap (UNa + UK − UCl). The urine anion will aid in estimating the kidneys’ ability to excrete acid. A positive urine anion gap is suggestive of renal causes of a normal anion gap metabolic acidosis, whereas a negative urine anion gap points toward a gastrointestinal source, such as diarrhea. The first step with this kind of presentation is to determine the type of acid-base and electrolyte disturbance. Once it is determined that it is a normal anion gap metabolic acidosis, the second step is to differentiate between renal causes versus extrarenal causes by calculating the urine anion gap; a positive urine anion gap points toward renal causes whereas a negative urine anion gap is mostly of gastrointestinal etiology. Reference Goldstein MB, Bear R, Richardson RMA, Marsden PA, Marsden ML, Haleperin ML. The urine anion gap: a clinically useful index of ammonium excretion. Am J Med Sci. 1986;292(4):198-202.

2. Correct Answer: A Rationale: The patient presents with an increased anion gap metabolic acidosis of 26—calculated anion gap = 
Na − (HCO3 + Cl). In addition, his serum bicarbonate level is reduced to 8 mEq/L with a plasma ​osmolal gap of 12 mOsm/kg H2O. Altogether, this points to ethylene intoxication. The osmolal gap is defined as the difference between the measured and the calculated plasma osmolality using the formula below: Calculated plasma osmolality = (2 × plasma sodium) + glucose/18 + BUN/2.8

Ethylene glycol is commonly found in automotive coolants and cleaners, and ingestion of ethylene glycol will cause an increased anion gap metabolic acidosis with increased plasma osmolal gap of >10 mOsm/kg H2O. Of note, methanol poisoning will also lead to similar findings. Isopropyl poisoning will also have an increased osmolal gap of >10 mOsm/L, but it does not usually cause an anion gap metabolic acidosis. Reference Jacobsen D, Bredesen JE, Eide I, Østborg J. Anion and osmolal gaps in the diagnosis of methanol and ethylene glycol poisoning. J Intern Med. 1982;212(1):17-20.

3. Correct Answer: E Rationale: The patient has a mixed acid-base disorder. Her pH is alkalotic. Upon closer look, it appears that she has a respiratory alkalosis (pCO2 20 mm Hg). To determine whether this is an acute or chronic respiratory alkalosis, the following formula can be used: 1-2-4-5 rule. Metabolic Compensation in Respiratory Acidosis/Alkalosis Every 10 mm Hg Change HCO3 − mmol/L in PaCO2 Acute Chronic PaCO2 (Increases)

1

4

PaCO2 (Decreases)

2

5

In acute respiratory alkalosis, for every decrease in PaCO2 by 10, the HCO3 − decreases by 2. In chronic respiratory alkalosis, for every decrease in PaCO2 by 10, the HCO3 − decreases by 5. Based on that, her expected HCO3 − is approximately 14 mEq/L. However, in this case, it is 21 mEq/L which is higher than expected. This suggests that there is concurrent metabolic alkalosis present. This is also confirmed by calculating the Δ-Δ ratio—(calculated AG – expected AG)/24 – measured HCO3 −—which aids in determining the presence of any other normal AG metabolic acidosis or if this is a pure high AG metabolic acidosis. In this case, the Δ-Δ ratio is >1, indicating that there is in fact a metabolic alkalosis present. 
A Δ-Δ ratio < 1 suggests a normal AG metabolic

acidosis. Reference

1. Berend K, De Vries APJ, Gans ROB. Physiological approach to assessment of acid–base disturbances. N Engl J Med. 2014;371:14341445.

4. Correct Answer: B Rationale: This patient likely has RTA type 1. In patients with RTA type 1, there is impairment of acidification in the distal part of the nephrons. Because the kidney is unable to excrete hydrogen ions, this defect leads to the secretion of NH4+. Patients with RTA type 1 usually have non–anion gap acidosis (which is also found in patients with RTA type 2 and 4). However, the main difference between the types of RTA is that type 1 will also have very low levels of HCO3 −, high urine pH of >5.5 (as the kidney is unable to maximally acidify the urine), and positive urine anion gap. In this case, the patient’s urine anion gap is 19 based on the formula, Urine anion gap = UNa + UK − UCl. Main causes of RTA type I include autoimmune diseases, such as rheumatoid arthritis and Sjögren syndrome, obstructive nephropathy, and nephrotoxins, including toluene and amphotericin B. There is a hereditary cause for RTA type 1 which is usually diagnosed in infants or childhood. Though the clear mechanism of acquired RTA type I is unclear, it has been suggested that this due to defective H+-K+ ATPase function at the apical surface of the α-type intercalated cells of the collecting duct. In children, this can manifest with polyuria, stunted growth, recurrent nephrocalcinosis, and hypercalciuria. In adults, RTA type 1 will also lead to nephrocalcinosis. Treatment consists of managing the underlying disorders but can sometimes include usage of thiazide diuretics and/or NaHCO3 and K+ supplementation. While all types of RTA lead to hyperchloremic non–anion gap metabolic acidosis, the main differences between types of RTA are laid out in the table that follows. Type 1 Distal

Type 2 Proximal

Type 4

Defect

Impaired H+ excretion in distal tubule

Impaired HCO3 − reabsorption in the proximal tubule

Urine pH

Usually >5.5

Usually 12. Corrected anion gap (AG) = AG + 2.5 × (4.5 − measured albumin [g/dL]) When calculating the anion gap, attention should be paid to patients with hypoalbuminemia and hypophosphatemia as this will increase the anion gap further. References

1. Zampieri FG, Park M, Ranzani OT, et al. Anion gap corrected for albumin, phosphate, and lactate is a good predictor of strong ion gap in critically ill patients: a nested cohort study. Rev Bras Ter Intensiva. 2013;25(3):205-211.

2. Lee S, Kang KP, Kang SK. Clinical usefulness of the serum anion gap. Electrolyte Blood Press. 2006;4:44-46.

7. Correct Answer: D Rationale: In patients who receive parental nutrition, close monitoring of electrolytes and acid-base status is required as complications such as refeeding syndrome and other metabolic disturbances can occur. Parenteral nutrition consists of various cations, such as sodium, potassium, and calcium along with other anions such as chloride. However, in lieu of chloride, acetate is a solution commonly used in parenteral nutrition as a substitute for chloride (as a buffer) as it reduces the incidence of metabolic acidosis and hyperchloremia. In this case, the patient has developed a metabolic alkalosis after TPN was started. In TPN, if the acetate content is too high, this can lead a metabolic alkalosis because acetate is metabolized to bicarbonate. Similarly, low chloride in TPN solutions will also lead to a metabolic alkalosis. Therefore, to correct this, the chloride content should be increased whereas the amino acid levels should be reduced to aid in reducing the acetate concentration in the TPN. References

1. Peters O, Ryan S, Matthew L, Cheng K, Lunn J. Randomized controlled trial of acetate in preterm neonates receiving parenteral nutrition. Arch Dis Child. 1997;77:12-15.

2. Johnson P. Review of micronutrients in parenteral nutrition for the NICU population. Neonatal Netw. 2014;33(3):155-161.

8. Correct Answer: B Rationale: The patient has primary metabolic alkalosis which is most likely due to diuretic usage. Diuretics such as furosemide increase sodium and water delivery to the distal nephron, which subsequently increases the urinary hydrogen and potassium secretion, thereby leading to metabolic alkalosis and hypokalemia. Furthermore, the contraction of extracellular fluid (ECF) leads to renin and aldosterone secretion, which slows the sodium loss but in turn increases the secretion of potassium and hydrogen ions. This is also known as “contraction alkalosis” which is due to the loss of low bicarbonate–containing extracellular fluid. Patients with Bartter syndrome will have similar findings, but with the recent usage of diuretics and lack of previous history, this is less likely the cause here. Treatment consists of replacement of potassium chloride. References

1. Greenberg A. Diuretic complications. Am J Med Sci. 2000;319(1):10-24. 2. Sica DA, Carter B, Cushman W, Hamm L. Thiazide and loop diuretics. J Clin Hypertens. 2011;13(9):639-643.

9. Correct Answer: A Rationale: The patient has a hypokalemic hypochloremic metabolic alkalosis caused by excessive gastrointestinal losses. This can be caused with prolonged nasogastric suctioning, vomiting, and high ileostomy ostomy output. Of note, despite the low serum chloride, the patients develop a paradoxical aciduria which is due the sodium exchange for hydrogen ion in the kidney.

The above arrows point to “U wave” which can be seen on ECGs in the presence of hypokalemia. References

1. Galla JH. Metabolic alkalosis. J Am Soc Nephrol. 2000;11(2):369-375. 2. Aspelund G, Langer JC. Current management of hypertrophic pyloric stenosis. Semin Pediatr Surg. 2007;16(1):27-33.

10. Correct Answer: E Rationale: Both diabetic and alcoholic ketoacidosis can lead to increased anion gap metabolic acidosis. However, given the patient’s elevated blood glucose and ketones in the urine, this is more suggestive of DKA. Patients with DKA generally present with nausea, vomiting, and abdominal pain. Patients with diabetes can develop DKA due to a reduction in effective insulin leading to increased conversion of free fatty acids into ketones, including β-hydroxybutyrate and acetoacetic acid, thereby leading to ketoacidosis. In this case, the patient’s elevated β-hydroxybutyrate also points toward DKA, though it may not be present in all cases. Treatment includes insulin and fluid administration as most of the patients are volume depleted. References

1. Dunger DB, Sperling MA, Acerini CL, et al. ESPE/LWPES consensus statement on diabetic ketoacidosis in children and adolescents. Arch Dis Child. 2004;89:188-194.

2. Viallon A, Zeni F, Lafond P, et al. Does bicarbonate therapy improve the management of severe diabetic ketoacidosis? Crit Care Med. 1999;27(12):2690-2693.

11. Correct Answer: C Rationale: This patient has metabolic alkalosis with respiratory compensation due to his recent diuretic use. Diuretics such as thiazide and loop diuretics will lead to a net loss of chloride with free water without affecting the bicarbonate excretion. This therefore leads to a so-called “contraction alkalosis.” As a compensatory mechanism, the elevated pH will depress the respiratory centers, thereby leading to an increase of PaCO2 to correct the pH. The formula below can be used to assess for the expected respiratory compensation:

In other words, for every 1 mEq/L rise in HCO3 −, there will be a 0.7 mm Hg increase in PCO2. References

1. Palmer BF, Naderi ASA. Metabolic complications associated with use of thiazide diuretics. J Am Soc Hypertens. 2007;1(6):
381-392.

2. Sood P, Paul G, Puri S. Interpretation of arterial blood gas. Indian J Crit Care Med. 2010;14(2):57-64.

12. Correct Answer: A Rationale: The patient has a respiratory acidosis due to Guillain-Barre syndrome. Her pH is acidic and her inappropriately elevated pCO2 points toward a primary respiratory etiology. To determine whether there is any compensation, the following formula can be used.

In other words, for every increase in PaCO2 by 10, the HCO3 − increases by 1 in acute respiratory acidosis and in chronic respiratory acidosis, for every increase in PaCO2 by 10, the HCO3 − increases by 4. In this case, the patient’s expected HCO3 − is around 27 mEq/L, but it is measured higher 
(HCO3 − 32 mEq/L). The higher HCO3 − in the patient suggests chronic respiratory acidosis. However, the low pH points towards an additional uncompensated acute component to the respiratory acidosis. hence the patient has acute on chronic respiratory acidosis. Reference Rose B, Post T. Clinical Physiology of Acid-Base and Electrolyte Disorders. 5th ed. USA: McGraw-Hill; 2001.

13. Correct Answer: B Rationale: Based on the patient’s pH, there is acidosis which is likely metabolic given his low HCO3 −. The next step is to calculate the anion gap (AG). In this case, the patient has an AG of 19. The next step is to see whether there is any respiratory compensation via Winter ’s formula as follows:

When using Winter ’s formula, the expected PaCO2 is 21 to 25. However, in this patient, the PaCO2 is higher than that, suggesting that there is respiratory acidosis present in addition to the anion gap metabolic acidosis. In this case, patient has aspirin overdose which explains the tinnitus and metabolic acidosis along with COPD leading to a respiratory acidosis. Reference

1. Kellum JA. Disorders of acid-base balance. Crit Care Med.

2007;35(11):2630-2636.

14. Correct Answer: E Rationale: The patient has a metabolic alkalosis that is caused by citrate from numerous blood transfusions. Citrate is used as an anticoagulant in blood bags as a preservative and is converted from citrate to form three moles of sodium bicarbonate via the liver. Another mechanism that leads to metabolic alkalosis includes the depletion of chloride due to the reduced chloride concentration content of the donor ’s blood. Both mechanisms lead to hypochloremic metabolic alkalosis. This is best treated with acetazolamide as it is carbonic anhydrase inhibitor and cause NaHCO3 − diuresis by inhibiting the reabsorption of bicarbonate ions from renal tubules. Other complications from massive blood transfusions include hypocalcemia due calcium chelation and hypomagnesemia. References

1. Calladine M, Gairdner D, Naidoo BT, Orrell DH. Acid-base changes following exchange transfusion with citrated blood. Arch Dis Child. 1965;40:626-631.

2. Li K, Xu Y. Citrate metabolism in blood transfusions and its relationship due to metabolic alkalosis and respiratory acidosis. Int J Clin Exp Med. 2015;8(4):6578-6584.

15. Correct Answer: C Rationale: The patient has increased anion gap metabolic acidosis of 20—calculated anion gap = Na − (HCO3 + Cl). 
However, her osmolal gap is 90%.

Which of the following is the next BEST step in management? A. Administration of tocilizumab while ruling out infection B. Immediate administration of empiric vancomycin and cefepime C. Fluid and vasopressor management to support hemodynamics while cytokine release syndrome resolves D. IV hydrocortisone

6. A 53-year-old man with history of chronic hepatitis C and acquired immune deficiency syndrome (history of cryptococcal meningitis, last CD4 count 250 cells/µL on antiretroviral therapy with undetectable viral load) presents for open liver resection after he was diagnosed with liver cancer. Following an uncomplicated operation, he is recovering on surgical floor. On postoperative day 3, he develops fever (temperature, 38.9°C). Other vitals are notable for blood pressure of 85/52 mm Hg, heart rate of 110 beats/min, respiratory rate of 26 breaths/min, and oxygen saturation of 92% on 4 L nasal cannula. Abdominal examination is benign. Chest x-ray reveals likely infiltrate and/or atelectasis in the right lower lobe. Lactate level is 3 mmol/L.

Which of the following is the MOST appropriate initial antibiotic regimen for this patient? A. Vancomycin, cefepime, azithromycin B. Levofloxacin C. Trimethoprim and sulfamethoxazole (TMP-SMX) D. Meropenem

7. A 45-year-old female is admitted to the surgical ICU following liver transplantation. On postoperative day (POD) 3, she develops a tonic-clonic seizure which is terminated by intravenous midazolam. However, she requires intubation for persistently poor mental status. CT scan of the head is obtained immediately and is negative for bleeding or mass lesion. Lumbar puncture is deferred due to

coagulopathy (INR 1.9, Plt 50). A brain MRI performed 6 hours later shows left temporal increased signal intensity in T2 and FLAIR weighted images.

Which of the following treatment is most effective to decrease patient mortality? A. Intravenous acyclovir B. Gancyclovir C. Levetiracetam D. Corticosteroids

8. A 52-year-old female with severe COPD is admitted to the ICU for hypercarbic respiratory failure. She requires mechanical ventilation and is treated with methylprednisolone and azithromycin. She is extubated on ICU day 4. After extubation, she complains of leftsided flank pain, but her examination and labs are normal. On day 6, a vesicular rash with erythematous base develops diffusely across her abdomen, and additional lesions are noted on her face and arms.

Which of the following is the next best step to diagnose her condition? A. Full ophthalmologic examination B. Unroofing vesicle and sending varicella-zoster virus and herpes virus direct fluorescence antibody stain (DFA) and reflex culture C. Punch biopsy for dermatopathology D. Skin testing for latex allergy

Chapter 70 Answers 1. Correct Answer: B Rationale: Patient has clinical symptoms and laboratory and radiographic signs consistent with pneumocystis jirovecii pneumonia (PJP), an opportunistic infection typically affecting immunocompromised hosts. These may include patients with acquired immune deficiency syndrome (AIDS), those with malignancies, stem cell and solid organ transplant recipients, and patients receiving high-dose corticosteroids and other immunosuppressants. PJP classically presents with fever, dry cough, and hypoxemic respiratory failure with exertional oxygen desaturations. Positive PJP PCR or silver stain from induced sputum or bronchoalveolar lavage (BAL) samples are confirmatory. LDH elevation is common, particularly in HIV-positive patients in whom the sensitivity is reported to be 100%; however, sensitivity is lower (∼60%) in HIV-negative individuals. Radiographic findings include bilateral patchy infiltrates with cysticappearing opacities on chest x-rays (as seen in the above chest x-ray). On high-resolution CT scans, the most common findings are ground glass opacities with cysts that are seen in approximately one third of patients. First-line treatment is intravenous trimethoprim/sulfamethoxazole (Bactrim) which should be continued for at least 7 to 10 days or until clinical improvement is seen, when transition to oral antibiotics is considered. Total duration of therapy is typically 3 weeks. PJP prophylaxis should be initiated in patients with CD4 count less than 200 cells/mm3 and those receiving high-dose steroids (>20 mg/d) for more than one month, particularly if the patient is receiving additional cytotoxic agents, as in this clinical vignette. References

1. Kanne JP, Yandow DR, Meyer CA. Pneumocystis jiroveci pneumonia: high-resolution CT findings in patients with and without HIV infection. Am J Roentgenol. 2012;198(6):555-561.

2. Limper AH, Knox KS, Sarosi GA. An Official American Thoracic Society Statement: treatment of fungal infections in adult pulmonary and critical care patients. Am J Resp Crit Care Med. 2011;183(1):96-128.

3. Smith DE, Forbes A, Davies S, Barton SE, Gazzard BG. Diagnosis of

Pneumocystis carinii pneumonia in HIV antibody positive patients by simple outpatient assessments. Thorax. 1992;47(12):1005-1009.

4. Vogel MN, Weissgerber P, Goeppert B, et al. Accuracy of serum LDH elevation for the diagnosis of Pneumocystis jiroveci pneumonia. Swiss Med Wkly. 2011;141:w13184.

5. Wieruszewski PM, Barreto JN, Frazee E, et al. Early corticosteroids for pneumocystis pneumonia in adults without HIV are not associated with better outcome. Chest. 2018;154(3):636-644.

2. Correct Answer: A Rationale: This patient has neutropenic fever which is a serious complication of chemotherapy. It is defined by an isolated temperature >38.3°C (101°F) or a temperature of >38.0°C (100.4°F) lasting for >1 hour in a patient with neutropenia. If not treated promptly, sepsis and septic shock can develop. Severe neutropenia is defined as an absolute neutrophil count (ANC) of less than 500 cells/mm3. Neutrophils prevent bacterial and fungal infections; therefore, neutropenic patients are especially prone to these types of infections. Adequate antibiotic therapy is the key intervention to decrease mortality of this patient. The initial empiric antibiotic therapy for febrile neutropenia is a beta-lactam with pseudomonas coverage (such as ceftazidime, cefepime, piperacillin-tazobactam) or a carbapenem (meropenem, imipenem). While vancomycin is not typically a part of the empiric antibiotic regimen, it should be added in patients with evidence of hemodynamic instability (such as in this patient), pneumonia, skin or soft tissue infection, or catheter-related infection. Vancomycin or alternative gram-positive coverage may be discontinued at 48 hours if there is no confirmation of gram-positive pathogens. Antibiotic therapy should be further tailored to resistance patterns of bacteria previously isolated from a patient and colonization with resistant organisms such as methicillinresistant Staphylococcus aureus (MRSA), vancomycin-resistant enterococcus (VRE), extended-spectrum β-lactamase (ESBL) gram-negative organisms, or carbapenemase-producing organisms. Patients should be evaluated for the presence of indwelling lines both as a source of infection and for removal depending on the isolated pathogen. In this clinical scenario, immediate removal of central line is not indicated.

In patients that are considered to be high risk for infectious complications (hospitalized at the time of fever, age >65 years, expected protracted neutropenia lasting >10 days, or ANC 1 or 40%). Distinguishing CRS from other syndromes with similar presentations, such as septic shock, is challenging. Therefore, broad-spectrum antibiotics should be the first-line therapy administered to each CAR T patients with new-onset hemodynamic or respiratory instability. Delaying antibiotic treatment while awaiting the clinical benefit of tocilizumab or confirming infection results in high mortality rates. Corticosteroids are the mainstay of treatment for high-grade CAR T neurotoxicity. In this patient, somnolence is likely related to hypotension, fever, and CRS, and if improved with fluid, vasopressors, and antibiotics, treatment with corticosteroids is not indicated. References

1. Le RQ, Li L, Yuan W, et al. FDA approval summary: tocilizumab for treatment of chimeric antigen receptor T cell-induced severe or lifethreatening cytokine release syndrome. Oncologist. 2018;23(8):943-947.

2. Neelapu SS, Tummala S, Kebriaei P, et al. Chimeric antigen receptor Tcell therapy—assessment and management of toxicities. Nat Rev Clin Oncol. 2018;15(1):47-62.

3. Porter D, Frey N, Wood PA, Weng Y, Grupp SA. Grading of cytokine release syndrome associated with the CAR T cell therapy tisagenlecleucel. J Hematol Oncol. 2018;11(35):e1-e12.

6. Correct Answer: A

Rationale: This patient most likely developed hospital-acquired pneumonia complicated by sepsis. Initial antimicrobial therapy should be broad while awaiting additional workup with consideration about localizing symptoms (pulmonary). The patient has been admitted to hospital for greater than 48 hours prior to infectious symptom onset, and treatment should cover MRSA and Pseudomonas. Therefore, the Correct Answer is A. Initial therapy with vancomycin (MRSA coverage), cefepime (gram-negative including pseudomonas), and azithromycin (atypical coverage) is appropriate. Although patient previously had an AIDS-defining illness (cryptococcal meningitis), a CD4 count above 200 cells/µL makes him less susceptible to opportunistic infections such as PJP. Based on description, his CXR is not suggestive of PJP pneumonia. Therefore, empiric TMP-SMX is not indicated at this point. Treatment with TMP-SMX alone would leave gaps in gram-negative coverage including pseudomonas and therefore would not be the correct empiric regimen. Meropenem and levofloxacin are not correct treatments for potential MRSA infection. There are some unique considerations in critically ill patients with HIV/AIDS. Prior antibiotic prophylaxis (ie, azithromycin for mycobacterium avium complex) and the potential for resistance should be considered when choosing an empiric regimen. Mycobacterium tuberculosis infection (TB) occurs in patients with HIV/AIDS. If there is clinical concern for TB, empiric fluoroquinolones (ie, levofloxacin) should be avoided. Fluoroquinolones may result in short-term improvement through partial treatment of TB followed by later clinical decompensation and bacterial resistance. Patients with HIV/AIDS and/or intravenous drug use are at greater risk of fungemia. Fungal coverage should be initiated in individuals who fail to improve with initial antimicrobial therapy. Additional caution for drug interactions is needed in patients taking antiretroviral therapy. Azoles and macrolides interact with some forms of antiretroviral therapy and consultation with a pharmacist should occur prior to initiation of these medications. References

1. Gade ND, Qazi MS. Fluoroquinolone therapy in Staphylococcus aureus infections: where do we stand? J Lab Physicians. 2013;5(2):109-112.

2. Japiassú AM, Amâncio RT, Mesquita EC, et al. Sepsis is a major

determinant of outcome in critically ill HIV/AIDS patients. Crit Care. 2010;14(4):e1-e8.

3. Panel on Opportunistic Infections in HIV-Infected Adults and Adolescents. Guidelines for the prevention and treatment of opportunistic infections in HIV-infected adults and adolescents: recommendations from the Centers for Disease Control and Prevention, the National Institutes of Health, and the HIV Medicine Association of the Infectious Diseases Society of America. Available at http://aidsinfo.nih.gov/contentfiles/lvguidelines/adult_oi.pdf. Accessed 11/3/2018.

4. Silva JM, dos Santos Sde S. Sepsis in AIDS patients: clinical, etiological and inflammatory characteristics. J Int AIDS Soc. 2013;16(1):e1-e8.

7. Correct Answer: A Rationale: This patient most likely developed herpes simplex virus (HSV) encephalitis—the most common cause of viral encephalitis with significant morbidity and mortality. MRI brain is the most specific imaging modality for encephalitis. Findings consistent with HSV encephalitis on neuroimaging include temporal or inferior frontal lobe edema and increased signal intensity on T2 and fluid-attenuated inversion recovery (FLAIR) images. Suspected HSV is treated with empiric acyclovir while diagnostic CSF studies are pending. In this case, imaging and clinical presentation is highly suggestive and warrants treatment. Patients who are seropositive for HSV prior to transplant are often on HSV prophylaxis with oral acyclovir. In this case, high-dose intravenous acyclovir should be initiated. Ganciclovir is used to treat viral encephalitis from varicella-zoster virus, human herpes virus 6, and cytomegalovirus (in conjunction with foscarnet) but is not the treatment for HSV. Treatment with levetiracetam may be initiated but would not be alone sufficient to improve patient outcome. Steroids are used to treat some types of encephalitis including Epstein-Barr virus and varicella-zoster virus, but they are not recommended for HSV encephalitis. References

1. A Report from the British Society for Antimicrobial Chemotherapy Working Party on Antiviral Therapy; Management of herpes virus infections following transplantation. J Antimicrob Chemotherapy. 2000;45(6):729-748.

2. Guenette A, Husain S. Infectious complications following solid organ transplantation. Crit Care Clin. 2019;35(1):151-168.

3. Tunkel AR, Glaser CA, Bloch KC, et al. The management of encephalitis: clinical practice guidelines by the Infectious Diseases Society of America. Clin Infect Dis. 2008;47(3):303-327.

4. Venkatesan A, Tunkel AR, Bloch KC, et al. Case definitions, diagnostic algorithms, and priorities in encephalitis: consensus statement of the international encephalitis consortium. Clin Infect Dis. 2013;57(8):11141128.

5. Wilck MB, Zuckerman RA; AST Infectious Diseases Community of Practice. Herpes simplex virus in solid organ transplantation. Am J Transplant. 2013;13(4):121-127.

8. Correct Answer: B Rationale: Herpes zoster is caused by reactivation of latent varicella-zoster virus in cranial nerves or dorsal root ganglia. Reactivation of the virus is caused by decreased T-cell immunity and may result from increased age, primary infection during the time of an immature immune system, transplantation, immunosuppressive agents (such as steroids), or HIV infection. A diagnosis of herpes zoster is made by unroofing a vesicle and sending the fluid for direct fluorescent antibody testing and reflex viral culture. Distinguishing reactivation of varicella-zoster virus from the vesicular rash of disseminated herpes virus may pose challenges clinically. Therefore, testing for both is appropriate. Latex allergies are common with manifestations ranging from contact dermatitis to anaphylaxis. While skin testing is used to diagnose a variety of allergens, a proportion of patients with latex allergy will develop anaphylaxis. Therefore, the best test for suspected latex allergy would be serologic evaluation with IgE-specific latex antibody. The prodromal pain makes a latex allergy less likely. Punch biopsy may be useful in a variety of dermatologic diseases but is not necessary to diagnose herpes zoster. A full dilated eye examination is necessary to rule out herpes zoster

ophthalmicus but would not confirm the diagnosis of herpes zoster in the absence of ocular involvement. References

1. Cohen JI. Herpes zoster. N Engl J Med. 2013;369(3):255-263. 2. Kelly KJ. Skin and serologic testing in the diagnosis of latex allergy. J Allergy Clin Immunol. 1993;91(6):1140-1145.

3. Naldi L, Venturuzzo A, Invernizzi P. Dermatological complications after solid organ transplantation. Clin Rev Allergy Immunol. 2018;54(1):185212.

4. Pavlopoulou ID, Poulopoulou S, Melexopoulou C, Papazaharia I, Zavos G, Boletis IN. Incidence and risk factors of herpes zoster among adult renal transplant recipients receiving universal antiviral prophylaxis. BMC Infect Dis. 2015;15(1):e1-e8

5. Pergam NA, Limaye AP; AST Infectious Diseases Community of Practice. Varicella zoster virus in solid organ transplantation. Am J Transplant. 2013;13(4):138-146.

C H AP T E R 7 1

Infections in the Immunocompromised Host Francisco Jesús Marco Canosa, Fatima I Adhi, Ceena N. Jacob and Teny M. John

1. A 38-year-old female with no significant past medical history was brought to the emergency department with confusion and lethargy. Her family noted that the patient had been complaining of worsening headaches for the last several days. On admission, she was afebrile and hemodynamically stable. Initial laboratory workup showed leukopenia (white blood cell [WBC] count of 3 000 cells/mL) with absolute lymphopenia. Human immunodeficiency virus (HIV) testing was positive with a ribonucleic acid (RNA) viral load of 68 000 copies/mL and CD4 count of 25 cells/µL. Magnetic resonance imaging (MRI) of the brain revealed multiple ring-enhancing lesions of different sizes with surrounding edema and mass effect. What is the NEXT BEST step to diagnose her disease process?

A. India ink staining B. Toxoplasma gondii IgG antibody C. Stereotactic brain biopsy D. Cytomegalovirus CSF polymerase chain reaction (PCR) E. Quantiferon tuberculosis testing

2. A 29-year-old male is admitted from the emergency department with fevers. He complained of night sweats and painful cervical lymphadenitis for the last 7 days. He was diagnosed with HIV/AIDS 1 month ago when he was admitted with an episode of communityacquired pneumonia. His CD4 count was 80 cells/µL and HIV RNA viral load was 1 million copies/mL at the time of diagnosis. He was

started on anti-retroviral therapy (ART) with tenofovir-emtricitabine and raltegravir. On examination, his blood pressure is 90/45 mm Hg and pulse rate is 106 beats per minute. Blood cultures are in process. He is appropriately fluid resuscitated and started on vancomycin and piperacillin-tazobactam to cover potentially hospital-acquired pathogens. Immune reconstitution inflammatory syndrome (IRIS) secondary to disseminated mycobacterium avium complex (MAC) infection is suspected. What is the NEXT BEST step regarding his ART during this admission?

A. Hold ART and resume in 1 week B. Hold ART and resume in 2 weeks C. Continue ART D. Optimize ART by increasing the dose of current medications E. Optimize ART by changing the ART regimen to include two new medications Infections in the neutropenic patient

3. A 65-year-old male is admitted to the hospital with malaise and fatigue for the past week. His WBC count on admission was 27 000 cells/mL. The patient was ultimately diagnosed with high-grade acute promyelocytic leukemia. A tunneled central venous catheter was placed and the patient was started on all-trans-retinoic acid, daunorubicin, and cytarabine. Four days after initiation of chemotherapy, he developed a fever of 39.1°C (102.3°F) and altered mental status. His blood pressure was 90/54 mm Hg, heart rate 108 beats per minute, respiratory rate 24 breaths per minute, and oxygen saturation 94% on 5 L of supplemental oxygen. Laboratory evaluation was significant for neutropenia (absolute neutrophil count 300 cells/µL). His chest x-ray showed a focal consolidation in the right middle lobe. He was admitted to the intensive care unit and blood cultures were obtained. What is the BEST empiric intravenous antibiotic regimen for this patient?

A. Ciprofloxacin and ampicillin-sulbactam B. Meropenem alone C. Piperacillin-tazobactam alone D. Piperacillin-tazobactam and vancomycin E. Meropenem and vancomycin and micafungin

4. A 60-year-old female was admitted to the ICU with acute hypoxic respiratory failure. She endorsed malaise, fevers, and neck swelling for 2 weeks prior to presentation. She received a bilateral lung transplant [cytomegalovirus (CMV) donor negative/recipient negative, Epstein-Barr virus (EBV) donor negative/recipient positive] 4 months ago for idiopathic pulmonary fibrosis and is currently on immunosuppression with azathioprine 200 mg daily and tacrolimus 2 g twice daily. Her antimicrobial prophylaxis includes trimethoprim-sulfamethoxazole one double-strength tablet thrice weekly and itraconazole 200 mg daily. On admission, she was alert and oriented, afebrile, and hemodynamically stable. Cervical lymphadenopathy was present. Her WBC count was 5600 cells/µL, hemoglobin 9.6 g/dL, alkaline phosphatase 125 U/L, aspartate aminotransferase (AST) 100 U/L, and alanine aminotransferase (ALT) 130 U/L. Her chest x-ray showed bilateral diffuse infiltrates. Blood cultures showed no growth on culture at 24 hours of collection. Serum EBV quantitative deoxyribonucleic acid (DNA) PCR was 100 000 copies/mL (undetectable on prior measurement 1 month ago). Posttransplant lymphoproliferative disease is suspected. What is the NEXT BEST step in the management of this patient?

A. Start treatment with rituximab B. Reduce the current dose of immunosuppressants C. Start treatment with rituximab and chemotherapy D. Start treatment with acyclovir E. Start treatment with valganciclovir

5. A 62-year-old male who underwent bilateral lung transplantation (CMV donor positive/recipient negative, EBV donor positive/recipient negative) for end-stage lung disease due to chronic obstructive pulmonary disease is admitted to the ICU with left lower quadrant abdominal pain, diarrhea, and hypotension. His symptoms started 5 days ago and have been progressively worsening. Diarrhea is mainly watery and frequency ranges from four to five times per day. On examination, the patient was afebrile with blood pressure 84/60 mm Hg, heart rate of 100 beats per minute, and dry oral mucosa. His abdomen was diffusely tender to palpation. An x-ray of the abdomen demonstrated colonic ileus with no evidence of gas under the diaphragm. Stool Clostridium difficile PCR, stool ova and parasites testing, and stool cultures are negative. CMV is undetectable by PCR in the plasma. What is the next BEST step in managing this patient?

A. CT scan of the abdomen with contrast B. Colonoscopy and biopsies C. Check serum EBV quantitative viral load D. 24-hour stool fat test E. Empiric treatment with micafungin and valganciclovir

6. A 50-year-old female underwent allogeneic stem cell transplantation for acute myeloid leukemia 10 days ago. She is brought to the ICU with fevers and hypotension. She is neutropenic. She noticed a red rash on her trunk and extremities yesterday. On examination, she appears ill and is febrile (38°C). Her blood pressure is 84/60 mm Hg, heart rate 120 beats per minute, and respiratory rate 36 breaths per minute. Her central venous catheter site is clean and nontender. Physical examination demonstrates grade 3 mucositis of her buccal mucosa, a diffuse erythematous, blanchable rash, and bilateral crackles on auscultation of her posterior lung fields. Her chest x-ray demonstrated bilateral diffuse infiltrates. A bedside echocardiogram showed normal valves with preserved left ventricular function. Her blood cultures grow gram positive cocci in

pairs and chains in both sets of aerobic and anaerobic bottles collected on transfer. What is the MOST LIKELY diagnosis?

A. Catheter-related infection due to coagulase-negative staphylococci B. Staphylococcal toxic shock syndrome C. Engraftment syndrome D. Infective endocarditis due to Enterococcus faecalis E. Septic shock due to viridans group Streptococci

7. A 39-year-old male who underwent a haploidentical allogeneic hematopoietic stem cell transplantation (HSCT) for acute myeloid leukemia 2 years ago is admitted to the ICU with acute hypoxic respiratory failure requiring supplemental oxygen through a highflow nasal cannula. His HSCT was complicated by graft-versus-host disease (GVHD) of the skin and gastrointestinal tract 6 weeks ago for which he was treated with pulse dose steroids. He was recovering from an upper respiratory tract infection caused by rhinovirus (nasopharyngeal swab PCR positive) 3 weeks ago when he started to experience shortness of breath that progressively worsened. His WBC count was 4.5 cells/µL with 80% neutrophils on admission. His creatinine was elevated at 2.1 mg/dL (baseline: 1 mg/dL). A CT scan of the chest without contrast revealed multifocal nodular opacities with right-sided predominance. Blood cultures, urine histoplasma antigen, serum cryptococcal antigen, and serum Aspergillus galactomannan were negative. His serum β-1,3-d-glucan assay was positive. The patient’s sputum culture grew normal respiratory flora. He was started on intravenous vancomycin and piperacillintazobactam, but his respiratory status continued to decline eventually requiring intubation 2 days into his admission. Bronchoscopy was performed with bronchoalveolar lavage (BAL). Initial stains on the BAL fluid showed nonpigmented, septate hyphae branching at right angles. What is the NEXT step in the antimicrobial management of this patient?

A. Add IV micafungin B. Add PO voriconazole C. Add IV voriconazole D. Start IV amphotericin B E. Continue current management and wait for final pathogen identification

8. A 26-year-old female with a history of sickle cell disease complicated by multiple sickle cell crises in the past year is admitted to the ICU with acute hypoxic respiratory failure and shock. On arrival to the ICU, she is febrile to 39.1°C with a blood pressure of 82/36 mm Hg and heart rate 110 beats per minute. She is intubated and mechanically ventilated. Initial laboratory evaluation demonstrates a neutrophilic- predominant leukocytosis to 14 000 cells/µL. Her chest x-ray on admission shows a left lower lung infiltrate with an associated pleural effusion. Blood cultures are in process. What would be the NEXT BEST STEP to confirm this patient’s diagnosis?

A. CT scan of the chest with and without IV contrast B. Sputum cultures C. Streptococcus pneumoniae urine antigen D. Legionella urine antigen E. Bronchoscopy with BAL

9. A 45-year-old female who underwent bilateral lung transplantation 6 days ago is brought to the ICU intubated following a seizure episode. Her transplantation was uneventful and she was transferred to a regular nursing floor on postoperative day 4. She was intubated at bedside for airway protection and brought to the ICU. On examination, the patient is sedated and her pupils were mildly dilated but equally reactive to light. Her blood pressure was 110/90 mm Hg and heart rate 120 beats per minute. Mild purulence is noted from the lower part of sternotomy site with no obvious

instability or bony crepitations. The output from her chest drains was nonpurulent. An arterial blood gas shows an elevated lactate of 2.5 mmol/L, partial pressure of oxygen of 92 mm Hg, and partial pressure of carbon dioxide of 38 mm Hg. Her laboratory results demonstrate a WBC count of 16 500 cells/µL, hemoglobin of 9.1 g/dL, platelet count of 350 000/µL. Blood cultures are collected. Wound cultures sent from the regular nursing floor prior to transfer show numerous neutrophils but a negative gram stain. A CT scan of the brain did not show any acute abnormalities. Debridement of sternal wound is done and the patient is started on empiric vancomycin and piperacillin-tazobactam. What is the NEXT BEST step in the management of this patient?

A. Wait for final culture results, no additional antibiotics B. Lumbar puncture, empirical IV acyclovir to treat Herpes simplex encephalitis C. Check serum ammonia level; start IV doxycycline to cover Mycoplasma hominis D. Start IV micafungin for empiric fungal coverage E. Order an MRI of the brain to rule out posterior reversible leukoencephalopathy

10. A 65-year-old former Vietnam War veteran male was admitted to the intensive care unit with shock. He had underwent orthotopic liver transplantation 3 months ago for cirrhosis due to alcohol abuse and hepatitis C infection. His posttransplant course had been complicated by graft-versus-host disease treated with pulse dose methylprednisolone for 3 days followed by prednisone 60 mg daily, which he was currently on. He presented to the emergency room with complaints of headache and wheezing for the last 2 days and was also found to be somnolescent. On the first day of admission, he was noted to have intermittent bouts of cough with two episodes of small volume hemoptysis. Chest x-ray showed bilateral patchy nodular opacities for which he was started on vancomycin and piperacillin/tazobactam. However, on the second day of admission,

his condition acutely worsened with tachycardia, hypoxemia, and hypotension requiring vasopressor support. Lactate was elevated raising suspicion of sepsis, and workup for an infectious source was initiated. Urinalysis was unremarkable, stool Clostridioides difficile PCR was negative, and CMV PCR was undetectable in blood. CT scan of the head was unremarkable. Two sets of blood cultures were sent. Gram stain of cerebrospinal fluid (CSF) showed gram-negative rods identified later the same day as Escherichia coli by PCR, and consequently piperacillin/tazobactam was changed to cefepime. Chest imaging the next day showed marked worsening of opacities on the left upper lobe and right lower lobe, and he was intubated for worsening hypoxia. The tracheal aspirate was sent for culture; however, you received a call from the microbiology lab the same day informing you of an unexpected finding on the gram stain of tracheal aspirate. A representative image follows.

Which of the following describes the best treatment plan for this patient? A. Continue cefepime B. Continue cefepime and add ivermectin C. Discontinue cefepime and start ivermectin D. Discontinue cefepime, reduce the dose of prednisone and start ivermectin E. Continue cefepime, reduce the dose of prednisone and start ivermectin

Chapter 71 Answers 1. Correct Answer: B Rationale: This patient with newly diagnosed HIV/AIDS, and a CD4 count 50% crescents on renal biopsy, serum creatinine >7 mg/dL, and requirement of renal replacement therapy within 72 hours of presentation. References

1.

Orsi D, Correa-Lopez W, Cavagnaro J. Rheumatologic and inflammatory conditions in the ICU. In: Oropello JM, Pastores SM, Kvetan V, eds. Critical Care. New York, NY: McGraw-Hill. http://accessanesthesiology.mhmedical.com/content.aspx? bookid=1944§ionid=143518884. Accessed March 14, 2019.

2.

Cortese G, Nicali R, Placido R, Gariazzo G, Anrò P. Radiological aspects of diffuse alveolar haemorrhage. Radiol Med. 2008;113(1):16-28.

Hematologic and Oncologic Disorders 76.RBC Disorders 459​ 77.White Blood Cell Disorders 464​ 78.Platelet Disorders 469​ 79.Coagulopathies 473​ 80.Hypercoagulable States 479​ 81.Transfusion Medicine 482​ 82.Solid Tumors 486​ 83.Oncological Syndromes 490​ 84.Hemopoietic Cell Transplantation 494​ 85.Complications of Immunosuppressive Drugs and Chemotherapy 498​

C H AP T E R 7 6

RBC Disorders Jean Kwo

1. Which of the following statements regarding anemia in hospitalized patients is MOST correct?

A. A low serum iron, normal or low serum ferritin, and normal or high total iron-binding capacity are associated with iron-deficiency anemia. B. Blood draws for diagnostic studies is an infrequent cause of anemia in hospitalized patients. C. Microcytic anemia is commonly associated with acute blood loss. D. The reticulocyte percentage can be artificially decreased in severe anemia.

2. A 57-year-old man has been in the ICU for 10 days with acute respiratory distress syndrome (ARDS) and sepsis due to pneumonia. His hemoglobin level is 7.5 g/dL. Which of the following statements is MOST correct regarding management of his anemia associated with critical illness?

A. Use of recombinant human erythropoietin (EPO, epoetin alfa) will reduce his need for red-cell transfusion. B. Iron supplementation will reduce his need for red-cell transfusion. C. He should be transfused with red blood cells as patients with sepsis have better outcomes with a target hemoglobin >9. D. Strategies to minimize blood loss associated with phlebotomy such as use of pediatric tubes, point-of-care testing, or use of blood conservation devices can decrease blood loss and transfusion requirements.

3. Which of the following statements regarding red blood cell transfusion in critically ill patients is MOST correct?

A. Transfusion of leukoreduced red blood cells (RBCs) is associated with decreased risk of ARDS in trauma patients. B. Transfusion of RBCs that have been stored for a longer period of time is associated with increased infection, organ dysfunction, and mortality. C. A liberal transfusion goal may be associated with an increased risk of nosocomial infections. D. Patients with coronary artery disease should have a transfusion threshold of hemoglobin 10 g/dL.

4. A 63-year-old man is admitted to the ICU after exploratory laparotomy, superior mesenteric artery (SMA) thrombectomy, and small bowel resection. He received 2 L of crystalloid resuscitation in the operation room, and his estimated blood loss was 150 mL. He did not receive transfusion in the operating room. His white blood count is 20,000 cells/microL with 90% neutrophils, hemoglobin 19.1 g/dL, hematocrit 57%, and platelet count 265,000 platelets/microL. Which of the following statements regarding his laboratory data is MOST correct?

A. His complete blood count (CBC) likely reflects a volume depleted state and no further workup is necessary. B. He cannot have polycythemia vera (PV) if the JAK2V617F mutation is not detected. C. If he has PV, he should be treated with aspirin alone. D. His serum EPO level should be low if he has PV.

5. Which of the following statements regarding hemoglobinopathies is MOST correct?

A. Deoxygenation of sickle hemoglobin (HbS) results in polymerization that produces sickling of the red cell which is reversible. B. The alpha-thalassemias are usually caused by the deletion of one or more beta-globin genes. C. Fetal hemoglobin (HbF) increases the polymerization of HbS and promotes sickling of RBCs. D. Pulmonary embolism is the most common cause of death in βthalassemia major (TM).

Chapter 76 Answers 1. Correct Answer: A Rationale: Reasons for anemia in hospitalized patients are myriad and can include exacerbation of an underlying disease, blood loss from procedures, hemodilution from fluid administration, and impaired erythropoiesis. Phlebotomy for diagnostic purposes is also major cause of anemia. One study showed that 74% of patients developed anemia during their hospitalization. Evaluation of the RBC indices can help determine the cause of anemia. The mean corpuscular volume (MCV) is the average volume of the patient’s RBC and can be low, normal, or elevated. Microcytic RBCs are formed because of decreased production of hemoglobin, which can be due to abnormal globin (thalassemias) or heme (sideroblastic anemias) production or lack of iron (iron-deficiency, anemia of inflammation). Iron studies can help elucidate the cause of microcytic anemias. Iron-deficiency anemia is characterized by low serum iron, a high transferrin, and low ferritin levels. Anemia of inflammation is associated with low iron levels due to reduced iron absorption from the gastrointestinal tract as well as decreased release of iron from body stores. The serum transferrin is usually normal to low and serum ferritin is usually normal to high. Serum iron and ferritin levels are usually normal to high in sideroblastic anemias and thalassemias. An elevated MCV (macrocytic) is usually due to red cell membrane defects or DNA synthesis defects. Defects in DNA synthesis is associated with folate or vitamin B12 (cobalamine) deficiency, abnormal RBC maturation (eg myelodysplastic syndrome), or certain chemotherapeutic medications. Liver disease or hypothyroidism can cause red cell membrane defects. However, the RBCs are normal sized (normocytic) in many cases. In these cases, it may be helpful to determine the mechanism underlying the anemia. Mechanisms leading to anemia include decreased RBC production, increased RBC destruction, and blood loss. These mechanisms are not mutually exclusive and can be operating at the same time in a patient. The reticulocyte count can help distinguish between decreased RBC

production and increased RBC destruction. However, because the reticulocyte count is often reported as a percentage of all RBCs, it can be falsely elevated in anemia. Furthermore, younger reticulocytes with a longer lifespan are released into the circulation in the setting of anemia. The reticulocyte production index is a calculated index that corrects for both hematocrit and reticulocyte lifespan. References

1. Cascio MJ, DeLoughery TG. Anemia. Med Clin North Am. 2017;101:263284

2. DeLoughery TG. Microcytic anemia. N Engl J Med. 2014;371:1324-1331. 3. Koch CG, Li L, Sun Z, et al. Hospital-acquired anemia: prevalence, outcomes, and healthcare implications. J Hosp Med. 2013;8:506-512.

4. Thavendiranathan P, Bagai A, Ebidia A, et al. Do blood tests cause anemia in hospitalized patients? The effect of diagnostic phlebotomy on hemoglobin and hematocrit levels. J Gen Intern Med. 2005;20:520524.

2. Correct Answer: D Rationale: Anemia is common in ICU patients. Ninety-seven percent of critically ill patients are anemic by day 8. Anemia results from RBC loss from injury, phlebotomy, procedures, etc., and decreased RBC production. Hepcidin plays a central role in iron homeostasis. Its synthesis is upregulated by inflammatory cytokines, resulting in decreased iron absorption and decreased release of iron from body stores creating an iron-deficiency–like state. Decreased renal function and proinflammatory cytokines decrease EPO production. In a prospective, multicenter, randomized, double-blind, placebocontrolled trial involving 1460 patients, the use of recombinant human EPO (epoetin alfa) to treat anemia in critically ill patients was not associated with decreased red transfusions using a target hemoglobin concentration between 7 and 9 g/dL. In this study, patients who received EPO had a higher rate of thrombotic events if they did not receive

prophylactic or therapeutic doses of heparin. Overall mortality was the same between the group that received EPO and the group that received placebo. The use of iron supplementation is controversial in critically ill patients because it can promote bacterial growth and infection. Hepcidin’s upregulation by inflammatory cytokines may be protective. One multicenter, randomized, placebo-controlled trial of intravenous iron supplementation in critically ill trauma patients showed no difference between groups in hemoglobin concentration, packed red blood cell transfusion requirement, risk of infection, length of stay, or mortality at 14 days. A meta-analysis of five randomized controlled trials involving 665 patients showed iron supplementation did not reduce RBC transfusion. However, the strength of this conclusion is limited by moderate heterogeneity between the studies. In a study comparing a transfusion threshold of 9 versus 7 g/dL in patients with septic shock, there was no difference in 90-day mortality, rates of ischemic events, or use of life support between the two groups. Patients assigned to the lower transfusion group received fewer transfusions. The Surviving Sepsis guidelines recommend not transfusing RBCs in adults with sepsis until the hemoglobin falls below 7.0 g/dL. Phlebotomy can result in a daily loss of 40 to 70 mL of blood in a critically ill patient exceeding the basal RBC formation rate of 15 to 20 mL/d under normal conditions. Strategies to minimize blood loss such as use of small volume phlebotomy tubes, point-of-care testing, reinfusion of discard sample from indwelling lines, and reducing the number of laboratory studies obtained can decrease this source of blood loss. References

1. Corwin HL, Gettinger A, Fabian TC, et al. Efficacy and safety of epoetin alfa in critically ill patients. N Engl J Med. 2007;357:965-976.

2. Hayden SJ, Albert TJ, Watkins TR, Swenson ER. Anemia in critical illness: insights into etiology, consequences, and management. Am J Respir Crit Care Med. 2012;185:1049-1057.

3. Holst LB, Haase N, Wetterslev J, et al. Lower versus higher hemoglobin threshold for transfusion in septic shock. N Engl J Med. 2014;371:13811391.

4. Shah A, Roy NB, McKechnie S, et al. Iron supplementation to treat anemia in adult critical care patients: a systematic review and metaanalysis. Crit Care. 2016;20:306-316.

3. Correct Answer: C Rationale: The primary goal of red blood cell transfusion is to improve oxygen delivery. However, potential adverse effects include transfusion reactions, infection, transfusion-related acute lung injury (TRALI), transfusionrelated circulatory overload (TRCO), and transfusion-related immunomodulation (TRIM). Transfusions are associated with immunosuppression (TRIM) and may result in an increased risk of nosocomial infections in hospitalized patients. One small, retrospective study found a dose-related response between the number of transfusions, and the risk for infection showed a dose-response relation such that the risk of infection increased by a factor of 1.5 for each unit transfused. A meta-analysis of 17 randomized trials comparing restrictive versus liberal RBC transfusion strategies involving 7456 patients found an increased risk of serious infections among patients treated with a liberal transfusion strategy with a number needed to treat (NNT) of 48 with a restrictive strategy in order to prevent serious infections. Leukoreduction not only removes donor leukocytes from packed RBCs but also filters inflammatory mediators (eg, tumor necrosis factor [TNF-α], interleukin-1 [IL-1]) and viruses transmitted via leukocytes (eg, EpsteinBarr virus [EBV], cytomegalovirus [CMV]), and reduces human leukocyte antigen (HLA) alloimmunization. Multiple studies have been performed looking at the effect of transfusion leukoreduced RBCs on infection, organ dysfunction scores, mortality, and risk of ARDS. Many of these studies are limited by size and study design but show no advantage to using leukoreduced RBCs. The maximum storage period for RBC units is 42 days as mandated by the US Food and Drug Administration. However, the storage of blood for longer periods results in changes in the RBC membrane, which can impede microvascular flow and trigger inflammation, decreased 2,3-DPG concentrations, which can make red cells ineffective as oxygen carriers, and

increased concentrations of proinflammatory cytokines. Two recent studies have looked at the effect of age of transfused blood in critically ill patients. The Age of Blood Evaluation (ABLE) trial was a multicenter trial that randomized 2430 critically ill patients (mean APACHE score 21.8 ± 7.6) to receive either fresh red cells (stored a mean of 6.1 ± 4.9 days) or standardissue red cells (stored a mean of 22 ± 8.4 days). There was no difference in 90-day mortality (primary outcome) or duration of respiratory, hemodynamic, or renal support, hospital length of stay, and transfusion reactions (secondary outcomes) between the two groups. The Standard Issue Transfusion versus Fresher Red-Cell Use in Intensive Care (TRANSFUSE) trial randomized 4994 critically ill patients (mean APACHE III score 72.9 ± 29.4, median APACHE III risk of death of 21.5%) to receive either blood that was stored for a mean of 11.8 days or blood that was stored for a mean of 22.4 days. There was no difference between the two groups in the primary outcome of 90-day mortality and secondary outcomes of organ dysfunction, need of mechanical ventilation and renal replacement therapy, blood stream infection, transfusion reactions, and ICU and hospital length of stay. The Transfusion Requirements in Critical Care (TRICC) trial found no difference in 30-day mortality in 838 euvolemic patients with normal baseline hemoglobin and no active ischemia or bleeding randomized to either a restrictive or liberal transfusion strategy (threshold hemoglobin, 7 vs 10 g/dL). However, there was a higher mortality that was not statistically significant in patients with coronary artery disease receiving a restrictive transfusion strategy. Although not done in critically ill patients, two studies suggest that a transfusion threshold of a hemoglobin of 8 is safe in patients with cardiovascular disease. The Transfusion Requirements after Cardiac Surgery (TRACS) showed that a restrictive transfusion strategy (maintain hematocrit ≥24%) was noninferior to a liberal transfusion strategy (maintain hematocrit ≥30%) in terms of a composite end-point consisting of 30-day all-cause mortality and severe morbidity (cardiogenic shock, ARDS, or acute renal injury requiring dialysis or hemofiltration). A second study randomized 2016 patients with either a history of or risk factors for cardiovascular disease undergoing hip fracture surgery, either a liberal transfusion strategy (if hemoglobin 49% in men, >48% in women OR Red cell mass >25% above normal predicted value

Subnormal serum erythropoietin level

References

1. Khan FA, Khan RA, Iqbai M, et al. Polycythemia vera: essential management protocols. Anaesth Pain Intensive Care. 2012;16:
91-97.

2. Tefferi A, Barbul T. Polycythemia vera and essential thrombocythemia: 2017 update on diagnosis, risk-stratification, and management. Am J Hematol. 2017;92:95-108.

3. Tefferi A, Vannucchi A, Barbui T. Polycythemia vera treatment algorithm 2018. Blood Cancer J. 2018;8:3.

5. Correct Answer: A Rationale: Hemoglobin is the major protein responsible for oxygen transport and is usually composed of two ​alpha-globin chains and two beta-globin chains. The synthesis of alpha and beta chains must also be closely matched as free globin units are toxic to red cells. Hemoglobinopathies arise from either (1) a quantitative defect (either reduction or total absence) in the production of one of the globin chains or (2) a structural defect in one of the globin chains. Quantitative disorders of globin chain synthesis ​result in the thalassemia syndromes. Most mutations that result in structural defects in the one of the ​globin chains are clinically silent and are discovered as an incidental finding. Those that are clinically relevant can cause the sickle cell disorders, anemia due to hemolysis, changes in oxygen affinity resulting in ​polycythemia or cyanosis, or methemoglobinemia. The thalassemia syndromes are inherited disorders that result in either decreased or absence of either the alpha- or beta-globin chains. Under normal circumstances, the synthesis of alpha- and beta-globin chains is highly regulated to prevent excess of one or the other chain. If synthesis of one globin chain is decreased or absent, there is accumulation of the unaffected globin chain that precipitates and leads to hemolysis and decreased red cell survival. The clinical manifestations of the thalassemia syndromes range from asymptomatic carrier status to profound abnormalities including severe anemia, extramedullary hematopoiesis, and skeletal and growth deficits. The alpha-thalassemias are usually caused by the deletion of one or more alpha-globin genes. Deletion of one or two alpha-globin genes is not associated with severe hematologic abnormalities; a mild hypochromic, microcytic anemia is seen with deletion of two alpha-globin genes. Deletion of three alpha-globin genes (eg, hemoglobin H [HbH] disease) results in a microcytic, hypochromic anemia with hemoglobin levels between 8 and 10 g/dL. The anemia can be exacerbated by acute infections, oxidative stress, and pregnancy and is treated with transfusions as needed. Deletion of all four alpha-globin chains results in hydrops fetalis and is

usually fatal during late pregnancy or shortly after birth. There are two beta-globin genes and beta-thalassemias are usually caused by point mutations in one or both genes. The mutations can result in decreased production or absence of beta-globin. Severity of disease depends on how much beta-globin is made with the most severe disease in homozygotes that make no beta-globin (TM). These patients have severe anemia (Hb range 1-7 g/dL), hemolysis, and ineffective erythropoiesis, resulting in skeletal abnormalities due to expanded marrow cavities and extramedullary hematopoiesis. Iron overload occurs because of increased intestinal iron uptake secondary to ineffective erythropoiesis and from transfusions. Excess iron stores can cause toxicity in the liver, heart, and endocrine organs, with resulting organ dysfunction. Heart failure is the most common cause of death in TM and primarily results from cardiac iron accumulation. HbS results from an amino acid substitution on the beta-globin chain. Patients with sickle cell disease are homozygous for HbS. Deoxygenation of HbS results in polymerization that distorts the shape of the red cell, which is reversible with reoxygenation of HbS. Sickled RBCs increase blood viscosity and obstruct capillary flow causing vaso-occlusion and pain. Furthermore, repeated cycles of sickling damage the RBC membrane, resulting in premature destruction of RBCs and a chronic hemolytic anemia. The polymerization of deoxygenated HbS is inhibited by HbF. Treatment of sickle cell disease includes pain medications to treat pain associated with vaso-occlusive crises, transfusions, hydroxyurea to increase HbF concentrations, and hematopoietic stem cell transplantation. References

1. Bunn HF. Pathogenesis and treatment of sickle cell disease. N Engl J Med. 1997;337:762-769.

2. Forget BG, Bunn HF. Classification of the disorders of hemoglobin. Cold Spring Harb Perspect Med. 2013;3:a011684.

3. Pennell DJ, Udelson JE, Arai AE, et al. Cardiovascular function and treatment in β-thalassemia major: a consensus statement from the American Heart Association. Circulation. 2013;128:281-308.

4. Rachmilewitz EA, Giardina PJ. How I treat thalassemia. Blood 2011;118:3479-3488.

5. Yawn BP, Buchanan GR, Afenyi-Annan AN, et al. Management of sickle cell disease: summary of the 2014 evidence-based report by expert panel members. JAMA. 2014;312:1033-1048.

C H AP T E R 7 7

White Blood Cell Disorders Jean Kwo

1. A 57-year-old male with history of alcohol abuse is admitted to the intensive care unit after a fall from a ladder. He has sustained a left ankle fracture and several rib fractures. His white blood cell count is 1600/µL. Initial workup for possible causes of leukopenia includes all of the following EXCEPT:

A. Obtain complete blood count (CBC) with differential B. Review patient’s medications C. Obtain screening studies for rheumatologic disorders D. Review results of CBCs/differential counts from prior hospitalizations and the ambulatory setting

2. A 52-year-old female presents with fever of 38.3°C (101°F) and malaise. Her white blood cell count is 710/µL with 70% neutrophils. She was recently diagnosed with an urinary tract infection and is currently taking trimethoprim-sulfamethoxazole. Which of the following statements is TRUE?

A. Her neutropenia is unlikely to be due to idiosyncratic drug–induced acute neutropenia because her absolute neutrophil count (ANC) is 10% clonal plasma cells. Other criteria for the diagnosis of multiple myeloma include all the following EXCEPT:

A. MRI showing two or more focal bone or bone marrow lesions at ≥5 mm in size B. Hypercalcemia C. Recurrent infections D. Creatinine clearance 11 mg/dL Renal insufficiency: Creatinine clearance 2 mg/dL Anemia: Hemoglobin 2 g/dL below normal Bone lesions: One or more osteolytic lesions ≥5 mm in size. MRI, CT, or PET/CT is preferred over skeletal survey for detection of bone lesions.

Biomarke r associate d with progre ssion to malignancy and e nd-organ damage ≥60% clonal plasma cells in bone marrow Involved:uninvolved free light chain ratio >100 MRI with more than one bone or bone marrow focal lesion

In 2014, the International Myeloma Working Group updated the diagnostic criteria for multiple myeloma with three biomarkers to identify those patients who are at imminent risk of progression of their malignancy to intervene prior to the development of end-organ damage. The biomarkers are: 1. 2. 3.

Bone marrow plasma cells of 60% or greater. Involved:uninvolved light chain ratio of >100 and involved free light chain ≥100 mg/dL. Excess production of one free light chain type (κ or λ) due to clonal plasma cell disorder results in increased light chain ratio.

MRI with two or more focal bone or bone marrow lesions that are 5 mm in size or greater.

References

1. Rajkumar SV. Updated diagnostic criteria and staging system for multiple myeloma. Am Soc Clin Oncol Educ Book. 2016;35:e418-e423.

2. Rajkumar SV, Dimopoulos MA, Palumbo A, et al. International myeloma working group updated criteria for the diagnosis of multiple myeloma. Lancet Oncol. 2014;15:e538-e548.

8. Correct Answer: B Rationale: Hypercalcemia is one of the myeloma-defining events and is caused by the osteolytic tumor lesions. Symptoms of hypercalcemia include nausea, vomiting, weakness, abdominal pain, constipation, confusion. Severe hypercalcemia (defined at serum calcium >14 mg/dL) can result in cardiac arrhythmias and coma. Treatment of hypercalcemia includes hydration with isotonic saline with possible use of loop diuretics to both augment excretion of calcium as well as inhibit calcium reabsorption in the thick ascending limb. Calcitonin decreases bone resorption and works within 4 to 6 hours to lower calcium levels. However, because of tachyphylaxis, its efficacy is limited to about 48 hours. Bisphosphonates also decrease bone resorption by inhibiting osteoclasts but its onset of action is 24 to 72 hours. Zoledronic acid or pamidronate are can be given intravenously and are recommended for hypercalcemia due to malignancy. Oral third-generation bisphosphonates such as risedronate and alendronate are not recommended for the treatment of severe or acute hypercalcemia. Denosumab is a monoclonal antibody that inhibits RANKL and decreases osteoclast activity. It is recommended for the treatment of hypercalcemia in patients who do not respond to bisphosphonates. Unlike bisphosphonates, it is not cleared by the kidney and can be used in patients with kidney disease. Its onset of action is 4 to 10 days. Hemodialysis may be needed in patients with severe hypercalcemia and neurologic symptoms (eg, coma) or in patients with renal failure or heart failure who cannot tolerate aggressive volume expansion. References

1. Cicci JD, Buie L, Bates J, et al. Denosumab for the management of hypercalcemia of malignancy in patients with multiple myeloma and renal dysfunction. Clin Lymphoma Myeloma Leuk. 2014;14:e207-e211.

2. Goldner W. Cancer-related hypercalcemia. J Oncol Pract. 2015;12:426-432. 3. Shane E, Berenson JR. Treatment of hypercalcemia. In: Rosen CJ, ed. UpToDate. Waltham, MA: UpToDate, Inc. 
http://www.uptodate.com. Accessed on December 28, 2018.

C H AP T E R 7 8

Platelet Disorders Sean M. Baskin and Kunal Karamchandani

1. A 72-year-old male is admitted to the intensive care unit after undergoing aortic valve replacement for severe aortic stenosis. He was successfully weaned from bypass, required no blood products, and received appropriate protamine reversal. In the intensive care unit, he has been having persistent drainage from the thoracotomy tubes. A complete blood count (CBC) shows a hematocrit of 40% and platelet count of 150,000/mm3. Coagulation studies including a thromboelastogram (TEG) are sent.

TEG results: Prothombin Time: 11 seconds (11-13.5 seconds) Partial Thromboplastin Time: 38 seconds (30-40 seconds) R-time: 8 minutes (5-10 minutes) K-time: 4 minutes (3-6 minutes) Alpha angle: 52° (45-55°) Maximum Amplitude (MA): 37 mm (50-70 mm) Based on the results of the TEG, which of the following would be the best treatment option A. Platelet B. Protamine C. Fibrinogen D. Fresh Frozen Plasma

2. A 63-year-old female with known lower-extremity deep vein thrombosis (DVT) is admitted to the intensive care unit after presenting with shortness of breath and chest pain. Imaging is negative for a pulmonary embolism, and blood tests including a

CBC and serum electrolytes are normal. She is started on supplemental oxygen and a heparin infusion titrated to aPTT of 60 to 90 seconds. On admission day 7, her platelet count is found to have decreased from 150,000/mm3 to 62,000mm/3 over a 24-hour period. To assist in making the diagnosis, you calculate the patient’s 4-T score. Which of the following is NOT a part of the 4-T score?

A. Timing of Platelet decrease B. Presence of Thrombosis C. Severity of thrombocytopenia D. Tachycardia

3. After calculating a 4-T score of 6, you proceed to obtain confirmatory testing to support your diagnosis. A Heparin PF-4 antibody test is positive, and heparin-induced serotonin release assay (SRA) is under process. Given the presumptive diagnosis, which of the following treatment strategies would be most appropriate?

A. Discontinue Heparin and start low-molecular weight Heparin B. Discontinue Heparin and start Argatroban C. Discontinue Heparin and transfuse platelets D. Start warfarin

4. A 32-year-old male presents to the hospital complaining of progressive fatigue, productive cough, and intermittent epistaxis. His initial evaluation is significant for left lower lobe consolidation on chest X-ray and a temperature of 38.7°C. During attempts to obtain peripheral venous access, he bruises easily and missed attempts bleed for over 2 minutes. A CBC and coagulation studies are obtained and are as follows:

WBC: 1.2 K/µL Hgb: 6.5 mg/dL Hct: 19.5% Platelet count: 74000/µL PT: 22 seconds (normal 10-14) aPTT: 56 seconds (normal 25-40) Fibrinogen: 65 mg/dL (normal 140-400 mg/dL) He is subsequently admitted to the ICU where broad spectrum antibiotics are initiated, and a bone marrow aspirate (BMA) is obtained, which shows finding consistent with acute myeloid leukemia. Which of the following is the best initial treatment option for this patient? A. Platelet transfusion B. All-trans-retinoic acid (ATRA) C. Packed RBC transfusion D. Administer IV Vitamin K and Fresh Frozen Plasma

5. A 29-year-old male is admitted to the intensive care unit after undergoing an uncomplicated laparoscopic splenectomy for idiopathic thrombocytopenic purpura. On post-op day 2, he is recovering well, is afebrile, is hemodynamically stable, and has no complaints. Routine laboratory test results are sent, which are significant for a platelet count of 654,000/µL. What is the most likely cause of thrombocytosis in this patient?

A. Sepsis B. Reactive thrombocytosis C. Cancer D. Lab error

Chapter 78 Answers 1. Correct Answer: A Rationale: Thomboelastogram, or TEG, is a whole blood, point-of-care test that analyses viscoelastic proprieties of evolving clot in the patient’s whole blood and can provide information about fibrin formation, platelet activation, and clot retraction, therefore assisting in identifying the cause of coagulopathy. Unlike the more traditional laboratory tests such as PT and PTT, which give a general state of the extrinsic and intrinsic pathways, TEG provides distinct values as well as a graphical representation of various stages in clot formation (Figure 1). R-time is the duration of time from the application of the blood sample until the clot reaches a graphical amplitude of 2 mm, in other words, the time elapsed between clot formation and a predetermined and consistent size. The initiation of clot formation is dependent on circulating, function clotting factors from both the intrinsic and extrinsic pathways. A normal Rtime is 5 to 10 minutes, whereas a value less than 5 minutes implies hypercoagulability, and conversely, a value greater than 10 minutes indicates either a quantitative or qualitative deficiency in clotting factors. Treatment for prolonged R-time typically is Fresh Frozen Plasma, or factor concentrate in the setting of known factor deficiency (ie Hemophilia). K-time represents the time elapsed between the conclusion of R-time and a graphical amplitude of 20 mm. Because the K-time is the duration between an initial amplitude (2 mm) and final amplitude (20 mm), this provides a quantitate assessment of the speed of clot formation. Thrombin catalyzes the conversion of fibrinogen to fibrin, which adheres the platelet plug, strengthening it, and finalizing the clot formation. A prolonged Ktime indicates inadequate circulating fibrin and is treated with either fibrinogen concentrate, or cryoprecipitate. Alpha angle is a measurement of the speed of clot strengthening via fibrin cross-linking. It is formed by a tangential line originating from the start of K-time and intersecting the upward slope of the graph. A decreased alpha angle is similar to a prolongation of K-time and is treated in a similar manner. MA is the greatest width between the two arms of the curve and

represents overall clot strength. As the available platelets continue to form clot, which is subsequently stabilized by fibrin cross-links, the strength of the clot increases and the arms of the curve move farther apart. Eventually, the clot begins to degrade and the arms start sloping downwards. This transition point marks the MA. MA is a surrogate for platelet function, where a decreased MA suggests either inadequate supply (ie postcardiac bypass thrombocytopenia) or suboptimal function (ie liver disease). A decreased MA is treated with platelets. This patient has a decreased MA suggesting poor clot strength likely from platelet deficiency. So, the best treatment option based on the TEG would be to administer platelets. Sample TEG demonstrating normal coagulation morphology with normal values:

References Scarpelini S, Rhind SG, Nascimento B, et al. Normal Range values for thromboelastography in healthy adult volunteers. Braz J Med Biol Res. 2009;42(12):1210-1217. Verma A, Hemlata. Thromboelastography as a novel viscoelastic method for hemostatis monitoring: its methodology, applications and constraints. Glob J Transfus Med. 2017;2:8-18.

2. Correct Answer: D 3. Correct Answer: B Rationale: Under normal conditions, Heparin binds to Antithrombin III (AT-3), which inactivates thrombin and Factor Xa causing a relative state of

hypocoagulability. Intermittent bolus dosing is beneficial for patients at risk for thromboembolic events such as patients undergoing prolonged surgery. For patients with known DVT/PE, or other vascular thrombi, a continuous infusion may be utilized to prevent additional clot formation as well as enhance existing clot degradation. However, exposure to heparin may cause an autoimmune reaction where IgG-mediated antibodies bind to Heparin-PF4 complexes resulting in platelet activation and the formation of microthrombi and eventual thrombosis. The microthrombi consume existing platelets and results in thrombocytopenia. This entity is referred to as Heparin-induced thrombocytopenia/thrombosis (HIT). One of the ways to predict the likelihood of HIT is to calculate the 4-T score. The 4-T score consists of four criteria graded on a 0 to 2 scale as shown in the table below. Scores of 0 to 3, 4 to 5, and 6 to 8 correspond to low, moderate, and high risk for HIT respectively. The negative predictive value of a low 4-Ts score has been found to be as high as 99%; meanwhile the positive predictive value of a high score is only 64%. For patients with thrombocytopenia and a moderate or high 4-Ts score, further testing is necessary to establish the ​diagnosis of HIT. Estimating the likelihood of HIT: the “4-Ts” score Category

2 Points

1 Point

0 Point

Thrombocytopenia

>50% fall, or nadir of 20100 × 10 9/L

30%-50% fall, or nadir of 1019 × 10 9/L

Timing of platelet count fall

Days 5-10 or ≤1 d if prior heparin exposure within the last 30 d

Thrombosis or other sequelae

Proven thrombosis, skin necrosis, or, after heparin bolus, acute systemic reaction None evident

Onset after day 10 or unclear (but fits with HIT) or ≤1 d if heparin exposure within past 30-100 d Progressive, recurrent, or silent thrombosis; erythematous skin lesions Possible

30% fall, or nadir of 50% fall or nadir ≥20 K

30%-50% fall or nadir 
1019 K

Timing of platelet decrease

Days 5-10 or ≤1 d if heparin exposure in past 30 d

>10 d or timing unclear, or 25 mm Hg warrants surgical compression, but the overall clinical picture should correspond with ACS to maximize benefits of an open abdomen and reduce morbidity. It is important to note that even with an open abdomen, ACS can occur, and it is important to remain vigilant when the constellation of symptoms exist. Reference

1. Ferreira JD. Chapter: abdominal compartment syndrome. In: Ferri FF, ed. Ferri’s Clinical Advisor. Elsevier; 2019.

11. Correct Answer: B Rationale: To obtain accurate IAP measurements, the needed steps must be performed correctly. The patient must be pharmacologically paralyzed, therefore preventing abdominal contractions. The patient must be completely supine, with the IAP transducer leveled at the mid-axillary line. This usually correlates at the level of the iliac crests. It is important to obtain multiple readings at the time of measurement and wait at least 30 to 60 seconds after instillation of room temperature of saline as cold fluids cause detrusor contractions. A maximum of 25 mL of sterile saline should be used as larger amounts will falsely elevate IAP. References

1. McBeth PB, Zygun DA, Widder S, et al. Effect of patient positioning on

intra-abdominal pressure monitoring. Am J Surg. 2007;193:644-647.

2. De Waele J, Pletinckx P, Blot S, Hoste E. Saline volume in transvesical intra-abdominal pressure measurement: enough is enough. Intensive Care Med. 2006;32:455-459.

3. Chiumello D, Tallarini F, Chierichetti M, et al. The effect of different volumes and temperatures of saline on the bladder pressure measurement in critically ill patients. Crit Care. 2007;11:R82.

12. Correct Answer: A Rationale: ACS encompasses an increased IAP in conjunction with the presence of end-organ dysfunction notably decreased renal function, decreased pulmonary function, and reduced preload. Typically, urine sodium in ACS is reduced in addition to decreased mixed venous saturation, which correspond to the reduced cardiac output. Cardiac ultrasound should correspond with hypovolemia in the setting of ACS, but a significant drop of hemoglobin complicates the picture as the patient has ongoing bleeding. Therefore, the best answer is A. References

1. Malbrain ML, De Iaet IE. Intra-abdominal hypertension: evolving concepts. Clin Chest Med. 2009;30(1):45-70.

2. Ke L, Tong Z, Ni H, et al. The effect of intra-abdominal hypertension incorporating severe acute pancreatitis in a porcine model. PLoS One. 2012;7(3):e33125. doi:10.1371/journal.pone.0033125.

C H AP T E R 9 8

Skin, Soft Tissue, and Extremities Casey McBride Luckhurst and April E. Mendoza

1. A 25-year-old male with no significant past medical history presents to the emergency department complaining of pain and “redness” of his left lower extremity. He denies any history of trauma or injury to the area. His vital signs are notable for Temp: 102.2F, HR 110, BP 95/65, RR 25, Sat 100% on room air. On examinaton, he is clearly agitated and in pain. His left lower extremity is erythematous and tender to palpation without any noticeable skin breakdown, crepitus, or evidence of external trauma. All of the following statements about the evaluation and diagnosis of a necrotizing soft tissue infection (NSTI) are true EXCEPT:

A. Clinical manifestations of a NSTI can include erythema, edema, pain out of proportion to examination, overlying skin changes, and systemic signs of infection including fever and hypotension B. Symptoms are typically acute in onset and rapidly progressive over a short period of time C. The LRINEC score, developed specifically to aid in differentiation of NSTI from other soft tissue infections, has high sensitivity and specificity and thus a negative score rules out NSTI D. If clinical suspicion for a NSTI is high, surgical intervention should not be delayed for further diagnostic evaluation E. Computed Tomography (CT) is the best radiographic imaging modality in the evaluation of a NSTI

2. A 55-year-old male with a history of methicillin-resistant Staphylococcus aureus (MRSA) colonization presents to the emergency department with complaints of purulent drainage from his surgical incision. He is now 7 days postop from his sigmoid

colectomy. On examination, his temperature is 102°F and hemodynamically within normal limits. His incision has skin staples in place and moderate surrounding erythema extending >5 cm from the wound edge and associated induration. Purulent drainage is easily expressed from the most inferior aspect of the incision. Which of the following is the most appropriate management of this patient?

A. Open the incision, obtain a fluid culture, and start on an empiric course of IV Vancomycin and Piperacillin-Tazobactam B. Open the incision, obtain a fluid culture and start on an empiric course of IV Vancomycin alone C. Discharge home on a 7-day course of oral Cephalexin D. Open the incision, obtain a fluid culture, and start on an empiric course of IV Piperacillin-Tazobactam alone E. Open the incision, obtain a fluid culture, and hold off on starting antimicrobial therapy until culture data returns

3. A 30-year-old male presents with significant erythema overlying his right arm and a fever to 101.0°F. He explains that he sustained an abrasion over the affected area while at work, and when the redness started 2 days ago he was prescribed a course of antibiotics by the Urgent Care Clinic, which he has been taking without improvement of his symptoms. An ultrasound shows edema and a phlegmon without an obvious abscess collection. He is admitted to the hospital and started on IV cefoxitin. Over the next 48 hours it is noted that even though there is some regression of the erythema, a firm, fluctuant area can be palpated in the subcutaneous tissue measuring >2 cm. What is the most appropriate next step in management?

A. Broaden antibiotics to IV Piperacillin-Tazobactam B. Broaden antibiotics to IV Vancomycin C. Repeat ultrasound

D. CT scan of the arm E. No change in current management

4. A patient is admitted to the surgical critical care unit after being found down for an unknown period of time following presumed assault. The patient was intubated at the scene. In addition to a multitude of other injures, the patient is noted to have an acute kidney injury (AKI) with an elevated creatine kinase (CK) to 10,000. On examination, all extremity compartments are soft. With regards to the management of AKI from traumatic rhabdomyolysis, which of the following statements is true?

A. In the absence of other indications for renal replacement therapy, prophylactic hemodialysis is recommended in the setting of elevated CK above 5,000 U/L B. Commonly seen laboratory abnormalities include elevated CK, hypocalcemia, hypokalemia, and hypophosphatemia C. Resuscitation with sodium bicarbonate solution, titrated to urine pH, can prevent acute renal failure D. If the patient is oligo-anuric, the addition of mannitol is recommended E. Early, aggressive resuscitation with normal saline solution fundamental to treatment

5. A 60-year-old male with a past medical history notable for obesity, hypertension, hyperlipidemia, and type 2 diabetes presents to the emergency department with complaints of significant scrotal and perineal pain with associated drainage. He explains that the symptoms started about 24 hours before presentation and have been rapidly progressive since. His vital signs are as follows: Temp 102.5°F, HR 102 bpm, BP 90/65 mm Hg, RR 18, Sat 100% on room air. On examination, he is diaphoretic and noticeably uncomfortable. His scrotum and perineum are diffusely erythematous and tender to palpation, with a pinpoint area draining dishwasher color fluid that

is malodorous. All of the following are appropriate next steps in the management of a patient presenting with Fournier ’s gangrene EXCEPT:

A. Aggressive fluid resuscitation to markers of end organ perfusion as dictated by early goal-directed therapy B. After acquiring appropriate culture data, initiation of broad spectrum antimicrobial therapy with empiric coverage for NSTI causing organisms C. Obtaining radiographic imaging to delineate the extent of infection and to aid in operative planning D. Early surgical consultation for urgent evaluation and operative debridement E. Postoperative admission to a surgical intensive care unit for ongoing monitoring and hemodynamic support

6. In addition to aggressive fluid resuscitation and urgent surgical intervention, the initiation of appropriate antimicrobial therapy is crucial in the setting of a NSTI. Which of the following represents an appropriate antibiotic regimen for the given clinical situation?

A. A 25-year-old male presenting with clinical findings concerning for NSTI of his left arm: Vancomycin, Piperacillin-Tazobactam B. A 25-year-old male with penicillin allergy, presenting with clinical findings concerning for NSTI of his left arm: Vancomycin, Meropenem, Clindamycin C. A 25-year-old male presenting with documented group A streptococcus NSTI: Piperacillin-Tazobactam D. A 30-year-old fisherman presenting with findings concerning for an NSTI of his left leg: Vancomycin, Ceftriaxone, Clindamycin E. A 30-year-old fisherman presenting with findings concerning for an NSTI of his left leg: Vancomycin, Doxycycline

7. A 34-year-old female with a history of IV substance abuse presents to the hospital via EMS after being found down for some unknown amount of time. While in the ED, she was treated with naloxone for presumed IV heroin overdose. On further examination, she was noted to have stigmata of recent IV drug use, and her right forearm is significantly swollen with tense compartments. When asked to move her fingers she is able to do so, but reports decreased sensation over the dorsal aspect of her hand as well as significant pain in her forearm with passive range of motion of her wrist. All of the following statements regarding the diagnosis and management of acute compartment syndrome (ACS) are true EXCEPT:

A. Mechanisms of development of ACS include fracture, thermal burns, prolonged compression, crush injuries, and revascularization following procedures B. Muscle breakdown leads to elevations in serum CK C. Although not necessary for diagnosis of ACS, compartmental pressures can be used to aid in the diagnosis, specifically when used to calculated extremity perfusion pressures D. Management involves early surgical consultation for compartmental decompression with fasciotomies E. Clinical diagnosis is classically made with the “five Ps”: Pain, Pallor, Paresthesias, Poikilothermia, Pulselessness

8. A 20-year-old male presents with a closed fracture of his left tibia and fibula sustained while playing basketball. He undergoes splinting by Orthopedic Surgery and 3 hours later he begins to complain of pain in his distal lower extremity. His pain is worsened with passive range of motion, and he notes decreased sensation over the dorsal aspect of his foot. His pulse examination is symmetric over bilateral lower extremities but his capillary refill time is delayed. What is the most appropriate next step in the management of this patient?

A. Increase dose of IV morphine to provide pain relief B. Remove splint and if symptoms are not improved, proceed to the operating room for emergent fasciotomies C. Invasive measurement of compartment pressures D. Computed tomography angiography (CTA) of the affected extremity E. Elevate lower extremity and serial examinations

Chapter 98 Answers 1. Correct Answer: C Rationale: NSTIs are severe infections that can be found in any layer of soft tissue and are associated with a high mortality rate. These infections can be classified based on imaging findings, specifically the presence of gas in the tissues, as well as microbiology (polymicrobial vs monomicrobial). Clinically, NSTIs classically present with erythema, edema, pain out of proportion to examination, overlying skin changes, and systemic signs of infection including fever and hypotension (Answer A). Symptoms are typically acute in onset and rapidly progressive over a short period of time, making the time from presentation to diagnosis crucial in the overall outcome of the patient (Answer B). When clinical suspicion is high, surgical consultation should be obtained immediately with a low threshold to proceed to the operating room for surgical evaluation and extensive debridement as indicated (Answer D). More commonly, the differentiation between NSTI and severe cellulitis is not clear. The Laboratory Risk Indicator for Necrotizing Fasciitis (LRINEC) score was developed to aid in differentiation between a severe cellulitis and NSTI with initial studies showing high negative predictive value. Of note, subsequent evaluation has called into question the sensitivity of the scoring system, which includes white cell count, hemoglobin, sodium, glucose, creatinine, and C-reactive protein. Although helpful in the evaluation, a negative score does not replace clinical evaluation and alone cannot rule out the presence of an NSTI (Answer C). Other diagnostic tools CT imaging with intravenous contrast, which has been deemed the best radiographic imaging modality in the evaluation of a NSTI. Findings include gas in the soft tissues, fluid collections, heterogeneous tissue enhancement, and inflammatory changes beneath the fascia (Answer E). References

1. Stevens DL, Bisno AL, Chambers HF, et al. Practice guidelines for the diagnosis and management of skin and soft tissue infections: 2014 update by the Infectious Diseases Society of America. Clin Infect Dis.

2014;59(2):147.

2. Wong CH, Khin LW, Heng KS, Tan KC, Low CO. The LRINEC (Laboratory Risk Indicator for Necrotizing Fasciitis) score: a tool for distinguishing necrotizing fasciitis from other soft tissue infections. Crit Care Med. 2004;32(7):1535.

3. Zacharias N, Velmahos GC, Salama A, et al. Diagnosis of necrotizing soft tissue infections by computed tomography. Arch Surg. 2010;145:452.

2. Correct Answer: A Rationale: Surgical site infection following an abdominal operation can be further classified as superficial or deep infections. Common features include periincisional tenderness, erythema, and induration, with purulent drainage present at the site of the incision. More significant infections can present with systemic signs including fever, tachycardia, and hypotension in addition to the localized findings. Once diagnosed, the treatment of a surgical site infection involves opening the incision to allow for adequate irrigation and drainage, obtaining a fluid culture of the purulent fluid and initiation of antibiotic therapy if there is evidence of extension of the infection into the surrounding tissues or systemic signs of infection (Answer E). Choice of antimicrobial therapy depends on the type of operation (in this case an abdominal operation involving the colon, which would be considered a clean contaminated case) and risk factors for MRSA (prior colonization or infection, recent hospitalization, recent antibiotics). In this case, it would be appropriate to cover for gram negative organisms as well as anaerobes. Given his recent hospitalization and known MRSA colonization status, the most appropriate choice listed above would be Vancomycin and Piperacillin-Tazobactam. Of the other choices listed above, Vancomycin alone and Cephalexin alone do not provide appropriate gram negative and anaerobic coverage (Answer B, Answer C). Although Piperacillin-Tazobactam does provide appropriate coverage for gram negative and anaerobic organisms, the addition of MRSA coverage is 
 recommended given that he is MRSA colonized and his recent hospitalization (Answer D). Answer A is best answer given that it recommends opening the incision, culturing the purulent fluid obtained,

and initiating appropriate antimicrobial therapy. References

1. Stevens DL, Bisno AL, Chambers HF, et al. Practice guidelines for the diagnosis and management of skin and soft tissue infections: 2014 update by the infectious diseases society of America. Clin Infect Dis. 2014;59(2):147.

2. Mizell JS. Complications of Abdominal Surgical Incisions. UpToDate; May 15, 2018.

3. Correct Answer: C Rationale: An initial presentation of presumed uncomplicated cellulitis can be treated with a trial of oral antibiotic therapy, with or without coverage for MRSA as indicated. Failure of improvement in symptoms and systemic signs of infection (including fever greater than 100.5°F) are indications for parenteral antibiotic therapy. In the patient above, an ultrasound was obtained that showed no underlying abscess collection initially, but his new examination findings are concerning for interval development of an abscess. The most appropriate next step listed above would be to repeat an ultrasound to assess for the formation of an abscess or drainable fluid collection (Answer C, Answer E). There is no indication to further broaden the antibiotic regimen given that the cellulitis is improving and the treatment for an abscess is drainage (Answer A, Answer B). Although a CT scan will provide a more detailed image, it is not necessary in this scenario and subjects the patient to unnecessary radiation (Answer D). References

1. Stevens DL, Bisno AL, Chambers HF, et al. Practice guidelines for the diagnosis and management of skin and soft tissue infections: 2014 update by the Infectious Diseases Society of America. Clin Infect Dis. 2014;59(2):147.

2. Raff AB, Kroshinsky D. Cellulitis: a review. JAMA. 2016;316(3):325-337.

4. Correct Answer: E Rationale: AKI following rhabdomyolysis is one of the more serious complications and is seen in an estimated 20% to 33% of patients. Accumulation of myoglobin from muscle injury combined with hypovolemia can lead to a mixed acute tubular necrosis picture and resultant AKI. Causes of rhabdomyolysis are not limited to, but include traumatic injury or compression, exertional (metabolic myopathies, hyperthermia) and infections, toxins, or pharmacologic agents. Commonly seen laboratory abnormalities include elevated CK, hypocalcemia, hyperphosphatemia, and hyperkalemia, which can be potentially life threatening (Answer B). The only effective treatment of rhabdomyolysis-induced AKI is aggressive fluid resuscitation with treatment of the associated metabolic and electrolyte abnormalities as they arise (Answer E). Adjuncts such as the addition of mannitol and sodium bicarbonate solution have not been shown to be effective and are currently not recommended as mainstays of treatment (Answer C, D). Additionally, the use of mannitol in the oligoanuric patient is contraindicated, and in that setting, consideration for the need for renal replacement therapy should be undertaken. The indications for dialysis are no different than the usual indications, including severe acidosis, uremia, volume overload, and refractory hyperkalemia. There has been no convincing evidence to show the benefit of prophylactic initiation of hemodialysis in the setting of rhabdomyolysis (Answer A). References

1. Torres PA, Helmstetter JA, Kaye AM, Kaye AD. Rhabdomyolysis: pathogenesis, diagnosis and treatment. Ochsner J. 2015;15(1):
58-69.

2. Brown CV, Rhee P, Chan L, et al. Preventing renal failure in patients with rhabdomyolysis: do bicarbonate and mannitol make a difference? J Trauma. 2004;56(6):1191-1196.

5. Correct Answer: C Rationale:

NSTIs are rapidly progressive, life-threatening bacterial infections that can present following trauma, a surgical procedure, or even minor breaches of the skin. NSTIs can be further classified as Type 1 (polymicrobial) or Type II (monomicrobial) with Type 1 infections being more common. Diagnosis is made with a high index of suspicion and can be aided with use of the LRINEC Score. Initial management involves initiation of fluid resuscitation, starting broad spectrum antibiotic coverage with empiric coverage for NSTI-causing organisms after obtaining culture data, and early surgical consultation for emergent operative debridement (Answer A, Answer B, Answer D). Postoperatively, these patients require close hemodynamic monitoring and support, typically requiring an ICU setting (Answer E). Given the rapidly progressive nature of NSTIs, operative debridement should not be delayed. Once a diagnosis is made, there is no indication to obtain further imaging and delay operative intervention (Answer C). Reference

1. Stevens DL, Bryant AE. Necrotizing soft-tissue infections. N Engl J Med. 2017;377(23):2253.

6. Correct Answer: B Rationale: When a patient presents with signs and symptoms of an aggressive infection and associated systemic toxicity, concern for an aggressive NSTI must be high. In addition to intravenous fluid resuscitation and urgent surgical consultation, initiation of broad spectrum antibiotic therapy is prudent. Initial coverage with broad spectrum antibiotic therapy is recommended given that infections may be polymicrobial (Type 1 NSTI) or monomicrobial (Type 2 NSTI) in nature. Initial coverage includes vancomycin or linezolid plus piperacillin-tazobactam or a carbapenem. Addition of Clindamycin provides antitoxin coverage should the pathogen include a toxin-producing strain of streptococcus or staphylococcus. Of the choices above, Answer B provides appropriate broad coverage, which can later be de-escalated based on culture data. Although the vignette in Answer E raises concern for Vibrio vulnificus, which would be appropriately

covered with Doxycycline, it would be remiss to not empirically treat for a polymicrobial infection until further culture data are obtained. Reference

1. Stevens DL, Bisno AL, Chambers HF, et al. Practice guidelines for the diagnosis and management of skin and soft tissue infections: 2014 update by the Infectious Diseases Society of America. Clin Infect Dis. 2014;59(2):147.

7. Correct Answer: E Rationale: Acute compartment syndrome (ACS) of an extremity occurs when elevated pressures within a fascial compartment result in compromised circulation and muscle death. The causes of ACS of the extremity are varied, but include trauma, specifically following long bone fractures, ischemia-reperfusion injuries, thermal burns, and crush injuries, among others (Answer A). The resultant muscle necrosis and breakdown can be measured by increases in serum CK (Answer B). The diagnosis is clinical, and an early high index of suspicion is critical. Early signs and symptoms include pain out of proportion that is worsened with passive range of motion and swollen, tense compartments. Decreased sensation and muscle weakness are later findings, suggestive of nerve and muscle ischemia. The “classic findings” associated with ACS, the “five Ps” are overall inaccurate, and waiting for these symptoms before intervention will result in irreversible damage and morbidity (Answer E). Instead, diagnosis is made by performing serial physical exam and may be aided by the measurement of compartmental pressures, specifically when used to calculate extremity perfusion pressures rather than used as an absolute number (Answer C). Management involves early surgical consultation for compartmental decompression and fasciotomies (Answer D). References

1. Via AG, Oliva F, Spoliti M, Maffulli N. Acute compartment syndrome. Muscles Ligaments Tendons J. 2015;5(1):18-22.

2. Velmahos GC, Toutouzas KG. Vascular trauma and compartment syndromes. Surg Clin North Am. 2002;82(1):125-141.

8. Correct Answer: B Rationale: The patient above is presenting with acute compartment syndrome (ACS) of his lower extremity following a closed, traumatic fracture. Although rare, it is important to remember that these patients are at risk of developing ACS and delays in diagnosis can result in irreversible damage, morbidity, and potential limb loss. The patient above has developed increased pain hours after splinting of his lower extremity, which is not relieved by pain medication and is worsened with passive range of motion. These signs, in conjunction with his decreased sensation and prolonged capillary refill time, are all clinical signs concerning for the development of ACS. It would be inappropriate to treat his symptoms with increased doses of narcotic pain medication, which may mask his symptoms and does not address the underlying problem (Answer A). Similarly, elevating the extremity will not address the underlying problem, and although serial examinations are important for monitoring, the patient above requires more aggressive intervention to avoid irreversible damage (Answer E). Invasive measurement of compartment pressures may aid in the diagnosis when it is not clinically apparent and should not delay surgical intervention in cases where clinical suspicion is sufficiently high (Answer C). There is no role in this clinical situation for obtaining a CTA and doing so would only delay the necessary intervention (Answer D). References

1. Shadgan B, Pereira G, Menon M, et al. Risk factors for acute compartment syndrome of the leg associated with tibial diaphyseal fractures in adults. J Orthop Traumatol. 2015;16(3):185-192.

2. Frink M, Hildebrand F, Krettek C, Brand J, Hankemeier S. Compartment syndrome of the lower leg and foot. Clin Orthop Relat Res. 2010;468(4):940-950.

C H AP T E R 9 9

Polytrauma Casey McBride Luckhurst and April E. Mendoza

1. A 40-year-old male is admitted to the Surgical Intensive Care Unit (SICU) following a high-speed motor vehicle collision with an extensive trauma burden. He has no significant past medical history, and on arrival to the SICU he is noted to be hemodynamically stable. However, his respirations are shallow and he is currently requiring 4 L nasal cannula to maintain an oxygen saturation above 92%. On review of his imaging, you note multiple right-sided rib fractures including ribs 2 to 8 with fractures of ribs 3 to 6 in two places. Which of the following statements is true regarding the diagnosis and management flail chest?

A. Flail chest is primarily a radiographic diagnosis B. A flail segment has minimal impact on overall pulmonary mechanics and thoracic volume C. Rib fixation and plating is indicated in all diagnosed cases of flail chest D. The diagnosis of flail chest is made with radiographic evidence of fractures of three or more ribs in two or more places and clinically apparent paradoxical chest movement E. Flail chest is most commonly seen in blunt trauma and has equal prevalence in the adult and pediatric populations

2. A 50-year-old male with a past medical history of hypertension and long-standing tobacco use is being admitted to the SICU after being involved in a motorcycle accident resulting in multiple bilateral rib fractures with underlying pulmonary contusions. On arrival to the ICU the patient is noted to be in obvious discomfort, taking in shallow breaths and requiring 4 L nasal cannula for oxygen

supplementation. All of the following statements about the management of a polytrauma with pulmonary contusions are true EXCEPT:

A. Colloid is preferred over crystalloid in the resuscitation of trauma patients with pulmonary contusions B. Pulmonary contusions may not be present on initial plain radiograph but can develop in days following the initial trauma C. Initial management involves judicious use of IVF fluid resuscitation and multimodal pain management D. Pulmonary contusions secondary to blunt trauma result in decreased lung compliance and ventilation-perfusion inequalities E. Pulmonary contusions may be present in certain patient populations without associated overlying rib fractures Hemothorax:

3. All of the following are appropriate methods to evaluate for the presence of an acute traumatic hemothorax in the setting of blunt thoracic trauma EXCEPT:

A. Chest tube placement B. Plain film chest X-ray C. Computed Tomography (CT) D. Ultrasound E. Magnetic Resonance Imaging (MRI)

4. A 25-year-old male is brought to the emergency department after being involved in a multiple vehicle motor collision. On primary survey his ABCs are intact and his initial vital signs are as follows: Temp 99°F, HR 100 bpm, BP 130/70 mm Hg, RR 19, Sat 95% on room air. On completion of his trauma workup he is noted to have an extensive trauma burden, including a traumatic right sided hemo-

pneumothorax with associated overlying rib fractures for which a tube thoracostomy is performed. Which of the following statements regarding acute traumatic hemothoraces and pneumothoraces is true?

A. All traumatic hemothoraces should be evaluated further via chest tube thoracostomy and Video-Assisted Thorascopic Surgery (VATS) B. Diagnosis of an occult pneumothorax is an indication for chest tube placement C. Persistent air-leak and/or recurrent pneumothorax should prompt evaluation for intrathoracic tracheobronchial injury D. Given the risk of progression, patients requiring positive pressure ventilation in the presence of a pneumothorax require chest tube placement E. Chest tube output >250 mL on initial placement is an indication to proceed to the operating room for thoracotomy

5. A 55-year-old female is the unrestrained passenger in a rollover motor vehicle accident. Among other injuries, she sustains multiple facial fractures, fractures involving C2-4 and multiple rib fractures bilaterally. According to the Denver Criteria, which of the following is NOT an indication for Computed Tomography Angiography (CT Angiography or CTA) as part of the comprehensive trauma evaluation?

A. High-impact mechanism resulting in significant polytrauma B. Neurological examination not congruent with findings on noncontrast CT of the head C. LeFort Fracture type 2 or 3 D. Base of skull fractures involving the carotid canal E. Cervical spine fractures involving C1-3 vertebrae

6. In a patient presenting with blunt trauma and obvious head involvement, a CT angiography of the head is obtained based on the Denver Modification Screening Criteria for blunt cerebrovascular injury (BCVI). The patient is subsequently diagnosed with a Grade 3 injury. Which of the following below correctly describes a Grade 3 BCVI according to the widely accepted Biffl scale?

A. Transection B. Complete Occlusion C. Dissection or intramural hematomas with ≥25% luminal narrowing D. Pseudoaneurysm or hemodynamically insignificant arteriovenous fistula E. Intimal irregularity or dissection with 65 years of age, the recommended dose is 5 mg PO BID. Oxandrolone, a synthetic analogue, administered orally, offers only 5% of the masculinizing effects of testosterone and is safe for both genders. Oxandrolone, when administered at a dose of 0.1 mg/kg twice daily, improved net muscle protein synthesis and protein metabolism in severely burned patients. During the acute phase post burn and up to 1 year of treatment, oxandrolone increased lean body mass, bone mineral content, and muscle strength. In addition, it decreased length of stay by decreasing time between operations for patients randomized to receive oxandrolone plus standard of care. Oxandrolone results in considerable improvements in lean body mass, protein synthesis, and overall growth in burn patients, mitigating the 1% risk of hirsutism and hepatic dysfunction that can be seen with treatment. It must be noted that although anabolic steroids can increase lean body mass, exercise is essential to developing strength. References

1. Porter C, Tompkins RG, Finnerty CC, Sidossis LS, Suman OE, Herndon DN. The metabolic stress response to burn trauma: current understanding and therapies. Lancet. 2016;388(10052):1417-1426.

2. Sabiston DC, Townsend CM, eds. Sabiston Textbook of Surgery: The Biological Basis of Modern Surgical Practice. Philadelphia, PA: Elsevier Saunders; 2012.

3. Lam NN, Tien NG, Khoa CM. Early enteral feeding for burned patients – an effective method which should be encouraged in developing countries. Burns. 2008;34:192.

4. Branski LK, Herndon DN, Barrow RE, et al. Randomized controlled trial to determine the efficacy of long-term growth hormone treatment in severely burned children. Ann Surg. 2009;127:145-154.

5. Williams FN, Jeschke MG, Chinkes DL, et al. Modulation of the hypermetabolic response to trauma: temperature, nutrition, and drugs. J Am Coll Surg. 2009;208:489-502.

6. Langouche L, Van den Berghe G. Glucose metabolism and insulin therapy. Crit Care Clin. 2006;22:119-129.

C H AP T E R 1 0 3

Disaster Management Todd A. Jaffe and Jarone Lee

1. A 57-year-old government employee who works to counter bioterrorism without any pertinent medical history is exposed to a small amount of an unknown white powder on his skin. Over 4 days, he develops a flulike illness with symptoms including malaise, fever, and fatigue. On presentation to the emergency department, he has a temperature of 38.5°C (101.3°F), HR of 102/min, BP of 148/86 mm Hg, and O2 of saturation of 98% on room air. Physical examination is notable for the skin lesion figure that follows.

Image courtesy of Balachandrudu B, Amrutha Bindu SS, Kuman CN, Malakondaiah P. An outbreak of cutaneous anthrax in a tribal area of Visakhapatnam district, Andhra Pradesh. J NTR Univ Health Sci. 2018;7:49-53.

Laboratory studies are notable for a WBC of 10,800. Gram staining of the substance reveals large, spore-forming, gram-positive bacilli. The patient is placed in isolation and admitted to the general medical service. Twelve hours after admission, the patient becomes increasingly

altered. He endorses nuchal rigidity and photophobia, is unable to formulate complete sentences, and is transferred to the ICU. Lumbar Puncture demonstrates amber colored fluid, an opening pressure of 35 cm H2O, 4800 WBCs/mm3 with 85% PMNs, 30 RBCs/mm, protein of 400 mg/dL, and a glucose of 24 mg/dL. In addition to antitoxin, which of the following antibiotic(s) should be empirically started on this patient in the ICU? A. Moxifloxacin and clindamycin B. Ciprofloxacin, meropenem, and linezolid C. Cefepime and vancomycin D. Levofloxacin and acyclovir

2. Twelve employees of a clothing factory are transported to the emergency department after a fire at their factory. The factory utilizes wool, silk, and multiple other clothing products. The group was in the building for 2 hours while the fire was ablaze. A 55-yearold male who was found located close to the fire presents with headache, vomiting, and altered mental status. He also reports a “bitter almond” smell. Initial examination is notable for T37.0°C (98.6°F), HR 121/min, BP 160/96 mm Hg, O2 saturation of 92% on room air. The patient appears flushed. His clothing is removed, and his skin is rinsed with soap and water.

As laboratory test results are pending, the patient has multiple episodes of convulsions and is intubated for airway protection. Laboratory test results are shown in the table that follows. Which of the following treatments for cyanide toxicity should be AVOIDED in this patient? Lab

Patient Value

Hgb Hct WBC Platelets Sodium Chloride Glucose Potassium BUN

13.6 g/dL 44% 14 000/µl (H) 420 000/µl 142 mEq/L 101 mEq/L 121 mg/dL 4.2 mEq/L 16 mg/dL

Creatinine Cyanide Methemoglobin

1.1 mg/dL 32 µmol/L (H) 18% (H)

A. Sodium nitrite B. Sodium thiosulfate C. Hydroxocobalamin D. 100% Oxygen

3. Multiple villagers in a war-torn Middle Eastern country are exposed to an unknown toxic agent. They present to a medical facility complaining of abdominal pain, diarrhea, frequent urination, and excessive tearing in their eyes. Physical examination is notable for bradycardia, miosis, and salivary secretions. Which of the following is the most likely agent and what is the appropriate antidote?

A. Sarin Nerve Gas; Atropine and pralidoxime B. Atropa belladonna; Physostigmine C. Benzodiazepines; Flumazenil D. Arsenic; Dimercaprol

4. A 56-year-old male who was working at a nuclear power plant presents to the emergency department complaining of significant nausea and vomiting. Initial vitals are temperature of 36.8°C (98.2°F), HR of 108/min, BP of 118/76 mm Hg, O2 saturation of 96% on room air. Physical examination is otherwise unremarkable. He is decontaminated, placed in isolation, and admitted to the ICU for further management. It is estimated that he was exposed to 4 -6 Gy of radiation.

On day 8 of admission, his laboratory studies are notable for WBC of 1500/µl with ANC of 300/µl, Hgb of 5.8 g/dL, Hct of 23%, and Plt of 23000/ µL.

In patients with hematopoietic radiation injury, which of the following is NOT a recommended component of treatment? A. Granulocyte colony-stimulating factor for patients exposed to >2 Gy B. Empiric antibiotics for patients with ANC 10 Gy D. Administration of blood products that have undergone leukoreduction and irradiation

5. A 55-year-old male presents to an emergency department in China with a 5-day history of cough, fever, and myalgias. On presentation, his temperature of 39.8°C (103.6°F), HR of 112/min, BP of 98/66 mm Hg, O2 saturation of 88% on room air, which improves to 94% on 2 L of nasal cannula. Chest X-ray demonstrates a left lower lobe pneumonia. He receives 2 L of lactated ringers, is started on empiric broad spectrum antibiotics with cefepime and vancomycin, and is admitted to an isolated bed on the general medicine floor for further management. Two days after admission, he develops a worsening oxygen requirement, is transferred to the ICU, where he is ultimately intubated for hypoxic respiratory failure. His CXR just before intubation is shown below.

Image adapted from: Fan E, Brodie D, Slutsky AS. Acute respiratory distress syndrome: advances in diagnosis and treatment. JAMA. 2018;319:698-710. A nasopharyngeal aspirate and endotracheal aspirate are ultimately positive for Avian Influenza A H7N9. Which of the following is true regarding the management of H7N9? A. Inhaled zanamivir should be reserved for patients with underlying airway disease B. Treatment with an antiviral agent such as oseltamivir or zanamivir is recommended, even if more than 48 hours have elapsed since illness onset C. Treatment with IV zanamivir is preferred over PO oseltamivir D. Antiviral administration should be held until laboratory testing has

confirmed the viral pathogen

6. A tropical storm strikes a town with a large nuclear reactor, and there is concern for ongoing leaking of radioactive material, with potential exposure to both the building inhabitants and nearby population. Representatives from the police force, fire rescue, medical personnel, and public health arrive on the scene to coordinate a response. The responders utilize the National Incident Management System (NIMS) and Incident Command System (ICS). When utilizing these guidelines, which of the following roles is responsible for managing the overall response to the potential emergency described above?

A. Operations Section Chief B. Incident Commander C. Logistics Section Chief D. Safety Officer

Chapter 103 Answers 1. Correct Answer: B Rationale: This patient likely has been exposed to anthrax. Bacillus anthracis is a gram-positive, aerobic, capsulated, spore-forming, rod-shaped bacterium. It can be transmitted through cutaneous exposure, inhalation, ingestion, and injection, with each transmission method posing the potential for systemic progression. Patients first develop a prodromal flulike illness. However, there are fatal complications of anthrax, most notably hemorrhagic mediastinitis and meningitis. Anthrax meningitis is almost invariably fatal. Patients may present with fever, neck rigidity, altered mental status, and with LP findings consistent with bacterial meningitis: elevated opening pressure, elevated WBCs with neutrophil predominance, elevated protein, and low glucose. In addition, the presence of frank blood or RBCs is also common given that anthrax meningitis may have a hemorrhagic component. Anthrax meningitis requires broad spectrum antibiotics with multiple antibiotic modalities and CSF penetration (B). This treatment should include at least one protein synthesis inhibitor to reduce exotoxin production. Moxifloxacin and clindamycin (A) would be appropriate empiric coverage for cutaneous anthrax without evidence of meningitis. However, linezolid has significantly better CNS penetration compared with clindamycin. Cefepime and vancomycin (C) are broad spectrum antibiotics with strong gram-positive and -negative coverage; however, studies have demonstrated that cephalosporins, including cefepime, have unreliable coverage of B. anthracis. Levofloxacin and other fluoroquinolones are often first line treatment for cutaneous anthrax; however, they would not be adequate coverage for anthrax meningitis. References

1. Lanska DJ. Anthrax meningoencephalitis. Neurology. 2002;59:327-334. 2. Sejvar JJ, Tenover FC, Stephens DS. Management of anthrax meningitis. Lancet Infect Dis. 2005;5:287-295.

3. Hendricks KA, Wright ME, Shadomy SV, et al. Centers for disease control

and prevention expert panel meetings on prevention and treatment of anthrax in adults. Emerg Infect Dis. 2014;20.

4. Bakici MZ, Elaldi N, Bakir M, Dokmetas I, Erandac M, Turan M. Antimicrobial susceptibility of Bacillus anthracis in an endemic area. Scand J Infect Dis. 2002;34:564-566.

2. Correct Answer: A Rationale: This patient presents with headache, vomiting, and altered mental status after prolonged smoke exposure, which is concerning for cyanide toxicity. Initial treatment for cyanide toxicity includes removal from the exposure, decontamination, and airway protection. Patients with inhalation cyanide toxicity secondary to smoke exposure often copresent with carbon monoxide poisoning. There are multiple modalities to definitively treat cyanide toxicity, including sodium nitrite, sodium thiosulfate, hydroxocobalamin, and 100% or hyperbaric oxygen. Sodium nitrite (A) is effective in treating cyanide by inducing methemoglobinemia. This leads to the oxidation of iron in hemoglobin to form the ferric ion. Cyanide can bind with the formed methemoglobin structure, which creates cyanomethemoglobin, which is less toxic than other cyanide products. However, the formation of methemoglobin shifts the oxygen dissociation curve to the left, which can be harmful for patients with concurrent carbon monoxide poisoning. As such, the induction of methemoglobinemia in patients with both cyanide poisoning and carbon monoxide poisoning has the potential to be lethal and should be avoided in these patients. Sodium thiosulfate (B) works by providing sulfur donors, which can convert cyanide to thiocyanate. Thiocyanate is readily excreted through the kidneys and is a reasonable treatment for this patient. Hydroxocobalamin works by binding to intracellular cyanide with greater affinity than cytochrome oxidase and forms cyanocobalamin. This structure can be excreted in the urine and works rapidly. Studies have found that 100% oxygen and hyperbaric oxygen can improve outcomes in cyanide poisoning. This patient may have concomitant carbon monoxide poisoning, with which 100% oxygen would be a cornerstone of treatment.

References

1. Bryson PD. Comprehensive Review in Toxicology for Emergency Clinicians. 3rd ed. Denver: Taylor and Francis; 1996:352.

2. Dumestre D, Nickerson D. Use of cyanide antidotes in burn patients with suspected inhalation injuries in North America: a cross-sectional survey. J Burn Care Res. 2014;35:e112-e117.

3. Dries DJ, Endorf FW. Inhalation injury: epidemiology, pathology, treatment strategies. Scand J Trauma Resusc Emerg Med. 2013;21:31.

4. Takano T, Miyazaki Y, Nashimoto I, Kobayashi K. Effect of hyperbaric oxygen on cyanide intoxication: in situ changes in intracellular oxidation reduction. Undersea Biomed Res. 1980;7:191-197.

3. Correct Answer: A Rationale: These patients are presenting with cholinergic toxidrome consistent with organophosphate poisoning. Sarin nerve gas is a colorless and odorless nerve agent that blocks the acetylcholinesterase enzyme that leads to the accumulation of acetylcholine at the neuromuscular junction. This can lead to a cholinergic crisis with the symptomatology described above. Symptoms include salivation, lacrimation, urination, diarrhea, GI pain, and emesis. The extreme of organophosphate poisoning is the sequelae of bradycardia, bronchorrhea, and bronchospasm, which can be fatal. The villagers in the question stem would likely need close monitoring of their airway with a low threshold for intubation, as they have evidence of increased secretions on physical examination. Atropa belladonna (B) is also known as deadly nightshade and contains atropine, hyocyamine, and scopolamine. It can lead to an anticholinergic toxidrome characterized by tachycardia, dry and flushed skin, altered mental status, and mydriasis, with the potential for significant neurologic dysfunction at high doses. Benzodiazepine overdose (C) would be characterized by lethargy, ataxia, and respiratory depression. It is unlikely in this group of people, and the clinical symptoms would differ from those described in this question. Arsenic ingestion (D) is characterized by prominent GI symptoms as well as jaundice, hematuria, and altered

mental status. Subacute poisoning would present with anemia and peripheral neuropathy, which are not characterized by the individuals in the given question. References

1. Chai PR, Boyer EW, Al-Nahhas H, Erickson TB. Toxic chemical weapons of assassination and warfare: nerve agents VX and sarin. Toxicol Commun. 2017;1:21-23.

2. Mundy SW. Arsenic. In: Goldfrank’s Toxicologic Emergencies. In: Hoffman RS, Lewin NA, Howland MA, et al, eds. 10th ed. New York: McgrawHill Education; 2015:1169.

3. Kwakye GF, Jimenez J, Jimenez JA, Aschner M. Atropa belladonna neurotoxicity: implications to neurological disorders. Food Chem Toxicol. 2018;116:346-353.

4. Correct Answer: C Rationale: Radiation injuries are largely dependent on the exposure burden for patients, and a patient’s clinical course can often be predicted based on exposure. Patients exposed to 0 -2 Gy are unlikely to have significant complications from their radiation exposure. Patients with 2 -9 Gy may have significant sequelae from their exposure and require significant monitoring and care. Hematopoietic cell transplantation has been described as potentially beneficial for patients with Gy exposure between 2 and 9. Exposure greater than 10 Gy is almost invariably fatal (see Radiation Injury Doses and Syndromes and Phases of Radiation Injury tables). Considering the fatality rates of greater exposures and the limited resources available for transplantation, it has been documented that it is unwise to transplant patients with greater exposure (C). Granulocyte colony-stimulation factor (A) has been shown to be beneficial for patients with significant exposure. Recommendations include daily administration of G-CSF until neutropenia resolves. For patients with significant cytopenia, including those with ANCs 10 Gy) Multiple organ failure probable death

(8-10 Gy) Consider stem cell transplant 3-7 Gy

GI syndrome onset

0-2 Gy

Hematopoietic syndrome onset

(6-7 Gy) LD50/60 with supportive care (3-5 Gy) LD50/60 without treatment (0-2 Gy) ∼100% survival without treatment

Adapted from Lopez M, Martin M. Medical management of the acute radiation syndrome. Rep Pract Oncol Radiother. 2011;16:138-146. Phases of Radiation Injury

Dose 
Range, Gy

Prodrome

Manifestation of Illness

Prognosis (Without Therapy)

0.5-1.0

Mild

Slight decrease in blood cell counts

1.0-2.0

Mild to moderate

Early signs of bone marrow damage

2.0-3.5

Moderate

3.5-5.5

Severe

Moderate to severe bone marrow damage Severe bone marrow damage; slight GI damage

5.5-7.5

Severe

Almost certain survival Highly probably survival (>90% of victims) Probable survival Death within 3.5-6 wk (50% of victims) Death probable within 2-3 wk

Pancytopenia and moderate GI damage

7.5-10.0

Severe

Marked GI and bone marrow damage, hypotension

10.0-20.0

Severe

20.0-30.0

Severe

Severe GI damage, pneumonitis, altered mental status, cognitive dysfunction Cerebrovascular collapse, fever, shock

Death probable within 12.5 wk Death certain within 5-12 d Death certain within 2-5 d

Adapted from Waselenko JK, MacVittie TJ, Blakely WF, et al. Medical management of the acute radiation syndrome: recommendations of the Strategic National Stockpile Radiation Working Group. Ann Intern Med. 2004;140:1037-1051.

References

1. Dainiak N. Medical management of acute radiation syndrome and associated infections in a high-casualty incident. J Radiat Res. 2018;59:ii54-ii64.

2. Dainiak N, Gent RN, Carr Z, et al. First global consensus for evidencebased management of the hematopoietic syndrome resulting from exposure to ionizing radiation. Disaster Med Public Health Prep. 2011;5:202-212.

3. Gafter-Gvili A, Fraser A, Paul M, Leibovici L. Meta-analysis: antibiotic prophylaxis reduces mortality in neutropenic patients. Ann Intern Med. 2005;142:979-995.

5. Correct Answer: B Rationale: As confirmed by laboratory testing, this patient has avian bird flu, H7N9. H7N9 is highly virulent form of influenza with the first cases of this new strain of influenza reported in 2013 in China. Patient’s clinical presentations may vary but have often included the acute onset and rapid progression of common influenza symptoms. In addition, many patients have presented with leukopenia, lymphopenia, and thrombocytopenia. The severity of the illness often correlates with the baseline health of the infected patient; however, over 75% of patients were reported to be admitted to the ICU while hospitalized, and over 25% of patients have died.

The CDC published interim guidelines in 2013 regarding treatment for patients with H7N9. For those admitted to the hospital, antiviral treatment with neuroaminidase inhibitors (oseltamivir or zanamivir) is recommended. It is recommended that these antiviral agents are included in treatment even after 48 hours of illness onset, especially for patients admitted to the hospital. For other forms of influenza, it is debated whether oseltamivir should be included in treatment after 48 hours. Inhaled Zanamivir should be avoided in patients with underlying airway disease (A), and instead oseltamivir or IV formulations should be used. There is no difference in outcomes when comparing IV zanamivir and PO oseltamivir (C). Zanamivir would only be preferred if the patient is unable to tolerate PO medications. Additionally, for patients with presumed avian flu, antivirals should be included in treatment even while laboratory testing is pending. The recommended test for H7N9 is the utilization of real-time reverse-transcriptase polymerase chain reaction for avian influenza A H7N9 on an oropharyngeal or nasopharyngeal aspirate. This study will likely include send-out laboratory testing, and treatment should not be delayed while results are pending. References

1. Emerging pathogens: influenza - H7n9. Dis Mon. 2017;63:251-256. 2. Gao HN, Lu HZ, Cao B, et al. Clinical findings in 111 cases of influenza A (H7N9) virus infection. N Engl J Med. 2013;368:2277-2285.

3. Centers for Disease Control and Prevention. Interim Guidance on the Use of Antiviral Agents for Treatment Of Human Infections With Avian Influenza A (H7n9) Virus. http://www.cdc.gov/flu/avianflu/h7n9antiviral-treatment.htm.

4. Jefferson T, Jones MA, Doshi P, et al. Neuraminidase inhibitors for preventing and treating influenza in healthy adults and children. Cochrane Database Syst Rev. 2014:CD008965.

5. Centers for Disease Control and Prevention. Interim Guidance For Specimen Collection, Processing, and Testing For Patients Who May Be Infected With Avian Influenza A (H7N9) Virus. http://www.cdc.gov/flu/avianflu/h7n9/specimen-collection.htm? s_cid=seasonalflu-govd-003.

6. Correct Answer: B Rationale: The ICS is a component of the NIMS and serves to guide the response to public crises. Since 2004, the federal government has mandated that organizations utilize this system to conduct emergency management activities. ICS is one aspect of the broader NIMS, the latter being a national initiative to standardize emergency medicine activities across the entire United States. ICS functions as the executional component of NIMS related to specific incidents and has the aim of streamlining multiple aspects of disaster response including organizational structure, communication patterns, and resource allocation. The system is designed to be adaptable for localized incidents and large, regional crises. One major component of the ICS is the role of an incident commander, whose function is to manage the overall response to an incident. This may be one individual or could include representation from multiple affected organizations (eg police commissioner and public health representative). Furthermore, the role may change depending on the magnitude of the incident, with incident commanders playing more of a hands-on role in smaller-scale incidents. The typical organizational structure of the ICS is included in Figure 103.1 below.

FIGURE 103.1 Incident command system organizational structure.

Medical professionals are often a key component of ICS in multiple facets. This can vary from acting as incident commander to safety officer or technical specialist as warranted by the nature of specific instances. Within medical facilities, there exists a Hospital ICS, which is geared toward addressing hospital-related incidents such as infection control, power outages, and security concerns. The operations section chief (A) and logistics section chief (C) may directly report to the incident commander or other officers. They are responsible for directing actions to accomplish objectives and resource management, respectively. The safety officer (D) is responsible for ensuring the safety of personnel involved in the incident as well as those responding to the incident. References

1. Jensen J, Thompson S. The incident command system: a literature review. Disasters. 2016;40:158-182.

2. Adapted from “National Incident Management System”. FEMA. December 2008:53. https://www.fema.gov/pdf/emergency/nims/NIMS_core.pdf. Accessed September 18, 2018.

3. Furin M, Goldstein S. EMS, Incident Command System (ICS). Treasure Island, FL: StatPearls;2018.

4. Rogers FB, McCune W, Jammula S, et al. Emergency operations program is an excellent platform to deal with in-hospital operation disaster. Am J Disaster Med. 2017;12:267-273.

5. Federal Emergency Management Agency “FEMA Glossary”. FEMA; October 28, 2010. https://training.fema.gov/emiweb/is/icsresource/assets/icsglossary.pdf. Accessed September 18, 2018.

C H AP T E R 1 0 4

Transplantation Reem Almuqati, Sandeep Khanna and Roshni Sreedharan

1. 30-year-old multiparous female with left ventricular noncompaction cardiomyopathy presents to the intensive care unit (ICU) from the operating room after undergoing an orthotopic cardiac transplantation. The donor organ was retrieved from a 20-year-old brain dead man. Preoperatively, the recipient was on chronic intravenous dobutamine therapy. Intravenous basiliximab was administered immediately prior to her transplant for induction of immunosuppression. Post termination of cardiac bypass in the operating room, temporary pacing was initiated in view of persistent bradyarrhythmia. The MOST likely cause of her posttransplant bradyarrhythmia is:

A. Basiliximab induction B. Surgical trauma C. Donor age and gender D. Preoperative dobutamine therapy

2. A 40-year-old male with known nonischemic cardiomyopathy and pulmonary hypertension returns to the ICU intubated and sedated after undergoing an orthotopic heart transplant. Graft ischemia time was 4 hours. On arrival, he is on high-dose intravenous vasopressor support including 0.5 µg/kg/min norepinephrine, 0.5 µg/kg/min epinephrine, and 0.1 units/h of vasopressin. He is receiving inhaled nitric oxide 40 ppm. Monitoring reveals the following: Cardiac index of 1.6 L/min/m2, invasive arterial pressure of 80/40 mm Hg, central venous pressure of 18 mm Hg, and heart rate 130 beats/min. The LEAST likely cause of his cardiogenic shock is:

A. T-lymphocyte–mediated rejection of allograft B. Mediastinal bleeding with regional cardiac tamponade C. Ischemia-reperfusion injury–related primary graft failure D. Acute on chronic pulmonary hypertension

3. 50-year-old male with end-stage lung disease and pulmonary hypertension secondary to emphysema is admitted to ICU after undergoing bilateral lung transplantation on cardiopulmonary bypass. His body mass index (BMI) is 22 kg/m2. The lungs were retrieved from a 25-year-old brain dead man. The donor was a nonsmoker. On POD 2, the recipient’s PaO2/FiO2 ratio is 150 and bilateral lung opacities consistent with pulmonary edema are noted on chest x-ray suggesting a diagnosis of grade 3 primary graft dysfunction (PGD). The risk factor MOST likely associated with PGD in this scenario is:

A. Donor ’s nonsmoker status B. Recipient and donor age mismatch C. BMI less than 25 kg/m2 D. Preexisting pulmonary hypertension

4. 40-year-old male underwent liver transplant 20 years ago due to biliary cirrhosis. He is now listed for a redo liver transplant in view of recurrent cirrhosis. He is admitted to the ICU with upper gastrointestinal variceal bleeding requiring massive transfusion. He is intubated and on mechanical ventilation. Bedside echocardiography reveals an ejection fraction of 70% and absence of diastolic dysfunction. Electrocardiogram exhibits sinus tachycardia with a normal QT interval. Chest x-ray appears normal. Patient’s invasive arterial pressure is 100/60 mm Hg and central venous pressure is 8 mm Hg. A pulmonary artery catheter is inserted and the following values are obtained: Cardiac output of 10 L/min, mean pulmonary artery pressure of 30 mm Hg, and pulmonary artery occlusion pressure of 10 mm Hg. This is MOST likely due to:

A. Porto pulmonary hypertension B. Hyper dynamic circulation C. Transfusion-associated circulatory overload D. Cirrhotic cardiomyopathy

5. A 45-year-old male with hepatitis C–related cirrhosis presents to the ICU from the operating room after undergoing orthotopic liver transplantation. At reperfusion, he suffered a brief asystolic arrest due to hyperkalemia. The donor liver was MOST likely preserved in which preservative solution:

A. Celsior B. Histidine-tryptophan-ketoglutarate (HTK) C. Institut Georges Lopez-1 (IGL-1) D. University of Wisconsin

6. 50-year-old male received a kidney transplant 3 months ago and is on immunosuppression with prednisone, tacrolimus, and mycophenolate. Antiviral prophylaxis includes valganciclovir. He recently attended a wedding in Mexico and was noncompliant with his medication regimen. He now presents with persistent diarrhea, nausea, and abdominal pain. He is admitted to ICU in view of need for ongoing fluid resuscitation. Nuclear acid testing is suggestive of cytomegalovirus (CMV) disease. Treatment of this condition MOST likely includes:

A. Reduction in doses of all immunosuppressants B. Initiation of oral valganciclovir C. Initiation of intravenous ganciclovir D. Initiation of intravenous cidofovir

7. After initiation of appropriate drug therapy, the patient in the above stem demonstrates clinical improvement but develops leukopenia. He will MOST likely benefit from:

A. Discontinuation of antiviral treatment B. Discontinuation of tacrolimus C. Addition of foscarnet D. Addition of filgrastim

8. 40-year-old male arrives to ICU from the operating room after undergoing an orthotopic combined liver kidney transplant. Intraoperative course was complicated by severe vasoplegic shock necessitating initiation of high-dose norepinephrine infusion. Which of the following is LEAST likely to increase mean arterial pressure (MAP) in this patient?

A. Angiotensin I B. Hydroxocobalamin C. Methylene blue D. Vasopressin

9. 30-year-old female is admitted to the ICU in view of progressive shortness of breath and increasing oxygen requirements. She had bilateral lung transplant secondary to cystic fibrosis 3 month ago. The LEAST likely cause for this presentation is:

A. Opportunistic infection B. Acute rejection C. Bronchial airway stenosis D. Pulmonary vein stenosis

10. 25-year-old male undergoes a combined pancreas kidney transplant. Maintenance immunosuppression includes tacrolimus, mycophenolate, and prednisone. Infection prophylaxis includes valganciclovir and trimethoprim-sulfamethoxazole. Two months later, he presents to the emergency department with seizures necessitating intubation for airway protection. Per his family, recent medication changes include the addition of pantoprazole and diltiazem; they are unaware if he has been compliant. He has also been drinking grapefruit juice for the last 1 week instead of orange juice. He is admitted to the ICU for further workup. Computed tomography reveals hypodensity in the posterior white matter. Elevations in plasma levels of which drug is MOST likely responsible for this condition?

A. Tacrolimus B. Pantoprazole C. Valganciclovir D. Trimethoprim-sulfamethoxazole

Chapter 104 Answers 1. Correct Answer: B Rationale: Sinus node dysfunction occurs in up to 50% of patients following cardiac transplantation. Such dysfunction commonly manifests as bradycardia, is usually temporary, and spontaneously resolves in a majority of patients within the first 3 months of transplantation. Temporary atrial pacing often suffices in the early postoperative period as atrioventricular (AV) node conduction is usually preserved. Unlike sinus node dysfunction, AV nodal conduction abnormalities are uncommon and tend to occur late after cardiac transplantation. Various risk factors for developing sinus node dysfunction and ensuing posttransplant bradycardia have been described. These include surgical trauma to the sinus node, perinodal atrial tissue, or sinoatrial artery; ischemia-reperfusion injury; pretransplant use of amiodarone; older donors; and rejection. While robust evidence for most risk factors is lacking, the most likely cause of sinus node dysfunction is surgical trauma at the time of transplantation. Use of bicaval anastomotic technique instead of the biatrial surgical approach for orthotopic cardiac transplantation has substantially decreased the incidence of posttransplant bradyarrhythmia and nearly eliminated the need for permanent pacing. Donor gender and choice of immunosuppressive induction agent are not risk factors for sinus node dysfunction. Key point: Sinus node dysfunction commonly occurs early after cardiac transplantation and is often related to surgical trauma. References

1. DiBiase A, Tse TM, Schnittger I, et al. Frequency and mechanism of bradycardia in cardiac transplant recipients and need for pacemakers. Am J Cardiol. 1991;67:1385.

2. Heinz G, Hirschl M, Buxbaum P, et al. Sinus node dysfunction after orthotopic cardiac transplantation: postoperative incidence and longterm implications. Pacing Clin Electrophysiol. 1992;15:731.

3. Heinz G, Kratochwill C, Koller-Strametz J, et al. Benign prognosis of early sinus node dysfunction after orthotopic cardiac transplantation. Pacing Clin Electrophysiol. 1998;21:422.

4. Melton IC, Gilligan DM, Wood MA, Ellengbogen KA. Optimal cardiac pacing after heart transplantation. Pacing Clin Electrophysiol. 1999;22:1510-1527.

2. Correct Answer: A Rationale: Early allograft dysfunction after orthotopic heart transplant can be apparent in the intraoperative period or can develop within 24 hours after transplant surgery. It can manifest as left ventricular (LV) dysfunction, isolated right ventricular (RV) dysfunction, or biventricular dysfunction. It is associated with significantly increased 30-day and 1-year mortality. Multiple factors can contribute to early graft dysfunction and include hyperacute rejection, pulmonary hypertension, prolonged graft ischemic time, cardiac tamponade, and suboptimal donor heart. PGD is defined as ventricular dysfunction that occurs within 24 hours after surgery and is not associated with a discernible cause. Hyperacute cellular rejection can present with immediate cardiogenic shock post transplantation and is commonly mediated by preformed B-cell antibodies. Acute rejection commonly occurs weeks to months after transplantation and is mediated by T-lymphocyte activation. Most cases of acute rejection are diagnosed by routine surveillance endomyocardial biopsy at a time when the patient is asymptomatic and ventricular function is normal. Acute rejection commonly occurs weeks to months after transplantation, is mediated by T-lymphocyte activation, and is NOT a cause for cardiogenic shock in the immediate postoperative period. Differentials for immediate posttransplant cardiogenic shock include PGD, exacerbation of pulmonary hypertension, cardiac tamponade, and hyperacute rejection. References

1. Kobashigawa J, Zuckermann A, Macdonald P, et al. Report from a consensus conference on primary graft dysfunction after cardiac transplantation. J Heart Lung Transplant. 2014;33(4):327-340.

2. McNamara D, Di Salvo T, Mathier M, Keck S, Semigran M, Dec GW. Left ventricular dysfunction after heart transplantation: incidence and role of enhanced immunosuppression. J Heart Lung Transplant. 1996;15(5):506-515.

3. Michaels PJ, Espejo ML, Kobashigawa J, et al. Humoral rejection in cardiac transplantation: risk factors, hemodynamic consequences and relationship to transplant coronary artery disease. J Heart Lung Transplant. 2003;22(1):58.

3. Correct Answer: D Rationale: PGD after lung transplantation develops in the first 72 hours after transplantation and is characterized by hypoxemia with radiographic appearance of diffuse pulmonary opacities. Multiple risk factors for PGD have been identified. Donor and recipient characteristics, presence of preoperative disease, and intraoperative risk factors may contribute to PGD. Donor risk factors include smoking, aspiration, lung contusion, undersized donor relative to recipient, heavy alcohol use, fat embolism, and thromboembolism. Recipient factors include female gender, elevated recipient BMI (≥25 kg/m2), and being African American. Pretransplant diseases with increased risk of PGD include idiopathic pulmonary fibrosis, sarcoidosis, and pulmonary arterial hypertension (PAH). Intraoperative risk factors include large volume intraoperative blood product transfusion, prolonged ischemic time, and use of cardiopulmonary bypass (CPB). PGD post lung transplantation is characterized by hypoxemia and the radiographic appearance of diffuse pulmonary opacities. Multiple risk factors for this condition exist. References

1. Snell GI, Yusen RD, Weill D, et al. Report of the ISHLT working group on primary lung graft dysfunction, part I: definition and grading-A 2016 consensus group statement of the International Society for Heart and Lung Transplantation. J Heart Lung Transplant. 2017;36(10):1097.

2. Liu Y, Su L, Jiang SJ. Recipient-related clinical risk factors for primary graft dysfunction after lung transplantation: a systematic review and meta-analysis. PLoS One. 2014;9(3):e92773.

3. Bermudez CA, Shiose A, Esper SA, et al. Outcomes of intraoperative venoarterial extracorporeal membrane oxygenation versus cardiopulmonary bypass during lung transplantation. Ann Thorac Surg. 2014;98(6):1936-1942

4. Correct Answer: B Rationale: Portopulmonary hypertension (PoPH) is PAH arising in the setting of portal hypertension with or without liver cirrhosis. The definition of PoPH comprises three essential elements: 1. 2.

Mean pulmonary arterial pressure (mPAP) greater than 25 mm Hg Pulmonary vascular resistance (PVR) greater than 240 dynes/s/cm5 or 3 Wood Units 3. Pulmonary arterial occlusion pressure (PAOP) less than or equal to 15 mm Hg The cause of PoPH is unknown. It has been hypothesized that humoral substances such as interleukin-1, endothelin-1, glucagon, secretin, thromboxane B2, and vasoactive intestinal peptide, which would normally be metabolized by the liver, are able to access the pulmonary circulation through portosystemic collaterals, resulting in PoPH. In the stem above, calculated PVR is 2 Wood Units. Although our patient’s mPAP is greater than 25 mm Hg with a PAOP less than 15 mm Hg, he would not meet the criteria for PoPH given his current PVR. Cirrhotic cardiomyopathy is a cardiac condition observed in patients with end-stage liver disease regardless of etiology. It is characterized by normal to increased cardiac output and contractility at rest but impaired systolic response to stress. It is commonly associated with diastolic dysfunction and electrophysiological abnormalities such as QT interval prolongation. Diagnosis of cirrhotic cardiomyopathy requires presence of both systolic and diastolic dysfunction with or without electrophysiological abnormalities. Our patient in the stem would not qualify. Transfusion-associated circulatory overload commonly presents with respiratory distress and hypertension within 6 hours of receiving transfusion. Evidence of fluid overload is often present in a chest x-ray. Although our patient received massive blood transfusion, he has no evidence of circulatory overload. A hyperdynamic circulation can lead to increased mean pulmonary artery pressures in the presence of a normal PVR. Elevated mean pulmonary artery pressures can occur for various reasons in cirrhosis with portal hypertension and should be cautiously interpreted. References

1.

Badesch DB, Champion HC, Sanchez MA, et al. Diagnosis and assessment of pulmonary arterial hypertension. J Am Coll Cardiol. 2009;54:S55.

2.

Mandell MS, Groves BM. Pulmonary hypertension in chronic liver disease. Clin Chest Med. 1996;17(1):17.

3.

Rodríquez-Roisin R, Krowka MJ, Hervé P, Fallon MB. Highlights of the ERS Task Force on

pulmonary-hepatic vascular disorders (PHD). J Hepatol. 2005;42(6):924-927.

4.

Zardi EM, Abbate A, Zardi DM, et al. Cirrhotic cardiomyopathy. Am Coll Cardiol. 2010;56(7):539.

5.

Chayanupatkul M, Liangpunsakul S. Cirrhotic cardiomyopathy: review of pathophysiology and treatment. Hepatol Int. 2014;8(3):308-315.

5. Correct Answer: D Rationale: The University of Wisconsin solution is the standard criterion static cold preservation for the procurement of liver, kidney, pancreas, and intestine. Other preservation solutions include HTK, IGL-1, and Celsior. University of Wisconsin solution is a potassium-rich (125 mmol/L), sodium-depleted, osmotically active fluid, with ion composition comparable with the intracellular milieu. Potential disadvantages of using University of Wisconsin solution include: a. b. c.

Its high viscosity that may impede flushing of organs Potential for hyperkalemic cardiac arrest given its high-potassium content Increased incidence of ischemic-type biliary complications

HTK solution is cheaper, has low viscosity, and a low-potassium content (9 mmol/L). Celsior is another cold storage solution, which has been studied as an alternative to University of Wisconsin solution. Celsior has less viscosity and greater buffering potential for acidosis than University of Wisconsin solution. Celsior solution has high-sodium and low-potassium content (15 mmol/L), with impermeants lactobionate and mannitol which limit cellular edema. IGL-1 is a new preservation solution, with a composition resembling that of UW with inversed potassium/sodium concentrations and hydroxyethyl starch substituted with polyethylene glycol. It has 30 mmol/L of potassium. University of Wisconsin solution is a potassium-rich, sodium-depleted, osmotically active fluid and can potentially precipitate hyperkalemic cardiac arrest during reperfusion in patients undergoing liver transplantation. References

1.

Kalayoglu M, Sollinger HW, Stratta RJ, et al. Extended preservation of the liver for clinical transplantation. Lancet. 1988;1:
617-619.

2.

Voigt MR, Delario GT. Perspectives on abdominal organ preservation solutions: a comparative literature review. Prog Transplant. 2013;23:383-391.

3.

Schneeberger S, Biebl M, Steurer W, et al. A prospective randomized multicenter trial comparing histidine–tryptophane–ketoglutarate versus University of Wisconsin perfusion solution in clinical pancreas transplantation. Transpl Int. 2009;22:217-224.

4.

Karam G, Compagnon P, Hourmant M, et al. A single solution for multiple organ procurement and preservation. Transpl Int. 2005;18:657-663.

6. Correct Answer: C Rationale: CMV can lead to either active CMV infection or CMV disease in transplant patients. Active CMV infection is defined as detection of CMV replication in the blood regardless of

whether signs or symptoms are present. Tissue-invasive CMV disease is defined as the demonstration of CMV in tissue biopsy specimens by histopathology in the presence of clinical symptoms and signs of end-organ disease (enteritis, colitis, hepatitis, nephritis, pneumonitis, meningitis, encephalitis, and retinitis). CMV disease can lead to allograft loss and mortality. Kidney transplant patients commonly receive CMV prophylaxis with oral valganciclovir for 3 to 6 months after transplantation. Oral valganciclovir has great bioavailability unlike oral ganciclovir. Oral ganciclovir is not available in the United States. Patients with life-threatening CMV disease, high viral loads, or moderate to severe gastrointestinal disease are preferably treated with intravenous ganciclovir. While reduction of immunosuppression is reasonable in patients with CMV disease, it does increase the risk of rejection. Commonly, the antimetabolite mycophenolate is stopped during treatment of CMV disease while tacrolimus is usually continued. Patients with mild CMV disease are often treated with oral valganciclovir. Patients with ganciclovir-resistant CMV may require intravenous foscarnet or cidofovir. These drugs are intensely nephrotoxic and have a worse side-effect profile as compared to ganciclovir. Life-threatening CMV disease with high viral loads or moderate to severe gastrointestinal disease should be treated with intravenous ganciclovir. References

1.

Razonable RR, Humar A. Cytomegalovirus in solid organ transplantation. Am J Transplant. 2013;13(S4):93-106.

2.

Beam E, Razonable RR. Cytomegalovirus in solid organ transplantation: epidemiology, prevention, and treatment. Curr Infect Dis Rep. 2012;14:633-641.

7. Correct Answer: D Rationale: Ganciclovir and valganciclovir may lead to bone marrow suppression and leukopenia. Caution is advised if the absolute neutrophil count is under 500 cells/µL or when the platelet count is under 25,000/µL. CMV disease in itself can suppress bone marrow production, but antiviral therapy with ganciclovir typically results in improvement of hematologic parameters. Worsening or unchanged leukopenia during ongoing anti-CMV treatment with ganciclovir necessitates use of hematopoietic growth factors such as granulocyte colony-stimulating factor (G-CSF) or filgrastim to stabilize neutrophil counts. Growth factors also reduce the risk of bacterial infections which may be secondary to improved neutrophil chemotaxis and phagocytosis. Hematopoietic growth factors such as G-CSF or granulocyte-macrophage colony-stimulating factor, GM-CSF, are widely used to counter the effects of myelosuppressive drugs. References

1.

Markham A, Faulds D. Ganciclovir. An update of its therapeutic use in cytomegalovirus infection. Drugs. 1994;48(3):455.

2.

Kotton CN, Kumar D, Caliendo AM, et al. Updated international consensus guidelines on the management of cytomegalovirus in solid-organ transplantation. Transplantation. 2013;96:333.

8. Correct Answer: A Rationale: Vasoplegic syndrome during liver transplantation is potentially lethal. It is characterized by

hypotension, normal or elevated cardiac index, decreased systemic vascular resistance, and an attenuated response to vasoactive medications. The incidence of vasoplegic syndrome in end-stage liver disease is unknown, but its etiology is most likely multifactorial. Vasodilation of the splanchnic circulation in liver failure reduces systemic vascular resistance and contributes to a hyperdynamic cardiovascular profile. Patients with liver failure also have a deficiency of endogenous vasopressin when compared to healthy subjects. Vasopressin is a potent vasoconstrictor that binds to receptors on vascular smooth muscle and can be added to increase MAP. Abnormal nitric oxide (NO) metabolism also has been shown to play a prominent role in vasoplegic syndrome. Methylene blue has gained widespread use for the treatment of vasoplegic syndrome in both cardiac surgery and liver transplant patients because of its actions as an NO synthase inhibitor and guanylate cyclase inhibitor. Hydroxocobalamin (Vitamin B12a) is used for the treatment of acute cyanide toxicity and often leads to hypertension. The exact mechanism is not well defined, but it is believed it may act as a scavenger of NO. Utility of hydroxocobalamin for vasoplegia has been demonstrated in patients undergoing cardiac surgery and liver transplantation. Angiotensin I has minimal hemodynamic effects. Angiotensin-converting enzyme converts angiotensin I to angiotensin II. The latter is a potent vasoconstrictor. Hydroxocobalamin may be a suitable alternative treatment of vasoplegic syndrome when methylene blue is ineffective or contraindicated. References

1.

Roderique JD, VanDyck K, Holman B, et al. The use of high-dose hydroxocobalamin for vasoplegicsyndrome. Ann Thorac Surg. 2014;97:1785-1786.

2.

Burnes ML, Boetthcer BT, Woehlck HJ, et al. Hydroxocobalamin as a rescue treatment for refractory vasoplegic syndrome after prolonged cardiopulmonary bypass. J Cardiothorac Vasc Anesth. 2017;31:1012-1014.

3.

Boettcher BT, Woehlck HJ, Reck SE, et al. Treatment of vasoplegic syndrome with intravenous hydroxocobalamin during liver transplantation. J Cardiothorac Vasc Anesth. 2017;31:13811384.

9. Correct Answer: D Rationale: The differential diagnosis of a decline in allograft function after initial recovery includes infection, humoral rejection, airway complications (including stenosis at the bronchial anastomosis, bronchomalacia, and granulation tissue), chronic rejection (bronchiolitis obliterans syndrome), thromboembolism, and recurrent primary disease. Anastomotic pulmonary arterial stenosis (PAS) is a rare complication with a reported incidence of less than 2%. A mild degree of stenosis at the arterial anastomosis without hemodynamic significance is a normal finding after transplantation secondary to donor-receptor size discordance or secondary to the suture technique. Pulmonary venous stenosis (PVS) is a rare complication that occurs usually in early postoperative period (first 48 hours) after lung transplantation. If left untreated PVS may lead to venous thrombosis and transplant failure. Thrombus formation at the pulmonary venous/left atrial anastomotic suture line carries the risk of systemic embolization and cerebrovascular accident. The incidence of venous thrombosis is about 15%. The inferior pulmonary veins and particularly the left lower pulmonary vein are most commonly involved secondary to their anatomical position and predisposition to suture stenosis. Complications of pulmonary arterial and venous anastomoses are less frequently seen than airway anastomotic complications. Pulmonary venous vascular complications including stenosis and

thrombosis typically occur in early postoperative period. References

1.

Machuzak M, Santacruz JF, Gildea T, Murthy SC. Airway complications after lung transplantation. Thorac Surg Clin. 2015;25:55.

2.

Anaya-Ayala JE, Loebe M, Davies MG. Endovascular management of early lung transplantrelated anastomotic pulmonary artery stenosis. J Vasc Interv Radiol. 2015;26(6):878-882.

10. Correct Answer: A Rationale: The clinical syndrome of reversible posterior leukoencephalopathy syndrome (RPLS) is characterized by headaches, altered consciousness, visual disturbances, and seizures. Although the pathogenesis of RPLS has not been completely elucidated, acute hypertension is often the precipitating event. Acute hypertension leads to disordered cerebral autoregulation and endothelial dysfunction resulting in vasogenic edema. Drugs that have commonly been implicated include cyclosporine, tacrolimus, sirolimus, cisplatin, and bevacizumab. Tacrolimus is a potent calcineurin inhibitor currently used for prophylaxis and treatment of allograft rejection. Tacrolimus metabolism occurs via the cytochrome P450-3A4 (CYP3A4) and coadministration of drugs that inhibit this enzyme lead to increased plasma levels of tacrolimus. Such drugs include protease inhibitors, diltiazem, and most proton pump inhibitors with the exception of pantoprazole. Transplant recipients are instructed to avoid grapefruit juice as it is a potent inhibitor of CYP3A4 enzyme. Tacrolimus often causes renal artery vasoconstriction leading to hypertension. Diltiazem is commonly employed to treat hypertension in this setting as it reverses the renal vasoconstriction and allows for a lower dose of tacrolimus for immunosuppression. Trimethoprim-sulfamethoxazole does not have significant interactions with tacrolimus, diltiazem, pantoprazole, and grapefruit juice. Tacrolimus has multiple drug interactions. Increased plasma levels have been associated with RPLS. References

1.

Singh N, Bonham A, Fukui M. Immunosuppressive associated leukoencephalopathy in organ transplant recipients. Transplantation. 2000;69(4):467-472.

2.

Li XQ, Andersson TB, Ahlstrom M, et al. Comparison of inhibitory effects of the proton pumpinhibiting drugs Omeprazole, Esomeprazole, Lansoprazole, Pantoprazole, and Rabeprazole on human cytochrome P450 activities. Drug Metab Dispos. 2004;32:821-827.

3.

Kothari J, Nash M, Zaltzman J, et al. Diltiazem use in tacrolimus-treated renal transplant recipients. J Clin Pharm Ther. 2004;29(5):425-430.

4.

Peynaud D, Charpiat B, Vial T, et al. Tacrolimus severe overdosage after intake of masked grapefruit in orange marmalade. Eur J Clin Pharmacol. 2007;63:721-722.

5.

Liu C, Shang YF, Zhang XF, et al. Co-administration of grapefruit juice increases bioavailability of tacrolimus in liver transplant patients: a prospective study. Eur J Clin Pharmacol. 2009;65(9):881-885.

Pharmacology and Toxicology 105.Basic Pharmacologic Principles 615​ 106.Adverse Effects of Drugs 620​ 107.Toxins and Poisoning 625​ 108.Drug Overdoses 635​ 109.Metabolism and Drug Interactions 640​ 110.Toxicology and Drugs of Addiction 646​ 111.Psychoactive Medications 654​

C H AP T E R 1 0 5

Basic Pharmacologic Principles Sarah Welch and Avneep Aggarwal

1. A 52-year-old woman is admitted to the ICU, who is s/p orthotopic liver transplant 2 weeks ago, for septic shock requiring vasopressor support. Pertinent past medical history includes an unprovoked pulmonary embolism 3 months ago for which she takes apixaban 5 mg twice daily at home. Review of systems reveals normal renal function, active bowel sounds with two recent bowel movements, and a Glasgow Coma Score of 15. In the presence of distributive shock requiring vasopressor support, with regard to absorption, which scheduled medications would you be most concerned about?

A. Subcutaneous enoxaparin B. Sublingual tacrolimus C. Intravenous hydrocortisone D. Oral mycophenolate

2. A 49-year-old man is admitted to the ICU for hemodynamic instability and ventilator weaning following surgery for a bowel perforation. His past medical history includes Crohn disease for which he takes adalimumab 40 mg once weekly. He was hospitalized 2 weeks ago for medical management of a bowel obstruction. He is started empirically on piperacillin/tazobactam and vancomycin, and 5 µg/min of norepinephrine to maintain a mean arterial pressure of at least 65 mm Hg. Over next 24 hours, the patient decompensates and is now requiring 37 µg/min of norepinephrine and 0.03 units/min of vasopressin to maintain a mean arterial pressure of 65 mm Hg, despite fluid resuscitation with 3 L of lactated ringers. He is currently anuric, is not responsive to fluids, and appears volume overloaded. Given his risk factors for multidrug-resistant organisms, his antibiotic regimen is broadened to meropenem,

amikacin, daptomycin, and micafungin. When determining dose of amikacin for this patient, what considerations should be made?

A. Decrease the dose because of diminished renal clearance B. Increase the dosing frequency because of the increased volume of distribution C. Increase the dose because of the increased volume of distribution D. Decrease the dose because of decreased protein binding

3. Which pathophysiological change seen in critically ill patients most frequently makes medication dosage adjustment necessary?

A. Decreased plasma protein binding B. Decreased renal clearance C. Diminished GI or subcutaneous perfusion D. Inhibition of hepatic enzymes

4. Fentanyl, a drug with a high extraction ratio, will be most influenced by which critical illness–related metabolic abnormality?

A. Decreased hepatic blood flow B. Decreased intrinsic clearance C. Decreased protein binding D. Decreased functional hepatocytes

5. Phenytoin, a drug with a low extraction ratio, will be most influenced by which critical illness–related metabolic abnormality?

A. Decreased hepatic blood flow

B. Decreased intrinsic clearance C. Decreased protein binding D. Decreased acetylation

6. A 76-year-old woman is admitted to the ICU for atrial fibrillation with rapid ventricular rate after an exploratory laparotomy. You elect to use Drug A for rate control. The oral formulation of Drug A undergoes significant first-pass metabolism. With this knowledge, what dosage adjustment should be made to the intravenous form of Drug A?

A. Increase the dose B. Decrease the dose C. Make no dosage adjustment D. Increase the dosing interval

7. A patient is admitted to the cardiac ICU after suffering a cardiac arrest. She is nonresponsive and therapeutic hypothermia is initiated. In addition to cooling, she receives a midazolam infusion for sedation, intermittent hydromorphone boluses for pain, and an atracurium infusion for shivering. What is your concern with this patient’s current medication regimen?

A. Therapeutic hypothermia may affect CYP450 activity decreasing the metabolism of midazolam and therefore prolong its sedative affect B. Therapeutic hypothermia may affect CYP450 activity decreasing the metabolism of atracurium and therefore prolong its neuromuscular blocking affects C. Therapeutic hypothermia may affect Hofmann elimination decreasing the metabolism of hydromorphone and therefore prolong its sedative affect D. Therapeutic hypothermia may affect Hofmann elimination

decreasing the metabolism of midazolam and therefore prolong its sedative affect

8. A 43-year-old woman is admitted to the ICU with communityacquired pneumonia. She is mechanically intubated and requires vasopressor support. Pertinent past medical history includes seizure disorder for which she takes phenytoin 100 mg by mouth three times daily. On day 3 in the ICU, patient remains intubated, is off vasopressors, and enteral tube feeding is started. Pertinent lab values on day 3 are as follows: ALT 154 U/L, AST 95 U/L, albumin 1.5 g/dL, SCr 2.3 mg/dL. On day 5 in the ICU patient suffers a seizure. What is the most likely cause of her seizure?

A. Kidney dysfunction B. Enteral tube feeds C. Elevated liver enzymes D. Hypoalbuminemia

9. On ICU day 3, the patient grows a multidrug-resistant Klebsiella pneumoniae from her bronchoscopy culture. She is initiated on a recently approved drug to treat carbapenem-resistant enterobacteracieae. She is given a reduced dose based on a calculated creatinine clearance of less than 20 mL/min. On ICU day 4, she is started on continuous veno-venous hemodialysis (CVVHD). You cannot find any dosing recommendations for CVVHD; however, you’re able to find that this new drug is 60% renally eliminated as unchanged drug, volume of distribution is 3 L/kg, its ∼25% bound to protein, and has a small molecular weight. What change should you make to the dosing regimen?

A. Increase the dose because of low protein binding and small molecular weight B. Increase the dose because of large volume of distribution

C. Maintain current dose because of low protein binding and small molecular weight D. Maintain current dose because of large volume of distribution

10. A 23-year-old man is admitted to the ICU for management of salicylate toxicity. His arterial blood gas on arrival shows the following: pH 7.54, PaCO2 22 mm Hg, PaO2 93 mm Hg, HCO3 – 18 mEq/L, oxygen saturation 90%. Sodium bicarbonate 100 mEq IV push is administered followed by a continuous infusion of sodium bicarbonate 150 mEq/L at 250 mL/h. Hypokalemia is corrected as appropriate. What physiochemical property of aspirin counters or supports sodium bicarbonate therapy?

A. Sodium bicarbonate therapy is contraindicated because of a pH of 7.54 B. In cases of salicylate toxicity, metabolic acidosis follows respiratory alkalosis. Sodium bicarbonate therapy can prevent severe acidemia C. Salicylic acid is a weak acid. Sodium bicarbonate will alkalinize the urine, enhancing renal tubular excretion D. Salicylic acid is a weak acid; therefore sodium bicarbonate therapy is contraindicated as urine alkalization will enhance renal tubular reabsorption

Chapter 105 Answers 1. Correct Answer: A Rationale: Critically ill patients receiving vasopressors may have reduced subcutaneous absorption of drugs. Presence of shock and/or use of vasopressors decrease peripheral tissue perfusion, resulting in impaired subcutaneous absorption. Studies evaluating low-molecular-weight heparins demonstrate lower antifactor Xa levels in critically ill patients receiving vasopressors when compared with critically ill patients not on vasopressors or those who are not critically ill. Sublingual absorption is not impaired in states of critical illness, and intravenous therapy always results in 100% bioavailability. Although blood is shunted to vital organs in states of hypotension and shock, the impact of splanchnic perfusion on drug absorption has not been well studied. References

1. Joachleberger S, Mayr V, Luckner G, et al. Anitfactor Xa activity in critically ill patients receiving antithrombotic prophylaxis with standard dosages of certoparin: a prospective, clinical study. Crit Care. 2005;9(5):R541-R548.

2. Dorffler-Melly J, de Jonge E, Pont AC, et al. Bioavailability of subcutaneous low-molecular weight heparin to patients on vasopressors. Lancet. 2002;359(9309):849-850.

2. Correct Answer: C Rationale: Hydrophilic molecules, such as amikacin, generally remain in the plasma water volume. Capillary leak due to septic shock and exogenous volume administration will contribute to an increase in the volume of distribution. Studies evaluating aminoglycosides in states of critical illness report increases in the volume of distribution anywhere from 25% to 50%. Additionally, aminoglycosides are concentration-dependent agents that are

most effective at concentrations 10 times the minimal inhibitory concentration of the offending pathogen. Therefore, the pharmacokinetic properties of the drug should lead the clinician to administering an increased dose. Although clearance of aminoglycosides will be decreased with impaired renal function, the initial dose should not be decreased because of the increased volume of distribution. Increasing the dosing frequency will increase the time above the minimal inhibitory concentration, but not result in target peak concentrations for some time. Amikacin is not protein bound, so will not be affected by decreased protein. References

1. Taccone FS, Laterre PF, Spapen H, et al. Revisiting the loading dose of amikacin for patients with severe sepsis and septic shock. Crit Care. 2010;14:R53.

2. Rhonda R, CapitanoB, BiesR, et al. Suboptimal aminoglycoside dosing in critically ill patients. Ther Drug Monit. 2008;30:674-681.

3. Correct Answer: B Rationale: Medications that undergo renal elimination generally have dosing recommendations based on GFR (glomerular filtration rate) estimated using the Cockroft-Gault formula. For renally eliminated medications, modifying dose regimens is essential in patients with acute or chronic kidney injury to prevent adverse drug events. Conversely, conditions such as sepsis, trauma, surgery, burns, and use of vasopressors can lead to an increase in renal blood flow, resulting in increased renal drug clearance. No empiric changes for augmented renal clearance are recommended by drug manufacturers. Although a decrease in plasma proteins can increase the free concentration of highly protein bound drugs, there are no empiric dosing changes that are recommended based on the degree of protein decrease. Similarly, although both ​enteral and subcutaneous absorption of medications is diminished in states of critical illness, no dosing adjustments exist for any medication empirically. Finally, there is no biomarker readily available in the clinical setting that can be used to

determine the degree of liver impairment for the purposes of drug dosing. Although the Child-Pugh classification uses patient-specific data to assess severity of hepatic disease, its purpose was to predict mortality. Although some drug manufacturers include dosing adjustment recommendations based on Child-Pugh score, such recommendations are neither available for many drugs nor are they validated in critically ill patients. References

1. Cantu TG, Ellerbeck EF, Yun SW, et al. Drug prescribing for patients with changing renal function. Am J Hosp Pharm. 1992;49(12):2944-4948.

2. Brown R, Babcock R, Talbert J, et al. Renal function in critically ill postoperative patients: sequential assessment of creatinine osmolar and free water clearance. Crit Care Med. 1980;8(2):68-72.

3. Marin C, Eon B, Saux P, et al. Renal effects of norepinephrine used to treat septic shock patients. Crit Care Med. 1990;18(3):282-285.

4. Pugh RN, Murray-Lyon IM, Dawson JL, et al. Transection of the esophagus for bleeding esophageal varices. Br J Surg. 1973;60(8):646649.

4. Correct Answer: A Rationale: The hepatic extraction ratio is the fraction of drug that is removed from the blood after one pass through the liver. Hepatic clearance of drugs with high extraction ratio (>0.7) primarily depends on liver blood flow and is less affected by changes in liver function (ie, intrinsic clearance or function hepatocytes). Conversely, hepatic clearance of drugs with low extraction ratio (80%) will have small plasma concentrations of unbound drug available for removal. Finally, a drug with a large volume of distribution is distributed widely through the

tissue with low amounts of drug available in the plasma for removal. With these properties considered, answer A is the most appropriate. Reference

1. Choi G, Gomersall CD, Tian Q, et al. Principles of antibacterial dosing in continuous renal replacement therapy. Crit Care Med. 2009;37:22682282.

10. Correct Answer: C Rationale: Urine alkalization will increase the reabsorption of basic drugs by making the drug nonionized and can enhance the elimination of acidic drugs by making the drug ionized. Data suggest that raising the urine pH 7.5 to 8 through the use of sodium bicarbonate enhances elimination of salicylates. An elevated pH is not a contraindication to use. In cases of salicylate toxicity, patients presenting in the early phase after ingestion generally have respiratory alkalosis due to direct stimulation of the respiratory center. As the absorption of the drug continues, an anion gap metabolic acidosis ensues. In this scenario sodium bicarbonate therapy is not used to prevent the development of a metabolic acidosis, but rather is used as a treatment to deprotonate the molecule, which both decreases concentration in the central nervous system and enhances excretion through renal tubular excretion. References

1. Proudfoot AT, Krenzelok EP, Brent J, et al. Does urine alkalization increase salicylate elimination? If so, why? Toxicol Rev. 2003;22(3):129136.

2. El-Sheikh AA, Masereeuw R, Russel FG. Mechanisms of renal anionic drug transport. Eur J Pharmacol. 2008;585(2-3):245-255.

3. Gabow PA, Anderson RJ, Potts DE, et al. Acid-base disturbances in the salicylate-intoxicated adult. Arch Intern Med. 1978;138(10):1481.

C H AP T E R 1 0 6

Adverse Effects of Drugs Anoop Chhina and Avneep Aggarwal

1. A 55-year-old woman undergoes elective ventral hernia repair. Past medical history is significant for hypertension and asthma. She had uneventful induction of anesthesia. Five minutes after cefazolin was started for perioperative prophylaxis, the patient becomes hypotensive and progressively hypoxic with high peak airway pressures. Diphenhydramine, steroids, a H2-blocker, bronchodilators, and epinephrine are administered, with clinical improvement. A decision is made to postpone surgery, and she is transferred to the intensive care unit (ICU) for further management. Which of the following laboratory levels can help determine whether the episode was related to anaphylaxis as compared to asthma exacerbation?

A. Pseudocholinesterase B. Tryptase C. Lipase D. Amylase

2. A 33-year-old woman is admitted to ICU s/p motor vehicle accident with traumatic brain injury, and CT scan of the head showed subdural hematoma without midline shift and bilateral frontal contusions. On day 2 of admission, she starts having seizures. She is given 1000 mg phenytoin and started on a maintenance dose. She remains in the ICU due to waxing and waning mental status. On day 5 as you are examining the patient, red-purple macules and papules are noticed on chest and abdomen, as well as on the bilateral upper and lower extremities. Similar lesions are also seen in mouth and genital area. Which of the following will help to differentiate Stevens-Johnson syndrome (SJS) from toxic epidermal necrolysis

(TEN)?

A. Presence of Nikolsky sign in SJS as compared to TEN B. Involvement of more than two mucosal surfaces in TEN as compared to two or less in SJS C. They are spectrum of same disease process; SJS is defined as affecting less than 10% body surface area (BSA), and TEN affects more than 30% BSA D. SJS occurs within 1 week of triggering factor, while TEN occurs more than 4 weeks after triggering agent

3. A 48-year-old woman with a history of poorly controlled hypertension, coronary artery disease, and chronic renal failure is admitted to ICU after presenting to emergency room (ER) with hypertensive emergency. Her blood pressure was controlled with sodium nitroprusside. After 32 hours of treatment, the patient develops agitation, confusion, and metabolic acidosis. Which of the following is used for the prevention/treatment of this adverse effect of nitroprusside?

A. Thiocyanate B. Cyanocobalamin C. Thiosulfate D. Methylene blue

4. A 28-year-old woman presents to the emergency department with sudden onset of generalized fatigue, fever, chills, and blurry vision. She was recently diagnosed with urinary tract infection and started on Bactrim 2 days ago. On physical examination, her skin is mildly jaundiced. She has multiple purpura over her extremities. Laboratory tests reveal: Hemoglobin 8.0 g/dL, platelet count 57/mm3, and creatinine of 2.8 mg/dL. Of note, renal function was normal 4 days ago. On peripheral smear, multiple schistocytes are present

(∼2%). She is admitted to ICU due to metabolic disarray and for additional workup. She reports no history of illicit drug use, recent diarrhea, or no other significant past medical history. Vitals are heart rate 90 beats/min, blood pressure 130/74 mm Hg, temperature 38.7°C, and respiratory rate 18 breaths/min. Serum lactate dehydrogenase (LDH) is elevated. Serum ADAMTS13 level was send from emergency department, showed a mild reduction. Based on these findings what is most probable diagnosis?

A. Immune thrombocytopenic purpura B. Thrombotic thrombocytopenic purpura C. Drug-induced thrombotic microangiopathy (DITMA) D. Disseminated intravascular coagulation

5. A 58-year-old woman is admitted to hospital with fever, productive cough, and shortness of breath. Chest x-ray is consistent with right lower lobe consolidation. She was treated as an outpatient for community-acquired pneumonia with oral ciprofloxacin, without improvement in symptoms. Medical history is significant for hypertension, gastroesophageal reflux disease (GERD), bipolar disorder, and depression. Her medications include carvedilol, omeprazole, aripiprazole, and amitriptyline. An ECG at time of admission shows normal sinus rhythm with prolonged QT interval.

Which of the following medications should be discontinued FIRST based on patient’s ECG findings? A. Aripiprazole B. Carvedilol C. Omeprazole D. Amitriptyline

6. A 48-year-old woman was recently admitted for pyelonephritis and

discharged home on trimethoprim-sulfamethoxazole based on urinary culture results. She returns to the ER 5 days later with fever, nausea, and vomiting. Right-sided hydronephrosis was seen on renal ultrasound and the patient was admitted to ICU for presumed urosepsis and postureteric stent placement. On examination, widespread morbilliform itchy rash was noted all over the body. On physical examination, the patient is found to be febrile. Her mucous membranes are normal. She has 3 cm lymph nodes in the anterior cervical and axillary regions and a liver edge palpable 4 cm below the costal margin.

Some facial swelling is also noted, but there was no difficulty in breathing. Medical history is otherwise unremarkable, and she was taking no other medications at home. Which of the following is the most appropriate next step in diagnosis of this patient? A. Skin biopsy B. No further testing C. Lymph node biopsy D. Complete blood count and liver chemistry

7. A 24-year-old man with no past medical history was scheduled for elective inguinal hernia repair under general anesthesia. Intraoperatively he becomes increasingly tachycardic and hyperthermic, and blood gas demonstrates severe metabolic acidosis. He is treated for malignant hyperthermia (MH) in the OR with good response and post procedure, is transferred to the ICU for close monitoring. Which of the following anesthetic agents used is most likely to trigger MH?

A. Propofol B. Rocuronium C. Midazolam

D. Succinylcholine

8. A 78-year-old woman with past medical history of hypertension, coronary artery disease, and stage III chronic kidney disease is admitted to ICU with symptoms of lethargy, altered mental status, hypotension, and bradycardia. On further history obtained from patient’s daughter, she has been experiencing constipation and was taking over-the-counter laxatives. Over the past week, she had increased the magnesium-based laxative uptake to several times a day. On physical examination, her blood pressure is 90/62 mm of Hg, heart rate is 48 beats/min, and respiratory rate is 8 breaths/min. Her temperature is 37.4°C, and ECG shows sinus bradycardia. Her chemistries reveal a creatinine that has worsen from baseline, and her serum magnesium is 8.4 mg/dL. What is the next best step in immediate management of this patient?

A. Hold more laxative use, no treatment required B. Urgent hemodialysis C. Calcium chloride D. Potassium chloride

Chapter 106 Answers 1. Correct Answer: B Rationale: Mast cells have preformed mediators, including tryptase, which can be used to measure systemic mast cell activation. Concentrations of α-tryptase correlate with mast cell number, whereas β-tryptase concentrations are associated with acute mast cell activation. Total serum tryptase can be used to confirm a diagnosis of anaphylaxis, although samples need to be collected within 4 hours of a suspected anaphylactic reaction. β-tryptase levels are thought to peak 30 to 60 minutes after a reaction, with a half-life of 2 hours. Normal total tryptase ranges from 1 to 10 ng/mL. If baseline tryptase is >20 ng/mL in a patient without acute symptoms of anaphylaxis, indolent systemic mastocytosis should be suspected and further evaluation sought. Histamine elevation is short-lived after an anaphylactic episode; however, metabolites, such as N-methyl histamine and prostaglandins, can be measured in the urine for 24 hours after an anaphylactic event and may be useful for diagnosis. Other potentially useful biomarkers are being studied, including platelet-activating factor, bradykinin, chymase, and others. Amylase, lipase, and pseudocholinesterase are not mast cell mediators, so not related to the diagnosis of anaphylaxis. Reference

1. Makhija M. Patterson’s Allergic Diseases. Lippincott Williams & Wilkins; 2018:868-869.

2. Correct Answer: C Rationale: This patient’s presentation is consistent with SJS and TEN. SJS and TEN are severe acute inflammatory exfoliative skin reactions with unclear etiology usually triggered by medications or, less frequently, by upper respiratory infections. SJS and TEN are believed to be variants of same condition; Nikolsky sign is almost always present in both. Both TEN and SJS occur 1 to 3 weeks after exposure to inciting agent. The difference between the two is related to the percentage of BSA affected: SJS affects less than 10% and TEN affects greater than 30%, while a range of 10% to 30% is referred to as SJS/TEN overlap.

Pharmacologic triggers can be divided into drugs administered for shorter durations (eg, antibiotics such as trimethoprim/sulfamethoxazole, sulfonamides, cephalosporins, quinolones, and aminopenicillins) and medications administered chronically (eg, carbamazepine, oxicam nonsteroidal anti-inflammatory drugs, phenytoin, phenobarbital, allopurinol, and valproic acid). Treatment consists of immediate discontinuation of the triggering agent and early transfer to a burn unit, which significantly reduces morbidity and mortality. Recent trials of immunosuppressive therapy, steroids, and immunoglobulins have not shown improvement in outcome. Application of silver nitrate may lead to cross-reactivity with antibiotics and is not recommended. Diagnosis is usually clinical and biopsy of affected areas is not required. References

1. Milliszewski MA, Kirchhof MG, Sikora S, Papp A, Dutz JP. StevensJohnson syndrome and toxic epidermal necrolysis: an analysis of triggers in implications for improving prevention. Am J Med. 2016;129(11):1221-1225.

2. Gerull R, Nelle M, Schaible T. Toxic epidermal necrolysis and StevensJohnson syndrome: a review. Crit Care Med. 2011;39(6):1521-1532.

3. Correct Answer: C Rationale: The signs and symptoms of the patient are consistent with cyanide toxicity from nitroprusside. Nitroprusside causes toxicity through release of cyanide and accumulation of thiocyanate. Most common symptoms of cyanide toxicity are changes in mental status, including convulsions, encephalopathy, coma, and even unexplained cardiac arrest. Metabolic acidosis can be present, though this may be a late event. Cyanide reacts in high affinity with metals such as ferric iron (Fe3+) and cobalt and also binds to numerous critical enzyme systems in the body. Cyanide inhibits oxidative phosphorylation and thereby causes central nervous system and cardiovascular dysfunction due to cellular hypoxia. It does this by primarily binding to and inactivating the enzyme cytochrome oxidase (cytochrome a3). Risk of cyanide toxicity can be decreased by utilizing nitroprusside at

recommended doses for short period of time. Thiosulfate is a specific antidote for cyanide toxicity associated with nitroprusside infusion. Sodium thiosulfate removes cyanide from the blood through the action of the enzyme rhodanese. It has also been recommended that thiosulfate infusions be used for patients receiving high doses of nitroprusside. Cyanocobalamin is not effective as an antidote and is not capable of preventing cyanide toxicity. Hydroxocobalamin, a precursor of vitamin B12, may also be used to treat cyanide toxicity. It contains a cobalt moiety that binds intracellular cyanide forming cyanocobalamin. Nitroprusside can cause a dose-dependent conversion of hemoglobin to methemoglobin. Methylene blue is used for treating methemoglobinemia, and not cyanide toxicity. References

1. Hall AH, Dart R, Bogdan D. Sodium thiosulfate or hydroxocobalamin for the empiric treatment of cyanide poisoning? Ann Emerg Med. 2007;49(6):806-813.

2. Curry SC. Sodium nitroprusside. In: Brent H, et al. Critical Care Toxicology. Philadelphia, PA: Mosby; 2005:843-850.

4. Correct Answer: C Rationale: Thrombotic microangiopathy is characterized by platelet microthrombi in small vessels leading to thrombocytopenia and microangiopathic hemolytic anemia. Microthrombi may also lead to systemic effects like acute kidney injury, neurologic abnormalities, and cardiac ischemia. In this patient, the presence of schistocytes (>1%) with hemolytic anemia (anemia, jaundice, elevated LDH) is suggestive of microangiopathic hemolytic anemia. This, in addition to thrombocytopenia and the systemic symptoms, supports a diagnosis of thrombotic microangiopathy. Schistocytes are not typical of disseminated intravascular coagulation and even, if present, are usually 1%) suggests thrombotic microangiopathy. Important causes of thrombotic microangiopathy include thrombotic thrombocytopenic purpura, hemolytic uremic syndrome, complementmediated thrombotic microangiopathy, and DITMA. Thrombotic thrombocytopenic purpura is associated with a lesser degree of renal

involvement (or no renal involvement), and the levels of ADAMTS13 (when available) are very low (0.7 hypotension, and/or arrhythmia events occur. Other ECG findings include QTc >415 to 430, wide/deep S in leads I and aVL, and acute right axis deviation. QRS >100 is predictive of seizure, whereas QRS >160 is predictive of ventricular arrhythmias. Although there are classic ECG findings as described above, sinus tachycardia is the most common ECG finding. If sodium bicarbonate is given, one must target a sodium concentration of 150 to 155 and pH of 7.5 to 7.55 and closely monitor for hypokalemia. Additionally, hyperventilation can aid in alkalizing the serum pH. Sodium bicarbonate displaces the TCA molecule from the cardiac myocytes as well as leading to decreased free TCA molecules in serum by increasing protein binding capacity. Large volumes of sodium bicarbonate or continuous infusions are

necessary at times to reverse the cardiotoxicity. Magnesium sulfate can be helpful in refractory arrhythmias with associated long QTc. Benzodiazepines are used to treat seizures in TCA overdose as compared with phenytoin, which should be avoided secondary to its proarrhythmic effects in setting of TCA overdose. Ammonium chloride acidifies the serum leading to more deleterious effects from TCAs owing to proarrhythmic nature in an acidic environment. Lidocaine is considered an adjunctive treatment of arrhythmias if the patient remains hemodynamically unstable despite initial treatment with bicarbonate. In theory, physostigmine would be appropriate for anticholinergic syndrome, but multiple studies have recommended to exclude its use in TCA overdose. In patients with suspected local anesthetic toxicity and continued circulatory collapse despite fluids, intralipid infusion should be considered, as the infusion will scavenge fat-soluble agents and slow the vasodilation. References

1. Verbee FC. Tricyclic antidepressant poisoning: cardiovascular and neurological toxicity. Neth J Crit Care. 2016;24(2):16-19.

2. Body R, Bartram T, Azam F, Mackway-Jones K. Guidelines in Emergency Medicine Network (GEMNet): guideline for the management of tricyclic antidepressant overdose. Emerg Med J. 2011;28(4):347-368.

3. Ward C, Sair M. Oral poisoning: an update. Continuing Education in Anesthesia Critical Care & Pain. 2010;10(1):6-11.

4. Goldstein JN, Dudzinski DM, Erickson TB, Linder G. Case 12-2018: A 30year-old woman with cardiac arrest. N Engl J Med. 2018;378(16):15381549.

C H AP T E R 1 0 9

Metabolism and Drug Interactions Jeremy T. Rainey and Avneep Aggarwal

1. Which of the following correctly pairs the pharmacodynamics with the antibacterial agents?

A. Oxacillin (time dependent), moxifloxacin (concentration dependent) B. Vancomycin (concentration dependent), cefazolin (time dependent) C. Daptomycin (time dependent), amikacin (concentration dependent) D. Levofloxacin (time dependent), meropenem (time dependent)

2. A 49-year-old male with a past medical history significant for hypertension and end-stage renal disease on hemodialysis is admitted to the ICU in septic shock from pneumonia. He is intubated and mechanically ventilated, and requires vasopressor support. Blood cultures grew multidrug–resistant Acinetobacter baumannii. You begin appropriate treatment. Over the course of several hours you begin to notice increasing vasopressor requirements and increasing peak airway pressures. Which of the following antimicrobial choices could adequately explain this scenario?

A. IV polymyxin B without renal adjustment B. Meropenem allergy C. Inhaled colistin hypersensitivity D. IV tobramycin-induced bronchospasm

3. A 54-year-old man with a past medical history of cirrhosis is in the

ICU 4 days after suffering severe burns to >40% of his body while at work. His respiratory status has been rapidly declining, and he is currently requiring BiPAP; however his P:F ratio is now 48 hours (risk can be elevated for up to 1 year after the burn), spinal cord injury >24 hours, strokes >72 hours, and patients at risk for exaggerated hyperkalemia (rhabdomyolysis, hyperkalemic patients with ECG changes). Non-depolarizing agents tend to have better safety profiles in the critically ill population. They do not cause fasciculations because of the competitive antagonism of the end plate. Rocuronium is the most commonly used alternative to succinylcholine for intubation in the critically ill because of its fairly rapid onset (approximately 60 seconds), intermediate duration, and low active metabolite production after metabolism. Rocuronium is excreted mainly through the biliary tract with minimal renal excretion. Thus, rocuronium infusion is not preferred in patients with cirrhosis. Vecuronium has a much longer onset time and has more renal excretion making it a poorer choice in the critically ill. Cisatracurium and atracurium are other non-depolarizing agents that do not undergo hepatic metabolism, nor do they have hepatic or renal clearance. Atracurium is metabolized through nonspecific plasma esterases and Hoffman elimination. Atracurium can also lead to histamine release, causing flushing, tachycardia, and hypotension, which may make it less suitable in the critically ill. Cisatracurium is metabolized exclusively via Hoffman elimination and is approximately four times as potent as atracurium. Cisatracurium has no active metabolites and does not lead to histamine release and has a faster onset than atracurium, making it a better choice than atracurium in the critically ill. References

1. Bittner EA. Clinical use of neuromuscular blocking agents in critically ill patients. In: Post T, ed. UpToDate. Waltham, Mass: UpToDate; 2018. www.uptodate.com. Accessed October 3, 2018.

2. Van Miert MM, Eastwood NB, Boyd AH, Parker CJR, Hunter JM. The pharmacokinetics and pharmacodynamics of rocuronium in patients with hepatic cirrhosis. Br J Clin Pharmacol. 1997;44(2):139-144. doi:10.1046/j.1365-2125.1997.00653.x.

3. Fan E, Brodie D, Slutsky AS. Acute respiratory distress syndromeadvances in diagnosis and treatment. JAMA. 2018;319(7):698710. doi:10.1001/jama.2017.21907.

4. Omera M, Hammad YM, Helmy AM. Rocuronium versus cisatracurium: onset of action, intubating conditions, efficacy and safety. Alexandria J Anaesth Intens Care. 2005;8:27-33.

4. Correct Answer: D Rationale: Tacrolimus is a calcineurin inhibitor that suppresses the immune system by preventing IL-2 production by T cells and is metabolized via CYP3A4. The adverse effects of the calcineurin inhibitors are primarily nephrotoxicity ranging from minimal to irreversible damage. In addition, tacrolimus may cause neurotoxicity, which can be severe enough to warrant immediate cessation of the medication. The need for frequent and specific monitoring of drug concentrations remains essential because the therapeutic dosing and pharmacokinetics of tacrolimus can demonstrate wide variability among recipients. Antifungals, such as fluconazole, and multiple classes of antibiotics, such as macrolides, are known inhibitors of of the CYP3A4 system and can lead to increased and toxic blood levels of tacrolimus if not carefully monitored. Normal-pressure hydrocephalus (NPH) is characterized by pathologically enlarged ventricular size with normal opening pressures on lumbar puncture. It classically presents with cognitive impairment, gait disturbance, and urinary incontinence. Secondary NPH can occur as a consequence of chronic meningitis or ongoing meningitis. Calcineurin inhibitor toxicity should be part of the differential diagnosis in patients presenting with new renal and neurological toxicity, especially when multiple CYP inhibitors are being used in conjunction with one another.

References

1. Paterson DL, Singh N. Interactions between tacrolimus and antimicrobial agents. Clin Infect Dis. 1997;25(6):1430-1440. doi:10.1086/516138.

2. Vanhove T, Bouwsma H, Hilbrands L, et al. Determinants of the magnitude of interaction between tacrolimus and voriconazole/posaconazole in solid organ recipients. Am J Transplant. 2017;17:2372-2380. doi:10.1111/ajt.14232.

3. Luo X, Zhu L, Cai N, Zheng L, Cheng Z. Prediction of tacrolimus metabolism and dosage requirements based on CYP3A4 phenotype and CYP3A5*3 genotype in Chinese renal transplant recipients. Acta Pharmacol Sin. 2016;37(4):555-560. doi:10.1038/aps.2015.163.

5. Correct Answer: B Rationale: The syndrome of MELAS is one of a complex group of heterogeneous multisystem disorders affecting the nervous system, which is maternally inherited and caused by mutations of mitochondrial DNA. Approximately 80% of MELAS cases are associated with an m.3243A>G mutation. Typical presenting symptoms include recurrent strokelike episodes resulting in hemiparesis, hemianopia, cortical blindness, generalized seizure activity, recurrent migraine headaches, short stature, hearing loss, muscle weakness, and cardiomyopathy. MELAS usually presents in patients 48 hours. Usually, doses at or exceeding 4 mg/kg/h are causative; however, there are case reports with smaller doses. Steroid use, vasopressors, low carbohydrate intake, poor tissue perfusion, sepsis, and cerebral injury have been shown to be associated PRIS; all of which may be seen in critical illness. The onset of PRIS is typically rapid and seen within 4 days of initiation. The mechanism of PRIS is poorly understood but may be related to direct mitochondrial respiratory chain inhibition or impaired mitochondrial fatty acid metabolism and blockage of beta-adrenoreceptors and cardiac calcium

channels. The best treatment is prevention and first-line therapy is immediate discontinuation of the infusion of propofol. Renal replacement therapy may become necessary because of metabolic acidosis, hyperkalemia, and rhabdomyolysis. Bradyarrhythmias can be managed with transthoracic or transvenous pacing and shock managed with the support of vasopressors and inotropes. Extracorporeal membrane oxygenation should be considered in cases of refractory PRIS. Ensuring adequate carbohydrate intake may also help to prevent PRIS development. References

1. Kam PC, Cardone D. Propofol infusion syndrome. Anaesthesia. 2007;62(7):690-701. doi: 10.1111/j.1365-2044.2007.05055.x.

2. Fodale V, La Monaca E. Propofol infusion syndrome: an overview of a perplexing disease. Drug Saf. 2008;31(4):293-303. doi:10.2165/00002018200831040-00003.

3. Mirrakhimov AE, Voore P, Halytskyy O, et al. Propofol infusion syndrome in adults: a clinical update. Crit Care Res Pract. 2015;2015:260385.

4. Laquay N, Prieur S, Greff B, Meyer P, Orliaguet G. Propofol infusion syndrome. Ann Fr Anesth Reanim. 2010;29(5):377-386. doi:10.1016/j.annfar.2010.02.030.

7. Correct Answer: C Rationale: Patients ingesting mushrooms, such as Amanita phalloides, typically do so incidentally when foraging for mushrooms and mistake its identity for an edible mushroom. Patients usually suffer intoxication from the amatoxin of the mushroom (most commonly alpha-amanitin, which is heat stable and insoluble in water) and endure four clinical stages. The first stage is usually observed during the first 6 to 12 hours after mushroom ingestion and is known as the latent stage, where patients usually will not experience any symptoms. Severe muscarinic symptoms might be evident during the second stage, typically with severe gastritis, nausea, vomiting,

abdominal pain, and severe diarrhea, which may be bloody or contain mucus. This second stage usually lasts between 12 to 24 hours. The third stage has been classified as a pseudo-remission period where the patient may begin to feel better, but unfortunately patients will rapidly progress to the fourth stage, which is characterized by acute liver failure with massive hepatocyte death. If not treated early, patients may go on to suffer multiorgan failure and death, which can be seen as early as 5 to 8 days after ingestion of the mushroom. Silibinin, which is a water-soluble silymarin (a flavonolignan from milk thistle), inhibits the amatoxin uptake and penetration into hepatocytes and improves cellular survival in human hepatocytes exposed to alphaamanitin. Silibinin is most effective when given within 24 to 48 hours of ingestion and is administered as an initial intravenous loading dose of 5 mg/kg followed by continuous infusion of 20 to 50 mg/kg/d for 6 to 8 days. Increased mortality or need for transplantation is seen when silibinin is administered >48 hours after ingestion. Other therapies that have been tried and abandoned include antibiotics, thioctic acids, steroids, hormones, and other antioxidants. References

1. Santi L, Maggioli C, Mastroroberto M, Tufoni M, Napoli L, Caraceni P. Acute liver failure caused by amanita phalloides poisoning. Int J Hepatol. 2012;2012:487480. doi:10.1155/2012/487480.

2. Cheung CW, Gibbons N, Johnson DW, Nicol DL. Silibinin-a promising new treatment for cancer. Anti Cancer Agents Med Chem. 2010;10:186195. doi: 10.2174/1871520611009030186.

3. Loguercio C, Festi D. Silybin and the liver: From basic research to clinical practice. World J Gastroenterol. 2011;17(18):2288-2301. doi:10.3748/wjg.v17.i18.2288.

4. Li Y, Mu M, Yuan L, Zeng B, Lin S. Challenges in the early diagnosis of patients with acute liver failure induced by amatoxin poisoning: two case reports. Medicine. 2018;97(27):e11288. doi:10.1097/MD.0000000000011288.

8. Correct Answer: D

Rationale: Typical antipsychotics exert their action by postsynaptic blockade of D2 dopamine receptors in the brain with extensive metabolism via the cytochrome P450 system, which makes this class quite dependent on adequate hepatic clearance to reduce systemic accumulation or drug-drug interactions. This class carries an increased risk for extrapyramidal side effects and tardive dyskinesia. Atypical antipsychotics also have postsynaptic blockade of brain dopamine D2 receptors but they tend to have more serotonin 5HT-2 affinity than their dopamine affinity. This primarily separates them from typical antipsychotics. This serotonin receptor affinity is also what is suggested as the mechanism behind the lower incidence of extrapyramidal side effects seen in this class. This class is metabolized through the cytochrome P450 system, but there is individual drug variability that alters the risk for accumulation and drug-drug interactions (which is beyond the specific scope of this question). Although there are clear differences in the extent of QTc prolongation between the two classes (haloperidol averages approximately 4.7 ms, quetiapine averages 14.5 ms), there does not seem to be a difference in the risk of arrhythmias, including Torsades de Pointes (TdP) between them. Patients taking either of these classes are approximately at a twofold increased risk of arrhythmias and TdP compared with patients not receiving these medications. Within the two classes, thioridazine (typical) and ziprasidone (atypical) have the highest risk of QTc prolongation and risk of TdP. Haloperidol does have a clinically significant risk; however, this is usually seen with doses of >35 mg/d. Of additional importance, QTc prolongation and TdP tend to occur when additional risk factors are present. These include age >65, preexisting cardiovascular disease, conduction disorders, brady or tachyarrythmias, female sex, electrolyte disturbances (hypokalemia, hypomagnesemia), supratherapeutic or toxic levels of accumulation, or concomitant administration of other drugs that interfere with cardiac conduction or drug metabolism. References

1. Huffman JC, Stern TA. QTc prolongation and the use of antipsychotics: a case discussion. Primary Care Companion J Clin Psychiatry.

2003;5(6):278-281.

2. Barr J, Fraser GL, Puntillo K, et al; American College of Critical Care Medicine. Clinical practice guidelines for the management of pain, agitation, and delirium in adult patients in the intensive care unit. Crit Care Med. 2013;41:263-306.

3. Ray WA, Chung CP, Murray KT, Hall K, Stein CM. Atypical antipsychotic drugs and the risk of sudden cardiac death. N Engl J Med. 2009;360(3):225-235. doi:10.1056/NEJMoa0806994.

4. Vieweg WVR. New generation antipsychotic drugs and qtc interval prolongation. Primary Care Companion J Clin Psychiatry. 2003;5(5):205215.

C H AP T E R 1 1 0

Toxicology and Drugs of Addiction Alexandra Plichta and Sheri M. Berg

1. The Rumack-Matthew nomogram is a risk stratification tool that should be employed when an overdose of which of the following medications is confirmed.

A. Warfarin B. Salicylic acid C. Acetaminophen D. Digoxin

2. Which of the following, if present upon initial presentation in a patient with acute digoxin overdose, portends a poor prognosis?

A. Visual disturbance B. An ECG showing a high-degree atrio-ventricular block C. Serum digoxin levels greater than 1 ng/mL D. A serum potassium concentration greater than 5.5 mEq

3. A patient is brought in to the emergency department after being found down by her mother. She admits to the attending physician that she ingested a large dose of her prescribed amitriptyline approximately seven hours prior. Which of the following pieces of information would be MOST helpful in risk stratification of likelihood of having a major cardiac or neurologic event due to her medication overdose?

A. Ingested dose B. Peak serum amitriptyline concentration C. Serum potassium level D. QRS duration

4. A 29-year-old female is transferred to the intensive care unit (ICU) with a temperature of 104.5°F. Her only medical history includes herpes labialis and migraines, for which she regularly takes sumatriptan combined with ondansetron or metoclopramide for the associated severe nausea. She was initially admitted to the hospital for a cholecystectomy, which was complicated by vancomycinresistant enterococcus (VRE) bacteremia, and she was treated with linezolid. She was started on appropriate empiric therapy and became acutely altered, tremulous, and febrile. Blood cultures were obtained and showed no growth. Which of the following would be the BEST initial therapy for this patient?

A. Cyproheptadine B. Change antibiotic regimen C. Dantrolene D. Acetaminophen

5. A 58-year-old female is admitted to the ICU for a suspected infection. She is started on moxifloxacin and fluconazole. She is intubated and sedated and subsequently treated for gastroparesis with erythromycin. Which ECG finding is MOST likely associated with this patient’s medication regimen?

A. Prolonged QRS B. Widened QT interval C. Prolonged PR D. Presence of Osborn wave

6. A 55-year-old male who has a history of alcohol abuse disorder is brought to the emergency department after experiencing a generalized tonic-clonic seizure in prison. He was incarcerated 24 hours prior to presentation. After 24 hours on intravenous midazolam therapy, he continues to have delirium tremens. Which of the following strategies is the LEAST appropriate?

A. Add gabapentin to current regimen B. Add phenobarbital to current regimen C. Continue administering midazolam until Clinical Institute Withdrawal Assessment from Alcohol—Revised (CIWA-Ar) score is less than 8 or until 50 mg midazolam is given D. Perform endotracheal intubation and start a propofol infusion

7. A 56-year-old male is admitted to the ICU overnight after admitting to snorting large quantities of cocaine. He is diaphoretic and agitated, with vital signs as follows: BP 223/150; rectal temperature 103.3°F. He endorses substernal chest pain, and his ECG shows ST segments that are depressed compared to his admission from one month ago. Which of the following is the LEAST appropriate treatment for this patient?

A. Lorazepam B. Phentolamine C. Nitroglycerin D. Labetalol

8. A 20-year-old male is brought in by ambulance from a college campus with an altered mental status and appears to be floridly hallucinating. His blood pressure is 190/110 mm Hg and heart rate 129 beats per minute. A standard urine toxicology panel is negative.

Which of the following is the MOST likely etiology?

A. “Ecstasy” intoxication B. Amphetamine intoxication C. Synthetic cathinone intoxication D. New-onset schizophrenia presentation

Chapter 110 Answers 1. Correct Answer: C Rationale: The Rumack-Matthew Nomogram helps to determine the risk of hepatotoxicity from acetaminophen (APAP) ingestion. It plots serum APAP concentration on a logarithmic scale against time (elapsed since acute ingestion). Note that the nomogram starts at 4 hours post-ingestion.

The nomogram was designed after observing two distinct populations of patients who had overdosed on acetaminophen: untreated patients that went on to develop transaminitis (AST or ALT >1,000 IU/L) untreated patients who maintained normal transaminase levels

A line divides these two populations. Those who fall above the line are high-risk and should be treated with N-acetylcysteine (NAC); those below are low-risk and do not need NAC therapy. References

1. Rumack BH, Peterson RC, Koch GG, Amara IA. Acetaminophen overdose. 662 cases with evaluation of oral acetylcysteine treatment. Arch Intern Med. 1981;141:380.

2. Rumack BH. Acetaminophen hepatotoxicity: the first 35 years. J Toxicol Clin Toxicol. 2002;40:3-20.

3. Hendrickson Robert G. Acetaminophen. In: Nelson LS, Lewin NA, Howland M, et al, eds. Goldfrank’s Toxicologic Emergencies. 9th ed. New York: McGraw-Hill; 2011:483-499:chap 34. Print.

4. Pharmaceutical drug overdoses. In: Marino PL. Marino’s The ICU Book. 4th ed. Philadelphia: Wolters Kluwer Health/Lippincott Williams & Wilkins; 2014:963-980:chap 54. Print.

2. Correct Answer: D Rationale: Hyperkalemia is a frequent finding in those with digoxin overdose. Serum potassium levels better predict mortality than either initial ECG changes or serum digoxin levels. However, successful treatment of the hyperkalemia does not change outcomes. The serum potassium level serves merely to prognosticate. This phenomenon was discovered in a 1973 study of patients with acute digoxin poisoning before digoxin-specific antibody fragment was available. The raw data can be seen in figure and table that follow.

From Bismuth C, Gaultier M, Conso F, Efthymiou ML. Hyperkalemia in acute digitalis poisoning: prognostic significance and therapeutic implications. Clin Toxicol. 1973;6:153162.

Serum K+ Level (mEq/L)

Mortality Rate

n

5.5

0% 40% 100%

58 15 18 91

Data from Bismuth C, Gaultier M, Conso F, Efthymiou ML. Hyperkalemia in acute digitalis poisoning: prognostic significance 
and therapeutic implications. Clin Toxicol. 1973;6:153-162.

References

1. Hack JB. Cardioactive steroids. In: Nelson LS, Lewin NA, Howland M, et al, eds. Goldfrank’s Toxicologic Emergencies. 9th ed. New York: McGrawHill; 2011:936-945:chap 64. Print.

2. Bismuth C, Gaultier M, Conso F, Efthymiou ML. Hyperkalemia in acute digitalis poisoning: prognostic significance and therapeutic implications. Clin Toxicol. 1973;6:153-162.

3. Manini AF, Nelson LS, Hoffman RS. Prognostic utility of serum potassium in chronic digoxin toxicity: a case-control study. Am J Cardiovasc Drugs. 2011;11(3):173-178.

4. Kelly RA, Smith TW. Recognition and management of digitalis toxicity. Am J Cardiol. 1992;69:108-109.

3. Correct Answer: D Rationale: The mechanism by which tricyclic antidepressants (TCAs) such as amitriptyline exhibit their toxicity is by blockade of sodium (Na+) channels. In the central nervous system, Na+ channel blockade ultimately manifests as seizures. In the heart, Na+ channel blockade leads to widening of the QRS and ventricular arrhythmias. The 1985 study by Boehnert stratified patients into high- and low-risk categories based on the duration of their QRS. In clinical toxicology, a QRS duration of 100 milliseconds after TCA poisoning is considered the upper limit of normal. As one can see from the original data, the negative predictive value of this is very high, while the positive predictive value is rather low (34% incidence of seizure, 14% incidence of ventricular arrhythmia in those with QRS >100 ms in above study). Also note that no ventricular arrhythmias occurred until the QRS duration was >160 ms.

Na+ channel toxicity can be identified on an ECG by recognizing “R-axis deviation” of the terminal 40 milliseconds frontal plane QRS axis, which includes the presence of both S wave in lead I or aVL R wave in aVR However, this finding is not specific to TCA overdose and can also be seen with pathophysiology leading to a large right ventricle or acute right ventricular strain. References

1. Boehnert MT, Lovejoy FH, Value of the QRS duration versus the serum drug level in predicting seizures and ventricular arrhythmias after an acute overdose of tricyclic antidepressants. N Engl J Med. 1985;313(8):474-479.

2. Clancy C. Electrophysiologic and electrocardiographic principles. In: Nelson LS, Lewin NA, Howland M, et al, eds. Goldfrank’s Toxicologic Emergencies. 9th ed. New York: McGraw-Hill; 2011:314-329:chap 22. Print.

3. Liebelt EL, Francis PD, Woolf AD. ECG lead aVR versus QRS interval in predicting seizures and arrhythmias in acute tricyclic antidepressant toxicity. Ann Emerg Med. 1995;26(2):195-201.

4. Correct Answer: A Rationale: A change of antibiotic regimen is likely unnecessary, as the question stem suggests control of infection with clearance of blood cultures. Linezolid was the antimicrobial initiated to appropriately treat VRE bacteremia. When combined with agents this patient may be taking to treat her migraines and nausea (triptans, ondansetron, and metoclopramide), linezolid may precipitate serotonin syndrome (SS). Of the answer choices listed, cyproheptadine, a nonspecific serotonin antagonist, would be the most appropriate therapy. Drugs implicated in SS Antidepressants

Other Medications

SSRIs: a. b. c. d. e. f.

Buspirone (Buspar) Valproate (Depakote)

Se rtraline (Zoloft) Paroxe tine (Paxil) Fluoxe tine (Prozac) Fluvoxamine (Luvox) Citalopram (Ce le xa) Escitalopram (Le xapro)

Analge sics: Meperidine (Demerol) Fentanyl (Sublimaze) Tramadol (Ultram)

Methylene blue, an MAOI Ondansatron (Zofran) Metoclopramide (Reglan) Triptans—sumatriptan (Imitrex) Ergot derivatives (ergotamine, methylergonovine) Linezolid (Zyvox), an MAOI Ritonavir (Norvir) Dextromethorphan MDMA (ecstasy, molly) St. John’s wort

SNRIs: a. Duloxe tine (Cymbalta) b. Ve nlafaxine (Effe xor) c. De sve nlafaxine (Pristiq) MAOIs a. Phe ne lzine (Nardil) b. Isocarboxazid (Marplan) c. Tranylcypromine (Parnate ) TCAs a. b. c. d.

Imipramine (Tofranil) Clomipramine (Anafranil) Doxe pin Se le giline

Mirtazapine (Remeron)

MAOI, monoamine oxidase inhibitors; MDMA, 3,4-methyl enedioxy methamphetamine (ecstasy); SNRI, serotonin and norepinephrine reuptake inhibitors; SSRI, selective serotonin reuptake inhibitors; TCA, tricyclic antidepressants.

Serotonin Syndrome (SS) Symptoms

Causes

Treatment

Fever Altered mental status Autonomic hyperactivity Akathisia Tremor Hyperreflexia, clonus (See above list)

Stop culprit meds Supportive care Benzodiazepines If severe, consider paralysis Cyproheptadine

NEUROLEPTIC MALIGNANT SYNDROME Fever Altered mental status Autonomic instability Rigidity

Neuroleptics Metoclopromide (Reglan) Cessation of DA agonist (amantadine, bromocriptine, levodopa)

Stop (or restart) culprit meds Supportive care Dantrolene (2-3 mg/kg/d up to 10 mg/kg/d)

While the high temperature in the question stem may lead the reader to suspect refractory infection, severity of fevers in hospitalized patients has

not been found to correlate with the likelihood of infection. The table that follows provides a framework for thinking about infectious and noninfectious causes of fever in the ICU. Noninfectious Causes of Fever in ICU Patients Alcohol/drug withdrawal Postoperative fever (48 h post-op) Blood transfusion re action fe ve rs b Drug fe ve rs b (ne urole ptic malignant syndrome b )

Neoplastic fevers Cerebral infarction/hemorrhage Subarachnoid hemorrhage Adrenal insufficiency (crisis) Myocardial infarction a Acute pancre atitis a Acalculous chole cystitis a

Ischemic bowel Cirrhosis Gastrointe stinal (GI) ble e d a

Aspiration pneumonitis Acute re spiratory distre ss syndrome a

Fat embolus Pulmonary e mbolism a /de e p ve in thrombosis Thrombophle bitis a He matoma a

Transplant rejection a

Indicates pathologies excluded by temperatures >102°F—per Cunha.

b

Nnoninfectious disease with temperatures >102°F in ICU patients.

Adapted from Marik PE. Fever in the ICU. Chest. 2000;117(3):855-869 and Cunha BA. Fever in the critical care unit. Crit Care Clin. 1998;14(1):1-14.

References

1. Boyer EW, Shannon M. The serotonin syndrome. N Engl J Med. 2005;352(11):1112-1120.

2. Graudins A, Stearman A, Chan B. Treatment of the serotonin syndrome with cyproheptadine. J Emerg Med. 1998;16(4):615.

3. Marik PE. Fever in the ICU. Chest. 2000;117(3):855-869. 4. Cunha BA. Fever in the critical care unit. Crit Care Clin. 1998;14(1):1-14. 5. Correct Answer: B

Rationale: A number of medications have a known risk of prolonging QTc, which is defined as a QTc greater than 450 ms in males and greater than 460 ms in females. However, note that while some medications are known to prolong QTc, they are not necessarily associated with torsades de pointes (TdP) (amiodarone). Others have a known risk of TdP (erythromycin, haloperidol, ondansetron). In this question, the patient was on amiodarone (a class III antiarrhythmic) and was started on moxifloxacin (a fluoroquinolone) and fluconazole, increasing her risk of developing a prolonged QTc. All three medications behave similarly to antipsychotics and produce this electrocardiographic finding by blocking potassium (K+) channels. Antipsychotics Channel blockade ECG finding

K+ channels

Na + channel Prolonged QTc

Possible results Other implicated medications

Torsades de pointes

Similar to Therapy

Hypocalcemia Defibrillate Overdrive pace Magnesium infusion (prevention)

Class IA antiarrhythmics Class III antiarrhythmics -azole antifungals Antimicrobials: macrolides, fluoroquinolones Protease inhibitors Methadone Citalopram (Celexa, SSRI) Diphenhydramine (Benadryl)

Tricyclic Antidepressants Widened QRS R-axis deviation (terminal 
40 ms) —see below Ventricular arrhythmia Class IA antiarrhythmics Class IC antiarrhythmics Phenothiazines Amantadine Diphenhydramine (Benadryl) Carbamazepine (Tegretol) Cocaine

Hyperkalemia NaHCO3

Xenobiotics that cause Na+ channel blockade prolong QTc by slowing

cellular depolarization during phase 0. Thus QT duration increases because QRS duration increases while the ST segment duration remains essentially unchanged. Xenobiotics that cause K+ channel blockade prolong QTc by prolonging phases 2 and 3 (the plateau and repolarization phases). The ST segment is prolonged. References

1. Beach SR, Celano CM, Noseworthy PA, et al. QTc prolongation, torsades de pointes, and psychotropic medications. Psychosomatics. 2013;54(1):113.

2. Rautaharju PM, Surawicz B, Gettes LS, et al. AHA/ACCF/HRS recommendations for the standardization and interpretation of the electrocardiogram: part IV: the ST segment, T and U waves, and the QT interval: a scientific statement from the American Heart Association Electrocardiography and Arrhythmias Committee, Council on Clinical Cardiology; the American College of Cardiology Foundation; and the Heart Rhythm Society: endorsed by the International Society for Computerized Electrocardiography. Circulation. 2009;119:e241-e250.

3. Holstege CP, Eldridge DL, Rowden AK. ECG manifestations: the poisoned patient. Emerg Med Clin N Am. 2006;24(1):159-177.

4. Woosley RL, Heise CW, Romero KA. www.Crediblemeds.org. QTdrugs List, Accessed March 2019, AZCERT, Inc. 1822 Innovation Park Dr., Oro Valley, AZ 85755.

6. Correct Answer: A Rationale: The severity of the alcohol withdrawal is frequently stratified using the Clinical Institute Withdrawal Assessment for Alcohol—Revised (CIWA-Ar) scale. While patients with mild withdrawal symptoms (CIWA score 0-15) may be appropriate for ambulatory therapy with medications including chlordiazepoxide, oxazepam, and gabapentin, patients with moderate to severe withdrawal symptoms (CIWA-Ar score >16) are usually more appropriately treated as inpatients, with some warranting admission to the ICU. Any patient with seizures or delirium tremens (DT) should be started on intravenous benzodiazepines. The choice of agents and schedule of

administration may be institution-dependent. Some patients’ symptoms may persist through frequent and high doses of benzodiazepines (BZDs). It would be appropriate to add a second agent such as phenobarbital or propofol. Given propofol’s propensity to depress ventilatory drive, especially when coadministered with BZDs, it would be most important to admit the patient to an ICU and secure the airway with an endotracheal tube prior to its initiation. References

1. Sullivan JT, Sykora K, Schneiderman J, et al. Assessment of alcohol withdrawal: the revised clinical institute withdrawal assessment for alcohol scale (CIWA-Ar). Br J Addict. 1989;84(11):1353.

2. Myrick H, Malcolm R, Randal PK, et al. A double-blind trial of gabapentin versus lorazepam in the treatment of alcohol withdrawal. Alcohol Clin Exp Res. 2009;33(9):1582-1588.

3. Victor M, Brausch C. The role of abstinence in the genesis of alcoholic epilepsy. Epilepsia. 1967;8(1):1-20.

4. Gold JA, Rimal B, Nolan A, Nelson LS. A strategy of escalating doses of benzodiazepines and phenobarbital administration reduces the need or mechanical ventilation in delirium tremens. Crit Care Med. 2007;35:724-730.

5. Gold JA, Nelson LS. Ethanol withdrawal. In: Nelson LS, Lewin NA, Howland M, et al, eds. Goldfrank’s Toxicologic Emergencies. 9th ed. New York: McGraw-Hill; 2011:1134-1142:chap 78. Print.

7. Correct Answer: D Rationale: This patient is displaying signs and symptoms of cocaine toxicity, including cocaine-related myocardial infarction. Of the choices listed, treatment with β-blockers is the least well-supported. First-line treatments may include sympatholysis with use of benzodiazepines such as lorazepam, α-adrenergic blockade with phentolamine, and vasodilation with nitroglycerin. The theoretical fear of use of β-blockers in cocaine overdose is the “unopposed alpha effect” whereby selective β-blockade leaves the patient with decreased cardiac chronotropy and inotropy (decreased cardiac output), while working against an increased afterload; a

combination that could precipitate cardiovascular collapse. Even nonspecific β-blockers (with both anti-α and anti-β activity) such as labetalol are thought to have this effect. Though a number of more recent studies have described the safe use of β-blockers in this setting, it is still not a recommended practice. References

1. McCord J, Jneid H, Hollander JE, et al. Management of cocaineassociated chest pain and myocardial infarction: a scientific statement from the American Heart Association Acute Cardiac Care Committee of the Council on Clinical Cardiology. Circulation. 2008;117(14):18971907.

2. Richards JR, Garber D, Laurin EG, et al. Treatment of cocaine cardiovascular toxicity: a systematic review. Clin Toxicol. 2016;54(5):345364.

3. Hoffman RS. Cocaine and beta-blockers: should the controversy continue? Ann Emerg Med. 2008;51(2):127.

8. Correct Answer: C Rationale: Synthetic cathinones (aka “bath salts”) are phenylethylamine derivatives, sharing the same core structure as amphetamines (methamphetamine, MDMA), as well as endogenous monoamines (epinephrine, norepinephrine, and dopamine). The effects of these drugs of abuse can be understood to some degree by comparing their ability to block reuptake of bioamines to different degrees: Differential effect on receptors: NET, DAT and SERT

Amphetamine Methamphetamine MDMA Methcathinone

Selectivity of NET Versus DAT

Selectivity of DAT > SERT

threefold NET > DAT twofold NET > DAT fivefold NET > DAT 1:1 NET = DAT

70-fold DAT > SERT 30-fold DAT > SERT 7-fold SERT > DAT 120-fold DAT > SERT

MDMA, 3,4-methylenedioxymethamphetamine.

Clinical Effect

Psychosis

Can Lead to a Life-

Associated With

Associated With

Methamphetamine

MDMA

Methcathinone

Hyperarousal, compulsivity, occasional paranoia “Easily controlled altered state of consciousness with emotional and sensual overtones” Stimulant and hallucinogenic effects. Increased energy, empathy, openness, and libido.

Threatening Hyperthermia?

Renal Failure?

Hyponatremia?

Common effect

Yes

Yes

No

Occasional

Yes

Yes

Yes

Most common effect

Yes

Yes

Yes

Methamphetamine, MDMA, and bath salts act as sympathomimetics, which results clinically in hyperalertness, hypertension, tachycardia, mydriasis, and diaphoresis. Acute psychosis is the most common effect of bath salt ingestion, and can be present in the absence of the sympathomimetic symptoms. Both MDMA and methamphetamine are highly likely to appear on a standard urine toxicology screen, whereas the heterogenous group of compounds within the singular group known as “bath salts” frequently evades detection on a standard assay. While new-onset schizophrenia may also present with similar clinical features, it would unlikely (by itself) present with perturbations in vital signs seen in this patient. Bath salt intoxication can be treated with benzodiazepines, antipsychotics, and supportive care, with particular attention paid to renal function. References

1. Banks ML, Worst TJ, Rusyniak DE, et al. Synthetic cathinones (“bath salts”). J Emerg Med. 2014;46(5):632-642.

2. Kramer J, Fischman VS, Littlefield DC. Amphetamine abuse: pattern and

effects of high doses taken intravenously. JAMA. 1967;201(5):305-309.

3. Winder GS, Stern N, Hosanagar A. Are “Bath Salts” the next generation of stimulant abuse? J Subst Abuse Treat. 2013;44(1):42-45.

C H AP T E R 1 1 1

Psychoactive Medications Archit Sharma and Anureet K Walia

1. A 55-year-old male with a past history of alcohol use disorder with a previous history of withdrawal seizures, hypertension, and cirrhosis presents to the emergency room requesting inpatient detoxification. The patient has been drinking 12 beers daily for the past 2 weeks. His initial lab work demonstrates a glomerular filtration rate >60, AST of 210 U/L, ALT of 152 U/L. Which of the following agents would be the best choice to use for this patient’s alcohol detoxification?

A. Chlordiazepoxide B. Diazepam C. Clorazepate D. Clonazepam E. Lorazepam

2. A 19-year-old male presents to the emergency department with agitation, altered mental status, and muscle rigidity. According to his girlfriend, his physician started him on a medication 2 weeks ago for schizophrenia and depression. During the clinical examination, he appears diaphoretic and has no clonus. His vital signs include a blood pressure of 185/90 mm Hg, temperature of 38.7°C, and pulse of 105 bpm. His lab data are significant for leukocytosis and elevated liver transaminases. Which of the following medications is most likely causing the patient’s severe adverse reaction?

A. Olanzapine 30 mg once daily B. Lithium 150 mg twice daily

C. Bupropion 100 mg twice daily D. Clomipramine 25 mg once daily E. Chlordiazepoxide 5 mg three times daily

3. An 85-year-old patient is brought into the emergency room, after he was found comatose at home by his son. The patient has a known history of bipolar disorder and epileptiform seizures and is on medication for that. Vital examination reveals a BP of 112/56 mm Hg, a pulse rate of 96 bpm, a respiratory rate of 24 breaths/min, and a temperature of 36.7°C. His serum sodium is 115 mmol per liter. Which of the following medications is the most likely cause of his sodium imbalance?

A. Lithium B. Olanzapine C. Carbamazepine D. Quetiapine E. Topiramate

4. A patient presents to the ED with confusion, myoclonus, diarrhea, hypotension, altered mental status, tachycardia, and a normal creatine phosphokinase (CPK). His family is able to confirm that he was recently started on a new medication by his psychiatrist for treatment of bipolar disorder, although they do not know the name of the medication or what kind of medication it was. Which one of the following is the most likely diagnosis and why?

A. Neuroleptic malignant syndrome (NMS), because the patient is confused B. NMS, because rigidity is more commonly a part of serotonin syndrome C. Serotonin syndrome, because it commonly presents with

hypertension D. NMS, because it usually presents with normal CPK E. Serotonin syndrome, because it more commonly presents with myoclonus and GI symptoms

5. A 37-year-old male with a past medical history of alcoholic cirrhosis presents to the ICU postoperatively, after having an open reduction and internal fixation of his femur fracture. He is currently intubated because of rib fractures, splinting, and concern for pulmonary contusions. He is currently sedated with dexmedetomidine and fentanyl infusions but is becoming more agitated and restless. The nurse requests a breakthrough dose of a benzodiazepine to keep him sedated. Which one of the following would be the best choice of medication in someone with impaired liver function?

A. Diazepam B. Oxazepam C. Clonazepam D. Prazepam E. Estazolam

Chapter 111 Answers 1. Correct Answer: E Rationale: In patients requiring alcohol detoxification who have evidence of liver disease or impairment, as evidenced in this patient by his elevated liver transaminases, lorazepam is the best option. Lorazepam does not have any active metabolites and is cleared from the patient’s system more rapidly than other benzodiazepines, making it a preferable choice in patients with liver dysfunction. The other options are benzodiazepines, which can protect against alcohol withdrawal symptoms but would have a prolonged effect in patients with liver dysfunction. They require normal functioning of liver for their primary metabolism (diazepam undergoes oxidation and clonazepam undergoes nitroreduction). References

1. Sachdeva A, Chandra M, Deshpande SN. A comparative study of fixed tapering dose regimen versus symptom-triggered regimen of lorazepam for alcohol detoxification. Alcohol Alcohol. 2014;49(3):287291.

2. McKeon A, Frye MA, Delanty N. The alcohol withdrawal syndrome. J Neurol Neurosurg Psychiatry. 2008;79(8):854-862.

3. Kumar CN, Andrade C, Murthy P. A randomized, double-blind comparison of lorazepam and chlordiazepoxide in patients with uncomplicated alcohol withdrawal. J Stud Alcohol Drugs. 2009;70(3):467-474.

2. Correct Answer: A Rationale: This patient is likely experiencing neuroleptic malignant syndrome (NMS), a life-threatening reaction to an antipsychotic medication such as olanzapine. Characteristic symptoms include fever, altered mental status, muscle rigidity, and autonomic dysfunction. He presented with the classic

symptoms of fever, altered mental status (mutism, agitation), muscle rigidity, and autonomic dysfunction (altered blood pressure, heart rate). Treatment involves stopping the medication and if clinically indicated, dantrolene, IV hydration, and benzodiazepines as needed. None of the other choices are antipsychotics, and hence do not cause NMS. References

1. Strawn JR, Keck PE Jr, Caroff SN. Neuroleptic malignant syndrome. Am J Psychiatry. 2007;164(6):870-876.

2. Trollor JN, Chen X, Sachdev PS. Neuroleptic malignant syndrome associated with atypical antipsychotic drugs. CNS Drugs. 2009;23(6):477-492.

3. Picard LS, Lindsay S, Strawn JR, Kaneria RM, Patel NC, Keck PE Jr. Atypical neuroleptic malignant syndrome: diagnostic controversies and considerations. Pharmacotherapy. 2008;28(4):530-535.

3. Correct Answer: C Rationale: Certain medications are known to cause syndrome of inappropriate antidiuretic hormone (SIADH) secretion, leading to hyponatremia. Carbamazepine acts like a vasopressin-agonist and has antidiuretic effects. Altered sensitivity to serum osmolality by the hypothalamic osmoreceptors appears likely, but an increased sensitivity of the renal tubules to circulating ADH cannot be excluded. As such it can cause hyponatremia, especially in the elderly. Another medication that causes the same side effect is oxcarbazepine. Treatment includes discontinuing offending drug and fluid restriction, in addition to monitoring and correction of electrolyte balance. The other choices listed here are not known to cause SIADH. References

1. Berghuis B, de Haan GJ, van den Broek MP, et al. Epidemiology, pathophysiology and putative genetic basis of carbamazepine-and oxcarbazepine-induced hyponatremia. Eur J Neurol. 2016;23(9):1393-

1399.

2. Van Amelsvoort TH, Bakshi R, Devaux CB, Schwabe S. Hyponatremia associated with carbamazepine and oxcarbazepine therapy: a review. Epilepsia. 1994;35(1):181-188.

3. Prakash S, Bhatia PS, Raheja SG, Pawar M. Carbamazepine-induced hyponatremia. Br J Anesth. 2016;117(eLetters suppl).

4. Correct Answer: E Rationale: Both NMS and serotonin syndrome may present with mental status changes, autonomic instability, diaphoresis, and mutism. Both can have elevated CPK, but high CPK is more common in NMS because of the muscular rigidity. The key differentiating features between the two are that serotonin syndrome presents with myoclonus, hyperreflexia and GI symptoms, whereas NMS presents with muscle rigidity. Hence, this patient is most likely to have serotonin syndrome. References

1. Perry PJ, Wilborn CA. Serotonin syndrome vs neuroleptic malignant syndrome: a contrast of causes, diagnoses, and management. Ann Clin Psychiatry. 2012;24(2):155-162.

2. Dosi R, Ambaliya A, Joshi H, Patell R. Case report: serotonin syndrome versus neuroleptic malignant syndrome: a challenging clinical quandary. BMJ Case Rep. 2014;2014

3. Sokoro AA, Zivot J, Ariano RE. Neuroleptic malignant syndrome versus serotonin syndrome: the search for a diagnostic tool. Ann Pharmacother. 2011;45(9):e50.

5. Correct Answer: B Rationale: Oxazepam is generally considered safer than many other benzodiazepines in patients with impaired liver function, primarily

because it is metabolized by glucuronidation and does not require hepatic oxidation. Hence, oxazepam is less likely to accumulate and cause adverse reactions in the elderly or people with liver disease. Other options for patients with liver dysfunction are temazepam and lorazepam. They have short half-lives and do not have any active metabolites. Other benzodiazepines are less desirable in patients with hepatic dysfunction because they require the liver for their primary metabolism (diazepam undergoes oxidation and clonazepam undergoes nitroreduction) or have longer half-lives (prazepam and estazolam). References

1. Shull HJ, Wilkinson GR, Johnson R, Schenker S. Normal disposition of oxazepam in acute viral hepatitis and cirrhosis. Ann Intern Med. 1976;84(4):420-425.

2. Klotz U, Avant GR, Hoyumpa A, Schenker S, Wilkinson GR. The effects of age and liver disease on the disposition and elimination of diazepam in adult man. J Clin Invest. 1975;55(2):347-359.

3. Greenblatt DJ, Divoll M, Harmatz JS, Shader RI. Oxazepam kinetics: effects of age and sex. J Pharmacol Exp Ther. 1980;215(1):
86-91.

Research, Administration, and Ethics 112.Research and Biostatistics 659​ 113.Administration 665​ 114.Teaching 674​ 115.Psychosocial Issue Among Providers 677​ 116.Ethical Considerations 679​ 117.Patient Confidentiality, Healthcare Policy 682​ 118.Palliative Care and End of Life 685​ 119.Organ Donation 692​

C H AP T E R 1 1 2

Research and Biostatistics Edward A. Bittner

1. A study involving 100 patients assessed the efficacy of a high-flow nasal cannula (HFNC) administration in preventing intubation after chest trauma. Among 40 patients who received HFNC, 35 did not require intubation. Among the 60 patients who did not receive HFNC, 55 did not require intubation. The statistical test MOST appropriate to determine if HFNC use is associated with a reduction in the need for intubation is

A. Chi-square test B. Analysis of variance C. Log-rank test D. A relative risk

2. An intensivist performs a study which examines the impact of a chlorhexidine bathing protocol on the incidence of new methicillinresistant Staphylococcus aureus (MRSA) infections in the ICU. Based on her sample size calculation, 300 patients are needed for randomization to the two study groups (chlorhexidine vs. standard care), to detect a clinically meaningful difference with power of 0.8. However, due to cost constraints, only 200 patients are ultimately enrolled in the study. Statistical testing of the data reveals a lower incidence of MRSA infection in the chlorhexidine group compared with the standard care group (P < .001). Which of the following statements regarding interpretation of the findings is MOST correct?

A. The chlorhexidine protocol is effective

B. The chlorhexidine protocol is not effective C. A type 1 error is likely D. A type 2 error is likely

3. Statistical analysis of 20 patients with acute respiratory distress syndrome shows a mean cardiac output (CO) of 5 L/min with a standard deviation of 1 L/min. The distribution of CO in the population is normal. Which of the following statements is MOST accurate regarding interpretation of these data?

A. Approximately 50% of the sample population would be expected to have a CO between 4 and 6 L/min B. Approximately 95% of the sample population would be expected to have a CO between 3 and 7 L/min C. Of the sample patients, 10 have a cardiac output greater than 5 L/min D. The mean and the median CO are the same in the sample of 20 patients

4. An investigator wants to compare a new noninvasive cardiac output measurement technology to cardiac output obtained from a pulmonary artery catheter using the thermodilution technique. Which of the following statistical methods is MOST appropriate for comparing the two techniques?

A. Cohen’s kappa statistic B. Correlation analysis C. Bland-Altman analysis D. Kaplan-Meier plot

5. A study is being designed to compare the efficacy of two analgesic

regimens on pain scores of patients in the ICU. Which factor is MOST likely to increase the number of patients required to detect a difference between the two analgesic regimens?

A. Low variability of individual responses to analgesic treatment B. Lack of randomization of patients to treatment regimens C. Little difference in the effectiveness of the two regimens D. Use of blinded rather than nonblinded observers

6. An investigator wants to evaluate the impact of a new antiinflammatory agent on survival from septic shock. He randomizes 300 patients that develop septic shock to either receive the antiinflammatory agent together with standard care or to receive standard care alone. He follows the two groups until ICU discharge. Which of the following is the most appropriate test to compare survival in the two groups?

A. Log-rank test B. T-test C. Chi-square test D. Wilcoxon Rank Sum test

7. Which of the following Pearson correlation coefficients represents the strongest linear relationship between two variables?

A. 0.85 B. –0.80 C. 1.25 D. –0.95

8. An investigator wishes to perform analysis of retrospective cohort data to examine the impact of goal-directed fluid management (independent variable) on the need for renal replacement therapy (dependent variable) in patients suffering from major thermal injury. Potential confounding variables which the investigator wishes to control for in the analysis are age, baseline creatinine level, and sequential organ failure assessment score. Which method of regression analysis is MOST appropriate for this analysis?

A. Simple linear B. Cox C. Multivariable linear D. Logistic

9. Receiver operator characteristic (ROC) curves comparing the performance of two biomarkers for early detection of sepsis are displayed in the figure that follows.

Based on the figure which of the following statements regarding the performance of the new biomarker is MOST likely true? A. The existing biomarker (red) is a better predictor of sepsis than the new biomarker (blue). B. There appears to be no difference in the performance of the two biomarkers for early detection of sepsis C. The difference in the areas under the ROC curves for the two biomarkers can be used to compare their performance D. The new biomarker (blue) appears to be a better predictor of septic

shock than the existing biomarker (red) but not for sepsis

10. A clinical trial is performed to evaluate the impact of a new drug therapy for prevention of contrast-induced nephropathy (CIN) in patients with stage 2 or 3 chronic kidney disease (CKD). Three hundred patients are randomized to receive either the new drug therapy in combination with standard care versus standard care alone. After analyzing the trial data, the number needed to treat (NNT) is calculated to be 5. Which of the following statements regarding interpretation of the NNT in this study is most correct?

A. The relative risk reduction in CIN associated with receiving the new therapy is 0.1. B. For every 5 patients receiving the new therapy, CIN will be prevented in one additional patient as compared with standard therapy alone. C. The new therapy is five times more effective in preventing CIN than standard therapy. D. The new therapy is more effective than an alternative prevention of CIN in patients with stage 4 CKD with an NNT of 15.

Chapter 112 Answers 1. Correct Answer: A Rationale: The chi-square test measures the association between two categorical variables. In the question above, the categorical variables are HFNC and intubation and each patient can be classified as “YES” or “NO” based on these variables. Analysis of variance is used to examine the differences in mean values of more than two groups. The log-rank test is used in analysis of time-to-endpoint data to test for differences in hazard rates. Relative risk is a ratio of the probability of an event occurring in the exposed group versus the probability of the event occurring in the nonexposed group. References

1. Bewick V, Cheek L, Ball J. Statistics review 8: qualitative data – tests of association. Crit Care. 2004;8:46-53.

2. Fisher MJ, Marshall AP, Mitchell M. Testing differences in proportions. Aust Crit Care. 2011;24:133-138.

2. Correct Answer: A Rationale: The power of a study is the likelihood that it will distinguish an effect of a certain size from chance. The study in the question was at risk for being underpowered given that the calculated sample size was not achieved. Nonetheless a difference was detected between groups, which suggests that the power was adequate for the effect measured. In statistical hypothesis testing, a type I error is the rejection of a true null hypothesis (also known as a “false-positive” finding or conclusion), while a type II error is the failure to reject a false null hypothesis (also known as a “false-negative” finding or conclusion). Although a type 1 error is possible, it is unlikely given that p-value is 5.0 L/min, resulting in a mean CO of 5. The median and the mode are the same within the population from which the sample is drawn. However, these measures may differ in a sample drawn from the population due to the effects of random sampling. References

1. Whitley E, Ball J. Statistics review 2: samples and populations. Crit Care. 2002;6:143-148.

2. Moran JL, Solomon PJ. Statistics in review part I: graphics, data summary and linear models. Crit Care Resusc. 2007;9:81-90.

4. Correct Answer: C Rationale: Bland-Altman analysis is used to compare a measurement technique

against a reference value, especially when the reference value may not be a true gold standard. Bland and Altman suggest that when a new technology has bias and precision comparable with the previous technology, then it may be accepted in the clinical setting. The Bland-Altman graph plots the difference between two techniques against their averages.

The resulting scatter diagram allows the clinician to determine the following: Bias—average difference, (ideal bias = 0) Precision—1 standard deviation that describes range for 68% of comparison points Limits of agreement—2 standard deviations that describe the range for 95% of comparison points

Correlation analysis (eg, the Pearson correlation coefficient) can be misleading in method agreement studies, since correlation measures linear association rather than agreement. Two methods of measurement can correlate well yet disagree greatly, as would occur if one method read consistently higher than the other. Furthermore, correlation typically depends on the range of measures being assessed, with wider ranges being assessed often resulting in higher correlations but not as a result of better agreement between the methods of measurement being assessed. Cohen’s kappa statistic uses Cohen’s kappa coefficient (κ) to measure inter-rater agreement of qualitative (categorical) items. The Kaplan-Meier

plots are used for estimating the survival function from lifetime data. The Mann-Whitney U-test is used to determine whether two independent samples of observations are drawn from the same or identical distributions. References

1. Odor PM, Bampoe S, Cecconi M. Cardiac output monitoring: validation studies-how results should be presented. Curr Anesthesiol Rep. 2017;7:410-415.

2. Cecconi M, Rhodes A, Poloniecki J, Della Rocca G, Grounds RM. Benchto-bedside review: the importance of the precision of the reference technique in method comparison studies-with specific reference to the measurement of cardiac output. Crit Care. 2009;13:201.

5. Correct Answer: C Rationale: When designing a clinical trial to compare two or more groups, a key consideration is to know how many subjects must be enrolled to have adequate power to detect a difference between groups if such a difference exists. Even the most rigorously executed trial may fail to answer its research question if the sample size is too small. On the other hand, a trial with an inappropriately large sample will be more difficult to carry out, and it will not be cost effective. Factors that influence the sample size include the following: Expected effect size (average difference between groups in the outcome of interest) Standard deviation in the population Acceptable level of significance Power of the study Underlying event rate in the population

A small difference in the effectiveness of treatment regimens (ie, effect size) will increase the sample size required to detect the difference as compared to detection of a larger effect size. Randomization of treatment regimens is a means of controlling for confounders but does not affect sample size needed. Use of blinded rather than nonblinded observers can reduce the bias in a study but does not affect the sample size required.

References

1. Whitley E, Ball J. Statistics review 4: sample size calculations. Crit Care. 2002;6:335-341.

2. Rosner B. Fundamentals of Biostatistics. 8th ed. Boston: Brooks/Cole, Cengage Learning; 2016.

6. Correct Answer: A Rationale: Studies are often designed to compare the survival of two (or more) groups of patients. While the proportions of patients surviving at any specific time could be calculated, the weakness of this approach is that it does not provide a comparison of the total survival experience of the two groups, but rather gives a comparison at an arbitrary time point. The logrank test is a popular method of comparing the survival of groups which takes the whole follow-up period into account. It accomplishes this by comparing estimates of the hazard functions of the two groups at each observed event time. The null hypothesis is that there is no difference between the groups in the probability of death at any time point. The logrank test has the considerable advantage that it does not require knowledge about the shape of the survival curve or the distribution of survival times. References

1. Bewick V, Cheek L, Ball J. Statistics review 12: survival analysis. Crit Care. 2004;8:389-394.

2. Bland JM, Altman DG. The logrank test. BMJ. 2004;328(7447):1073. 7. Correct Answer: D Rationale: Correlation is a measure of a monotonic association between two variables in which a change in the magnitude of one variable is associated with a change in the magnitude of another variable, either in the same or in the opposite direction. Most often, the term “correlation” is used in the context of such a linear relationship between two continuous, random variables, known as a Pearson product-moment correlation. To quantify the

strength of the relationship, the correlation coefficient “r ” from sample data is calculated. The value of “r ” always lies between –1 and +1. A value of the correlation coefficient close to +1 indicates a strong positive linear relationship (ie, one variable increases with the other). A value close to –1 indicates a strong negative linear relationship (ie, one variable decreases as the other increases). A value close to 0 indicates no linear relationship; however, there could be a nonlinear relationship between the variables. References

1. Bewick V, Cheek L, Ball J. Statistics review 7: correlation and regression. Crit Care. 2003;7:451-459.

2. Schober P, Boer C, Schwarte LA. Correlation coefficients: appropriate use and interpretation. Anesth Analg. 2018;126:1763-1768.

8. Correct Answer: D Rationale: A common question in the analysis of study data is whether a there is a statistical relationship between a dependent variable (Y) and independent variables (X1, X2, …). A common method to answer this question is to employ regression analysis in order to model the relationship between the variables. The type of the regression model depends on the distribution of the dependent variable Y; if it is continuous and approximately normal, linear regression is used to model the relationship between Y and Xi. Simple linear regression is used when there is a single independent variable (X) and multiple linear regression when there are two or more independent variables (X1, X2, …). If the dependent variable Y is dichotomous (eg, YES/NO), then logistic regression is used; if modeling time-to-event (survival-type) data, Cox regression is used. For analysis of the cohort study data in which the outcome (dependent) variable (eg, need for renal replacement therapy), is dichotomous, logistic regression is the regression model most appropriate to use. References

1. Bewick V, Cheek L, Ball J. Statistics review 7: correlation and regression. Crit Care. 2003;7:451-459.

2. Bewick V, Cheek L, Ball J. Statistics review 12: survival analysis. Crit Care. 2004;8:389-394.

3. Bewick V, Cheek L, Ball J. Statistics review 14: logistic regression. Crit Care. 2005;9:112-118.

4. Bender R. Introduction to the use of regression models in epidemiology. Methods Mol Biol. 2009;471:179-195.

9. Correct Answer: C Rationale: ROC curve analysis is used to assess the diagnostic accuracy of a test. It can also be used to compare the performance of more than one test for the same outcome. With ROC analysis, the sensitivity is plotted against 1specificity for every cutoff point of the diagnostic test. Generally, a tradeoff is made between sensitivity and specificity, and a decision must be made regarding their relative performance. A perfect test (biomarker) would have sensitivity and specificity both equal to 1. The better the test the closer the ROC curve is to this ideal. The global performance of a diagnostic test can be quantified by calculating the area under the ROC curve (AUC). A greater AUC indicates better test performance. The ideal test would have an AUC of 1 whereas a random guess would have an AUC of 0.5. In the figure, the position of the ROC curve for the new biomarker (blue) above and toward the left corner of the graph relative to the ROC curve for the existing biomarker (red) suggests that it may be a better in predicting early sepsis. A formal statistical test would be needed to determine whether the difference in AUCs of the two ROC curves is significant. In the question, the performance of the two biomarkers was assessed for the early diagnosis of sepsis and not septic shock. References

1. Bewick V, Cheek L, Ball J. Statistics review 13: receiver operating characteristic curves. Crit Care. 2004;8:508-512.

2. Søreide K, Kørner H, Søreide JA. Diagnostic accuracy and receiveroperating characteristics curve analysis in surgical research and decision making. Ann Surg. 2011;253:27-34.

10. Correct Answer: B Rationale: The number needed to treat (NNT) estimates the number of patients who would need to be treated in order to obtain one more success than that obtained with a control treatment. The smaller the NNT, the more successful the intervention. The NNT is always in reference to a comparison group (in which patients receive placebo, no treatment, or some other treatment), a particular treatment outcome, and a defined period of treatment. The NNT is calculated as the reciprocal of the absolute risk reduction (ARR) for a given treatment: NNT = 1/ARR. While calculation of NNT is useful for describing the impact of a clinical treatment, it has some important limitations. First, an NNT is estimated from data obtained in a clinical trial, and therefore, the true value of the NNT may be higher or lower than the point estimate. A confidence interval for the NNT is useful in this regard because it can provide an indication of the range in which the true value of the NNT falls. Second, it is inappropriate to compare NNTs across disease conditions, particularly when the outcomes of interest differ. Only if we have NNTs for different interventions for the same condition (and severity) and with the same outcome is it appropriate to directly compare NNTs. Third, the NNT for a specified intervention in an individual patient depends not only on the nature of the treatment but also on the patient’s risk at baseline. The concept of NNT assumes that a given intervention produces the same relative risk reduction whether the patient’s risk at baseline is low, intermediate, or high. Since that risk may not be the same for all patients, an NNT that is provided by the literature may have to be adjusted for a given patient. For the question, option B is the correct answer based on the definition of NNT. Option A is incorrect since the relative risk reduction cannot be calculated based on the data provided. The absolute risk reduction based on the NNT of 5 is 0.2 (1/5). Option C is an incorrect interpretation of the NNT while option D incorrect since it compares the NNTs of patient groups with different risks of CIN at baseline (stage 2-3 CKD vs. stage 4) References

1. Bewick V, Cheek L, Ball J. Statistics review 11: assessing risk. Crit Care.

2004;8:287-291.

2. Cook RJ, Sackett DL. The number needed to treat: a clinically useful measure of treatment effect. BMJ. 1995;310:452-454.

C H AP T E R 1 1 3

Administration Jason K. Bowman and Jarone Lee

1. A 75-year-old male is admitted to the ICU with combined heart failure exacerbation and multifocal pneumonia. He is currently on BiPAP and his only lines are two small peripheral IVs. You anticipate that he many require numerous procedures in the ICU within the first 24 to 48 hours. You consider talking to the patient and his family to obtain anticipatory consent for numerous specific common ICU procedures (intubation, bronchoscopy, arterial line, central venous access, etc), or just seeking a blanket consent for “all routine ICU interventions.” What generally accepted aspects of informed consent may be violated by either of these methods of obtaining consent?

A. Assessment of decision-making capacity B. Description of the proposed intervention(s) C. Discussion of risks and benefits of the proposed intervention(s) and alternatives D. Assessment of patient / proxy understanding and shared decisionmaking E. Ensuring voluntariness of consent

2. You are the director of an ICU in an academic center and are considering hiring NP and/or PA providers to work alongside residents under the supervision of the ICU attendings and fellows. Based on existing medical literature, ALL of the following appear to be true about the effect of integrating advanced practice providers in this manner EXCEPT for:

A. Improved patient clinical outcomes B. Increased hospital costs for patients C. Decreased length of ICU admissions D. Similar or improved efficacy at invasive procedures E. Improved resident adherence to duty hours and conference attendance

3. Use of the Awakening and Breathing Coordination, Delirium Monitoring/ Management, and Early Exercise/Mobility (“ABCDE”) Bundle has been shown to have which of the following effects on ICU patients?

A. Increased need for sedative medications B. Decreased risk of delirium C. More frequent need for reintubation D. Increased risk of patient self-extubation E. Shorter time to discharge

4. On morning rounds in the ICU, you wish to assess a patient for delirium. You would like to use the Confusion Assessment Method for ICU (CAM-ICU) scale. Which of the following is most important to assess in the patient, before using the CAM-ICU tool on them?

A. Heart rate and blood pressure B. Fluid status C. If the patient has a history of psychiatric disease D. The patient’s level of consciousness E. If the patient has risk factors for delirium in their history

5. During a busy shift as an ICU attending, you receive several

requests for admission to your ICU from your hospital’s emergency department and inpatient medical floor, as well as from another smaller hospital nearby that does not have an ICU. However, your unit currently only has one bed available. Effective and ethical ICU triage involves all of the following EXCEPT for:

A. Assessing for specific patient need(s) that can only be addressed in an ICU B. Considering the patient’s diagnosis, medical/surgical condition(s), and prognosis C. Balancing the ethical principle of distributive justice with obligation to individual patients D. Preferential use of standardized triage tools over case-by-case decisions from experienced clinicians E. Multidisciplinary discussion between ICU, emergency medicine, hospitalist medicine, surgery, and other allied health providers

6. You are leading a project to try and develop machine learning and decision tools in your ICU. Your team wishes to start by studying a topic for which machine learning has already been shown to demonstrate promising potential. All of the following topics fit this description EXCEPT:

A. Sepsis detection B. Predicting complications in postsurgical ICU patients C. Deciding need for admission in patients diagnosed with pneumonia D. Predicting ICU readmission E. Predicting the need for prolonged mechanical ventilation

7. A 56-year-old female is admitted to the ICU for influenza and acute hypoxemic respiratory failure. On further review of her chart you note she also has advanced HIV complicated by AIDS and

disseminated candidiasis. However, she has consistently declined antiretroviral therapy, but is still sexually active with her husband per prior notes, and previously reported intermittent use of protection. She arrives to the unit on BiPAP but you are able to briefly talk with her to try and assess her wishes and goals. The patient informs you that her husband is her healthcare proxy but tells you that he does not know about her HIV status, asks you to not disclose this to him, and further tells you that you cannot share this information without her consent because of HIPAA. A short time later her respiratory status further declines; she is successfully intubated, placed on a mechanical ventilator, and sedated. How should you proceed in subsequent conversations with her husband?

A. Do not under any circumstances disclose the patient’s HIV status to him because it’s immoral to do so B. Tell him about her HIV status to allow him to better understand her medical condition and make informed decisions about her care C. Tell him about her HIV status so that he can obtain HIV testing himself D. Do not tell him because doing so is forbidden by HIPAA E. Both B and C

8. What is an example of a core aspect of critical care medicine that is not currently addressed by the US Department of Health and Human Services’ contracted National Quality Forum (NQF), in the existing quality measures they have put forth?

A. ICU length of stay B. In-hospital mortality rate C. Rate of catheter-associated bloodstream infections D. Delirium E. Sepsis management bundle

9. You have been tasked with researching and implementing tools to help better predict outcomes for patients in the ICU—both to improve clinical care and assist biomedical research. What are three of the major ICU predictive scoring systems in use today?

A. APACHE (Acute Physiology and Chronic Health Evaluation), BISAP, and SAPS (simplified acute physiology score) B. MPM0 (mortality probability model), SAPS, and TRISS C. APACHE, MPM0, and SAPS D. BISAP, MPM0, and TRISS E. APACHE, SAPS, and TRISS

10. Which of the following realms appears to often be the largest source of nosocomial actual/potential patient harm in the ICU?

A. Medication error B. Problems with lines/drains/catheters C. Equipment failure D. Obstruction or leakage of artificial airway E. In appropriate silencing of an alarm

Chapter 113 Answers 1. Correct Answer: C Rationale: Ethically obtained consent comprises numerous components, including (1) assessment of decision-making capacity, (2) discussion of pertinent information, (3) assessing comprehension, (4) ensuring voluntariness, (5) joint deliberation, (6) obtaining consent, and (7) documentation. Furthermore, discussion of pertinent information should include description of the proposed intervention (including listing the providers who will be participating), potential benefits and risks of the intervention, reasonable alternatives, and potential benefits and risks of those alternatives. The manner in which information is provided should be similar to how other clinicians would do so (“professional standard”), give details that a typical well-reasoning individual would want to know (“reasonable person standard”), and also answer the specific questions the patient/proxy has (“subjective standard”). Obtaining consent for ICU patients presents numerous additional challenges. The patient is often unable to provide consent, proxies are not always present or available, a single ICU patient often needs numerous different procedures, and when an intervention is required it is often timesensitive and needs to be done urgently or emergently. One method ICUs have adapted to try and address this challenge is to use “bundled consents” where a patient/proxy consents upfront to several commonly needed ICU procedures (intubation, central line, arterial line, etc). Another method is to use “blanket consents,” which typically cover almost everything an ICU provider may need to do. These methods have been shown to result in markedly higher rates of consent obtained for procedures (increased from 53%-90% in one study) and improved family satisfaction. However, some argue that bundled or blanket consents may not allow for truly informed consent because the provider is not explaining the specific benefits and risks of an intervention (or alternatives to it) as they relate to the patient at a specific point in time in their ICU course. One potential way around this during conversations with a patient/proxy would be to describe for each intervention the common clinical scenario(s) in

which it might be considered, such as intubation for respiratory failure, and the benefits / risks / alternatives that would likely exist in that situation. References

1. Davis N, Pohlman A, Gehlbach B, et al. Improving the process of informed consent in the critically ill. JAMA. 2003;289(15):1963-1968.

2. Dhillon A, Tardini F, Bittner E, et al. Benefit of using a “bundled” consent for intensive care unit procedures as part of an early family meeting. J Crit Care. 2014;29(6):919-922.

2. Correct Answer: B Rationale: Use of NP and/or PA providers in the ICU is becoming increasingly common in the United States. Acute care/critical training for NPs has existed since 1995, and over 5000 have been certified to date. In contrast, PA providers all complete a broad, nonspecialized training and limited data exist on the number employed in an ICU setting. Collectively, only 4% to 6% of all NP/PA providers are acute care trained. Despite this, over 150 studies have examined the role of NP and/or PA providers in acute and critical care settings, and more than 30 have specifically examined their impact on patient care management. There is no evidence that using NP/PA providers results in increased costs for patients or the hospital. To the contrary, it appears that integrating them into multidisciplinary provider teams along with physicians and nurses leads to decreased hospital costs as well as cost savings to the patient. Numerous studies have compared NP/PA provider clinical outcomes to those of resident physicians in caring for acute care / critically ill patients, and shown similar outcomes or improved outcomes. One suggested reason for this is that resident physicians spend a relatively short portion of their residency working in the ICU, whereas in surveys it appears that the average NP/PA ICU provider reports having worked in that setting for several years or more. Furthermore, integration of NP/PA providers helps to provide clinical coverage and allow residents and fellows to attend didactics, conferences, fulfill other requirements outside

of the ICU, and meet duty hours. The increased ICU experience of many NP/PA providers compared to resident physicians rotating in the unit may also explain why integration of the former appears to have numerous other positive impacts on patient outcomes. However, many of the studies that have examined this topic had small sample sizes and short duration, and further study is needed. References Kleinpell R, Ely W, Grabenkort R. Nurse practitioners and physician assistants in the intensive care unit: an evidence-based review. Crit Care Med. 2008;36(10):2888-2897. Gershengorn H, Johnson M, Factor P. The use of nonphysician providers in adult intensive care units. Am J Respir Crit Care Med. 2012;185(6):600605. Kleinpell R, Ward N, Lynn K, et al. Provider to patient ratios for nurse practitioners and physician assistants in critical care units. Am J Crit Care. 2015;24(3):e16-e21.

3. Correct Answer: B Rationale: Medical care bundles typically consistent of a small collection of evidence-based practices that, when performed consistently and collectively, have been shown to improve patient outcomes. Examples of these that are frequently used in ICUs include bundles intended to decrease urinary catheter-associated UTIs and ones aimed at reducing central line associated blood stream infections. The ABCDE bundle was first described by Vasilevskis et al in 2010 and incorporates numerous evidence-based interventions with the collective stated goal of (1) improving collaboration among ICU team members, (2) standardizing care processes, and (3) breaking the cycle of oversedation and prolonged ventilation, which appear to cause delirium and weakness. In subsequent studies, the ABCDE bundle has been shown to be beneficial in both mechanically ventilated and nonventilated ICU patients. Risk of delirium in particular appears to be markedly lower, with an adjusted odds ratio of 0.55 (risk reduced by almost half) for patients

receiving ABCDE bundle care. Some of the other demonstrated benefits include reduced need for sedation medications, increased rate of out-ofbed ambulation, reduced ventilator days, and decreased hospital mortality. Use of the ABCDE bundle does not appear to result in more frequent need for reintubation or increased risk of patient self-extubation. It also does not appear to reduce time to discharge. References Vasilevskis E, Speroff T, Pun B, et al. Reducing iatrogenic risks: ICUAcquired delirium and weakness—crossing the quality chasm. Chest. 2010;138(5):1224-1233. Balas C, Vasilevskis E, Olsen M, et al. Effectiveness and safety of the awakening and breathing coordination, delirium monitoring/management, and early exercise/mobility bundle. Crit Care Med. 2014;42(5):1024-1036. Morandi A, Piva S, Ely E, et al. Worldwide survey of the “assessing pain, both spontaneous awakening and breathing trials, choice of drugs, delirium monitoring/management, early exercise/mobility, and family empowerment” (ABCDEF) bundle. Crit Care Med. 2017;45(11):e1111e1122. Marra A, Ely E, Pandharipande P, et al. The ABCDEF bundle in critical care. Crit Care Clin. 2017;33(2):225-243. Bounds M, Kram S, Speroni K, et al. Effect of ABCDE bundle implementation on prevalence of delirium in intensive care unit patients. Am J Crit Care. 2016;25(6):535-544. Ren X, Li JH, Peng C, et al. Effects of ABCDE bundle on hemodynamics in patients on mechanical ventilation. Med Sci Monit. 2017;23:46504656.

4. Correct Answer: D Rational: Delirium is a commonly diagnosed condition in the ICU, with reported incidences in various studies ranging from 45% up to as high as 87%. Delirium is defined in The Diagnostic and Statistical Manual of Mental Disorders, Fifth Edition (DSM-5) in part as “a disturbance in attention (ie, reduced ability to direct, focus, sustain, and shift attention) and awareness

(reduced orientation to the environment).” This typically develops over a short period of time (usually hours to a few days), represents a change from baseline attention and awareness, and tends to fluctuate in severity during the course of a day. Delirium can be further subclassified into hyperactive, hypoactive, or mixed type—with the latter two being the most commonly diagnosed in ICU patients. Despite the fact that ICU delirium is quite common, the complex pathophysiology with regard to how it actually develops is not well understood. Some broad categories that are thought to potentially contribute to delirium include neurotransmitter imbalance, inflammation, impaired oxidative metabolism, and availability of large neutral amino acids. Numerous risk factors have been reported, which includes advanced age, infection, hypotension, metabolic disturbance, respiratory illness, immobilization, and critical illness. ICU patients who develop delirium appear to be at markedly higher risk for both morbidity and mortality. One recent systemic review and meta-analysis showed that ICU patients with delirium had a “significantly higher risk of mortality” (odds ratio 2.2), as well as longer durations of mechanical intubation, ICU stay, and overall hospital stay. Given this, guidelines recommend daily assessment for delirium in ICU patients. The CAM-ICU is one of five currently validated screening tools to screen for delirium in adults. This and another tool, called the Intensive Care Delirium Screening Checklist (ICDSC), are the two best studied and most widely accepted. However, before using either method, clinicians must first assess the patient’s level of sedation. Perhaps the most common way to do this is the Richmond Agitation–Sedation Scale. If patients have a score greater than −3, then they can be assessed using the CAM-ICU tool. The information listed in the other question answers above is also important to assess in your patients but is not specifically needed to screen for delirium with CAM-ICU. References Kupfer D, Regier D, Narrow W, et al. Diagnostic and Statistical Manual of Mental Disorders. 5th ed. Washington, DC: American Psychiatric Publishing; 2013. Salluh J, Wang H, Schneider E, et al. Outcome of delirium in critically ill patients: systematic review and meta-analysis. Br Med J.

2015;350(8011):h2538. Girard T, Pandharipande P, Ely E. Delirium in the intensive care unit. Crit Care. 2008;12:s3. Arumugam S, El-Menyar A, Al-Hassani A, et al. Delirium in the intensive care unit. J Emerg Trauma Shock. 2017;10(1):37-46. Hayhurst C, Pandharipande P, Hughes C. Intensive care unit delirium: a review of diagnosis, prevention, and treatment. Anesthesiology. 2016;125(6):1229-1241. Khan B, Guzman O, Campbell N, Comparison and agreement between the richmond agitation-sedation scale and the riker sedation-agitation scale in evaluating patients’ eligibility for delirium assessment in the ICU. Chest. 2012;142(1):48-54.

5. Correct Answer: D Rationale: Critical care resources are often simultaneously vital, limited, and expensive. When this is combined with the inherent medical complexity of critically ill patients, it can make effectively and ethically triaging potential ICU admissions challenging. ICU care has been demonstrated to reduce mortality in patients who are critically ill, in particular within the first 3 days, suggesting that there is a “window of critical opportunity” and further emphasizing the importance of timeliness in ICU admissions. Numerous critical care organizations have released guidelines intended to help aid clinicians in approaching the difficult task of ICU triage. Examples of these include the 2016 Society of Critical Care Medicine’s “ICU Admission, Discharge, and Triage Guidelines,” the 1997 American Thoracic Society’s Bioethics Taskforce publication “Fair Allocation of Intensive Care Unit Resources,” and the 2016 World Federation of Societies of Intensive and Critical Care Medicine report entitled “Triage Decisions in ICU Admission.” Triage decisions should adhere to numerous fundamental principles including: 1.

ICU care, when medically appropriate, is an essential component of a basic package of healthcare services that should be available to all 2. The duty of healthcare providers is to benefit an individual patient;

3. 4. 5. 6. 7. 8. 9.

when doing so unfairly they compromise the availability of resources needed by others Providers should advocate for patients Members of the provider team should collaborate Care must be restricted in an equitable system Decisions to give care should be based on expected benefit Mechanisms for alternatives should be planned Explicit policies should be written Prior public notification is necessary

The actual process of ICU triage should consider in part the following: 1. 2. 3. 4.

Likelihood of benefit to the patient if admitted to the ICU Impact of ICU treatment in improving the patient’s quality of life Duration of potential benefit Urgency of the patient’s condition (ie how close the patient is to death) 5. Amount of resources required for successful treatment Three general models for triage have been proposed in previous critical care guidelines. These include the (1) prioritization model, where patients are categorized into four levels of priority based on likelihood of benefit from ICU admission, the (2) diagnosis model, where a list of specific diseases and conditions is used to decide which patients should be admitted to the ICU, and the (3) parameter model, where specific vital signs, laboratory findings, imaging or EKG results, or physical examination findings are used to guide decisions. At present, there is a dearth of validated tools to assist in triage decisions. Although such tools will likely appear in coming years, no policy or tool can replace the clinical judgment of experienced clinicians such as those on a critical care multiprofessional team. References Nates JL, Nunnally MS, Kleinpell RS, et al. ICU admission, discharge, and triage guidelines: a framework to enhance clinical operations, development of institutional policies, and further research. Crit Care Med. 2016;44(8):1553-1602. Blanch L, Abillama FF, Amin P, et al. Triage decisions for ICU admission:

report from the task force of the world federation of societies of intensive and critical care medicine. J Crit Care. 2016;36(C):301-305. Klein D, Olivier A, Milner B, et al. Fair allocation of intensive care unit resources. American Thoracic Society. Am J Respir Crit Care Med. 1997;156(4 Pt 1):1282-1301.

6. Correct Answer: C Rationale: Machine learning is a rapidly expanding scientific field that focuses on both how computers learn from data as well as with the development of algorithms to help make this learning possible and more efficient. Given the incredible amount of patient data generated by a modern ICU each day, there is growing interest in using machine learning to analyze these massive datasets to provide more timely and effective care. However, critical care medicine presents numerous challenges to effectively implementing machine learning, including in part (1) compartmentalization of data across multiple systems, (2) data corruption, and (3) the incredible complexity inherent in most ICU patients. After years of mostly hypothetical discussion on the value of machine learning in improving critical care medicine, within the past year or two numerous studies have been published reporting promising potential in specific topics. Some of these include sepsis detection (machine learning model detected sepsis in ICU patients 4-12 hours before clinical detection), predicting complications in postsurgical ICU patients (serious bleeding, kidney failure, mortality), predicting ICU readmission (which is associated with markedly worse outcomes), and predicting the need for prolonged mechanical ventilation (and need for tracheostomy). However, great care must also be taken when designing, creating, testing, and implementing machine learning–based decision support systems (ML-DSS), as they can have unforeseen consequences and generate unexpected results. For example, studies examining computeraided detection of radiology and EKG findings showed decreased sensitivity and accuracy when using them compared to non–computeraided detection. Similarly, in a study of over 14 000 pneumonia patients that was presented in 2015, the researchers reported that their ML-DSS analyzed the provided data and decided that the patients at lowest risk of

complication, and thus the safest to discharge on outpatient management, were those with asthma (C). In particular, machine learning systems struggle when dealing with intrinsic uncertainties and clinical scenarios where there is not a clear “right” answer but expert variation in practice style (“the art of medicine”)—both of which are common in critical care medicine. Finally, because ML-DSS use algorithms that are typically inscrutable to clinicians (and with machine-learning / neural networks, that can be inscrutable even to the engineers who designed the system), there is concern that with cases less extreme than the pneumonia/asthma one mentioned above, incorrect results generated by the computer may not be detected by clinicians and could lead to patient harm. To help prevent these types of negative outcomes, ML-DSS products should offer explanations to clinicians of their results/recommendations, and clinicians must become familiar with the ML-DSS products they use—including possible limitations and sources of error. Although the few existing machine learning generated ICU models (listed above) are still novel, initial comparisons of each of them with existing previously validated detection/prediction tools appear to suggest that they perform significantly better. Machine learning generated tools such as these, particularly once validated (preferably with prospective studies), have the potential to be integrated into ICU electronic health records to better facilitate early and excellent provider interventions. However, care must be taken in designing and implementing these MLDSS tools to avoid potential harm to patients. References Nemati S, Holder A, Razmi F, et al. An interpretable machine learning model for accurate prediction of sepsis in the ICU. Crit Care Med. 2018;46(4):547-553. Meyer A, Zverinski D, Pfahringer B, et al. Machine learning for real-time prediction of complications in critical care: a retrospective study. Lancet Respir Med. 2018;6(12):905-914. Rojas J, Carey K, Edelson D, et al. Predicting intensive care unit readmission with machine learning using electronic health record data. Ann Am Thorac Soc. 2018;15(7):846-853. Desautels T, Das R, Calvert J, et al. Prediction of early unplanned intensive care unit readmission in a UK tertiary care hospital: a cross-

sectional machine learning approach. BMJ Open. 2017;7(9):e017199. Parreco J, Hidalgo A, Parks J, Kozol R, Rattan R. Using artificial intelligence to predict prolonged mechanical ventilation and tracheostomy placement. J Surg Res. 2018;228:179-187. Sanchez-Pinto L, Luo Y, Churpek M. Big data and data science in critical care. Chest. 2018;154(5):1239-1248. Johnson A, Ghassemi M, Nemati S, et al. Machine learning and decision support in critical care. Proc IEEE Inst Electr Electron Eng. 2016;104(2):444-466. Pollard T, Celi L. Enabling machine learning in critical care. ICU Manage Pract. 2017;17(3):198-199. Cabitza F, Rasoini R, Gensini GF. Unintended consequences of machine learning in medicine. JAMA. 2017;318(6):517-518. Shortliffe E, Sepúlveda M. Clinical decision support in the era of artificial intelligence. JAMA. 2018;320(21):2199-2200.

7. Correct Answer: E Rationale: The Health Insurance Portability and Accountability Act (HIPAA) was passed in August 1996 and was intended to both make health care delivery more efficient and increase the percentage of Americans covered by health insurance. Several years later, in 1999 the US Department of Health and Human Services (HHS) proposed an additional HIPAA Privacy Rule, and the final iteration was issued in 2002. The HIPAA Privacy Rule controls how personally identifiable patient information can be stored, accessed, and used. In general, a patient’s personally identifiable information cannot be stored, accessed, or used without their explicit permission. However, the HIPAA Privacy Rule was carefully designed to still allow healthcare researchers to access such data for research purposes—provided that they follow certain requirements and protocols. This hypothetical case presents several challenging legal and ethical issues. First, providers are ethically obliged to provide proxies with all relevant information about the patient’s medical condition and potentially contributory medical history to empower the proxy to make the bestinformed decision possible on behalf of the patient. Because the patient’s

critical illness is likely due at least in part to her untreated AIDS, this is important information that would likely help her husband better understand her overall condition and prognosis. In general, disclosure is justified when: (1) the patient is at significant risk of unnecessary suffering; (2) the fully informed action of others (such as the medical proxy) is needed to prevent this suffering; (3) the action of others has a high probability of preventing suffering; and (4) the benefit that the patient can be expected to gain outweighs any harms, costs, or burdens that others are likely to incur by the proxy being fully informed. Furthermore, the patient’s AIDS and intermittently unprotected sex with her husband places him at serious risk for harm, and the ICU clinician has a duty to inform him so that he can get testing and/or treatment if desired by him. HIPAA does not forbid sharing of patient information for either of these purposes. Furthermore, there is legal precedent (including the Tarasoff case of 1976) that healthcare professionals are not only protected but have a duty to act in cases such as this, even if doing so breaches confidentiality with the patient, in cases where not doing so would result in an imminent threat to the patient or to an identifiable third party. In this case, not fully informing the husband/proxy poses an imminent threat to the patient (not fully informed medical decision-making by the proxy) and to the proxy. Finally, in challenging situations such as this one, remember that most hospitals have an ethics committee and legal representative, both of whom are typically available on-call and can help you do right by your patients and their families and provide the most clinically excellent, ethically, and legally defensible care possible. References Nass S, Levit L, Gostin L, et al. Beyond the HIPAA privacy rule: enhancing privacy, improving health through research. In: Institute of Medicine (US) Committee on Health Research and the Privacy of Health Information: The HIPAA Privacy Rule. Washington, DC: National Academies Press (US); 2009. Available at https://www.ncbi.nlm.nih.gov/books/NBK9576/. Vernillo A, Wolpe P, Halpern S. Re-examining ethical obligations in the intensive care unit: hiv disclosure to surrogates. Crit Care. 2007;11(2):125.

8. Correct Answer: D Rationale: In 2009, the US Department of Health and Human Services tasked the nonprofit organization National Quality Form (NQF) with developing quality and efficiency metrics for use within the US healthcare system. Over 600 of these metrics have been created to date, though only a small percentage of these are unambiguously attributable and/or relevant to ICU medicine. Some of these include ICU length of stay, in-hospital mortality rate, rate of catheter-associated bloodstream infections, and sepsis management bundle—but not avoidance of delirium. The subsequent passage of the US Affordable Care Act in 2010 brought with it increased focus on incentivizing efficient and coordinated care, and subsequent federal legislative developments, such as the passage of the Medicare Access and CHIP Reauthorization Act of 2015, further prioritized value-based repayment models. As of this year (2018), it is estimated that 90% of all Medicare payments will be performance based and in the near future up to 10% of physicians’ annual salary adjustments will be tied to performance. Critical care medicine is a particularly important target for these national initiatives as ICU costs are high. The estimated total annual cost of ICU beds in the United States increased from $56.6 billion in 2000, to $81.7 billion in 2005, to approximately $108 billion in 2010. The number of ICU beds increased nationally during the same periods of time, but at a much lower rate—and roughly 75% of these new beds were created in only a small percentage (25%) of regions. However, it’s important to also note that costs vary widely between different ICUs, also vary by day of ICU admission (the first day or two of an ICU stay typically costs more than the subsequent days), and are only one component of value in healthcare for the critically ill. Given the lack of standardized metrics for evaluating quality and efficiency in ICU patients, some national critical care organizations are advocating to develop and test these metrics themselves instead of being constrained to those developed by a nonmedical third party (such as NQF). Others suggest that individual ICUs should be allowed to choose their own metrics—perhaps from a list of approved options. Although the best pathway forward into value-based critical care delivery

remains unclear and fiercely debated, what is certain is that it will profoundly impact the way future generations of ICU clinicians provide (and are reimbursed for) their expert subspecialty care. References

Nguyen A, Hyder J, Wanta B, et al. Measuring intensive care unit performance after sustainable growth rate reform: an example with the national quality forum metrics. J Crit Care. 2016;36:81-84. Murphy D, Ogbu O, Coopersmith C. ICU director data: using data to assess value, inform local change, and relate to the external world. Chest. 2015;147(4):1168-1178. Rubenfield G. Do We Need More Public Reporting? Critical Care Canada Forum. Presented November 5, 2011. Available at https://criticalcarecanada.com/presentations/2011/do_we_need_more_public_r Chrusch C, Martin C. Quality improvement in critical care: selection and development of quality indicators. Can Respir J. 2016;2016:11. Churpek M, Hall J. Measuring and rewarding quality in the ICU: the yardstick is not as straight as we wish. Am J Respir Crit Care Med. 2012;185(1):3-4. Pastores S, Halpern N. Insights into intensive care unit bed expansion in the United States. National and Regional Analyses. Am J Respir Crit Care Med. 2015;191(4):365-366. Halpern N, Pastores S. Critical care medicine beds, use, occupancy, and costs in the United States: a methodological review. Crit Care Med. 2015;43(11):2452-2459.

9. Correct Answer: C Rationale: The concept of using scoring systems based on physiologic data to try and predict ICU patient outcomes was first introduced over three decades ago. Since then, the three most commonly used ICU scoring systems have been the APACHE, MPM0, and SAPS. Each of these has been updated numerous times over the years and offers various benefits and insights, their use is widely supported, and some European countries have made

their use mandatory. They have each been validated in numerous regions of the world and in distinct different groups of ICU patients as well as specific subgroups such as critically ill cancer patients, cardiovascular, surgical, acute kidney injury requiring renal replacement therapy, and patients in need of extracorporeal membrane oxygen. Of note though, only the APACHE provides both mortality and length of stay predictions. Despite these benefits, in the United States only 10% to 15% of ICUs report using scoring systems such as these. Ideally, a scoring system would have the following characteristics (1) scores calculated on the basis of easily/routinely recordable variables, (2) well calibrated and validated, (3) a high level of discrimination, (4) applicable to all patient populations in ICU, (5) can be used in different countries, health systems, or patient cohorts, and (6) the ability to predict mortality, functional status, or quality of life after ICU discharge. Scoring systems are typically described in terms of their “discrimination” and “calibration.” Discrimination in this case refers to the ability of the scoring system to acutely predict the patients at highest risk for mortality. Discrimination is often described using a “receiver operating characteristic (ROC) curve,” with ROC of 0.5 being no better than chance, and values >0.7, 0.8, and 0.9 being considered acceptable, excellent, and outstanding, respectively. Calibration measures how well actual outcomes match their predicted incidence across different groups and is often calculated with the Hosmer-Lemeshow C statistic. Ideally, a scoring system would perform equally well across all risk strata. One metric generated by scoring systems and commonly used to evaluate ICU performance is the “standardized mortality ratio (SMR),” which is the ratio of the observed or actual patient mortality and the predicted general mortality for that hospital/unit/team over the same period of time. However, SMRs are significantly affected by outcomes in high-risk patient populations, as well as differences in patient population. For example, facilities that frequently transfer their sickest patients out to a quaternary facility may have a falsely low SMR, even if their ICU performance is not in fact as high. Simply reporting SMR does not account for differences in the patient population and does not convey details such as presenting acuity or patient comorbidities. In summary, scoring systems such as the APACHE, MPM0, and SAPS have the potential to help ICU clinicians better understand and predict patient outcomes to guide both clinical care and research. However, it is

important to understand how a given scoring system works, when using it, and realize its potential weakness and limitations. References Breslow MJ, Badawi O. Severity scoring in the critically ill: part 1– interpretation and accuracy of outcome prediction scoring systems. Chest. 2012;141(1):245-252. Breslow MJ, Badawi O. Severity scoring in the critically ill: part 2: maximizing value from outcome prediction scoring systems. Chest. 2012;141(2):518-527. Vincent JL, Moreno R. Clinical review: scoring systems in the critically ill. Crit Care. 2010;14(2):207. Kuzniewicz M, Vasilevskis E, Lane R, et al. Variation in ICU risk-adjusted mortality: impact of methods of assessment and potential confounders: impact of methods of assessment and potential confounders. Chest. 2008;133(6):1319-1327. Salluh J, Soares M. ICU severity of illness scores: APACHE, SAPS and MPM. Curr Opin Crit Care. 2014;20(5):557-565. Singh J, Gupta G, Garg R, et al. Evaluation of trauma and prediction of outcome using TRISS method. J Emerg Trauma Shock. 2011;4(4):446-449. Wu B, Johannes R, Sun X, et al. The early prediction of mortality in acute pancreatitis: a large population-based study. Gut. 2008;57(12):16981703.

10. Correct Answer: A Rationale: Patients are at particularly high risk of inadvertent nosocomial harm while in the ICU. This is likely due to numerous factors including, in part, the complex medical condition(s) they are suffering from, the number of interventions performed on them daily by ICU staff (estimated to be over 140 interventions per day for each patient), and the busy and challenging environment of the ICU. Sentinel events, which could or do harm the patient, are frequently underreported. The data that do exist, while thought to be an

underestimate, suggest the rate of these is between 40 and 80 per 100 patient days in the ICU. 
The largest study to date that has examined these events is the “Sentinel Events Evaluation (SEE) study,” which studied patients in 205 ICUs around the world. In it, the following were shown to place the patient at higher risk for experiencing a sentinel event: (1) any organ failure, (2) a higher intensity in level of care, and (3) and time of exposure. Given the relative frequency of potential harmful events in ICU patients, a great deal of research has gone into how to best improve timely and accurate reporting of these mistakes as well as how to avoid them. Nurses appear to typically be the most frequent reporters of potentially harmful events that occurred in the ICU, and reporting rates appear to improve when the method of reporting is simplified. (One study showed a three- to fourfold improvement in response rates by switching from a digital reporting system back to a simplified paper card system.) Numerous factors have been suggested as potentially decreasing the rate of potentially harmful events. Some of these include improved nurse to patient ratio, increased number of physicians, full-time ICU specialists, and others. However, some of the best evidence for beneficial effect appears to be around multidisciplinary ICU rounds and digital prescribing. References Moreno R, Rhodes P, Donchin A. Patient safety in intensive care medicine: the declaration of Vienna. Intensive Care Med. 2009;35(10):1667-1672. Valentin A, Capuzzo M, Guidet B, et al. Patient safety in intensive care: results from the multinational sentinel events evaluation (SEE) study. Intensive Care Med. 2006;32:1591-1598. Rothschild J, Landrigan C, Cronin J, et al. The critical care safety study: the incidence and nature of adverse events and serious medical errors in intensive care. Crit Care Med. 2005;33(8):1694-1700. Rothen H, Stricker U, Einfalt K, et al. Variability in outcome and resource use in intensive care units. Intensive Care Med. 2007;33(8):1329-1336. Ilan R, Squires M, Panopoulos C, et al. Increasing patient safety event reporting in 2 intensive care units: a prospective interventional study. J Crit Care. 2011;26(4):431.e11-431.e18.

C H AP T E R 1 1 4

Teaching Ryan J. Horvath

1. Several residents approach you, an intensivist, to mentor them through a Quality improvement (QI) project. You advise them that the three components of quality of care are structure, process, and outcome. Which of the following is an accepted model for approaching QI that you can suggest the residents use?

A. Quality-adjusted life years (QALY) B. Structure-Process-Outcome cycle (SPO cycle) C. Plan-Do-Study-Act cycle (PDSA cycle) D. I-PASS E. Situation-Background-Assessment-Recommendation (SBAR)

2. During rounds in the intensive care unit (ICU), you begin discussing closed versus open ICU structure and processes that improve delivery of critical care with a group of residents and fellows. Which of the following has been associated with lower patient mortality across ICUs with divergent hospital structure (teaching hospital vs private practice), care team structure (solo intensivist vs mixed care team), and patient mix (surgical vs medical ICU)?

A. Use of daily goals forms B. Dedicated ICU nursing managers and medical directors C. Avoidance of protocols (ie ventilator management, antibiotic usage, venous thromboembolism preventions, hypoglycemia management) D. Staffing models that rely heavily on residents, nurse practitioners, physician assistants, or other care extenders

E. Multidisciplinary rounds at least twice daily

3. Recently, resident evaluations of their ICU rotations have revealed significant dissatisfaction with the traditional didactic teaching sessions. The director of the ICU asks you to develop a “flipped classroom” educational series for your ICU. Which of the following would be considered a model of a “flipped classroom”?

A. Moving the location of teaching sessions out of the ICU and into an area away from clinical care and more conducive to learning. B. Instead of having attendings lecture to residents, allow residents to drive their own learning by asking questions of their staff during didactic sessions. C. Provide answers to problem sets or clinical questions to residents before teaching sessions to focus their learning. D. Completely abolish didactic classroom learning and initiate onlineonly learning modules. E. Have students first complete independent, self-directed education outside of the classroom and then do learning activities inside the classroom that builds upon prior work.

Chapter 114 Answers 1. Correct Answer: C Rationale: Governmental agencies with healthcare oversight and hospital administration have placed increased focus on the delivery of quality health care in recent years. The Institute of Medicine defines quality care by six metrics: care that is safe, timely, effective, efficient, equitable, and patient centered. Quality care is not only a method by which hospitals and healthcare systems are judged but increasingly plays a role in their financial reimbursement. This incentivizes healthcare professionals to improve the quality of care that they deliver to patients. The Institute for Healthcare Improvement recommends the PDSA cycle (answer C) model as an effective means of quality improvement. The first step in this model is to study the identified problem and come up with a Plan tailored to the environment and healthcare setting. The next steps are to implement a change or intervention (Do) and evaluate the results (Study). This information should be included in the improvement effort (Act), which can be further refined and improved in subsequent PDSA cycles. QALY (answer A) is an outcome measure for clinical trials, I-PASS (answer D) and SBAR (answer E) are handoff tools commonly used in the ICU, and Structure-Process-Outcome (answer B) are the components of quality care, but not an accepted model for approaching QI. References

1. Curtis JR, Cook DJ, Wall RJ, et al. Intensive care unit quality improvement: a “how-to” guide for the interdisciplinary team. Crit Care Med. 2006;34:211-218.

2. Hahn J, Cummings BM. Quality Improvement and Standardization of Practice. In: Wiener-Kronish JP, ed. Critical Care handbook of the Massachusetts General Hospital. 6th ed. Philadelphia: Wolters Kluwer; 2016:575-588.

2. Correct Answer: A

Rationale: There has been significant interest in the characteristics that improve outcomes in the ICUs. Across the Unites States, there is significant variation on the delivery of critical care including: ICU type: Medical ICU, surgical ICU, cardiac ICU, neuro ICU, burn ICU, etc. Structure: Open ICUs where the admitting physician remains the patient’s primary caregiver after transfer to the ICU, and intensivists are available for consult purposes versus closed ICUs where there is a dedicated ICU care team including an intensivist who assumes primary responsibility for patients on transfer to the ICU. Hospital location: Large city versus rural. ICU team structure: Private practice solo practitioner intensivist, mixed structures with nurse practitioners and physician assistants, teaching hospitals with academic practice and resident teams, etc. Intensivist coverage: 24-hour in-house intensivist coverage, night home call intensivist coverage, etc.

Many studies exist that show incremental improvement in patient outcomes (usually low-level outcomes such as ventilator-free days or total ICU stay and not higher level outcomes such as overall morbidity and mortality) in individual or groups of ICUs before and after an intervention. However, with such variation in the structure and delivery critical care across the United States, it has been a challenge to extrapolate these finding more broadly. In 2014, the United States Critical Illness and Injury Trails Group Critical Illness Outcomes Study (USCIITG-CIOS) studies the structure, process, and mortality across 69 divergent ICUs across the United States. Factors associated with lower mortality included use of a daily plan of care review (answer A) and lower bed-to-nurse ratios. 24-hour intensivist coverage and closed ICU status were not found to be associated with improved mortality. References

1. Checkly W, Martin GS, Brown SM, et al. Structure, process and annual intensive care unit mortality across 69 centers: United States Critical Illness and Injury Trials Group Critical Illness Outcomes Study (USCIITG-CIOS). Crit Care Med. 2014;42(2):344-356.

2. Frankel SK, Moss M. The effect of organizational structure and processes of care on ICU mortality as revealed by the USCIITG-critical illness outcomes study. Crit Care Med. 2014;42(2):463-464.

3. Correct Answer: E Rationale: Teaching in the ICU can be challenging for many reasons including: a wide range in learners from med students through residents and fellows; a wide range of past experiences from learners; varying acuity and volume of patients in the ICU; the fast-paced atmosphere that often allows little time without interruption. Additionally, learners are increasingly requesting more learner-centric opportunities as compared to more traditional didactic lectures. The “flipped classroom” model involves having students perform independent self-directed learning activities before ​education sessions, usually involving online modules, videos, etc. Educational sessions are then used to focus on reinforcing what was learned before the sessions. In this way, the method by which information is presented to learners is reversed or “flipped” compared to traditional educational sessions. References

1. Peets AD, McLaughlin K, Lockyer J, et al. So much to teach, so little time: a prospective cohort study evaluating a tool to select content for a critical care curriculum. Crit Care. 2008;12(5):R127.

2. Tainter CR, Wong NL, Cudemas-Desada GA, et al. The “Flipped Classroom” model for teaching in the intensive care unit: rationale, practical considerations, and an example of successful implementation. J Intensive Care. 2017;32(3):187-196.

C H AP T E R 1 1 5

Psychosocial Issue Among Providers Jennifer Cottral and William J. Benedetto

1. Classic symptoms of burnout syndrome (BOS) as defined by the Maslach Burnout Inventory (MBI) include all of the following EXCEPT:

A. Exhaustion B. Moral distress C. Depersonalization D. Reduced personal accomplishment

2. What is the most commonly misused substance among physicians with substance use disorders?

A. Alcohol B. Opioids C. Benzodiazepines D. Controlled stimulants

Chapter 115 Answers 1. Correct Answer: B Rationale: BOS is a pattern of nonspecific signs and symptoms that result from work-related stressors, often seen in individuals with no history of psychological or psychiatric disorders. BOS was first described in healthcare workers in the 1970s, and by the 1980s, evaluation criteria became available through the design of a standard measurement instrument called the Maslach Burnout Inventory (MBI). The MBI quantifies the presence and severity of the three classic symptoms of BOS that include exhaustion, depersonalization, and reduced personal accomplishment. Exhaustion is physical and/or emotional fatigue related to devoting disproportionate time and effort to a task that is not perceived by the person to be beneficial. Depersonalization is feeling disconnected from one’s work and colleagues and can manifest as negative, callous, and cynical behaviors. Reduced personal accomplishment is negatively evaluating one’s job performance and the worth of one’s work. Moral distress occurs when an individual knows the ethical action to take but feels constrained from taking this action. Moral distress can overlap with or contribute to BOS but is not a defined symptom of BOS as classically described by the MBI. References

1. Moss M, Good VS, Gozal D, et al. An official critical care societies collaborative statement: burnout syndrome in critical care healthcare professionals: a call for action. Crit Care Med. 2016;44(7):1414-1421.

2. Weber A. Burnout syndrome: a disease of modern societies? Occup Med (Lond). 2000;50(7):512-517.

3. Fourie C. Who is experiencing what kind of moral distress? Distinctions for moving from a narrow to a broad definition of moral distress. AMA J Ethics. 2017;19(6):578-584.

2. Correct Answer: A

Rationale: An often-quoted prevalence of substance use disorders among physicians is 10% to 12%, which is based on a paper published over 25 years ago. More recent volunteer survey data among attending and trainee physicians suggest that the prevalence of substance use disorders among physicians is much higher, though predominantly represented by alcohol use disorder. Indeed, over the course of over 3 decades of research, alcohol has been the most frequently misused substance among physicians surveyed. This is also true for those physicians treated for substance use disorders, followed by opioids. Physicians have an ethical and (depending on the state) legal responsibility to report impaired colleagues. However, many are reluctant to report a colleague, even when the suspect is impaired or incompetent. This is due to belief that someone else was taking care of the problem, that nothing would happen as a result of the report, or due to fear of retaliation. References

1. Berge KH, Seppala MD, Schipper AM. Chemical dependency and the physician. Mayo Clin Proc. 2009;84(7):625-631.

2. McLellan AT, Skipper GS, Campbell M, DuPont RL. Five year out- comes in a cohort study of physicians treated for substance use disorders in the United States. BMJ. 2008;337:a2038.

3. Oreskovich MR, Shanafelt T, Dyrbye LN, et al. The prevalence of substance use disorders in American physicians. Am J Addict. 2015;24(1):30-38.

4. Baldisseri MR. Impaired healthcare professional. Crit Care Med. 2007;35:S106-S116.

5. Taub S, Morin K, Goldrich MS, Ray P, Benjamin R; Council on Ethical and Judicial Affairs of the American Medical Association. Physician health and wellness. Occup Med. 2006;56:77-82.

6. DesRoches CM, Rao SR, Fromson JA, et al. Physicians’ perceptions, preparedness for reporting, and experiences related to impaired and incompetent colleagues. JAMA. 2010;304:187-193.

C H AP T E R 1 1 6

Ethical Considerations Theresa Barnes, William J. Trudo and Avneep Aggarwal

1. A 62-year-old man with new onset shortness of breath is admitted to the ICU with a large right-sided pleural effusion, requiring noninvasive positive pressure ventilation because of increased work of breathing. CT imaging is suggestive of lung cancer with metastasis to the liver. Prior to disclosing results of imaging to the patient, the patient’s daughter asks to speak with you privately. She states that her father does not wish to know anything regarding his condition. What is the MOST appropriate next step?

A. Determine why the daughter believes that her father does not want to know information regarding his diagnosis B. Ask the patient about his preferences for knowing results of imaging, likely diagnosis, and plans of care C. Further assess if the patient has capacity to make decisions D. Involve the ethics committee to determine if the patient is competent to make decisions

2. Which of the following is true regarding the concept of respect for autonomy?

A. A patient may be coerced into a treatment by using a credible threat B. A patient may be persuaded into a treatment by using a legitimate argument C. Physicians are obligated to “do whatever the patient wants” D. If a patient refuses a treatment, there is no way a physician can be liable for injuries occurred owing to lack of treatment

3. A 78-year-old woman is admitted for choledocholithiasis, diagnosed by magnetic resonance cholangiopancreatography (MRCP), with mildly elevated liver function tests and serum bilirubin of 1.5 mg/dL. Serum amylase and lipase are normal and temperature is 37.1°C. Which of the following is LEAST appropriate to discuss while consenting the patient for endoscopic retrograde cholangiopancreatography (ERCP)?

A. The diagnosis of choledocholithiasis and need for additional procedures after ERCP B. Propose and explain ERCP and alternative treatment options C. The contact information of family members not present D. The risks of refusing treatment

4. A 55-year-old male presents with a left middle cerebral artery stroke. Shortly after arriving at the hospital, his mental status declines, and he is intubated. His wife is his durable power of attorney for healthcare decisions. She notes that he did not want to end up, “connected to machines to keep him alive,” and his living will states: “I do not want aggressive measures to extend my life.” The neurointerventional team feels that his prognosis is likely good with an endovascular revascularization procedure, but the wife is hesitant to consent to this procedure. What is the most appropriate next step?

A. Explore the patient’s wishes with regard to medical care with his wife B. Proceed with the procedure without his wife’s consent because it is likely lifesaving, and any delay may make his outcome worse C. Consult the hospital ethics committee D. Consult other members of the family and seek their consent for the procedure

5. Which of the following is the correct order for highest to lowest priority of surrogate decision makers for an incompetent adult patient?

A. Individual appointed by patient, spouse, adult child, parent, adult sibling B. Individual appointed by patient, spouse, parent, adult child, adult sibling C. Individual appointed by patient, spouse, adult sibling, adult child, parent D. Individual appointed by patient, spouse, parent, adult sibling, adult child

Chapter 116 Answers 1. Correct Answer: B Rationale: Patient autonomy involves the right to determine which treatment will and will not be accepted. The exertion of autonomy requires that the patient be free to make their own decision with adequate knowledge and freedom from controlling influences about their medical care. If a patient chooses to make a family member a healthcare proxy, the patient must do so independently, with the understanding that further medical information may be withheld from them. In this particular situation, a discussion with the patient would need to occur first (Answer B). Family and group dynamics have a strong impact on decision making in many healthcare contexts. However, as the patient has not defined his daughter as a healthcare proxy, and there is no reason to question his decision-making capacity (no history of dementia, psychosis, intoxication, altered mental status, etc), it would be inappropriate at this time to relinquish the patient’s right to know his medical details (Answer A and C). With the information provided, there is no indication that the patient lacks decisional capacity. Legal determination of competency would need to be assessed if incapacity were established and the family was unable to come to a joint decision regarding care (Answer D). References

1. Entwistle VA, Carter SM, Cribb A, McCaffery K. Supporting patient autonomy: the importance of clinician-patient relationships. J Gen Intern Med. 2010;25(7):741-745.

2. Jonsen AR, Siegler M, Winslade WJ. Clinical Ethics: A Practical Approach to Ethical Decisions in Clinical Medicine. 8th ed. New York: McGrawHill; 2015.

2. Correct Answer: B Rationale:

A key point of informed consent of patients is the concept of respect for autonomy. In order to fulfill this principle, patients must be fully informed regarding risks, benefits, and alternative treatment options. Presenting information about different treatment options can include a component of persuasion, the act of influencing a decision with legitimate arguments (Answer B). For example, you might explain to a patient that the placement of a central venous catheter will ensure the safe delivery of vasoactive medications compared with a peripheral intravenous line. However, coercion with threats is not an appropriate method of communication when presenting treatment plans (Answer A). It would be inappropriate to say, “If you don’t let us put in the central line, and we continue to use the peripheral IV for these medications, you’ll lose your arm.” In this case, the (somewhat) credible threat is coercion. A physician cannot “do whatever the patient wants,” if the patient does not have a proper understanding of all medical options (Answer C). Furthermore, if a patient refuses a treatment, the patient must have adequate understanding of the complications that could be associated with their refusal. If the patient is not truly making an informed refusal, then the physician may be liable for injury resulting from a lack of information held by the patient. Reference

1. Waisel DB, Truog RD. Informed consent. Anesthesiology. 1997;87(4):968978.

3. Correct Answer: C Rationale: There is a legal and ethical responsibility of a physician to provide sufficient information about medical procedures to a patient, allowing him or her to process the information and then choose an appropriate decision. Informed consent should include: 1.

Reasoning behind and purpose of procedure, with potential need for further procedures 2. Explanation of procedure

3. 4. 5.

Alternative options Risks and benefits of procedure Risks of refusing treatment

Unless the patient is determined by the medical provider to lack decisional capacity (intoxicated, delirious, demented, comatose, or is unable to remember/understand what is being said) or has been legally determined to lack competency, family members or a healthcare proxy do not need to be contacted or available during consent for the procedure (Answer C). References

1. Entwistle VA, Carter SM, Cribb A, McCaffery K. Supporting patient autonomy: the importance of clinician-patient relationships. J Gen Intern Med. 2010;25(7):741-745.

2. Mandava A, Pace C, Campbell B, Emanuel E, Grady C. The quality of informed consent: mapping the landscape. A review of empirical data from developing and developed countries. J Med Ethics. 2012;38(6):356365.

3. White SM. Ethical and legal aspects of anesthesia for the elderly. Anesthesia. 2014;69(suppl 1):45-53.

4. Jonsen AR, Siegler M, Winslade WJ. Clinical Ethics: A Practical Approach to Ethical Decisions in Clinical Medicine. 8th ed. New York: McGrawHill; 2015.

4. Correct Answer: A Rationale: This patient lacks capacity to decide medical treatments secondary to his impaired mental status. Because his living will is somewhat vague regarding his wishes, the principle of substituted judgment should be followed. This principle calls for a surrogate decision maker to choose options for the patient based on their understanding of the patient’s beliefs and values. The patient’s wife, who is his durable power of attorney, is serving as the patient’s legal surrogate decision maker in this case, and

she has the final say with regard to medical treatments. It is appropriate to discuss the treatment with her and seek informed consent, and if she does not agree, explore her reasoning further (Answer A). She has the sole legal power to refuse any further medical treatments for her husband even if his prognosis is not necessarily terminal (Answer B). If other family members oppose this decision, they cannot overrule her choice (Answer D). If there is a lack of consensus among family members regarding treatment decisions, a hospital ethics committee consultation may be appropriate (Answer C). References

1. Bernat JL. Clinical ethics and the law. In: Ethical issues in neurology. 3rd ed. Philadelphia: Lippincott Williams & Wilkins; 2008:81-110.

2. Adelman EE, Zahuranec DB. Surrogate decision making in neurocritical care. Continuum (Minneap Minn). 2012;18(3):655-658.

5. Correct Answer: A Rationale: If an adult patient is deemed incapacitated, the hierarchy followed for surrogate decision makers is as follows: 1. 2. 3. 4. 5.

Individual appointed by patient Spouse Adult child Parent Adult sibling

If the patient has an advance directive, then the surrogate decision maker should use this document to help guide their medical choices for the patient. The surrogate should also employ the substituted judgment principle, making choices that the incompetent individual would choose if they were competent. References

1. American Bar Association Commission on Law and Aging. Default

Surrogate Consent Statutes. 2018. Available at https://www.americanbar.org/content/dam/aba/administrative/law_aging/2014_

2. Buchanan AE, Brock DW. Deciding for Others: The Ethics Of Surrogate Decision Making. Cambridge: Cambridge University Press; 1989.

C H AP T E R 1 1 7

Patient Confidentiality, Healthcare Policy Ryan J. Horvath

1. Which of the following is LEAST likely to reduce central line– associated bloodstream infections (CLABSI) in critically ill patients?

A. Use of full body drapes when inserting central venous catheters (CVCs) B. Performing hand hygiene before placement or manipulation of CVCs C. Limiting number of people in the room during placement of the CVC to only those needed for placement and immediate patient care D. Routinely culturing the tip of CVCs on removal E. Avoidance of femoral vein CVCs if subclavian or internal jugular sites are available

2. Which of the following is MOST likely to reduce catheter-associated urinary tract infection (CAUTI) in the intensive care unit (ICU)?

A. Routine use of urinary catheters for all postoperative patients B. Intermediate catheterization for patients with bladder emptying dysfunction C. Bladder irrigation with antimicrobial agents D. Maintain collecting systems above the level of the bladder E. Leave catheters unsecured to allow free movement of the collecting system

3. Under the Standards for Privacy of Individually Identifiable Health Information established under the Health Insurance Portability and Accountability Act of 1996 (HIPAA), which of the following is NOT a permitted disclosure of protected health information under “Public Interest and Benefit Activities”?

A. Disclosure of the results of beta-HCG blood tests to the parents of a 15-year-old female recently admitted to the ICU B. Disclosure of an incident of abuse or neglect to government authorities C. Disclosure of protected information to the medical examiner in the case of a death in the ICU D. Disclosure of a diagnosis of measles to state authorities and the Centers for Disease Control E. Disclosure of protected information to the United Network for Organ Sharing (UNOS)

4. Regarding infection control in the ICU, which of the following are standard precautions shown to be most effective in decreasing the transmission of hospital-acquired infections?

A. Positive pressure ICU rooms for all patients on “respiratory precautions” B. Limiting patient visitors to those patients on “contact precautions” C. Use of an alcohol-based hand cleanser or washing with soap and water between patient contacts D. Requiring ICU staff to wear dedicated clothing only for use in the ICU, and banning white coats as they are notorious fomites E. Use of sterile gloves and gowns as personal protective equipment during all patient interactions

5. The HIPAA Privacy Rule permits healthcare providers to disclose

protected healthcare information concerning a patient with major depression, who has capacity and requests that his or her family not be contacted in which of the following situations?

A. When the family member is the healthcare proxy B. When the healthcare provider perceives a serious patient safety concern C. When the family member is the spouse or next of kin D. When the healthcare provider does not agree with the patient’s healthcare decisions E. When the healthcare provider believes that the patient’s medical decisions are being influenced by their mental illness

Chapter 117 Answers 1. Correct Answer: D Rationale: Many patients in the ICU will require CVCs for reasons including delivery of vital medication (ie vasopressors), nutrition (ie parenteral nutrition), and monitoring (ie central venous pressure monitoring). Morbidity from CVCs include adverse events from placement (ie hematoma, inadvertent arterial puncture, pneumothorax), thrombus, and CLABSI. CLABSI can arise from intraluminal or extraluminal contamination at any time from placement through routine use. The cost of CLABSI has been estimated to be as high as $45 000 for an individual patient and over $1 billion in annual healthcare costs nationwide. CVC bundles have been developed to encourage methods that have been shown to reduce CLABSI. These methods include CVC training and education, centralizing CVC equipment into a central line cart; proper hand hygiene throughout placement and access of CVCs; proper personal protective equipment including hat and mask for everyone in the room and hat, mask, sterile gown, and gloves for practitioners placing CVCs; full body drapes; chlorhexidine-based antiseptic cleaning before placement, use of antimicrobial impregnated CVCs dressing sponges, changing dressing when they become loose or soiled, etc. Routine culture of the tip of CVC catheters (answer D) is not recommended as the tip can be easily contaminated by skin flora during removal. References

1. Hahn J, Cummings BM. Quality improvement and standardization of practice. In: Wiener-Kronish JP, ed. Critical Care handbook of the Massachusetts General Hospital. 6th ed. Philadelphia: Wolters Kluwer; 2016:575-588.

2. O’Grady NP, Alexander M, Burns LA, et al. Guidelines for the Prevention of Intravascular Catheter-Related Infections. Centers for Disease Control; 2011. https://www.cdc.gov/infectioncontrol/guidelines/bsi/.

2. Correct Answer: B

Rationale: Urinary catheters are commonplace in the ICU; however, infections related to their use can cause morbidity ranging from simple urinary tract infections to urosepsis. CAUTI-related costs are estimated to be only about $750 per patient; however, their high incidence leads to nearly $300 million in cost every year nationwide. Similar to CVC bundles, catheter bundles have been developed to encourage methods shown to reduce CAUTI. These methods include only using catheters when absolutely indicated (including acute retention or obstruction, accurate monitoring of urine production in critical illness, with epidural use, peroneal wounds 
or certain surgeries); removing catheters as soon as reasonably possible, use of aseptic technique when placing or 
changing catheters or collection systems; maintaining collecting systems below the level of the bladder to avoid flow from the collecting system back into the bladder; securing catheters to prevent movement in the urethra; and using antimicrobial-coated catheters in high-risk populations. Bladder irrigation with antimicrobials is not recommended. References

1. Gould CV, Umscheid CA, Agarwal RK, et al. Guideline for Prevention of Catheter-Associated Urinary Tract Infections 2009. Healthcare Infection Control Practices Advisory Committee, Centers for Disease Control. https://www.cdc.gov/infectioncontrol/guidelines/cauti/.

2. Hahn J, Cummings BM. Quality improvement and standardization of practice. In: Wiener-Kronish JP, ed. Critical Care handbook of the Massachusetts General Hospital. 6th ed. Philadelphia: Wolters Kluwer; 2016:575-588.

3. Correct Answer: A Rationale: The HIPAA was a revolutionary piece of legislation that, in addition to wide-ranging changes to health insurance regulation in the United States, established the Standards for Privacy of Individually Identifiable Health

Information. This “Privacy Rule” established standards for the disclosure of protected health information to “covered entities” (including health plans, healthcare providers, and healthcare clearinghouses). One of the permitted uses and disclosures comes under “Public Interest and Benefit Activities.” These activities include: when required by law; public health activities (answer D); victims of abuse, neglect, or domestic violence (answer B); health oversight activities (answer C); judicial and administrative proceedings; law enforcement purposes; decedents; cadaveric organ, eye, or tissue donations (answer E); research; serious threat to health or safety; essential government functions; and workers’ compensations. A juvenile’s right to privacy concerning reproduction and especially pregnancy testing has been well established and thus disclosure of this protected information is not covered by HIPAA. Reference The United States Department of Health and Human Services, Office for Civil Rights. Summary of the HIPAA Privacy Rule. Revised 05/03.

4. Correct Answer: C Rationale: Hospital-acquired infections have come under increased scrutiny because of their cost both in patient morbidity and mortality and in healthcare dollars. It is recognized that the major method of transmission of most hospital-acquired infections is from healthcare workers transmitting microorganisms between patients through lack of proper hand hygiene, personal protective equipment, or through fomites on their person (objects that are likely to carry infection from one patient to another including stethoscopes, white coats, ties, etc). Therefore, limiting patient visitors (answer B) to individual rooms is unlikely to directly reduce transmission of disease between patients in the ICU where most rooms are single occupancy. No strategy has proven more effective and cost-efficient as proper hand hygiene (answer C) in reducing the spread of hospital-acquired infections. In a positive pressure ICU room, the air pressure is higher in the room than the surrounding workspace, and although it can help to keep contagions from entering the room (as used in operating rooms), it can

promote the spread of respiratory contagions (answer A). Therefore, patients with particularly virulent respiratory pathogens, such as tuberculosis (TB), are placed in negative pressure rooms. Use of sterile personal protective equipment versus regular personal protective equipment (answer E) and dedicated hospital clothing for the ICU (answer D) would certainly be an effective way of limiting fomites; however, these are costlier to implement and not as effective as good hand hygiene. References

1. Hahn J, Cummings BM. Quality improvement and standardization of practice. In: Wiener-Kronish JP, ed. Critical Care handbook of the Massachusetts General Hospital. 6th ed. Philadelphia: Wolters Kluwer; 2016:575-588.

2. Siegel JD, Rhinehart E, Jackson M, et al. 2007 guideline for isolation precautions: preventing transmission of infectious agent in health care settings. Am J Infect Control. 2007;35(10 suppl 2):S65-S164. Last Updated February 15, 2017. Centers for Disease Control.

5. Correct Answer: B Rationale: The HIPAA allows for the disclosure of protected healthcare information concerning a patient with mental illness in a few very specific situations. If the patient is deemed competent, they retain agency to make their own healthcare decisions. Healthcare providers disagreeing with the patient’s decisions (answer D) or believing that their mental illness is affecting their medical decision-making (answer E) is not sufficient to allow disclosure against the patient’s wishes. Healthcare proxy status (answer A) or being a spouse or next of kin (answer C) does not supersede the patient’s request for no information to be shared as long as they are not incapacitated. HIPAA provides allowances for disclosure of protected healthcare information concerning a patient with mental illness if the healthcare provider perceives a “serious and imminent threat to the health or safety of the patient or others and the family members are in a position to lessen the threat.”

References

1. U.S. Department of Health and Human Services Office for Civil Rights. HIPAA Privacy Rule and Sharing Information. https://www.hhs.gov/sites/default/files/hipaa-privacy-rule-and-sharinginfo-related-to-mental-health.pdf.

2.

The United States Department of Health and Human Services, Office for Civil Rights. Summary of the HIPAA Privacy Rule. Revised 05/03.

C H AP T E R 1 1 8

Palliative Care and End of Life Jennifer Cottral and William J. Benedetto

1. A 61-year-old male is admitted to the intensive care unit (ICU) intubated after an out-of-hospital cardiac arrest due to myocardial infarction complicated by anoxic brain injury. He has a history of severe chronic obstructive pulmonary disease (COPD) with home oxygen, poorly controlled insulin-dependent type II diabetes, chronic kidney disease, and severe peripheral vascular disease status post left above-the-knee amputation. The patient was assessed to be too high-risk for percutaneous coronary interventions, and two cardiac surgeons have deemed the patient a poor surgical candidate. Over the past 24 hours, the patient has remained on multiple high-dose vasopressors, has become anuric, his creatinine has tripled, and his potassium is 7.0 mEq/dL. The patient has not filled out an advance directive or assigned a healthcare proxy, and his next of kin includes his three children. After multiple family meetings, the patient’s children continue to request renal replacement therapy despite concerns from the ICU team about its medical appropriateness. Which of the following is NOT a recommended step to conflict resolution based on the most recent multispecialty consensus statement regarding potentially inappropriate medical care in the ICU?

A. Obtain a second medical opinion B. Provide review by an interdisciplinary hospital committee C. Offer legal counsel to surrogates D. Offer surrogates the opportunity for transfer to an alternate institution E. Enlist expert consultation to aid in achieving a negotiated agreement

2. The available evidence suggests that the impact of ICU-based palliative care includes all the following EXCEPT:

A. Decrease ICU and hospital length of stay B. Decrease healthcare cost C. Improve communication with patient D. Increase in-hospital mortality

3. All the following have been shown to significantly reduce the rate of catheter-related bloodstream infections (CR-BSIs) occurring in the ICU EXCEPT:

A. Handwashing B. Use of full-barrier precautions C. Cleaning the skin with chlorhexidine D. Avoiding the femoral site for cannulation E. Performing a time-out before line insertion

4. What is the 1-year mortality risk in a critically ill 67-year-old patient with platelets of 100 × 109/L requiring hemodialysis and mechanical ventilation on hospital day 21?

A. 10% to 20% B. 30% to 40% C. 40% to 50% D. 60% to 70% E. >70%

5. Symptoms of postintensive care syndrome (PICS) include all of the

following except:

A. Cognitive dysfunction B. Psychiatric disturbances C. Metabolic syndrome D. Musculoskeletal weakness

6. A 64-year-old male with no known prior medical history was admitted 5 days ago with a large subarachnoid hemorrhage from a suspected ruptured cerebral aneurysm. His current vitals are blood pressure (BP) 105/70 mm Hg, heart rate 84 beats per minute, respiratory rate 16/min on volume control ventilation, and temperature 98.8°F. Laboratory data are all within normal limits. He has not received any central nervous system depressants or paralytics but makes no respiratory effort after 8 minutes of apnea. He is noted to have an intermittent bilateral finger tremor, though not in response to stimuli such as pain. All brainstem reflexes are absent. Does this patient meet the criteria for brain death based on the most recent recommendations by the Society of Critical Care Medicine?

A. Yes B. No

7. All of the following conditions in a suspected organ donor are contraindications to organ donation EXCEPT:

A. Bacteremia on appropriate antibiotics B. Grade II central nervous system tumor C. Hepatitis C virus (HCV) seropositive D. Human immunodeficiency virus (HIV) seronegative but meets the

high-risk behavioral criteria for HIV infection E. All of the above F. None the above

8. In patients who are being considered for organ donation after cardiac death (DCD), all of the following are considered part of the United Network for Organ Sharing (UNOS) criteria for prediction of death within 60 minutes of withdrawal of life-sustaining treatment (LST), EXCEPT:

A. Apnea B. Respiratory rate 30 breaths/min C. ≥3 vasopressors to maintain a mean arterial pressure (MAP) >65 mm Hg D. Fio 2 ≥0.5 and Sao 2 ≤92% E. Norepinephrine or phenylephrine ≥0.2 µg/kg/min

9. True or false: Withholding LST is permissible, but once started, it must be continued.

A. True B. False

10. Match the following terms with their definitions:

Terms a

Living will

Definitions 1

Medical orders addressing a range of topics likely to be relevant to the care of a patient

b

Advance care planning

2

c

Medical orders for lifesustaining treatment

3

d

Do-not-resuscitate orders, do not hospitalize, do not intubate orders

4

near the end of life; other healthcare professionals (including emergency personnel) are required to follow them The process of clarifying goals, values, and preferences, usually through written documents and medical orders, about the type(s) of medical care a patient does or does not want under certain medical conditions A type of advanced directive that specifies the type(s) of medical care a patient does or does not want under certain medical conditions, in the case that at a future time, the patient is not able to express those preferences Medical orders addressing the limitations of specific treatments near the end of life that are written within a healthcare facility but do not transfer to

other care settings and are not necessarily followed by outside healthcare professionals e

Durable power of attorney for health care

A. a3, b5, c4, d2, e1 B. a5, b2, c3, d2, e1 C. a5, b5, c1, d4, e3 D. a3, b2, c1, d4, e5

5

A type of an advanced directive that identifies the healthcare agent who will make medical decisions on behalf of a patient if the patient is incapable of making medical decisions

Chapter 118 Answers 1. Correct Answer: C Rationale: In 2015, a multispecialty group released a consensus statement regarding how ICU clinicians can and should respond to requests for potentially inappropriate treatment in the ICU with the goal to prevent intractable disagreements about the use of such treatments. This group recommended the use of the term “potentially inappropriate” rather than “futile” care to describe scenarios where a treatment may provide the effect desired by surrogates, but clinicians believe that certain ethical considerations justify not providing the treatment in question. If such a scenario arises and the surrogate(s) continue to request potentially inappropriate treatment despite intensive communication and negotiation with the ICU care team, the following approach recommended: 1. 2. 3. 4. 5. 6. 7.

Enlist expert consultation to continue negotiation during the dispute resolution process Give notice of the process to surrogates Obtain a second medical opinion Obtain review by an interdisciplinary hospital committee Offer surrogates the opportunity to transfer the patient to an alternate institution Inform surrogates of the opportunity to pursue extramural appeal Implement the decision of the resolution process

Although surrogates should be informed of their right to seek extramural appeal (usually through seeking judicial review), clinicians and hospitals are not required to offer and/or provide legal counsel to surrogates. Reference Bosslet G, Pope T, Rubenfeld G, et al. An official ATS/AACN/ACCP/ESICM/SCCM policy statement: responding to requests for potentially inappropriate treatments in intensive care units. Am J Respir Crit Care Med. 2015;191(11):1318-1330.

2. Correct Answer: D Rationale: Palliative care in the ICU involves interprofessional support for both patients and families, attending to the spiritual, physical, and emotional domains of critical illness. Palliative care is often provided concomitantly with life-prolonging care and unlike hospice care, is not based on prognosis. Efforts to define outcomes and quantify the impact of ICUbased palliative care interventions are ongoing. Various meta-analyses and systematic review of the effect of ICU-based palliative care on a variety of outcomes suggest that palliative care interventions largely decrease hospital and ICU length of stay and a reduction in healthcare cost. While some studies suggest a reduction in in-hospital mortality, others show no effect. But there is no evidence to suggest that mortality increases with palliative care interventions (answer D). In addition, the results suggested that ICU-based palliative care led to improvement in the quality, quantity, and content of communication, decrease in symptoms of distress and anxiety among family members, decrease in procedures, decrease in the time between admission and comfort measures only, withdrawal of LSTs, and do-not-resuscitate orders. References

1. Aslakson RA, Curtis JR, Nelson JE. The changing role of palliative care in the ICU. Crit Care Med. 2014;42:2418-2428.

2. Aslakson RA, Bridges JF. Assessing the impact of palliative care in the intensive care unit through the lens of patient-centered outcomes research. Curr Opin Crit Care. 2013;19:504-510.

3. Mularski RA. Defining and measuring quality palliative and end-of-life care in the intensive care unit. Crit Care Med. 2006;34(11 suppl):S309S316.

4. Mularski RA, Curtis JR, Billings JA, et al. Proposed quality measures for palliative care in the critically ill: A consensus from the Robert Wood Johnson Foundation Critical Care Workgroup. Crit Care Med. 2006;34:S404-S411.

5. Aslakson R, Cheng J, Vollenweider D, Galusca D, Smith TJ, Pronovost PJ.

Evidence-based palliative care in the intensive care unit: a systematic review of interventions. J Palliat Med. 2014;17:219-235.

3. Correct Answer: E Rationale: CR-BSIs are a costly and often preventable source of morbidity and mortality in the ICU. The average cost per CR-BSI in the adult surgical ICU is estimated to be between $54 000 and $75 000. Many CR-BSIs are preventable, and studies have shown that implementing a variety of prevention strategies can significantly decrease the incidence of CR-BSIs. In a 2006 landmark paper, Provonost et al. demonstrated a nearly 50% decrease in CR-BSIs over 18 months with the implementation of five evidence-based procedures and identified as having the greatest effect on the rate of CR-BSIs and the fewest implementation barriers. These five procedures include the following: handwashing, using full-barrier precautions during the insertion of central venous catheters, cleaning the skin with chlorhexidine, avoiding the femoral site if possible, and removing unnecessary catheters. The preprocedure “time-out” is an important step in preventing error but does not impact CR-BSI rate. References

1. Hollenbeak CS. The cost of catheter-related bloodstream infections: implications for the value of prevention. J Infus Nurs. 2011;34:309-313.

2. Mermel LA. Prevention of intravascular catheter-related infections [Erratum in Ann Intern Med. 2000;133:5]. Ann Intern Med. 2000;132:391402.

3. Blot K, Bergs J, Vogelaers D, Blot S, Vandijck D. Prevention of central line associated bloodstream infections through quality improvement interventions: a systematic review and meta-analysis. Clin Infect Dis. 2014;18(1):96-105.

4. Correct Answer: E Rationale:

The ProVent score (Prognosis for Prolonged Ventilation Score) is a tool that aims to quantify risk of 1-year mortality among critically ill patients requiring prolonged mechanical ventilation. It is comprised of four easily measured variables recorded at day 21 of ventilation (see tables below). The ProVent score was originally derived from a prognostic model for 1year mortality in medical, surgical, and trauma patients at a single academic hospital who required mechanical ventilation for at least 21 days. It was first described in 2008 by Carson et al. and over the past decade, has been validated at centers across the United States and overseas. The patient described above has a ProVent score of 4 (age >65 years, thrombocytopenia, hemodialysis). ProVent Model with Categorized Risk Variables Categorical Variable

No. (%)

Odds Ratio (95% Confidence Interval)

Beta Value

Points

Age ≥65 y Age 50-64 y Platelets ≤150 × 10 9/L Vasopressors Hemodialysis

80 (31%) 88 (34%) 65 (25%) 35 (13%) 34 (13%)

7.6 (3.8-15.5) 2.0 (1.0-3.9) 1.9 (0.9-3.9) 4.4 (1.6-12.6) 2.4 (1.0-6.0)

2.03 0.67 0.65 1.49 0.89

2 1 1 1 1

ProVent score and observed 1-year mortality ProVent Score

No.

Observed Mortality Percent 
(95% Confidence Interval)

0 1 2 3 4 or 5

72 60 78 36 14

20 (10-29) 36 (24-48) 56 (45-68) 81 (67-94) 100 (77-100)

References

1. Carson SS, Kahn JM, Hough CL, et al. A multicenter mortality prediction model for patients receiving prolonged mechanical ventilation. Crit Care Med. 2012;40(4):1171-1176.

2. Carson SS, Garrett J, Hanson LC, et al. A prognostic model for one-year mortality in patients requiring prolonged mechanical ventilation. Crit Care Med. 2008;36(7):2061-2069.

3. Leroy G, Devos P, Lambiotte F, Thévenin D, Leroy O. One-year mortality in patients requiring prolonged mechanical ventilation: multicenter evaluation of the ProVent score. Crit Care. 2014;18(4):R155. doi:10.1186/cc13994.

4. Jaiswal S, Sadacharam K, Shrestha RR, et al. External validation ofprognostic model of one-year mortality in patients requiring prolonged mechanical ventilation. J Nepal Health Res Counc. 2012;10:4751.

5. Correct Answer: C Rationale: PICS was first described in 2012 after a Society of Critical Care Medicine conference created to improve long-term outcomes after critical illness for patients and their families. PICS is characterized by new or worsening problems in cognitive, psychiatric, or physical health that occur after a critical illness and persist after acute care hospitalization. Common symptoms are generalized weakness, fatigue, decreased mobility, sexual dysfunction, sleep disturbances, and cognitive impairment. Reported cognitive dysfunction includes memory disturbance or loss, slow mental processing, and poor concentration. Psychiatric disturbances include anxiety, depression, and posttraumatic stress disorder. Metabolic syndrome has not been associated with PICS. Many studies have attempted to identify risk factors for and the prevention of PICS, though are still in the early stages of discovery. References

1. Needham DM, Davidson J, Cohen H, et al. Improving long-term outcomes after discharge from intensive care unit: report from a stakeholders’ conference. Crit Care Med. 2012;40:502-509.

2. Casaer MP, Van den Berghe G. Nutrition in the acute phase of critical illness. N Engl J Med. 2014;370:1227.

6. Correct Answer: A

Rationale: In 2015, the Society of Critical Care Medicine, the American College of Chest Physicians, and the Association of Organ Procurement Organizations released a joint consensus statement on the management of the potential organ donor in the ICU. Included in these guidelines is the mandate that clinicians should incorporate the most recent recommendations provided by the Quality Standards Subcommittee of the American Academy of Neurology (AAN). Within these AAN recommendations is the assertion that complex motor activity (such as finger tremor in the patient described in the question stem) can sometimes occur in patients who meet the criteria for brain death and as such, its presence does not exclude the diagnosis of brain death. References

1. Kotloff RM, Blosser S, Fulda GJ, et al. Management of the potential organ donor in the ICU: Society of Critical Care Medicine /American College of Chest Physicians/ Association of Organ Procurement Organizations Consensus Statement. Crit Care Med. 2015;43:1291-1325.

2. Wijdicks EFM, Varelas PN, Gronseth GS, et al.: Evidence-based guideline update: determining brain death in adults. Report of the Quality Standards Subcommittee of the American Academy of Neurology. Neurology. 2010;74:1911-1918.

7. Correct Answer: F Rationale: Based on the most recent multiorganizational guidelines regarding management of the potential organ donor in the ICU, all of the aforementioned conditions are NOT considered contraindications for organ donation consideration. Organs from bacteremic donors have been successfully donated with few transmitted infections when the donor received pathogen-specific antibiotics for a minimum of 48 hours before procurement. As such, the most recent guidelines recommend that bacteremic organ donors receive pathogen-specific antibiotic therapy for at least 48 hours, even if this

delays organ procurement. Patients with histologically low-grade central nervous system tumors (grades I–II) and no history of craniotomy, brain irradiation, or ventricular shunts have a low risk of tumor transmission. Organs from a HCV-seropositive donor can be used in HCV-positive recipients. Based on behavioral criteria that define donors at increased risk of having acquired HIV infection, the estimated risk of undetected viremia among seronegative donors is sufficiently low to not disqualify from organ donation consideration. For adults, the risk of undetected viremia among seronegative donors at increased risk of having acquired HIV infection is the following (per 10 000 donors): IV drug users (12.1), men who have had sex with other men (10.2), commercial sex workers (6.6), inmates of correctional facilities (2.3), persons exposed to HIV-infected blood in the past 12 months (1.5), for persons who have had sex with a high-risk individual in the past 12 months (0.7), and hemophiliacs (0.09). Reference Kotloff RM, Blosser S, Fulda GJ, et al. Management of the potential organ donor in the ICU: Society of Critical Care Medicine /American College of Chest Physicians / Association of Organ Procurement Organizations Consensus Statement. Crit Care Med. 2015;43:1291-1325.

8. Correct Answer: C Rationale: A widely adopted protocol for donation after circulatory determination of death (DCDD) involves transferring a patient being considered for DCD to an operating room where circulatory function is monitored and LST is withdrawn. The patient is observed until circulatory function ceases or until 60 minutes pass. If cessation of circulatory function does not occur within 60 minutes, the patient is returned to the ICU and organ procurement is aborted. Although the majority of deaths are determined using cardiac (rather than brain) death criteria, a disproportionately low number of donated organs are procured via DCD. One hypothesized reason is the difficulty in predicting death within 60 minutes of withdrawal of LST. Accurately predicting the timing of cardiac death also has emotional, logistical, and financial consequences on all those involved

including the donor family, recipient, and transplant teams. One of the widely used tools to predict cardiac death within 60 minutes of withdrawal of LST is the UNOS Consensus Committee Criteria (see below). These criteria were prospectively validated in a single-center study and demonstrated that among patients with 0, 1, 2, and 3 UNOS DCD criteria, 29%, 52%, 65%, and 82% died within 60 minutes of withdrawal of LST, respectively. UNOS Consensus Committee Criteria for Prediction of Death Within 60 Minutes of Withdrawal of LST Apnea Respiratory rate 30 breaths/min Dopamine ≥15 µg/kg/min Left or right ventricular assist device Venoarterial or venovenous extracorporeal membrane oxygenation Positive end-expiratory pressure ≥10 and Sao 2 ≤92% Fio 2 ≥0.5 and Sao 2 ≤92% Norepinephrine or phenylephrine ≥0.2 µg/kg/min Pacemaker unassisted heart rate 200 on 5 cm H2O PEEP B. P/F >300 on 10 cm H2O PEEP C. P/F >300 on 5 cm H2O PEEP D. P/F >400 on 7 cm H2O PEEP

Chapter 119 Answers 1. Correct Answer: A Rationale: The hypothalamic-pituitary axis (HPA) is particularly vulnerable to ischemia. Vasopressin, which is produced in the hypothalamus, transported to the posterior pituitary, then released by the posterior pituitary, is decreased in up to 80% of patients with brain death, leading to central diabetes insipidus (DI). Arginine vasopressin (AVP) deficiency can lead to diuresis and is characterized by dilute urine and hypertonic plasma. Urine osmolality is often less than 200 mOsm/L in central DI, and one can confirm the diagnosis with fluid restriction and failure of urine osmolality to increase more than 30 mOsm/L in the first few hours. Treatment of central DI consists of desmopressin administration. Desmopressin is a vasopressin analogue with greater affinity for the V2 receptor (present in the distal collecting duct) than the V1 receptor on vascular smooth muscle. Dosing can vary, but can be between 0.5 µg and 2 µg IV. Additional dosing is required approximately every 6 hours and should be titrated based on urine output, urine osmolality, and serum sodium. Reference

1. Kotloff RM, Blosser S, Fulda GJ, et al; Society of Critical Care Medicine/American College of Chest Physicians/Association of Organ Procurement Organizations Donor Management Task Force. Management of the potential organ donor in the ICU. Crit Care Med. 2015;43(6):1291-1325.

2. Correct Answer: C Rationale: Initial discussions with families regarding organ donation are an essential part of the organ donation process. Ideally, the person who speaks to family members for authorization (or consent) for organ

donation is someone who is familiar with the process and has experience in guiding and supporting family members through the organ donation process. As such, some centers have organ procurement organization coordinators who are ideally suited for this task. Although the ICU attending and bedside nurse may have developed a relationship with the family, conversations to consent for organ donation may be viewed as a conflict of interest. The same principle applies to transplant team members who wish to approach families of patients who may become organ donors. In the situation that a transplant team member is caring for or has cared for the patient, for instance, as the trauma critical care attending, that attending should transfer care to another physician in order to participate in the organ procurement. References

1. Kotloff RM, Blosser S, Fulda GJ, et al; Society of Critical Care Medicine/American College of Chest Physicians/Association of Organ Procurement Organizations Donor Management Task Force. Management of the potential organ donor in the ICU. Crit Care Med. 2015;43(6):1291-1325.

2. Gries CJ, White DB, Truog RD, et al. An Official American Thoracic Society/International Society for Heart and Lung Transplantation/Society of Critical Care Medicine/Association of Organ and Procurement Organizations/United Network of organ sharing statement: ethical and policy considerations in organ donation after circulatory determination of death. Am J Respir Crit Care Med. 2013;188(1):103-109.

3. Correct Answer: D Rationale: The primary hemodynamic goal of organ donors is to maximize organ perfusion for organ preservation by maintaining normovolemia, an adequate blood pressure, and adequate cardiac output. Specific goals include mean arterial pressure of 60 to 70 mm Hg, urine output of 1 to 3 mL/kg/h, minimization of pressor dosage, left ventricular ejection fraction of at least 45%, initial volume replacement with crystalloids or

colloids, and avoidance of HES for colloidal resuscitation. The use of HES can lead to acute kidney injury and has been shown to be associated with delayed graft function and graft failure. References

1. Cittanova ML, Leblanc I, Legendre C, Mouquet C, Riou B, Coriat P. Effect of hydroxyethylstarch in brain-dead kidney donors on renal function in kidney-transplant recipients. Lancet. 1996;348(9042):1620-1622.

2. Kotloff RM, Blosser S, Fulda GJ, et al; Society of Critical Care Medicine/American College of Chest Physicians/Association of Organ Procurement Organizations Donor Management Task Force. Management of the potential organ donor in the ICU. Crit Care Med. 2015;43(6):1291-1325.

3. Wood KE, Becker BN, McCartney JG, D’Alessandro AM, Coursin DB. Care of the potential organ donor. N Engl J Med. 2004;351(26):27302739.

4. Correct Answer: C Rationale: Generally, a P/F ratio greater than 300 mm Hg in the donor is considered acceptable for lung donation. In donors who fail to meet this requirement, donor management protocols that include diuresis, therapeutic bronchoscopy, chest physiotherapy, and lung recruitment can improve oxygenation. Of note, bronchoscopy should be performed in all potential lung donors, both to assess for occult aspiration and infection and to perform therapeutic airway clearance. Additional ideal lung donor criteria include: Age 92%. A bedside transthoracic echocardiography is performed and the left ventricle appears to be severely hypokinetic and dilated. The patient is admitted to the intensive care unit for further management. Which of the following statements is true regarding peripartum cardiomyopathy (PPCM)?

A. Approximately 50% of patients with PPCM regain normal left ventricular function by 6 months C. Beta blockers, diuretics, and ACE inhibitors are contraindicated in the postpartum period D. The risk of PPCM in subsequent pregnancies is very low E. The majority of patients do not recover normal LV function and require heart transplantation

4. A 27-year-old female with a past medical history of severe pulmonary hypertension secondary to a ventricular septal defect becomes pregnant despite warnings from expert obstetricians. At 27 weeks, the patient presents to the emergency room with shortness of breath and is admitted to an intensive care unit for hemodynamic monitoring. Several hours after admission, she

becomes comatose, apneic, and pulseless. Which of the following is NOT true regarding the treatment of cardiac arrest in pregnancy?

A. If possible, IV access should be obtained in the upper extremity B. Uterine displacement must be achieved by placing the patient 15 to 30° left lateral decubital position C. After 10 minutes of unsuccessful resuscitation efforts, a cesarean section should be performed in the operating room D. Chest compressions should be performed 1 to 2 cm higher on the sternum in pregnant patients E. Defibrillation energy doses should remain the same as in nonpregnant patients

5. A 30-year-old G4P3 female presents at 35 weeks’ gestation with changes in vision and headaches. At triage, her blood pressure is elevated at 149/91 mm Hg and her urine analysis is positive for protein. Initial blood work is within normal limits. Decision is made to treat with steroids to improve fetal lung maturity. Two days later, the patient complains of severe right upper quadrant pain with nausea and vomiting. A repeat metabolic panel reveals elevated aspartate aminotransferase, alanine aminotransferase, and lactate dehydrogenase. Platelet count and hemoglobin are downtrending. One day after an uneventful cesarean section, the patient again complains of severe right upper quadrant pain and becomes hypotensive and lethargic. The critical care team responds to a code, and STAT labs show a hemoglobin level of 5.2 g/dL, with a bedside ultrasound showing free fluid in the abdominal cavity. Which of the following is the most likely diagnosis?

A. Uterine hemorrhage B. Splenic rupture C. GI bleeding D. Ruptured liver hematoma

E. Intravascular hemolysis

Chapter 121 Answers 1. Correct Answer: D Rationale: Postpartum hemorrhage continues to present a great challenge to obstetricians and anesthesiologists and represents a major cause of maternal mortality (between 3.5% and 12%). In the last decade, the incidence of postpartum hemorrhage has shown an uptrend, mainly because of an accompanying increase in maternal morbidity. Patient’s risk factors include uterine atony and placenta accreta, percreta, and increta. The four T’s classify etiologies into: Tone—atony Tissue—retained products of conception or abnormal placental attachment (accreta, percreta, increta) Trauma—rupture, laceration Thrombin—preexisting or acquired coagulation abnormalities Along with replacing blood loss with packed red blood cells, fresh frozen plasma, and platelets in a 1:1:1 ratio, fibrinogen levels in obstetrical patients are of utmost importance in evaluating/predicting the severity of postpartum hemorrhage. Normal fibrinogen levels at the end of pregnancy are higher (373-619 mg/dL) than in nonpregnant patients (233-496 mg/dL). Decrease in fibrinogen level below 200 mg/dL has been found to be a strong predictor for severe postpartum hemorrhage. Replacement of fibrinogen with either cryoprecipitate or fibrinogen concentrate has been shown to reduce the need for transfusion of blood products by up to 33%. References

1. Landau R, Ciliberto CG, Vuilleumier PH. Management of obstetric hemorrhage. In: Santos AC, Epstein JN, Chaudhuri K. eds. Obstetric Anesthesia. 1st ed. New York, NY: McGraw-Hill 2015.

2. Butwick AJ. Postpartum hemorrhage and low fibrinogen levels: the past, present and future. Int J Obstet Anesth. 2013;22:87-91.

3. Bell SF, Rayment R, Collins PW, Collis RE. The use of fibrinogen

concentrate to correct hypofibrinogenaemia rapidly during obstetric haemorrhage. Int J Obstet Anesth. 2010;19:218-223.

4. Ahmed S, Harrity C, Johnson S, et al. The efficacy of fibrinogen concentrate compared with cryoprecipitate in major obstetric haemorrhage—an observational study. Transfus Med. 2012;22:344-349.

5. Wikkelsoe AJ, Afshari A, Stensballe J, et al. The FIB-PPH trial: fibrinogen concentrate as initial treatment for postpartum haemorrhage: study protocol for a randomised controlled trial. Trials. 2012;13:110.

2. Correct Answer: D Rationale: Diagnosis of amniotic fluid embolism (AFE) remains a diagnosis of exclusion and can easily be mistaken for many other syndromes including eclampsia, high neuraxial block, local anesthetic toxicity, pulmonary embolism, hemorrhage, oxytocin reaction, and anaphylaxis, among others. The condition is characterized by initial cardiovascular collapse, which may or may not include seizures, followed by disseminated intravascular coagulation and clinical bleeding, leading ultimately to end organ damage. Contrary to historical belief, the term “embolism” is a misnomer, and the phenomenon is now thought to be an immunological reaction resulting from cytokine activation and inflammation, rather than an “embolic phenomenon,” which causes obstructive shock. Nonetheless, there are echocardiographic findings that aid in the diagnosis of AFE. Initially, TEE may reveal severe right ventricular dilation and systolic dysfunction, secondary to pulmonary vasoconstriction and right ventricular pressure overload. In short-axis view at the midpapillary level, there is a D-shaped left ventricle in systole and diastole, with the interventricular septum deviated to the left, because of the increased right ventricular pressure. Because the changes in the geometry of the right ventricle occur abruptly, there is no time for the right ventricle to adapt to the new loading conditions and systolic failure is imminent. With the decrease in the right ventricular ejection, the left ventricular preload and stroke volume decreases as well. In case of acute increase in the right ventricular afterload, the right ventricular systolic pressure is expected to be low,

because of the inability of the acutely overloaded right ventricle to eject against high pulmonary artery resistance. High right ventricular systolic pressure is typically observed with chronic pulmonary arterial hypertension and hypertrophy of the right ventricle, which occurs over long period of time. As a late sign of amniotic fluid embolism, due to myocardial ischemia, TEE may show worsening left ventricular systolic function. Reference

1. James CF, Feinglass NG, Menke DM, et al. Massive amniotic fluid embolism: diagnosis aided by emergency transesophageal echocardiography. Int J Obstet Anesth. 2004;13:279-283. 3. Correct Answer: A Rationale: PPCM occurs in approximately 1:3200 deliveries in the United States. The condition is defined as new onset systolic heart failure presenting in the last month of pregnancy up to 5 months postpartum in a female with no history of cardiac disease. African American women, advanced maternal age, and multiparity are all risk factors for postpartum cardiomyopathy. The diagnosis of PPCM can be delayed as symptoms such as mild shortness of breath, tachycardia, and lower extremity edema can be typical in pregnancy. The diagnosis is typically made by echocardiography revealing an LVEF of 65 mm Hg. His arterial pressure waveform follows.

Adapted from Michard F. Changes in arterial pressure during mechanical ventilation. Anesthesiology. 2005;103(2):419-428.

What is the next best step in management? A. 1 L lactated Ringer ’s B. Add epinephrine C. No further intervention D. Obtain STAT echocardiogram

15. A rapid response is called on an 18-year-old female due to seizure. She was recently diagnosed with acute lymphocytic leukemia, and chemotherapy has just been started. She is intubated on the floor and transferred to the unit. A basic metabolic panel is likely to show:

A. Hypocalcemia B. Hypophosphatemia C. Hypokalemia D. Hypouricemia

16. In assessing delirium using the Confusion Assessment Method for ICU Patients (CAM-ICU), which is NOT required to be present to diagnose delirium?

A. Acute onset of changes or fluctuations in the course of mental status B. Inattention C. Disorganized thinking D. Combative behavior

17. Which coagulation factor decreases in pregnancy?

A. Protein C B. Factor VII C. Factor XII D. Factor XIII

18. Which of the following statements about heparin-induced thrombocytopenia (HIT) is FALSE?

A. Patients of any age, receiving any type of heparin are at risk for developing HIT antibodies. B. Bleeding is commonly seen with HIT. C. Diagnosis of HIT is clinical with support from laboratory testing. D. Testing for heparin-PF4 antibody is recommended in patients with suspected HIT.

19. In conjunction with the infectious disease department, your unit is trying to improve compliance with hand washing in the setting of contact isolation patients. The team plans an intervention and proposes to employ Plan-Do-Study-Act (PDSA) cycles. Which of the following is NOT part of a PDSA cycle?

A. Plan B. Do C. Study D. Assess

20. An investigator wishes to study transthoracic echocardiograms performed by trainees. In the trial of 100 echocardiograms, 10 cases of endocarditis were correctly identified, 15 cases were incorrectly identified as endocarditis, 70 cases were correctly ruled out for endocarditis, and 5 cases of endocarditis were missed. What is the positive predictive value of this test?

A. 40% B. 67% C. 82% D. 93%

21. Which of the following criteria is part of the 2016 definition of sepsis (Sepsis-3)?

A. Systemic inflammatory response syndrome (SIRS) B. Hypotension unresponsive to fluids C. Confirmation of at least one positive blood culture D. Life-threatening organ dysfunction

22. A 26-year-old man is admitted to the ICU with septic shock from a perforated appendix. He is treated with abscess drainage, fluid resuscitation, broad-spectrum antimicrobials (vancomycin, cefepime, metronidazole), and norepinephrine for blood pressure support. Vitamin C, thiamine, and hydrocortisone are administered as adjuncts. While his shock improves, his blood glucose levels

remain elevated by point-of-care (POC) testing while laboratory testing reveals normoglycemia. What is the most likely cause of the discrepancy between the POC and laboratory glucose levels?

A. Hydrocortisone B. Cefepime C. Thiamine D. Vitamin C

23. Endogenous molecules released from damaged host cells that provoke inflammation in sepsis are referred to as:

A. Damage-associated molecular patterns (DAMPs) B. Pathogen-associated molecular patterns (PAMPs) C. Inflammasomes D. Toll-like receptors (TLRs)

24. Development of acute kidney injury (AKI) in sepsis is most likely attributable to which of the following etiologies?

A. Reduced renal blood flow B. Hypotension C. Acute tubular necrosis D. Microvascular dysfunction

25. A 70-year-old man has a PEA arrest immediately following intubation for respiratory failure. To facilitate the intubation etomidate and rocuronium had been administered. He received chest compressions and epinephrine and had ROSC after 10 minutes. To facilitate decision-making after ROSC, a neurologic

examination is desired. Administration of which of the following medications would be most appropriate to facilitate this examination?

A. Neostigmine B. Sugammadex C. Flumazenil D. Naloxone

26. A 27-year-old man presents to the emergency department with the “worst headache of his life,” and imaging reveals a subarachnoid hemorrhage. Which of the following statements regarding ICU triage is MOST appropriate for this patient?

A. Admit to a general ICU B. Triage to the ICU based on the patient’s severity of illness score C. Transfer to the ICU within 6 hours of presentation D. Admit to an ICU with 24/7 inhouse intensivist staffing

27. A 49-year-old, 70 kg man is admitted to the ICU after large volume aspiration resulting from a small bowel obstruction. He is intubated and initial ventilator settings are tidal volume 420 mL, respiratory rate 27 breaths/min, positive end-expiratory pressure (PEEP) 5 cm H2O, 100% FiO2, and the arterial blood gas is pH 7.30/ pCO2 39 mm Hg/ pO2 57 mm Hg on those settings. He undergoes bronchoscopy with evidence of and successful evacuation of aspirated material from the airways. Chest radiograph reveals diffuse bilateral alveolar opacities. A recruitment maneuver is performed, and the PEEP is increased to 10 cm H2O with subsequent arterial blood gas of pH 7.32/ pCO2 40 mm Hg/ pO2 60 mm Hg.

Which of the following is the next best step in diagnosis of the patient’s hypoxemia? A. Pulmonary embolism—protocol CT scan B. Cardiac catheterization C. Echocardiogram with bubble study D. Bronchoalveolar lavage

28. Which of the following statements regarding the performance of a passive leg raise (PLR) test for assessment of fluid responsiveness is MOST correct?

A. It should only be performed in a spontaneously breathing patient. B. It should start with the patient in the supine position. C. It should be assessed by a direct measurement of cardiac output. D. It should be assessed after waiting more than a minute for volumetric equilibration.

29. For the management of gastrointestinal dysmotility in critically ill patients, erythromycin differs from metoclopramide in which of the following ways?

A. Erythromycin is more effective as a prokinetic agent than metoclopramide. B. Combination therapy with erythromycin and metoclopramide is not more effective than administration of either agent alone. C. Erythromycin is a motilin agonist while metoclopramide acts as an agonist of dopamine receptors. D. Tachyphylaxis to the prokinetic effects develops with erythromycin but not metoclopramide.

30. A 55-year-old woman with ideal body of 50kg has developed acute

respiratory distress syndrome (ARDS) as a result of intra-abdominal sepsis. She is currently being mechanically ventilated with tidal volume 300 mL, respiratory rate 30 breaths/min, PEEP 8 cm H2O, and 40% FiO2. Her peak pressure is 37 cm H2O and plateau pressure is 34 cm H2O. Her most recent arterial blood gas is pH 7.31, PCO2 44 mm Hg, and PO2 80 mm Hg. Which of the following values is the BEST estimate of the driving pressure during mechanical ventilation?

A. 4 cm H2O B. 14 cm H2O C. 26 cm H2O D. 29 cm H2O

31. A 76-year-old male who has been in a rehabilitation facility for 3 months after sustaining a traumatic brain injury was brought to the emergency room with a 3-day history of nausea, vomiting, and unremitting abdominal pain. On examination, he was febrile with a maximum temperature of 102.5°F and hypotensive with a systolic BP of 82 mm Hg. His heart rate was 92 beats/min, in sinus rhythm. A midline laparotomy scar was evident. He has a history of hypertension, coronary heart disease, and atrial fibrillation and is currently on aspirin, simvastatin, and metoprolol and had a complete blood count (CBC) which showed a WBC of 13,000/mm3. Two large bore peripheral IV catheters were placed and 2 L of Ringer ’s lactate promptly administered, without any significant hemodynamic changes. A right internal jugular central venous line was promptly placed; he was pan-cultured, given a single dose of IV piperacillin-tazobactam and started on norepinephrine and vasopressin infusions. He was promptly admitted to the surgical intensive care unit (SICU) where a focused abdominal ultrasound showed a distended gallbladder with a 4 mm wall thickness, mild pericholecystic fluid, and gallbladder sludge. What is the next appropriate next step?

A. Laparoscopic cholecystectomy B. Percutaneous cholecystostomy tube (PCT) placement C. Open cholecystectomy D. Endoscopic retrograde cholangiopancreatography (ERCP) E. Resuscitation with 5% albumin

32. A 52-year-old male with a history of chronic alcoholism and liver cirrhosis presented to the emergency room with a 3-day history of difficulty breathing, abdominal distension, and altered mental status. His liver function enzymes were all significantly elevated. Abdominal ultrasound showed massive ascites with splinting of the right hemidiaphragm. He underwent a large volume paracentesis and was admitted to the ICU. While in the ICU, bleeding from the paracentesis site which was initially controlled with digital pressure was noted. The patient soon became hypotensive and was started on vasopressors. A right IJ central line was placed. However, this required two attempts as the right common carotid artery was inadvertently punctured on the first attempt. Three hours later, he was noted to have an expanding hematoma at the right IJ puncture site with deviation of his trachea to the left, increasing dyspnea and a systolic BP of 90 mm Hg. International normalized ratio (INR) was 4.5, with increased bleeding from the paracentesis site. The next best step in the management of this patient is:

A. Emergent neck exploration B. 1:1:1 transfusion of packed red blood cells (PRBCs), fresh frozen plasma (FFP), and platelets C. Endotracheal intubation D. CT angiogram of the neck E. Exploratory laparotomy and control of abdominal bleeding

33. A 63-year-old female with no significant past medical history

underwent exploratory laparotomy and extensive cytoreductive surgery with right hemicolectomy and ileotransverse anastomosis for mucinous adenocarcinoma of the appendix. She was admitted to the SICU for her immediate postoperative care with patientcontrolled epidural anesthesia for pain control and extubated on postoperative day 1. Beginning from postoperative day 3 into postoperative day 4, she was noted to be having persistent tachycardia that was unresponsive to volume resuscitation and adequate pain control. She had mild right lower quadrant tenderness. ECG showed sinus tachycardia, and bedside echocardiography was unimpressive. Noninvasive duplex ultrasonography was negative for deep venous thrombosis. The next step in management of this patient should be:

A. CT abdomen and pelvis with oral and intravenous contrast B. Abdominal ultrasound C. Formal transthoracic ultrasound D. Intravenous broad-spectrum antibiotics E. Esophagogastroduodenoscopy and colonoscopy

34. A 65-year-old female presented to the emergency department with a 5-day history of nausea, vomiting, and left lower quadrant abdominal pain. She had abdominal tenderness and rebound tenderness on physical examination. CT scan of the abdomen and pelvis showed perforated diverticulitis. She received 2 L of Ringer ’s lactate and is taken emergently to the operating room for exploratory laparotomy. Intraoperatively, she was found to have a perforated sigmoid diverticulitis with spillage of bowel contents into the abdomen. She had sigmoid colectomy, anastomosis of the descending colon to the upper rectum, and thorough irrigation of the abdominal cavity with 3 L of warm normal saline mixed with antibiotics. The abdomen was then closed. She received 3 L of crystalloids and 2 units of packed red cells intraoperatively and was transferred to the ICU. She remains intubated on mechanical ventilation. Her resuscitation was continued with Ringer ’s lactate at

150 mL/h. About 12 hours after arrival in the SICU, she became oliguric with elevated peak airway pressures. Her nasogastric tube output was negligible, and bladder pressure was 31 mm Hg. The next critical step in this patient’s care is what?

A. Increase in PEEP B. Increase in the dosage of intravenous sedative C. CT scan of the abdomen and pelvis with oral and intravenous contrast D. Decompressive laparotomy E. Administration of intravenous furosemide to improved urine output

35. A 73-year-old male had laparoscopic cholecystectomy for acute calculous cholecystitis 2 weeks ago followed by an uncompleted immediate postoperative course. He now presents to the emergency department with a 2-day history of upper abdominal pain, nausea, and vomiting. He is febrile to 101.5°F, and hypotensive with a systolic BP of 86 mm Hg. His abdomen is distended on physical examination with right upper quadrant tenderness and his WBC count was 14,000/mm3. A CT scan of the abdomen and pelvis showed a large amount of perihepatic, right paracolic, and left paracolic regions. He is given 3 L of intravenous crystalloids, started on broadspectrum antibiotics, and admitted to the ICU. What is the next best step in the patient’s 
management?

A. CT-guided drainage of intra-abdominal fluid collection B. Endoscopic retrograde cholangiopancreatography C. Magnetic resonance cholangiopancreatography D. Exploratory laparotomy E. Intravenous vitamin C, hydrocortisone, and thiamine

36. A 70-year-old male with a history of coronary artery disease successfully underwent an endovascular repair of a 7 cm abdominal aortic aneurysm and was admitted to the ICU for neurovascular monitoring. On postoperative day 2, he was started on clear liquid diet. About 6 hours later, he had increasing left upper quadrant pain and nausea with slight difficulty breathing. Physical examination revealed generalized abdominal distention and localized tenderness in the left upper quadrant. An abdominal x-ray showed a markedly dilated stomach with a nonobstructive bowel gas pattern. Which of the following interventions should be performed next?

A. Aggressive bowel regimen B. Placement of a nasogastric tube C. High-flow nasal oxygen D. Noninvasive ventilation with bilevel positive airway pressure (BiPAP) E. Upper gastrointestinal endoscopy

37. A 23-year-old male was involved in a high-speed motor vehicle crash. He sustained multiple pelvic and lower extremity fractures. He is hemodynamically stable and is admitted to the ICU. His serum creatine phosphokinase is 29,500 U/L. Which of the following measures should be instituted to reduce the risk of AKI?

A. Aggressive fluid resuscitation B. IV mannitol C. Alkalinization of the urine D. Hemodialysis E. Intravenous urogram

38. A 55-year-old female was having a right internal jugular central

venous line placed under ultrasound guidance. She required multiple attempts before successful cannulation of the vein. Following the central line placement, her oxygen decreased from 97% to 86% on 2 L/min nasal cannula with hypotension. Bedside ultrasound of the chest revealed absence of lung sliding on the right. What is the next best step in the patient’s management?

A. Administration of vasopressors B. Fluid resuscitation C. Immediate chest CT scan D. Right tube thoracostomy E. Endotracheal intubation

39. During a bedside open tracheostomy on a 75-year-old female in the ICU, the respiratory therapist noted extreme difficulty in ventilating the patient after placement of the tracheostomy tube and removal of the endotracheal tube. The patient’s peak airway pressures were elevated, along with hemodynamic instability and subcutaneous emphysema involving the head and neck. What is the most likely underlying pathophysiology?

A. Bleeding from the injury to the anterior jugular veins B. Rupture of preexisting pulmonary bullae C. Abdominal compartment syndrome D. Massive hemothorax E. Tension pneumothorax

40. A 71-year-old female patient is admitted to the intensive care unit with hypotension and shortness of breath. She received 2 L crystalloids over 2 hours, however, continues to be hypotensive and requires moderate doses of phenylephrine to maintain MAP above 65 mm Hg. Also, she is receiving supplemental oxygen via high-flow

nasal cannula (FiO2 50%, 50 L/min). Her past medical history is significant for congestive heart failure and chronic obstructive pulmonary disease (COPD). A bedside ultrasound is performed to assess fluid responsiveness. The following image is obtained:

M mode through inferior vena cava in subcostal view.

Which among the following is true regarding volume responsiveness in this patient? A. Patient is fluid responsive B. Unable to interpret fluid responsiveness C. Patient is fluid overloaded D. Patient is fluid responsive to 5% albumin

41. A 54-year-old woman is admitted to the ICU for hypoxemic respiratory failure secondary to pneumonia. Her oxygen saturation

is 90% on high-flow nasal cannula (FiO2 60%, 60 L/min). She is also in septic shock and needs a high-dose norepinephrine infusion to keep MAP above 65 mm Hg, despite adequate fluid resuscitation. You decide to place a central venous line for vasopressor infusion. During ultrasound-guided central line placement in the right internal jugular vein, the patient coughs; and blood pressure abruptly decreases from 110/54 to 90/45 mm Hg and her heart rate increases from 90 to 120 beats/min in sinus rhythm. A bedside transthoracic echocardiogram shows a hyperdynamic left ventricle and a mildly hypokinetic right ventricle. What is the next best step in management?

A. Discontinue PEEP B. Aspirate the central line C. Increase FiO2 to 100% D. Chest tube placement

42. A 59-year-old woman is placed on peripheral venoarterial extracorporeal membrane oxygenation (VA-ECMO) for pneumonia with septic shock complicated by stress cardiomyopathy with a decrease in left ventricular ejection fraction to 10%. On ECMO day 2, she develops frothy secretions from the endotracheal tube, and chest radiograph shows diffuse consolidations. Her arterial line waveform demonstrates decreased pulsatility, and inferior ST wave depressions are noted on ECG. What is the most likely transthoracic echocardiogram finding?

A. Left ventricle distension B. Papillary muscle rupture C. Right ventricle distension D. Patent foramen ovale (PFO)

43. A 67-year-old man with a history of dilated cardiomyopathy with an ejection fraction of 20% is admitted with septic shock secondary to pyelonephritis. He is placed on VA-ECMO for shock unresponsive to fluid resuscitation and vasopressors. The heparin infusion rate required to maintain a therapeutic activated clotting time increases from 10,000 units/24 hours to 20,000 units/24 hours over his admission. On ECMO day 6, the patient begins to bleed from venous access sites, and a clot is noted in the ECMO oxygenator. PreECMO labs were hemoglobin 9 g/dL and platelets 110,000/mm3; labs now show hemoglobin 8 g/dL, platelets 55,000/mm3, and coagulation studies remarkable only for an activated partial thromboplastin time (aPTT) of 70 seconds. What is the most likely diagnosis?

A. Heparin resistance B. Disseminated intravascular coagulation C. Supratherapeutic heparin dose D. Heparin-induced thrombocytopenia

44. A 34-year-old woman with a history of hereditary hemorrhagic telangiectasia (HHT) is admitted to the ICU intubated and sedated following an exploratory laparotomy for a stab wound to the abdomen. Sedation is weaned on postoperative day 1; the patient’s mental status is maintained at Richmond Agitation Sedation Scale (RASS) 0, and she passes a spontaneous breathing trial (SBT). During injection of hydromorphone in her central line, the luer-lock hub becomes disconnected and the line is left open to air. Moments later, blood pressure decreases to 85/45 mm Hg, heart rate rises to 110 beats/min in sinus tachycardia, and respiratory rate increases from 8 to 25 breaths/min. ST elevations are noted in leads II, III, and aVF on telemetry, and the patient suddenly exhibits a left facial droop. After hemodynamic stability is achieved, an emergent noncontrast head computed tomography shows no acute bleeding but did find an area of low attenuation in the left frontal lobe. In addition to ongoing hemodynamic support, what is the next best

step in management?

A. Cardiac catheterization with percutaneous coronary intervention B. Hyperbaric oxygen therapy (HBOT) C. Heparin infusion D. Tissue plasminogen activator initiation

45. A 78-year-old man with a history of COPD is admitted to the ICU intubated and sedated after colectomy with loop ileostomy creation for perforated diverticulitis. It is postoperative day 3, and despite broad-spectrum antibiotic therapy and fluid resuscitation, the patient continues to require a norepinephrine infusion to maintain MAP above 65 mm Hg. Blood glucose is 130 mg/dL. What is the best option for administering nutrition?

A. With normoglycemia, initiate nutrition after day 7 B. Total parenteral nutrition (TPN) C. Enteral tube feeds D. Dextrose infusion

46. A 67-year-old male with a history of severe COPD and morbid obesity remains intubated in the ICU POD 2 following an exploratory laparotomy and small bowel resection. Your respiratory therapist informs you that she has been having difficulty ventilating the patient despite changing from volume-control to pressurecontrol ventilation. You are then called stat to the room by his nurse, as his ECG demonstrates sinus bradycardia at 40 beats/min with associated hypotension (60/42 mm Hg). You are unable to palpate a carotid pulse. Your team initiates chest compressions and administers epinephrine. What is the most appropriate next step in managing this patient?

A. Administer a bronchodilator B. Remove the endotracheal tube and reintubate C. Needle decompression in the second intercostal space D. Disconnect the patient from the ventilator circuit E. Obtain a chest x-ray

47. An 84-year-old female is admitted to your ICU following surgical repair of her complex right hip fracture. She has a history significant for moderate aortic stenosis, hypertension, and COPD on 2 L/min oxygen via nasal cannula at home. All of the following interventions are recommended to reduce the risk of ventilator-associated pneumonia (VAP), EXCEPT:

A. Elevation of head of bed at least 30 degrees B. Early tracheostomy C. Daily SBTs D. Noninvasive positive-pressure ventilation (NIPPV) E. Minimize sedation

48. Your patient has been in the ICU for over 2 weeks and it is noted that her CVC has been in place since admission. You are concerned that it should be changed, as you are worried about a possible infection from the catheter. Which of the following statements regarding catheter-related bloodstream infections (CRBSIs) is true?

A. CVCs should be replaced weekly. B. CRBSI is diagnosed when blood cultures drawn from the catheter are positive. C. Arterial lines carry a negligible risk of CRBSI. D. Skin preparation with chlorhexidine/alcohol is more effective than povidone iodine in preventing CRBSI.

E. The most common pathogens in CRBSI are gram-negative rods.

49. A 67-year-old male is postoperative day 2 following a Whipple for pancreatic cancer. The surgeon requests that his Foley catheter be removed. Which of the following statements regarding catheterassociated urinary tract infection (CAUTI) is true?

A. External urinary catheters have similar rates of complications compared with indwelling catheters. B. Asymptomatic catheter-associated bacteriuria should be treated to reduce the risk of developing systemic infection. C. Intermittent catheterization does not reduce the risk of CAUTI. D. Indwelling catheters should be replaced at regular intervals to reduce the risk of CAUTI. E. Screening for bacteriuria is not effective in reducing CAUTI.

50. A 51-year-old female is admitted to the ICU following a pneumonectomy for cavitary tuberculosis. She reports that she may have received treatment “many years ago” but does not recall if she completed the full course of recommended therapy. All of the following statements regarding tuberculosis precautions are true EXCEPT:

A. The patient should be placed in a negative-pressure isolation room. B. Empiric therapy for tuberculosis with a four-drug regimen should be initiated immediately. C. Contact precautions are not necessary. D. Airborne precautions can be discontinued if a tuberculin skin test is negative. E. A bacterial filter should be placed on the ventilator circuit.

51. A 63-year-old female was admitted to the ICU following the development of septic shock, secondary to a presumed pneumonia. She has been treated with antibiotics for 1 week and has improved clinically. Your team is planning to extubate her this evening. When preparing to extubate this patient, all of the following criteria should be met EXCEPT?

A. Rapid shallow breathing index of less than 100 B. Minimal secretions C. Presence of cuff leak D. Successful SBT lasting 30 minutes using continuous positive airway pressure (CPAP) 5 E. Ability to follow commands

52. You care called to evaluate an 81-year-old male who has had an acute worsening of his already tenuous respiratory status. He has a remote history of a liver transplant, COPD, and recently diagnosed diffuse large B-cell lymphoma and is currently receiving a course of antibiotics for pneumonia. When speaking with his family, you discover he was admitted to the hospital 2 days ago via MedFlight from a community hospital in Georgia. You note that his oxygen saturation is 86% on high-flow nasal cannula and he is using accessory muscles to breathe. You obtain an arterial blood gas: pH 7.27, PaO2 56 mm Hg, PaCO2 49 mm Hg. His rhythm is atrial fibrillation with a heart rate of 115 beats/min and blood pressure of 80/55 mm Hg. You perform an urgent bedside echo:

Given what the beside echo shows, what is the most likely cause of this patient’s shock? A. Distributive B. Hypovolemic C. Cardiogenic D. Obstructive E. Unable to determine at this time

53. A 53-year-old female is admitted to the ICU in severe respiratory distress and circulatory shock. Her medical history is significant only for widely metastatic breast cancer to which she is currently undergoing systemic chemotherapy. Her vasopressor therapy has been steadily rising over the past 2 hours and you elect to place a PA catheter. Based on the image that follows, which of the following data sets would be obtained?

A B C D E

HR

CVP

PA

CO

SVR

120 80 50 120 150

1 7 25 25 2

10/5 20/10 45/30 45/30 12/8

4 4 3 3 6

1800 1300 1800 2000 500

54. A 56-year-old female with a history of tetraplegia following a bicycle accident over 10 years ago is admitted to the ICU following a ureteroscopy for multiple stone extraction. She has known recurrent UTIs and was recently in the ICU a few months ago for pyelonephritis complicated by urosepsis. The last urine culture obtained from that time was E coli, resistant to quinolones and cephalosporins. Her vital signs after a 2 L fluid bolus are temperature 38.8°C, heart rate 130 beats/min, blood pressure 75/50 mm Hg, and respiratory rate 25 breaths/min. She appears to be shivering and drowsy but does answer questions appropriately.

What is the next step in your management of this patient?

A. Call her primary urology team and recommend repeat ureteroscopy B. Start empiric vancomycin and cefepime and initiate vasopressor therapy C. Start empiric meropenem and repeat the fluid bolus D. Start empiric meropenem and initiate vasopressor therapy

55. A 31-year-old male is admitted to the ICU after he dived from a boat in shallow water and presumably hit his head on the bottom of the lake. He was pulled immediately from the water and reports that he “cannot feel” his arms or legs. Primary survey is notable for Glasgow Coma Scale (GCS) 15, absence of sensation below clavicles, and inability to move his arms and legs. His has weak cough and intermittent desaturations that resolve with deep nasal suctioning. Chest and abdominal CT scan are remarkable for a 5mm pneumothorax and grade 2 splenic laceration. MRI demonstrates a C6 subluxation injury with resulting spinal cord compression and edema at C4-C7 level. His blood pressure is 70/30 mm Hg, heart rate 50 beats/min, and O2 saturation is 94%, with supplemental oxygen via facemask. What is the primary etiology of his hemodynamic instability and impending shock state?

A. Spinal shock B. Neurogenic shock C. Obstructive shock D. Septic shock E. Hypovolemic shock

56. A 66-year-old male has been in the ICU for 5 days following a mandibular resection and partial glossectomy. He has failed a speech and swallow examination and is unable to safely intake

orally. Your team would like to start him on some form of nutrition and an enteral feeding tube is placed. Which of the following statements regarding nutrition in the ICU is true?

A. Gastric residual volumes should be frequently measured to monitor for feeding intolerance. B. Initiation of enteral nutrition should be delayed until signs of return of bowel function. C. Early enteral nutrition decreases mortality. D. Protein has the lowest respiratory quotient. E. A postpyloric tube should be placed for enteral feeding.

57. A 74-year-old female is in postoperative day 1 following a sigmoid resection for colon cancer. Her past medical history is significant for a right-sided stroke 5 years ago (she has some residual left upper extremity weakness), hypertension, and coronary artery disease (coronary stents placed 1 year ago). She takes a full-dose aspirin and clopidogrel at home, which were both held for her procedure. She now complains of chest pain and shortness of breath. Her ECG shows 2 mm ST elevations in V2-V4 leads. Her vital signs are heart rate 72 beats/min, blood pressure 92/55 mm Hg, and SpO2 95% on room air. Labs are remarkable for troponin 3.8 ng/mL and lactate 8 mmol/L. A cardiology consult is obtained, and an emergent cardiac catheterization is planned. What would be another appropriate initial treatment modality at this time?

A. Dopamine B. 1L crystalloid bolus C. Dobutamine D. Norepinephrine E. No other intervention necessary besides immediate cardiac catheterization

58. A 79-year-old male with a history significant for dementia, hypertension, and diabetes mellitus was admitted from his skilled nursing facility for failure to thrive. He was not able to safely take per orally due to risk of aspiration. A nasogastric tube is placed and enteral feeding initiated on hospital day 2. Labs the following day are notable for phosphate 0.6 mg/dL, total calcium 5.5 mg/dL, magnesium 1.2 mg/dL, potassium 2.6 mEq/L, and albumin 1.4 g/dL. Of the following choices, which is the most appropriate regarding the next step in management of this patient?

A. Increase enteral feeds B. Temporarily stop enteral feeding C. Switch to TPN D. Initiate workup for hyperparathyroidism E. Increase phosphate, magnesium, and potassium content in enteral feeds

59. You are called urgently for an intubation in the emergency department for a young woman who was found stumbling out of a subway terminal confused and with labored breathing. She has copious oral secretions and appears to have recently vomited. As you prepare to intubate, the patient has a seizure. You hear of other patients arriving from the same subway station with a similar presentation and suspect a terrorist attack with a biologic or chemical weapon. Which agent is most likely?

A. Sarin B. Chlorine C. Phosgene D. Cyanide E. Anthrax

60. A 32-year-old female with a history of severe depression and multiple suicide attempts is brought to the emergency department by her mother after she admitted to ingesting 10 pills of lorazepam. She is somnolent but does wake up following sternal rub. She is hemodynamically stable and is maintaining oxygen saturations of 94% on room air. What is the next step in managing this patient?

A. Administer 2 mg of flumazenil B. Intubate her for inability to protect her airway C. Continue to observe her with continuous telemetry monitoring D. Directly admit her to an inpatient psychiatry unit E. Obtain a stat head CT

61. A 34-year-old male was admitted to the intensive care unit 1 week ago after polytrauma sustained after a motor vehicle accident. Major injury burden includes multiple rib fractures, lung contusion, splenic laceration, and lower extremity long bone fractures. His ventilator settings are pressure support ventilation with a pressure support of 5 cm H2O over PEEP of 8 cm H2O and FiO2 of 30%. His minute ventilation remains high at 13 L/min. His vital are BP 130/70 mm Hg, HR 82 beats/min, and respiratory rate 30 breaths/min. An arterial blood gas is drawn to evaluate further.

pH 7.29 Bicarbonate 14 mEq/L pCO2 30 mm Hg pO2 105 mm Hg Additional lab results are:

Na 138 mEq/L K 3.5 mEq/L

Cl 110 mEq/L Albumin 3.8 mg/dL Creatinine 2.8 mg/dL Urine electrolytes:

Na 40 mEq/L K 10 mEq/L Cl 15 mEq/L What is the next best step in management of this patient? A. 1 L normal saline B. Enteral bicarbonate replacement C. Hemodialysis D. Loop diuretics

62. A 48-year-old female presents to the intensive care unit with 2-day history of abdominal pain and vomiting. Her past medical history is significant for recurrent small bowel obstruction and multiple abdominal surgeries in the past. Vitals are blood pressure 80/40 mm Hg, heart rate 112 beats/min, and temperature 37°C. An arterial blood gas is drawn with the following results:

pH 7.31 Bicarbonate 19 mEq/L pCO2 37 mm Hg Na 138 mEq/L K 4 mEq/L Cl 98 mEq/L Albumin 3.0 mg/dL What type of acid-base abnormality is seen in this patient? A. Anion gap and normal anion gap metabolic acidosis

B. Unable to determine with given information C. Metabolic acidosis and metabolic alkalosis D. Normal anion gap metabolic acidosis

63. A 65-year-old male with past medical history of congestive heart failure, diabetes, and COPD presents with shortness of breath. He reports progressive shortness of breath for the last 5 days. He is admitted to the intensive care unit for hypoxemia and is placed on noninvasive mechanical ventilation. On examination, the jugular venous pressure is elevated. His vitals are blood pressure 110/70 mm Hg, heart rate 102 beats/min, and respiratory rate 28 breaths/min.

Arterial blood gas shows pH 7.26 Bicarbonate 14 mEq/L pCO2 30 mm Hg Laboratory data Na 132 mEq/L Chloride 98 mEq/L K 4.5 mEq/L Albumin 3 mg/dL Blood glucose 105 mg/dL What is the next best step in investigating the cause of this acid-base abnormality? A. Urine anion gap B. Serum lactic acid C. Beta-hydroxy butyrate D. Urine chloride E. Serum osmolar gap

64. A 72-year-old patient is admitted to intensive care unit with altered

mental status. Her past medical history is remarkable for recent hospitalization for periprosthetic hip infection, for which she underwent a prosthesis explant. She was discharged to rehabilitation center 1 week ago, on a 6-week course of cefepime for hip osteomyelitis.

To evaluate the cause of the altered mentation, a CT scan of the head and lumbar puncture is done, both with negative results. After ruling out other etiologies, she is suspected to have cefepime-induced neurotoxicity. Which among the following is NOT useful in management of cefepimeinduced neurotoxicity? A. Hemodialysis B. Antiepileptic medications C. Discontinuation of cefepime D. Corticosteroids

65. A 66-year-old female presents to intensive care unit with altered mental status. Her past medical history is significant for diabetes on metformin, chronic back pain recently started on gabapentin, chronic kidney disease, COPD, and recent-onset hypertension on hydrochlorothiazide and lisinopril. She has no history of fall, and her vital signs at admission are within normal limits. Her laboratory data are as follows:

Na 118 mEq/L Cl 95 mEq/L Creatinine 1.3 mg/dL Glucose 85 mg/dL What medication is the moist likely culprit for this patient’s hyponatremia? A. Metformin B. Gabapentin

C. Lisinopril D. Hydrochlorothiazide

66. An 82-year-old frail looking female presents to the emergency room with failure to thrive. She reports having diarrhea for last 2 days. She currently weighs 45 kg, with 10 kg weight loss in last 6 months. Vitals are blood pressure 82/40 mm Hg, heart rate 108 beats/min, and respiratory rate 24 breaths/min. Lungs are clear to auscultation. She is lethargic but oriented to time, place, and person.

Laboratory blood work shows the following:

Na 121 mEq/L K 3.2 mEq/L Cl 95 mEq/L Creatinine 1.5 mg/dL Hemoglobin 9 g/dL Serum osmolality 270 mOsm/kg Urine Na 5 mEq/L What is the next best step in management of her hyponatremia? A. Resuscitation with normal saline B. Diuresis with loop diuretics C. Water restriction D. Start salt tablets

67. The previous patient receives 1 L normal saline with improvement in blood pressure. Repeat sodium in 4 hours shows sodium to be 118 mEq/L. Now, she appears more confused and is only oriented to self. You decide to start an infusion of hypertonic saline and check serum sodium levels every 2 hours.

What is the benefit of adding desmopressin to this patient’s treatment regimen? A. For rapid correction of hyponatremia B. To treat occult diabetes insipidus C. Prevent inadvertent overcorrection of hyponatremia D. For evaluating cause of hyponatremia

68. A 45-year-old male with Crohn disease presents to the intensive care unit after urgent exploratory laparotomy, extensive lysis of adhesions, and small bowel resection for small bowel obstruction. Past medical history is significant for bipolar disorder on quetiapine. He was on lithium previously but was stopped 5 years ago. On postoperative day 1, he was noted to have a high urine output (300-400 mL/h). Blood and urine electrolytes were sent to investigate the etiology. His serum sodium was 155 mEq/L and serum osmolarity, 315 mOsm/Kg. His urine sodium was 5 mEq/L and urine osmolarity, 214 mOsm/Kg. His preoperative serum sodium was 143 mEq/L.

What is the next best step in management? A. Decrease intravenous maintenance fluids by half B. Restart lithium C. Start 5% dextrose water intravenously D. Start desmopressin

69. A 66-year-old male presents with perforated diverticulitis and undergoes urgent exploratory laparotomy and sigmoid resection. His past medical history is significant for hypertension, diabetes on insulin, COPD, and bipolar disorder on lithium. The operative course is complicated by intra-abdominal spillage of bowel contents and hypotension requiring vasopressors. Postoperatively, he is brought to the intensive care unit for close monitoring. On postoperative day 2, he develops acute mental status changes and

slurred speech. Vital signs are with in normal limits. CT scan of the head was done which was unremarkable.

Laboratory data show:

Na 144 mEq/L Cl 100 mEq/L K 4.0 mEq/L Creatinine 1.8 mg/dL Urea 56 mg/dL Hb 10 g/dL What is the most likely cause for the neurological symptoms seen in this patient? A. Abrupt reduction in lithium levels B. Acute lithium toxicity C. Chronic lithium toxicity D. Unrelated to lithium

70. A 62-year-old male is admitted to intensive care unit with intraabdominal sepsis. He developed septic cardiomyopathy 2 days later, with a left ventricular ejection fraction of 25%. So, a pulmonary artery catheter was placed to guide fluid management. The pulmonary artery catheter is wedged to determine the left ventricular end-diastolic pressure. Which point in the waveform correctly represents the wedge pressure?

A. Point A B. Point B

C. Point C D. Point D

71. A 71-year-old male with is admitted to ICU with COPD exacerbation. At admission, his vitals are BP 110/72 mm Hg, heart rate 78 beats/min, normal sinus rhythm, temperature 38°C, respiratory rate 22 breaths/min, and oxygen saturation 93% on 5 L oxygen via nasal cannula. Next day, the patient complains of palpitations, and you find him to be tachycardic in 130 beats/min. His blood pressure is 100/68 mm Hg. The following rhythm is noticed.

Adapted from Appendix 1: Atlas of electrocardiography. In: Barash PG, Cullen BF, Stoelting RK, et al. Clinical Anesthesia. Philadelphia, PA: Lippincott, Williams & Wilkins; 2013.

What is the next best step in management? A. Cardioversion B. Beta blockade C. Digoxin D. Anticoagulation E. Echocardiogram

72. A 55-year-old male with chronic liver disease is admitted to ICU with decompensated cirrhosis. He is known to have portopulmonary hypertension and is on sildenafil. A pulmonary artery catheter is placed to guide therapy and the following measurements were

obtained. Calculate the pulmonary vascular resistance (PVR) in dyne-sec/cm5.

Cardiac output Mean arterial blood pressure Mean pulmonary arterial pressure Pulmonary capillary wedge pressure Central venous pressure

5.2 L/min 91 mm Hg 28 mm Hg 14 mm Hg 12 mm Hg

A. 215 B. 1215 C. 2.7 D. Cannot be calculated with the given information

73. A 47-year-old diabetic male presents to the intensive care unit in distributive shock. On examination, the likely source is soft-tissue infection of his leg. Which among the following is NOT a component of the 1-hour (Surviving Sepsis Campaign) sepsis bundle?

A. Obtaining blood cultures before antibiotics B. Starting 30 mL/kg of crystalloid resuscitation C. Serum procalcitonin before antibiotics D. Serum lactate concentration

74. Which one among the following parameters is NOT used in calculating the qSOFA (quick Sequential Organ Failure Assessment) score described in Sepsis-3 recommendations?

A. Respiratory rate B. Serum lactate C. Blood pressure

D. Mental status

75. A 55-year-old female patient is admitted to the intensive care unit with a diagnosis of septic shock. She received 30 mL/kg of crystalloid resuscitation in the emergency department. Currently, she is receiving a moderate dose (14 µg/min) of norepinephrine infusion to maintain a MAP above 65 mm Hg. Based on Surviving Sepsis Guidelines, additional fluid resuscitation can be based on which among the following parameters?

A. Central venous pressure (CVP) B. Central venous oxygen saturation C. Response to PLR D. Pulmonary capillary wedge pressure

76. A patient presents to the intensive care unit in atrial fibrillation with rapid ventricular rate and is short of breath. He is known to have paroxysmal atrial fibrillation which was first detected after he underwent mechanical mitral valve replacement for severe rheumatic mitral stenosis. He has been therapeutically anticoagulated with warfarin for the mechanical mitral valve.

In the past, he has underwent cardioversion three times for symptomatic atrial fibrillation. Now, he is started on amiodarone for rhythm control, since he is short of breath while in atrial fibrillation. What is the impact of coadministering amiodarone and warfarin? A. Warfarin and amiodarone cannot be coadministered B. Decrease warfarin dose C. Increase warfarin dose D. Amiodarone has no impact on warfarin dose

77. A 61-year-old male presents with altered mental status 6 days after craniotomy and resection of meningioma. CT head is unremarkable. A lumbar puncture is done which is suggestive of bacterial meningitis. He is empirically started on vancomycin and ceftriaxone for empiric coverage. His cerebrospinal fluid (CSF) culture grows Serratia marcescens; resistant to ampicillin and cefazolin, sensitive to ceftriaxone.

Which of the following is the next best step? A. Stop vancomycin, continue ceftriaxone B. Continue both vancomycin and ceftriaxone C. Stop vancomycin and ceftriaxone, start meropenem D. Stop vancomycin and ceftriaxone, start penicillin G

78. A 34-year-old male is admitted to the ICU after a motor vehicle crash and associated polytrauma. Major injuries include multiple rib fractures, lung contusions, and traumatic brain injury. He was intubated on arrival due to poor mental status. Five days into his ICU stay, he remains intubated and mechanically ventilated. He develops increased tan-colored respiratory secretions and increasing oxygen requirements and is febrile. He is empirically started on broad-spectrum antibiotics—vancomycin and cefepime. Preliminary results are available from respiratory culture, shows Stenotrophomonas. Antibiotic resistance patterns are not yet available.

Which among the following alterations in antimicrobial therapy is most appropriate? A. Stop all antibiotics B. Stop cefepime, and start meropenem C. Add trimethoprim-sulfamethoxazole D. Stop vancomycin and cefepime, and start penicillin G

E. No change to current regimen

79. A 55-year-old female has been admitted to the intensive care unit for hypoxemic respiratory failure presumably from exacerbation of COPD. She is being treated with steroids, broad-spectrum antibiotics, and diuresis. A urinary catheter is placed for monitoring urinary output after diuresis. Four days later, she continues to have leukocytosis, and hence, urine, blood, and respiratory cultures are sent to investigate leukocytosis. The urine analysis shows candida species.

What is the next best step, while urine culture data are pending? A. Start fluconazole B. Start micafungin C. CT scan of the abdomen D. Change the urinary catheter

80. A 55-year-old male patient is admitted to the intensive care unit with hypoxemic respiratory failure, AKI, altered mental status, and distributive shock. He is undergoing treatment for acute lymphocytic leukemia and received an infusion of chimeric antigen receptor modified T cells 3 days ago.

Which among the following therapies would be most beneficial? A. Tocilizumab B. Abciximab C. Intravenous immunoglobulin D. Plasma exchange

81. A 60-year-old patient was admitted in the intensive care unit with pneumonia, hypoxemic respiratory failure, and altered mental

status. He was intubated and mechanically ventilated upon arrival to the intensive care unit. His past medical history is significant for hypertension and COPD. Five days later, despite improvement in lung function, his mental status does not. Thyroid function tests are sent to evaluate for causes of altered mental status.

Which among the following is expected to be elevated in critical illness? A. Thyroid-stimulating hormone B. Total T4 C. T3 D. rT3

82. A 68-year-old male patient undergoes transvenous extraction of a right ventricular pacemaker lead due to lead malfunction and implantation of a new lead. His past medical history is significant for coronary artery disease, sick sinus syndrome, and diabetes. He is found to have a new small pericardial effusion after the lead explant and so is admitted to the ICU for hemodynamic monitoring. Over the next 8 hours, he is progressively hypotensive, and you perform a bedside echo. Left ventricular function is hyperdynamic, and the following images are obtained.

M mode through inferior vena cava in subcostal view

Parasternal long axis view

What is the next best step in management? A. Urgent pericardiocentesis B. Administer 1 L normal saline C. No further intervention D. Start inotropes

83. A 67-year-old male with no significant past medical history is admitted to the ICU following angiography and embolization of a lingual artery pseudoaneurysm after developing significant oropharyngeal hemorrhage following resection of a 1 cm oropharyngeal squamous cell carcinoma. The left common femoral artery was used as the access point. Six hours later, he developed coldness of his left foot with absence of the left popliteal, dorsalis pedis, and posterior tibial pulses. The most likely cause of this is:

A. Low-flow state B. Embolus to the left common femoral artery

C. Thrombosis of the left common femoral artery D. Left iliofemoral deep venous thrombosis E. Excessive compression with sequential compression devices

84. Which among the following statements accurately represent the latest 2016 Surviving Sepsis Guideline recommendations?

A. Volume resuscitation needs to be guided by early goal-directed therapy. B. All available sites should be sampled for microbial culture before initiating antibiotic therapy. C. Antibiotics need to be administered within 1 hour of presumed sepsis. D. Resuscitation and metabolic stabilization for at least 24 hours is recommended before operative intervention for source control.

85. A 55-year-old male patient is transferred to your ICU (from an outside hospital) on high-dose inotropes for cardiogenic shock. Urgent transthoracic echocardiogram on arrival shows:

Left ventricular hypertrophy, small left ventricular cavity, left ventricular ejection fraction of 25% with wall motion abnormalities in left circumflex distribution, mild right ventricular dysfunction, mild mitral regurgitation, no aortic regurgitation or aortic stenosis. An intra-aortic balloon pump (IABP) is inserted for left ventricular support as he is being prepared for cardiac catheterization. But the patient becomes severely hypoxemia and requires intubation. In this patient, which among the following conditions will cause worsening of hypoxemia after insertion of an IABP? A. Hypertrophic obstructive cardiomyopathy B. Ventricular septal defect C. Papillary muscle rupture

D. Left main coronary obstructive lesion

Appendix Answers 1. Correct Answer: B Rationale: The number 150 can help a clinician to distinguish between groups of patients who may and may not need NAC therapy after APAP ingestion.

A toxic dose is greater than 150 mg/kg. Start NAC therapy if APAP serum level is greater than 150 µg/mL 4 hours after ingestion (the first time point on the nomogram). Most guidelines outside of the United States use the “200 line” instead of this “150 line.” The 200 line was initially used in the United States but was lowered to 150 µg/mL to increase sensitivity. Patients with certain comorbidities including chronic alcoholism and other liver disease may benefit from a “100-line,” or a more sensitive threshold, before starting NAC.

The NAC loading dose is 150 mg/kg IV (140 mg/kg PO). References

1. Heard KJ. Acetylcysteine for acetaminophen poisoning. N Engl J Med. 2008;359(3):285-292.

2. Waring WS, Robinson ODG, Stephen AFL, et al. Does the patient history predict hepatotoxicity after acute paracetamol overdose? Q J Med. 2008;101(2):121-125.

3. Vale JA, Proudfoot AT. Paracetamol (acetaminophen) poisoning. Lancet. 1996;346(8974):547-552.

4. Hodgman MJ, Garrard AR. A review of acetaminophen poisoning. Crit Care Clin. 2012;28(4):499-516.

2. Correct Answer: B Rationale: This clinical scenario is most consistent with an ingestion of digoxin leading to toxicity. Virtually any arrhythmia is possible in digoxin overdose, save rapid supraventricular tachycardias. Bidirectional VT is almost pathognomonic of digoxin toxicity, but is a rare finding. Most commonly, one will see rhythms with AV junctional blocks of varying degrees and ventricular ectopy. Slow or regularized atrial fibrillation (AFib) or flutter (AFlutter) is also highly suggestive of digoxin overdose. Cardiac Dysrhythmias Associated With Digoxin Toxicity Myocardial Irritability Causing Dysrhythmias

AFib and AFlutter w/ AV block Nonparoxysmal atrial tachydysrhythmias w/ AV block Premature ventricular contractions (PVCs) Nonsustained ventricular tachycardia (NSVT) Ventricular bigeminy Bidirectional VT VT VFib

Primary Conduction System Dysfunction Causing Dysrhythmia AV dissociation Exit blocks High-degree AV block His-Purkinje dysfunction Junctional tachycardia SA nodal arrest Sinus bradycardia

Adapted from Cardiac dysrhythmias associated with cardioactive steroid poisoning. In: Hoffman RS, Howland MA, Lewin NA, Nelson LS, Goldfrank LR, eds. Goldfrank’s Toxicologic Emergencies. 10th ed.

McGraw-Hill Education; 2015.

Hemodialysis (HD) would be a poor choice of therapy, as digoxin cannot be dialyzed. It may be a more appropriate choice in certain instances of salicylate (acetylsalicylic acid [ASA]) or lithium toxicity, as those compounds are able to be hemodialyzed. Administration of intravenous bicarbonate, a way to alkalinize the urine in order to promote trapping and excretion, is a first-line therapy for ASA overdose if the patient does not meet criteria for HD. Hyperkalemia can be a common finding in digoxin overdose, though it serves only to prognosticate: its treatment will not lead to improvement in outcome. It has been classically thought, based on theoretical molecular mechanisms, that administering exogenous calcium can lead to “stone heart,” or impaired diastolic function leading to a hypocontractile state. Despite the fact that recent literature does not support this belief, many believe IV calcium to be potentially dangerous in situations of digoxin overdose. The best treatment would be administration of digoxin-specific antibody fragment derived from sheep, marketed as Digibind and DigiFab. Indications for Use of Digoxin-Specific Antibody Fragments Any digoxin-related life-threatening dysrhythmias (regardless of digoxin level) K+ >5 mEq/L (after acute digoxin poisoning) Chronic e le vation of se rum digoxin le ve l associate d with any of the following: Dysrhythmias Significant GI symptoms Altered mental status

Serum digoxin level >15 ng/mL at any time Se rum digoxin le ve l >10 ng/mL 6 hour post inge stion

Acute ingestion of 10 mg of digoxin in an adult Acute ingestion of 4 mg of digoxin in a child Poisoning with a nondigoxin cardioactive steroid

Adapted from Indications for administration of digoxin-specific antibody fragments (DSFab). In: Hoffman RS, Howland MA, 
Lewin NA, Nelson LS, Goldfrank LR, eds. Goldfrank’s Toxicologic Emergencies. 10th ed. McGraw-Hill Education; 2015.

References

1. Hack JB. Chapter 64: Cardioactive steroids. In: Nelson LS, Lewin NA, Howland M, et al, eds. Goldfrank’s Toxicologic Emergencies. 9th ed. New York: McGraw-Hill; 2011:936-945.

2. Howland M. Chapter A20: Digoxin-specific antibody fragments. In:

Nelson LS, Lewin NA, Howland M, et al, eds. Goldfrank’s Toxicologic Emergencies. 9th ed. New York: McGraw-Hill; 2011:946-951.

3. Levine M, Nikkanen H, Pallin DJ. The effects of intravenous calcium in patients with digoxin toxicity. J Emerg Med. 2011;40(1)41-46.

4. Kelly RA, Smith TW. Recognition and management of digitalis toxicity. Am J Cardiol. 1992;69(18):108G-118G.

3. Correct Answer: C Rationale: Antipsychotics include first-generation antipsychotics (typical antipsychotics, neuroleptics), as well as newer second-generation antipsychotics (atypical antipsychotics). Examples of each are included in the table below. First-Generation Antipsychotics Haloperidol (Haldol) Fluphenazine (Prolixin) Pimozide (Orap) Perphenazine (Trilafon) Chlorpromazine (Thorazine)

Second-Generation Antipsychotics Clozapine (Clozaril) Risperidone (Risperdal) Olanzepine (Zyprexa) Quetiapine (Seroquel) Lurasidone (Latuda) Paliperidone (Invega)

In addition to their primary (intended) effect as dopamine (D2) receptor antagonists, antipsychotics (atypicals > typicals) exhibit varying degrees of effects on other receptors. This is responsible for many of their adverse side effects. These include histamine (H1), adrenergic (α1), and muscarinic (M1) receptor antagonism, which can be thought of as the “HAM” effects. Receptor

Effects of Antipsychotic Medications (Receptor Blockade)

H1

Sedation

α1

Orthostatic hypotension (dizziness) → reflex tachycardia

M1

Anticholinergic side effects (dry mouth, blurry vision, constipation, urinary retention)

Of the choices listed, bradycardia is least likely, as the patient is more likely to experience orthostatic hypotension with a reflex tachycardia.

References

1. Voicu VA, Rădulescu F, Gheorghe M. Pharmacodynamic and therapeutic effects of antipsychotics. Modulation and neurobiological resetting. Ther Clin Pharmacol Toxicol. 2008;12(1):11-25.

2. Miron IC, Baroană VC, Popescu F, Ionică F. Pharmacological mechanisms underlying the association of antipsychotics with metabolic disorders. Curr Health Sci J. 2013;40(1):12-17.

4. Correct Answer: A Rationale: Tricyclic antidepressants (TCAs) act as sodium (Na+) channel blockers. As there are a large number of Na+ channels in the heart and central nervous system (CNS), the toxicity of TCAs are readily seen in these organs/systems. Drugs that cause cardiac Na+ channel blockade slow Na+ influx into the cell and delay the depolarization phase (phase 0) of the ventricular myocyte action potential. This results in widening of the QRS complex. In severe cases, this may lead to a sinusoidal pattern on electrocardiogram, possible ventricular tachycardia with degeneration to ventricular fibrillation, and eventual asystole. A similar phenomenon can be seen with hyperkalemia, as well as overdose with: Class IA antiarrhythmics Class IC antiarrhythmics Phenothiazines Cocaine Quinine (an isomer of quinidine), hydroxychloroquine (Plaquenil) Carbamazepine (Tegretol) Diphenhydramine (Benadryl) Bupropion (Wellbutrin) Sinus tachycardia is more commonly seen in TCA overdose, as many of these compounds have anticholinergic or sympathomimetic properties. Bradycardias are therefore rare but can be seen if Na+ channel blockade is so significant that the cardiac pacemaker tissue is affected, resulting in pacemaker cells which are slow to depolarize. Noting bradycardias in a TCA-overdose patient is an ominous sign and should be treated immediately with administration of IV sodium bicarbonate.

References

1. Clancy C. Chaper 22: Electrophysiologic and electrocardiographic principles. In: Nelson LS, Lewin NA, Howland M, et al, eds. Goldfrank’s Toxicologic Emergencies. 9th ed. New York: McGraw-Hill; 2011:314-329.

2. Holstege CP, Eldridge DL, Woden AK. ECG manifestations: the poisoned patient. Emerg Med Clin N Am. 2006;24(1):159-177.

3. Harrigan RA, Brady WJ. ECG abnormalities in tricyclic antidepressant ingestion. Am J Emerg Med. 1999;17(4):387-393.

5. Correct Answer: C Rationale: The triad of medications this patient is on increases the risk of a prolonged QT interval. Torsade de pointes (TdP) is a polymorphic VT that occurs in the setting of a prolonged QT and may be either nonsustained or sustained. It is usually a nonperfusing rhythm (as is suggested in our patient with hypotension), can rapidly degenerate into ventricular fibrillation, and therefore urgent treatment with defibrillation is warranted. Because TdP is a polymorphic ventricular tachycardia, synchronized cardioversion may not be possible, in which case defibrillation (unsynchronized cardioversion) may be performed. Magnesium sulfate (MgSO4) IV may be given in a conscious (hemodynamically stable) patient to treat self-limited TdP episode or prevent their recurrence. Magnesium boluses (1-2 g) are repeated or infusion is initiated until QTc duration drops below 500 ms. If patients are refractory to magnesium infusion, one may consider overdrive (transvenous, either atrial or ventricular) pacing at about 70 to 100 beats/min, as the pauses that trigger TdP are less likely to occur at faster heart rates. While not the BEST therapy at this point, ILE warrants further discussion. Guy Weinberg at University of Illinois, Chicago has done substantial work involving ILE therapy, the gold standard treatment for local anesthetic systemic toxicity (LAST). However, recent reviews detail the successful use of ILE in humans with toxicity from bupropion/lamotrigine-induced cardiac arrest, reversal of sertraline-/quetiapineinduced coma, and improvement of shock due to verapamil and beta-

blocker overdose. In one case report, a 17-year-old female with bupropion and lamotrigine overdose had cardiovascular collapse which was refractory to standard advanced cardiac life support (ACLS) algorithm. She had ROSC 1 minute after intralipid administration. References

1. Drew BJ, Ackerman MJ, Funk M, et al. Prevention of torsade de pointes in hospital settings: A scientific statement from the AHA and American College of Cardiology Foundation. Circulation. 2010;121(8):1047-1060.

2. Tzivoni D, Banai S, Benhorin J, et al. Treatment of torsade de pointes with magnesium sulfate. Circulation. 1988;77(2):392-397.

3. Yap YG, Camm AJ. Durg induced QT prolongation and torsades de pointes. Heart. 2003;89(11):1363-1372.

4. Ozcan MS, Weinberg G. Intravenous lipid emulsion for treatment of drug toxicity. J Intensive Care Med. 2014;29(2):59-70.

5. Weinberg GL. Lipid emulsion infusion: resuscitation for local anesthetic and other drug overdose. Anesthesiology. 2012;117(1):180-187.

6. Sirianni AJ, Osterhoudt KC, Calello DP, et al. Use of lipid emulsion in the resuscitation of a patient with prolonged cardiovascular collapse after overdose of bupropion and lamotrigine. Ann Emerg Med. 2008;51(4):412-415.

7. Cave G, Harvey M. Intravenous lipid emulsion as antidote beyond local anesthetic toxicity: a systematic review. Acad Emerg Med. 2009;16(9):815824.

6. Correct Answer: B Rationale: Alcoholic hallucinosis (AH) and delirium tremens (DTs) share many features, yet it is critical to differentiate the two clinically, as they carry different mortality risks. Presence of hallucinations is the uniting feature of both syndromes. However, where vital signs and sensorium are usually normal in AH, they are both usually abnormal in DTs. These features are summarized in the table below.

Alcoholic hallucinosis Delirium tremens

Hallucinations

VSs

Sensorium

YES—usually visual, sometimes auditory or tactile YES

Normal (usually)

Normal

Abnormal (usually)

CLOUDED— disoriented

inc HR inc BP inc temp

References

1. Adams VM. The effect of alcohol on the nervous system. Res Publ Assoc Nerv Ment Dis. 1953;32:526-573.

2. Gold JA, Nelson LS. Chapter 78: Ethanol withdrawal. In: Nelson LS, Lewin NA, Howland M, et al, eds. Goldfrank’s Toxicologic Emergencies. 9th ed. New York: McGraw-Hill; 2011:1134-1142.

7. Correct Answer: C Rationale: Cocaine is known to act on several different receptors to produce its clinical effects:

1.

2.

Biogenic amine (norepinephrine [NE], epinephrine [epi], dopamine [DA], and serotonin [5-HT]) reuptake inhibition, acting as an indirect sympathomimetic a. Increased levels of catecholamines (NE > epi) leads to increased stimulation of adrenergic receptors (α1 >> α2, β1, β2), which leads to the cardiac and peripheral vasoconstriction frequently seen with cocaine intoxication. b. Increased levels of 5-HT in the CNS lead to the euphoria welldescribed with cocaine intoxication. c. Increased levels of DA in the CNS fuel the reward pathway that frequently leads to cocaine addiction. Sodium (Na+) channel blockade a. Cocaine was the first local anesthetic, discovered (and subsequently abused) by pioneering surgeon, William Stewart Halsted. Like other local anesthetics, cocaine decreases the cell membrane permeability to Na+, thereby blocking conduction. In neurons, this blockade manifests as local anesthesia, while in the

3.

heart it manifests as a widened QRS complex on ECG, and as negative inotropy in cardiac myocytes. Increased glutamate (Glu) concentration a. Increased concentrations of Glu (an excitatory neurotransmitter) in the CNS leads to the psychomotor agitation seen with cocaine intoxication.

Downregulation of GABAA receptors is a well-known change in chronic alcohol use as an adaptation to overstimulation of the receptor by ethanol. In chronic alcohol use, Glu receptors (excitatory) are upregulated in response to increased GABAA (inhibitory) neurotransmission. References

1. Tella SR, Schindler CW, Goldberg SR. Cocaine: cardiovascular effects in relation to inhibition of peripheral neuronal monoamine uptake and central stimulation of the sympathoadrenal system. J Pharmacol Exp Ther. 1993;267(1):153.

2. Smith JA, Mo Q, Guo H, et al. Cocaine increases extraneuronal levels of aspartate and glutamate in the nucleus accumbens. Brain Res. 1995;683(2):264.

3. Most D, Ferguson L, Harris RA. Molecular basis of alcoholism. Handb Clin Neurol. 2014;125:89-111.

8. Correct Answer: B Rationale: While agonists of GABA receptors (ethanol, benzodiazepines, barbiturates) are classically known to be associated with a potentially fatal withdrawal syndrome, opioid withdrawal can also carry a significant mortality rate. Voluntary or involuntary abstinence from opioids (such as answer choice A) is known to result in an uncomfortable withdrawal syndrome, which includes the following signs and symptoms: Symptoms of Opioid Withdrawal Insomnia Yawning Mydriasis Lacrimation

Rhinorrhea Vomiting Diarrhea Piloerection Diaphoresis Myalgias Mild tachycardia and hypertension

In an opioid-dependent patient, an opioid withdrawal that is precipitated by the administration of an opioid antagonist like naloxone differs from one that is brought about by abstinence. One review article purports that “in patients treated for severe pain with an opioid, high-dose naloxone and/or rapidly infused naloxone may cause catecholamine release and consequently pulmonary edema and cardiac arrhythmias.” Fatal opioid withdrawal syndromes have been reported after ultrarapid opioid detoxification (UROD). Experts in the field support the use of naloxone in these patient populations with the desired goal of return of adequate spontaneous ventilation rather than full arousal. Most patients should respond to 0.04 to 0.05 mg of naloxone IV, with “rapid escalation as warranted by the clinical situation.” While a starting dose of 0.4 mg naloxone is appropriate in nonopioid-dependent patients, it should be avoided in opioid-dependent patients. References

1. Nelson LS, Olsen Dean. Chapter 38: Opioids. In: Nelson LS, Lewin NA, Howland M, et al, eds. Goldfrank’s Toxicologic Emergencies. 9th ed. New York: McGraw-Hill; 2011:559-578.

2. Howland M, Nelson LS, Chapter A6: Opioid antagonists. In: Nelson LS, Lewin NA, Howland M, et al, eds. Goldfrank’s Toxicologic Emergencies. 9th ed. New York: McGraw-Hill; 2011:579-585.

3. Bracken MB, Shepard MJ, Collins WF, et al. A randomized controlled trial of methylprednisolone or naloxone in the treatment of acute spinal cord injury. N Engl J Med. 1990;322(20):1405-1411.

4. Van Dorp EL, Yassen A, Dahan A. Naloxone treatment in opioid addiction: the risks and benefits. Expert Opin Drug Safety. 2007;6(2):125-132.

5. Kienbaum P, Thürauf N, Michel MC, Scherbaum N, Gastpar M, Peters J. Profound increase in epinephrine plasma concentration and cardiovascular stimulation following µ-opioid receptor blockade in opioid addicted patients during barbiturate anesthesia for acute detoxification. Anesthesiology. 1998;88(5):1154-1161.

6. Michaelis LL, Hickey PR, Clark TA, et al. Ventricular irritability associated with the use of naloxone hydrochloride. Ann Thorac Surg. 1984;18:608-624.

7. Hamilton RJ, Olmedo RE, Shah S, et al. Complications of ultrarapid opioid detoxification with subcutaneous naltrexone pellets. Acad Emerg Med. 2002;9:63-68.

8. Flacke JW, Flacke WE, Williams GD. Acute pulmonary edema following naloxone reversal of high-dose morphine anesthesia. Anesthesiology. 1977;47:376-378.

9. Prough DS, Roy R, Bumgarner J. Acute pulmonary edema in healthy teenagers following conservative doses of intravenous naloxone. Anesthesiology. 1984;60:485-486.

10. Schwartz JA, Koenigsberg MD. Naloxone-induced pulmonary edema. Ann Emerg Med. 1987;16:1294-1296.

11. Tanaka GY. Hypertensive reaction to naloxone. JAMA. 1974;228:25-26. 9. Correct Answer: A Rationale: This question asks you to differentiate between sympathomimetic (amphetamine) and anticholinergic (oxybutynin) toxidromes. Many similarities exist between the two toxidromes on physical examination, including presence of dilated pupils, tachycardia, warm skin to touch, and a flushed appearance. The provider must look to other examination findings to differentiate the two: Pupil reactivity Bowel sounds Bladder Mucous membranes Perspiration

Anticholinergic

Sympathomimetic

Unreactive Decreased Distended Dry Absent

Reactive Normal Normal Normal Present

Adapted from Vincent JL, Moore FM. Textbook of Critical Care. 7th ed. Philadelphia, PA: Elsevier; 2011.

One may remember the signs and symptoms of anticholinergic (muscarinic) toxicity with the popular mnemonic “blind as a bat, red as a beet, hot as a hare, dry as a bone, made as a hatter.” Mnemonic

Meaning

Blind as a bat

Blurry vision

Red as a beet

Flushed skin (red appearance)

Hot as a hare

Warm skin/hyperpyrexia

Dry as a bone

Dry mucous membranes

Muscarinic Mechanism Loss of ability to accommodate (adjust near and far vision by adjusting lens thickness via contraction of ciliary muscles) Ciliary muscles possess muscarinic receptors, and accommodation is thus blocked in an anticholinergic toxidrome and preserved in a sympathetic toxidrome

Peripheral vasodilation is a compensatory mechanism to facilitate cooling, as cholinergic blockade prevents ability to dissipate heat via sweating (see below) Adrenergic stimulation results in both sweating and peripheral vasoconstriction

Occurs as the patient loses ability to sweat as a cooling mechanism (see below)

Though sweat glands are part of the sympathetic nervous system, they are innervated by parasympathetic nerve fibers (both nicotinic and muscarinic neurotransmission) Anticholinergic overdose would inhibit sweating, while sympathomimetic overdose would preserve this function

Full as a flask

Full urinary bladder/urinary retention

Mad as a hatter

Altered mental status: 
agitation, delirium, psychosis (“Lilliputian hallucinations”)

Muscarinic stimulation will increase detrusor muscle contraction and decrease urethral sphincter contraction (promoting urination) Oxybutinin’s intended effect is to reduce urinary frequency in those with overactive bladder by antagonizing muscarinic receptors in the urinary bladder

By antagonism of CNS cholinergic receptors

References

1. Farmer B, Seger DL. Chapter 153: Poisoning: overview of approaches for evaluation and treatment. In: Vincent J, Abraham E, Moore FA, et al, eds. Textbook of Critical Care. 7th ed. Philadelphia: Elsevir; 2017:10701077.

2. Liebelt EL. Chapter 73: Cyclic antidepressants. In: Nelson LS, Lewin NA, Howland M, et al, eds. Goldfrank’s Toxicologic Emergencies. 9th ed. New York: McGraw-Hill; 2011:1049-1059.

10. Correct Answer: D Rationale: This patient had preexisting right ventricular dysfunction as one may interpret from the echocardiogram. The patient’s RVSP appears to be within normal limits, which can be due to a combination of factors including decompensated RV systolic function (an RV that is to weak to mount a significant forward flow) and volume loading (hypovolemia vs. euvolemia at the time of the examination). Intravenous administration of protamine has several deleterious effects which can include Anaphylaxis Urticaria Bronchospasm

Hypotension Transient increase in pulmonary artery pressure Cardiovascular collapse Death The cellular and molecular mechanisms by which these events occur are not completely understood but are thought to include IgE-mediated reactions Potentiation of IgE-mediated histamine release Direct, nonimmunologic histamine release from mast cells Complement activation Inhibition of serum carboxypeptidase Elevation of thromboxane B2 and 6-keto-PGF1a levels causing pulmonary artery pressure increases Reaction Type

CLINICAL PRESENTATION

Description

Type I

Hypotensive

Likely secondary to histamine release

Type II

Allergic Anaphylactic

Type IIA

Type IIB

Type IIC Type III

Anaphylactoid, early

Anaphylactoid, delayed

Acute pulmonary hypertension

Diagnosed clinically when the following is seen after protamine administration Increased PAP RV failure Decreased systemic BP Etiology unclear Immunological + nonimmunological factors contribute Risk factors Preexisting pulmonary HTN Prior exposure to protamine

or NPH insulin Vasectomy Allergy to vertebrate fish

All of the answer choices and all three types of protamine reactions can cause hypotension; systemic hypotension in a patient with an already failing right ventricle can lead to RV ischemia and cardiac arrest; however the most likely explanation in this patient is a type III reaction, which worsens the patient’s already-ailing right heart. Risk factors for anaphylaxis to or other adverse reactions from protamine include patients with A prior history of protamine-containing insulin (NPH) or IV protamine use (significantly increased risk) A vasectomy, as they have IgE to protamine due to immunologic response to systemic absorption of sperm that occurs after vasectomy (probably at risk) A fish allergy due to cross-reactivity between salmon and protamine (no great evidence) References

1. Pannu BS, Sanghavi DK, Guru PK, Reddy DR, Iyer VN. Fatal right ventricular failure and pulmonary hypertension after protamine administration during cardiac transplantation. Indian J Crit Care Med. 2016;20(3):185-187.

2. Gupta SK, Veith FJ, Ascer E, et al. Anaphylactoid reactions to protamine: An often lethal complication in insulin-dependent diabetic patients undergoing vascular surgery. J Vasc Surg. 1989;9(2):342-350.

3. Nicklas RA, Bernstein IL, Li JT, et al. Protamine. J Allergy Clin Immunol. 1998;101(6):S507-S509.

11. Correct Answer: C Rationale: The superficial branch of the radial nerve is often in close proximity to the cephalic vein of the wrist and can be damaged during blood draws or intravenous cannulation. This often presents as burning pain during the blood draw or peripheral intravenous line insertion. In this case, the

patient likely had damage to the superficial branch of the radial nerve during his IV placement. References

1. Horowitz SH. Peripheral nerve injury and causalgia secondary to routine venipuncture. Neurology. 1994;44(5):962-964.

2. Andrea A, Gonzales JR, Iwanaga J, Oskouian RJ, Tubbs RS. Median nerve palsies due to injections: a review. Cureus. 2017;9(5):e1287. Published 2017 May 29. doi:10.7759/cureus.1287.

12. Correct Answer: B Rationale: This patient presents with the classic tetrad of neuroleptic malignant syndrome (NMS), including fever, rigidity, mental status changes, and autonomic instability in the setting of haloperidol. NMS is associated with first-generation neuroleptic agents such as haloperidol and fluphenazine; however it is implicated in lower-potency and second-generation antipsychotic medications as well. The mainstay of treatment for NMS is stopping the causative agent—in this case, haloperidol. Additionally, patients with NMS should receive supportive care to avoid potentially lifethreatening complications of the syndrome. Reference Tse L, Barr AM, Scarapicchia V, Vila-rodriguez F. Neuroleptic malignant syndrome: a review from a clinically oriented perspective. Curr Neuropharmacol. 2015;13(3):395-406.

13. Correct Answer: D Rationale: The PICC line migrates into the azygos vein. Due to its size (azygos vein is 6-7 mm in diameter while the superior vena cava is typically 1.5-2 cm in diameter) and opposing blood flow, cannulation of azygos vein leads to a higher risk of perforation. It has been reported that up to 19% of azygos vein cannulations led to perforation. Additional potential complications of azygos vein cannulation are often sequelae of perforation and include pleural effusion and hemopericardium. Pneumothorax is not a typical

complication of azygos vein cannulation. References

1. Talari G, Oyewole-eletu S, Talari P, Parasramka S. Migration of peripherally inserted central catheter likely into the azygos vein: a conservative management. BMJ Case Rep. 2016;2016.

2. Langston CT. The aberrant central venous catheter and its complications. Radiology. 1971;100:55-59.

3. Granata A, Figuera M, Castellino S, Logias F, Basile A. Azygos arch cannulation by central venous catheters for hemodialysis. 
J Vasc Access. 2006;7:43-45.

14. Correct Answer: A Rationale: Latest Surviving Sepsis Guidelines 2016 recommend using dynamic indices of volume responsiveness to guide fluid therapy in patients with sepsis. Dynamic parameters of fluid responsiveness are calculated based on measurement of change in stroke volume resulting from breathingbased changes in left ventricular preload. If stroke volume increases significantly from intrathoracic pressure-induced increase in preload, administration of an external fluid bolus will also have the same effect. Since stroke volume is difficult to measure, changes in pulse pressure obtained from an arterial line tracing is a commonly used surrogate. It has been shown that variations in pulse pressure or stroke volume (called pulse pressure variation or stroke volume variation) of a magnitude greater than 15% are associated with positive response to fluid challenge (increase in blood pressure and cardiac output). Conversely, when variations are less than 10%, no hemodynamic benefit to volume resuscitation is seen. These measurements are best validated in patients without respiratory efforts (after neuromuscular blockade) and are on positive-pressure ventilation, as in our patient. Pulse pressure variation is calculated based on the equation:

The patient in the question has a pulse pressure variation of 41%, based on the figure. This favors administration of more intravenous fluids to

treat hypotension. Reference Rhodes A, Evans LE, Alhazzani W, et al. Surviving sepsis campaign: International guidelines for management of sepsis and septic shock: 2016. Crit Care Med. 2017;45(3):486-552.

15. Correct Answer: A Rationale: Tumor lysis syndrome is a disease-related emergency in patients with hematologic cancers. Patients with non-Hodgkin lymphoma or acute leukemia are at the highest risk of developing this syndrome; however its incidence is increasing among other cancers. Tumor lysis syndrome occurs due to tumor cell content release into the bloodstream, which leads to hyperuricemia, hyperkalemia, hyperphosphatemia, and hypocalcemia. These electrolyte derangements can lead to renal failure, cardiac arrhythmias, seizures, and death from multiorgan failure. Reference Howard SC, Jones DP, Pui CH. The tumor lysis syndrome. N Engl J Med. 2011;364(19):1844-1854.

16. Correct Answer: D Rationale: Delirium is a common pathology encountered in the ICU setting. The Confusion Assessment Method for ICU Patients is a validated delirium assessment instrument that allows for rapid, valid, and reliable assessment of delirium, even in mechanically ventilated patients. Combative behavior is not one of the primary features assessed in CAM-ICU.

Reference Ely EW, Inouye SK, Bernard GR, et al. Delirium in mechanically ventilated patients: validity and reliability of the Confusion Assessment Method for the Intensive Care Unit (CAM-ICU). JAMA. 2001;286(21):2703-2710. doi:10.1001/jama.286.21.2703.

17. Correct Answer: D

Rationale: The process of hemostasis is complex and is further complicated in pregnancy due to physiologic changes. In a normal pregnancy, most coagulation factors increase; however it is notable that factor XIII and protein S actually decrease. Hemostatic Parameter

Change at Term Pregnancy (% Change)

Factors II and V Fibrinogen Factor VII Factors VIII, IX, X, XII, and VWF Factor XI Factor XIII Protein C Protein S D-dimer Platelet count

No change Increases more than 100% Up to 1000% increase Increase more than 100% Variable Up to 50% decrease No change Up to 50% decrease Up to 400% increase Up to 20% decrease

From Katz D, Beilin Y. Disorders of coagulation in pregnancy. Br J Anaesth. 2015;115(suppl 2):ii75-ii88.

Reference Katz D, Beilin Y. Disorders of coagulation in pregnancy. Br J Anaesth. 2015;115(suppl 2):ii75-ii88.

18. Correct Answer: B Rationale: Heparin-induced thrombocytopenia (HIT) is an immune-mediated drug reaction caused by antibodies that activate platelets in the setting of heparin administration. Although patients will become thrombocytopenic, bleeding is rare. Thromboembolic complications are more common in HIT and can occur in both arterial and venous vascular systems. The “4 Ts” are commonly used to determine the pretest probability of HIT and include thrombocytopenia, timing of platelet count fall, thrombosis, and other potential causes for thrombocytopenia. Although laboratory testing is important, HIT is a clinical diagnosis. As part of laboratory workup, heparin-PF4 antibody testing is recommended; however, since these results may not be immediately available, treatment should not be delayed pending laboratory confirmation.

Reference Ahmed I, Majeed A, Powell R. Heparin induced thrombocytopenia: diagnosis and management update. Postgrad Med J. 2007;83(983):575582.

19. Correct Answer: D Rationale: Walter Shewhart and Edward Deming’s work on iterative processes later evolved into the four stages of the PDSA cycle. Application of PDSA cycles promotes small-scale interventions in order to provide quicker assessment and increased adaptability for change. In order to drive quality improvement in medicine, methods such as PDSA cycles have been introduced as a tool to use when crafting a quality improvement project. References

1. Deming WE. Out of the crisis, 1986. Cambridge, MA: Massachusetts Institute of Technology Center for Advanced Engineering Study, 1991; xiii, 507.

2. Taylor MJ, Mcnicholas C, Nicolay C, Darzi A, Bell D, Reed JE. Systematic review of the application of the plan-do-study-act method to improve quality in healthcare. BMJ Qual Saf. 2014;23(4):290-298.

20. Correct Answer: A Rationale: In this question, the true positive (TP) is 10, false positive (FP) is 15, true negative (TN) is 70, and false negative (FN) is 5. Positive on TTE Negative on TTE Total

Endocarditis

Not Endocarditis

Total

10 5 15

15 70 85

25 75 100

PPV = TP/(TP + FP) PPV = 10/(10 + 15) = 0.4 or 40% NPV = TN/(TN + FN) NPV = 70/(70 + 5) = 0.93 or 93%

Sensitivity = TP/(TP + FN) Sensitivity = 10/(10 + 5) = 0.67 or 67% Specificity = TN/(TN + FP) Specificity = 70/(70 + 15) = 0.82 or 82% Reference Preoperative assessment, premedication, & perioperative documentation. In: Butterworth JFIV, Mackey DC, Wasnick JD, eds. Morgan & Mikhail’s Clinical Anesthesiology. 6th ed. New York, NY: McGraw-Hill; 2018. Available at: http://accessanesthesiology.mhmedical.com/content.aspx? bookid=2444§ionid=193847252. Accessed June 02, 2019.

21. Correct Answer: D Rationale: The first consensus definition of sepsis was proposed at an international conference in 1992. Sepsis (Sepsis-1) was defined as the identification of two or more SIRS criteria, in addition to a known or suspected infection. SIRS criteria included four clinical signs that, when altered, were thought to induce an inflammatory response: temperature, heart rate, respiratory rate, and white blood cell count. The definition of sepsis was subsequently revised in 2001 (Sepsis-2) by expanding the criteria defining SIRS as well as defining organ dysfunction variables indicative of severe sepsis. In 2016, a new consensus definition was published (Sepsis-3) which defined sepsis as “a life-threatening organ dysfunction caused by dysregulated host response to infection.” In contrast to prior definitions, Sepsis-3 replaced “systemic inflammatory response” with “dysregulated host response,” and “systemic inflammatory response syndrome” (SIRS) has been replaced with “sequential organ failure assessment” (SOFA). The reasons for changing the definition from Sepsis-1 to Sepsis-3 included an overwhelming focus on inflammation in prior definitions; a misleading continuum between sepsis, severe sepsis, and septic shock; SIRS criteria lacking adequate sensitivity and specificity. References

1. Singer M, Deutschman CS, Seymour CW, et al. The third international

consensus definitions for sepsis and septic shock (sepsis-3). JAMA. 2016;315:801-810.

2. Nunnally ME, Patel A. Sepsis – What’s new in 2019? Curr Opin Anaesthesiol. 2019;32:163-168.

22. Correct Answer: D Rationale: Significant discrepancies between POC and central laboratory-analyzed blood glucose values have been reported in patients receiving high-dose vitamin C (mean of 225 vs. 138 mg/dL, respectively). High doses of vitamin C may falsely elevate glucose level readings when measured with certain POC glucometers employing glucose dehydrogenase-pyrroloquinoline quinone amperometric methods. These discrepancies are often inconsistent over time and can range widely. A falsely elevated POC blood glucose measurement may result in overly aggressive insulin therapy, resulting in iatrogenic hypoglycemia, and has been reported to have contributed to patient mortality in at least one case. References

1. Flannery AH, Bastin MLT, Magee CA, Bensadoun ES. Vitamin C in sepsis: when it seems too sweet, it might (literally) be. Chest. 2017;152:450-451.

2. Moskowitz A, Andersen LW, Huang DT, et al. Ascorbic acid, corticosteroids, and thiamine in sepsis: a review of the biologic rationale and the present state of clinical evaluation. Crit Care. 2018;22:283.

3. Kahn SA, Lentz CW. Fictitious hyperglycemia: point-of-care glucose measurement is inaccurate during high-dose vitamin C infusion for burn shock resuscitation. J Burn Care Res. 2015;36:e67-e71.

23. Correct Answer: A Rationale: In sepsis, the host loses local-regional containment of an infection, and the body is systemically exposed to microbes, microbial components, and products of damaged tissue. The innate immune system, composed mainly of macrophages, monocytes, granulocytes, natural killer cells, and

dendritic cells, has evolved to detect PAMPs including components of bacterial, fungal, and viral pathogens (e.g., endotoxin and 9-glucan) and DAMPs which are endogenous molecules released from damaged host cells, (e.g., ATP, mitochondrial DNA, and high mobility group box 1). DAMPs and PAMPs activate the innate immune through pattern recognition receptors (PRRs) on the surface of immune cells (e.g., TLRs or in the cytosol (e.g., NOD-like receptors), initiating transcription of type I interferons and proinflammatory cytokines such as tumor necrosis factor (TNF)-α, interleukin (IL)-1, and IL-6. Some of these pattern recognition receptors (mostly nucleotide-binding oligomerization domain [NOD]-like receptors) can assemble into molecular complexes termed inflammasomes, which are important in the maturation and secretion of the very potent cytokines IL-1β and IL-18, that can trigger programmed cell death termed pyroptosis. References

1. Gotts JE, Matthay MA. Sepsis: pathophysiology and clinical management. BMJ. 2016;353:i1585.

2. Delano MJ, Ward PA. The immune system’s role in sepsis progression, resolution, and long-term outcome. Immunol Rev. 2016;274:330-353.

24. Correct Answer: D Rationale: Acute kidney injury (AKI) is common in severe sepsis and substantially increases morbidity and mortality. Although in the past septic AKI has been attributed to reduced renal perfusion and widespread tubular necrosis, recent studies have challenged this notion, by showing, for example, that AKI occurs in the setting of normal or increased renal blood flow; and that it is characterized not by acute tubular necrosis or apoptosis, but rather by heterogeneous areas of colocalized sluggish peritubular blood flow and tubular epithelial cell oxidative stress. Evidence has also shown that microvascular dysfunction, inflammation, and the metabolic response to inflammatory injury are fundamental pathophysiologic mechanisms that may explain the development of sepsis-induced AKI. References

1. Gómez H, Kellum JA. Sepsis-induced acute kidney injury. Curr Opin Crit Care. 2016;22:546-553.

2. Prowle JR, Bellomo R. Sepsis-associated acute kidney injury: macrohemodynamic and microhemodynamic alterations in the renal circulation. Semin Nephrol. 2015;35:64-74.

25. Correct Answer: B Rationale: Sugammadex is the first selective antagonist to reverse neuromuscular blockade induced by rocuronium and vecuronium. Importantly, sugammadex has the capacity to rapidly and predictably reverse even deep neuromuscular blockade. Administration of sugammadex to the patient in this question will reverse the rocuronium received during intubation thereby facilitating neurologic examination. Acetylcholinesterase inhibitors, such as neostigmine, have traditionally been used for reversal of nondepolarizing neuromuscular blocking agents. However, these drugs have significant limitations, such as indirect mechanisms of reversal, limited and unpredictable efficacy, and undesirable autonomic responses. Flumazenil is a benzodiazepine antagonist, and naloxone is an opioid antagonist. Since the patient did not receive either benzodiazepines or opioids, the administration of these antagonists would not be indicated for this patient. References

1. Ezri T, Boaz M, Sherman A, Armaly M, Berlovitz Y. Sugammadex: an update. J Crit Care Med. 2016;2:16-21.

2. Karalapillai D, Kaufman M, Weinberg L. Sugammadex. Crit Care Resusc. 2013;15:57-62.

3. Pani N, Dongare PA, Mishra RK. Reversal agents in anesthesia and critical care. Indian J Anaesth. 2015;59:664-669.

26. Correct Answer: C Rationale: In 2016, the Society of Critical Care Medicine published ICU Admission, Discharge, and Triage (ADT) Guidelines to provide a comprehensive

framework to guide practitioners in making informed decisions in ADT process. The ADT guidelines contain recommendations classified as “strong” (grade 1) or “weak” (grade 2) based on certainty of the evidence, assessment of the balance of risks and benefits, relevant values and preferences, and burdens and costs of interventions. Guideline recommendations relevant to the patient in this question include: The admission of neurocritically ill patients to a neuro-ICU is suggested, especially for patients with a diagnosis of intracerebral hemorrhage or head injury (grade 2C). Evidence suggests that neurocritically ill patients show improved outcomes when compared with the treatment in a general ICU, especially for intracerebral hemorrhage and head injury. Scoring systems are not recommended for use in isolation to determine ICU admission or discharge because they are not accurate in predicting individual mortality (grade 2C). Minimizing the transfer time of critically ill patients from the emergency department to the ICU (35,000 units/24 hours) to maintain an aPTT or ACT in the therapeutic range. Antithrombin III and antifactor Xa levels can verify the diagnosis and assess whether the current heparin dose is therapeutic. A subtherapeutic aPTT combined with a low ATIII and an anti-Xa level in the therapeutic range would suggest that heparin resistance is present but that the current heparin dose is appropriate. The 4Ts of HIT Variable

Score 20

1

0

Thrombocytopenia

>50% fall, or nadir 
20100,000/mm 3

30%-50% fall, or 
nadir 1019,000/mm 3

Timing of onset after heparin initiation

5-10 days, or 10, or 4 mmol/L, and (5) vasopressors to maintain MAP > 65 mm Hg. Serum procalcitonin is not a required aspect of the bundle. Reference Levy MM, Evans LE, Rhodes A. The surviving sepsis campaign bundle: 2018 update. Crit Care Med. 2018;46(6):997-1000.

74. Correct Answer: B Rationale: The definitions of sepsis, septic shock, and organ dysfunction have largely remained unchanged for more than 2 decades prompting the

development of the Sepsis-3 guidelines. Sepsis is a syndrome defined as life-threatening organ dysfunction caused by a dysregulated host response to infection. SIRS criteria are no longer recommended to diagnose sepsis, since SIRS may simply reflect an appropriate host response which is frequently adaptive. A new screening criteria, qSOFA was therefore coined comprising of three components: altered mentation, systolic BP < 100 mm Hg, and respiratory rate > 22 breaths/min. In patients with a suspicion of infection, qSOFA score >2 identifies patients who are likely to have a prolonged ICU stay or in-hospital mortality, from sepsis. This simple scoring system will allow early identification and triage of patients with (or at risk for) sepsis, enabling early intervention. Lactate is a component of qSOFA assessment. References

1. Singer M, Deutschman CS, Seymour CW, et al. The third international consensus definitions for sepsis and septic shock (sepsis-3). JAMA. 2016;315(8):801-810.

2. Seymour CW, Liu VX, Iwashyna TJ, et al. Assessment of clinical criteria for sepsis: for the third international consensus definitions for sepsis and septic shock (sepsis-3). JAMA. 2016;315(8):762-774.

75. Correct Answer: C Rationale: Early effective fluid resuscitation is crucial for stabilization of sepsisinduced tissue hypoperfusion or septic shock. Early goal-directed therapy, based on the protocol published by Rivers et al., uses a series of “goals” that included CVP and central venous oxygen saturation (option A and B). This approach has now been challenged following the failure to show a mortality reduction in three subsequent large multicenter randomized controlled trials. The same holds true for other static measurements of right or left heart pressures or volumes (option D). Dynamic measures of assessing whether a patient requires additional fluid have been shown to be superior in predicting those patients who will respond to a fluid challenge by increasing stroke volume. After initial resuscitation (30 mL/kg crystalloids over 3 hours), further volume resuscitation in septic shock should be guided by dynamic indices of volume responsiveness. Dynamic indices of volume responsiveness include PLR (option C), fluid challenges

against stroke volume measurements, or the variations in systolic pressure, pulse pressure, or stroke volume to changes in intrathoracic pressure induced by mechanical ventilation. Reference Rhodes A, Evans LE, Alhazzani W, et al. Surviving sepsis campaign: international guidelines for management of sepsis and septic shock: 2016. Crit Care Med. 2017;45(3):486-552.

76. Correct Answer: B Rationale: Warfarin anticoagulation is challenging because of its narrow therapeutic range and the wide variation in interindividual response and consequent requirement to vary dosage. Warfarin is metabolized to inactive form predominantly by hepatic microsomal enzymes (cytochrome P450). Polymorphisms in the CYP2C9 gene affect the rate at which patients metabolize S-warfarin, the most active enantiomer of the drug. Amiodarone can interact with warfarin metabolism by inhibiting CYP2C9. It leads to increased anticoagulant effects of warfarin and thus warrants close monitoring and reduction in warfarin dose (option B). Some studies report effects within 1 week of coadministration, while other report effects many weeks later dependent on the final maintenance dose of amiodarone. Reference Santos PC, Soares RA, Strunz CM, et al. Simultaneous use of amiodarone influences warfarin maintenance dose but is not associated with adverse events. J Manag Care Spec Pharm. 2014;20(4):376-381.

77. Correct Answer: C Rationale: Serratia species can cause a wide spectrum of human infections that can involve the urinary tract, bloodstream, skin and soft tissue, bone, respiratory tract, CNS, and eye. Serratia is associated with nosocomial infections and hospital-associated outbreaks and should prompt evaluation for a potential common source outbreak. Serratia species are intrinsically resistant to ampicillin, amoxicillin, ampicillin-sulbactam,

amoxicillin-clavulanate, narrow-spectrum cephalosporins, cephamycins, cefuroxime, macrolides, tetracyclines, nitrofurantoin, and colistin. S. marcescens and several other Serratia species encode an inducible, chromosomal AmpC beta-lactamase. AmpC gene–mediated resistance is seen in Serratia, Pseudomonas, Acinetobacter, Citrobacter, and Enterobacter (SPACE ​organisms). It mediates resistance to several beta-lactam antibiotics, such as penicillins and first-generation cephalosporins (e.g., cefazolin), but resistance to later-generation cephalosporins may not be detected on initial antibiotic susceptibility tests. However, treatment with third-generation cephalosporins usually leads to selection of mutants which are resistant to higher-generation cephalosporins leading to treatment failures. This is especially important in CNS infections, where the risk of emergence of AmpC-mediated resistance during therapy is high due to need for prolonged antibiotic therapy. Hence in this patient, though resistance patterns show susceptibility to ceftriaxone, the potential for inducible resistance is high. So, escalating to meropenem is the best option. Vancomycin can be stopped since no grampositive organisms were isolated. Reference Meini S, Tascini C, Cei M, et al. AmpC β-lactamase-producing Enterobacterales: what a clinician should know. Infection. 2019;47(3):363-375.

78. Correct Answer: C Rationale: This patient likely has ventilator-associated pneumonia. Stenotrophomonas maltophilia is a multidrug-resistant gram-negative bacillus that is an opportunistic pathogen, particularly among hospitalized patients. Due to widespread multidrug-resistance, antibiotic options are limited and clinical data are limited regarding optimal therapy. High levels of resistance are seen to penicillin, cephalosporins, and carbapenems. More than 95% of the isolates are still susceptible to trimethoprimsulfamethoxazole and thus is recommended as first-line therapy. Alternatives include fluroquinolones, minocycline, and tigecycline. Vancomycin and cefepime needs to be continued until respiratory culture data are finalized, since multiple pathogens can be present in ventilator-

associated pneumonia. Reference Brooke JS. Stenotrophomonas maltophilia: an emerging global opportunistic pathogen. Clin Microbiol Rev. 2012;25(1):2-41.

79. Correct Answer: D Rationale: In many instances, a report from the clinical laboratory indicating candiduria represents colonization or procurement contamination of the specimen and not invasive candidiasis. Even if infection of the urinary tract by Candida species can be confirmed, antifungal therapy is not always warranted. When candiduria is found, changing or removing the catheter can be anticipated to clear the candiduria in 20% to 40% of individuals (option D). Elimination of predisposing factors, such as discontinuing antibiotics that are no longer necessary and removal of indwelling bladder catheters, is recommended whenever feasible. If candiduria fails to resolve despite these measures, a more deep-seated infection should be suspected, and imaging of the kidneys and collecting system is indicated. Treatment with antifungal agents is not recommended unless the patient belongs to a group at high risk for dissemination, that is, neutropenic patients, very low-birth-weight infants, and patients who will undergo urologic manipulation. Reference Pappas PG, Kauffman CA, Andes DR, et al. Clinical Practice Guideline for the Management of Candidiasis: 2016 Update by the Infectious Diseases Society of America. Clin Infect Dis. 2016;62(4):e1-e50.

80. Correct Answer: A Rationale: Chimeric antigen receptor therapy targeting CD19 is an effective treatment for refractory B-cell malignancies, especially acute lymphoblastic leukemia. As immune-based therapies for cancer become potent, more effective, and more widely available, optimal management of their unique toxicities becomes increasingly important. Cytokine release syndrome

(CRS) is a potentially life-threatening toxicity that has been observed following administration of natural and bispecific antibodies and, more recently, following adoptive T-cell therapies for cancer. CRS is characterized by fever, hypotension, and respiratory insufficiency associated with elevated serum cytokines, including interleukin-6 (IL-6). Tocilizumab is anti–IL-6 receptor monoclonal antibody approved for treatment of rheumatoid arthritis, juvenile idiopathic arthritis, and polyarticular juvenile rheumatoid arthritis. Retrospective data have shown that immunosuppression using tocilizumab with or without corticosteroids is effective against CRS and has been approved (by FDA) as the first-line therapy against severe life-threatening CRS (option A). Abciximab is a glycoprotein IIb/IIIa receptor antagonist used as an antiplatelet agent. Intravenous immunoglobulin and plasma exchange has no current role in treatment of CRS. References

1. Lee DW, Gardner R, Porter DL, et al. Current concepts in the diagnosis and management of cytokine release syndrome. Blood. 2014;124(2):188195.

2. Le RQ, Li L, Yuan W, et al. FDA approval summary: tocilizumab for treatment of chimeric antigen receptor t cell-induced severe or lifethreatening cytokine release syndrome. Oncologist. 2018;23(8):943-947.

81. Correct Answer: D Rationale: Thyroid function should not be assessed in seriously ill patients unless there is a strong suspicion of thyroid dysfunction. However, when measured, most with critical illness have low serum concentrations of both thyroxine (T4) and triiodothyronine (T3). Serum thyroid-stimulating hormone (TSH) concentration also may be low in patients who may have acquired transient central hypothyroidism. It is possible that the changes in thyroid function during severe illness are protective in that they prevent excessive tissue catabolism. Thyroid hormone metabolism pathways are depicted below, which will help understand thyroid dysfunction of critical illness.

T3 is the active form of thyroid hormone. T4 is converted to T3 by 5’ monodeiodinase and rT3 by 5 monodeiodinase. normally. Majority of T4 (90%) is converted to T3, the active form of the hormone. But in critical illness, cytokines and cortisol inhibit 5’ monodeiodinase, leading to low T3 levels. On the other hand, rT3 is the product of 5-monodeiodination of T4, and 5 monodeiodinase is induced in (nonthyroid) critical illness, especially in the setting of hypoxia or ischemia. This leads to high levels of rT3 and lower levels of T3. Also, total T4 is low in critical illness due to reduction in binding proteins (thyroxine-binding globulin, transthyretin, and albumin). Reference Van den Berghe G. Non-thyroidal illness in the ICU: a syndrome with different faces. Thyroid. 2014;24(10):1456-1465.

82. Correct Answer: B Rationale: Transvenous lead extractions are associated with a consistent rate of significant procedure-related complications. Some of them are lifethreatening and occur from damage to structures adherent to the pacemaker lead. These complications include vascular laceration,

pericardial effusion, cardiac tamponade, cardiac arrest, and tricuspid regurgitation, among others. A small, nonexpanding, hemodynamically insignificant pericardial effusion can be managed expectantly with close observation in the intensive care unit. Hemodynamically significant effusion with tamponade physiology requires immediate percutaneous pericardial drainage or sternotomy. Echocardiographic monitoring is instrumental in diagnosing cardiac tamponade in these patients. Important echocardiographic features of tamponade are right atrial collapse during systole and right ventricular collapse during diastole. The parasternal long-axis view of this patient shows a circumferential pericardial effusion. However, the image is in diastole (since aortic valve is closed), but WITHOUT right ventricular collapse. Another important feature of cardiac tamponade is jugular venous distension or elevated CVP. American Society of Echocardiography uses the following criteria to estimate right atrial pressure (CVP) in spontaneously breathing patients based on IVC diameter and collapsibility. IVC Diameter

IVC Collapsibility

Estimated Right Atrial Pressure

50%

3 mm Hg

2.1 cm >2.1 cm

50%